You are on page 1of 94

TOPNOTCH MEDICAL BOARD PREP ANATOMY SUPEREXAM

For inquiries visit www.topnotchboardprep.com.ph or email us at topnotchmedicalboardprep@gmail.com


DEAR TOPNOTCH FRIENDS:

PLEASE FOLLOW THESE INSTRUCTIONS:

1. These questions are previous diagnostic, midterm, and finals exams of Topnotch, almost all of them made by Topnotch Board Exam Topnotchers.
2. Answer this Topnotch Superexam seriously 100-items at a time. Cover the “Explanations” Column. Do not immediately look at the answers from the
answer key. That’s not the correct way of answering sample exams. You need to treat these MCQs as exercises and not as handouts.
3. Time yourself. 1.5 hours per 100-item block.
4. After answering each 100-item block, refer to the Topnotch Answer Key for the correct answers. Please be careful of “frameshift mutations” when
checking your answers – check every 10 items. (the format of the answer key was designed for you to practice against “frameshift mutations”)
5. The Topnotch Superexams are EXERCISES for the actual med boards. They will not appear verbatim in your future exams. More than knowing what’s
the correct answer, it’s more important for you to:
a. Know why the other choices are wrong
b. Know why the other choices were included in the first place
c. Know the explanation to the correct answer
6. Sharpen your mind by answering the Topnotch Superexams. Most of these questions based on past feedback are more difficult than the actual questions
in the med boards. In these exams made by Board Exam Topnotchers, if you’re getting a score of 60/100 , that’s already a good score. More than 80/100
is outstanding.

Item QUESTION EXPLANATION AUTHOR TOPNOTCH
# EXAM
1 Patient had improperly fitted axillary crutch and suffered The affected nerve with improperly fitted crutches KRISTEL DIAGNOSTIC
injury to a branch of the brachial plexus. Which is the most is the radial nerve. Yes, the first impulse for this TANHUI EXAM -
likely finding? question is to think axillary nerve is the answer (TOP 3 - AUG MARCH
A. Weakness of arm abduction because it’s an axillary crutch. BUT THAT’S 2015 MED 2016
B. Hand of benediction WRONG. The correct answer is radial nerve! BOARDS;
C. Ape hand TOPNOTCH
D. Cyclist palsy The axillary nerve, with the posterior humeral MD FROM LA
E. Weakness in extending the forearm and wristdrop circumflex artery actually courses much higher SALLE)
E posteriorly via the quadrangular space.
The radial nerve, along with the deep brachial
artery courses posteriorly via the triangular
interval and is more susceptible to injury from the
axilla.

Weakness of arm abduction - axillary nerve
Hand of benediction – Median nerve
Ape hand – median nerve
Cyclist palsy – ulnar nerve
Weakness in extending the forearm – radial nerve

Source: Topnotch handout and pearls in anatomy

2 Which of the following is not involved in the transmission of The fibula is just a bone for anchoring muscles. In KRISTEL DIAGNOSTIC
weight while standing? fact, it’s so insignificant, you can use it for bone TANHUI EXAM -
A. Sacrum grafting! The part used for grafting is the middle (TOP 3 - AUG MARCH
E B. Pelvis
C. Femur
third because this contains the nutrient artery.

2015 MED
BOARDS;
2016

D. Tibia Source: Moore Clinically oriented anatomy 4th ed TOPNOTCH


E. Fibula Chapter 5 (Ebook) MD FROM LA
SALLE)

3 Which is the equivalent layer of the Deep investing fascia of I can’t fit a table here but the ideal way to KRISTEL DIAGNOSTIC
the abdomen in the perineum? memorize the following is to put them in a table TANHUI EXAM -
A. Colles’ fascia side by side: (TOP 3 - AUG MARCH
B B. Gallaudet’s fascia 2015 MED 2016
C. Scarpa’s fascia Abdomen (superficial to deep) BOARDS;
D. Dartos fascia Camper’s fascia (fatty superficial layer) TOPNOTCH
E. Buck’s fascia Scarpa’s fascia (membranous superficial layer) MD FROM LA
Deep investing fascia SALLE)

Perineum (superficial to deep)
Fatty superficial layer
Colles’ fascia (membranous superficial layer)
Gallaudet’s fascia/investing fascia

Penis (superficial to deep)
Dartos muscle
Dartos fascia
Buck’s fascia

Source: Moore Clinically oriented anatomy 4th ed
Chapter 2 and 3 (Ebook)

TOPNOTCH MEDICAL BOARD PREP ANATOMY SUPEREXAM Page 1 of 94


For inquiries visit www.topnotchboardprep.com.ph or email us at topnotchmedicalboardprep@gmail.com
TOPNOTCH MEDICAL BOARD PREP ANATOMY SUPEREXAM
For inquiries visit www.topnotchboardprep.com.ph or email us at topnotchmedicalboardprep@gmail.com
Item QUESTION EXPLANATION AUTHOR TOPNOTCH
# EXAM
4 Which of the following is false of the Greater pelvis? Lesser pelvis (true pelvis, pelvis minor) – below KRISTEL DIAGNOSTIC
A. Obstetric significance the arcuate line, obstetric significance TANHUI EXAM -
B. Lies above the linea terminales (TOP 3 - AUG MARCH
C. Pelvis major Source: Moore Clinically oriented anatomy 4th ed 2015 MED 2016
A D. False pelvis
E. None of the above
Chapter 3 (Ebook) BOARDS;
TOPNOTCH
MD FROM LA
SALLE)

5 What is the shape of the left adrenal gland? Left crescent, Right triangle KRISTEL DIAGNOSTIC
A. Triangular TANHUI EXAM -
B. Kidney For those who are math geeks, my mnemonic is (TOP 3 - AUG MARCH
C. Crescent “right triangle” (a triangle with a right angle in it) 2015 MED 2016
C D. Oval BOARDS;
E. Round Source: Moore Clinically oriented anatomy 4th ed TOPNOTCH
Chapter 2 (Ebook) MD FROM LA
SALLE)

6 Which of the following cartilages is pyramidal in shape? Arytenoid: 3 sided pyramid KRISTEL DIAGNOSTIC
A. Epiglottis Apex superiorly – bears corniculate cartilage and TANHUI EXAM -
B. Arytenoid attaches to aryepiglottic fold (TOP 3 - AUG MARCH
C. Corniculate Vocal process anteriorly – posterior attachement 2015 MED 2016
D. Cuneiform for vocal cord BOARDS;
B E. Thyroid Muscular process laterally – attachment for TOPNOTCH
posterior and lateral cricoarytenoid muscle MD FROM LA
Base: cricoarytenoid joint SALLE)

Source: Moore Clinically oriented anatomy 4th ed
Chapter 8 (Ebook)

7 Which of the following is not true of the quadrangular The vestibular ligament forms the inferior border KRISTEL DIAGNOSTIC
membrane? of the quadrangular membrane. TANHUI EXAM -
A. Extends from the lateral aspects of the arytenoid and (TOP 3 - AUG MARCH
epiglottic cartilage Source: Moore Clinically oriented anatomy 4th ed 2015 MED 2016
E B. The aryepiglottic ligament/fold forms the free superior Chapter 8 (Ebook) BOARDS;
margin TOPNOTCH
C. The vestibular ligament/fold forms the free inferior MD FROM LA
margin SALLE)
D. The vestibular fold forms the false vocal cord
E. None of the above

8 What do you call the junction where the nasal bones and SIMILAR TO PREVIOUS BOARD EXAM KRISTEL DIAGNOSTIC
D frontal bones meet?
A. Inion
CONCEPT/PRINCIPLE.

TANHUI
(TOP 3 - AUG
EXAM -
MARCH
B. Rhinion Source: Moore Clinically oriented anatomy 4th ed 2015 MED 2016
C. Glabella Chapter 7 (Ebook) BOARDS;
D. Nasion TOPNOTCH
E. Pterion MD FROM LA
SALLE)

9 FSH is secreted by which type of cell in the pituitary gland? Acidophils – Growth hormone, prolactin KRISTEL DIAGNOSTIC
A. Acidophils Basophils – FSH, LH, TSH, ACTH TANHUI EXAM -
B B. Basophils Supraoptic nuclei – Vasopressin mainly (TOP 3 - AUG MARCH
C. Chromophobes Paraventricular nuclei – Oxytocin mainly 2015 MED 2016
D. Supraoptic nuclei BOARDS;
E. Paraventricular nuclei Source: Topnotch TOPNOTCH
MD FROM LA
SALLE)

10 A trauma patient suffers a tear at the junction of the SVC and The SA node is located in the right atrium at the KRISTEL DIAGNOSTIC
the right atrium. This tear would likely damage the: junction where it is joined by the SVC. TANHUI EXAM -
A. SA node (TOP 3 - AUG MARCH
A B. AV node Source: Moore Clinically oriented anatomy 4th ed 2015 MED 2016
C. AV bundle Chapter 1 (Ebook BOARDS;
D. Right bundle branch TOPNOTCH
E. Left bundle branch MD FROM LA
SALLE)

11 The lowest level of the lung on end expiration at the Lungs – 6-8-10 KRISTEL DIAGNOSTIC
midaxillary line is? Pleura – 8-10-12 TANHUI EXAM -
C A. 6th intercostal space (TOP 3 - AUG MARCH
B. 7th intercostal space Source: Topnotch handout on anatomy 2015 MED 2016
C. 8th intercostal space BOARDS;
D. 9th intercostal space TOPNOTCH
E. 10th intercostal space MD FROM LA
SALLE)

TOPNOTCH MEDICAL BOARD PREP ANATOMY SUPEREXAM Page 2 of 94


For inquiries visit www.topnotchboardprep.com.ph or email us at topnotchmedicalboardprep@gmail.com
TOPNOTCH MEDICAL BOARD PREP ANATOMY SUPEREXAM
For inquiries visit www.topnotchboardprep.com.ph or email us at topnotchmedicalboardprep@gmail.com
Item QUESTION EXPLANATION AUTHOR TOPNOTCH
# EXAM
12 While doing a spinal tap, which describes the most accurate Source: Topnotch handout on anatomy KRISTEL DIAGNOSTIC
sequence of layers from the skin to the subarachnoid space? TANHUI EXAM -
A. Skin, interspinous ligament, dura mater, subarachnoid (TOP 3 - AUG MARCH
space 2015 MED 2016
B. Skin, supraspinous ligament, interspinous ligament, BOARDS;
C posterior longitudinal ligament, dura mater, subarachnoid TOPNOTCH
space MD FROM LA
C. Skin, supraspinous ligament, interspinous ligament, dura SALLE)
mater, subarachnoid space
D. Skin supraspinous ligament, intertransverse ligament,
arachnoid space, subarachnoid space
E. Skin, interspinous ligament, anterior longitudinal
ligament, dura mater, subarachnoid space

13 A 5-year-old male presented with painless testicular mass. If Tumor cells from the gonads that metastasize KRISTEL DIAGNOSTIC
the mass proved to be a malignancy, the earliest affected through the lymphatics metastasize to the lumbar TANHUI EXAM -
lymph nodes would be: nodes initially. Remember that the origin of the (TOP 3 - AUG MARCH
A. Lumbar (aortic) lymph nodes gonadal arteries is the aorta, therefore the 2015 MED 2016
B. Inferior mesenteric nodes lymphatics will also course similarly. BOARDS;
C. Deep inguinal nodes TOPNOTCH
A D. Common iliac nodes Source: Moore Clinically oriented anatomy 4th ed MD FROM LA
E. Superficial inguinal nodes Chapter 2 (Ebook) SALLE)

14 A surgeon is about to make an incision through the fundus of Lesser curvature – right and left gastric artery KRISTEL DIAGNOSTIC
the stomach, which of the following requires clamping to Greater curvature – right and left gastroomental TANHUI EXAM -
prevent bleeding? artery (TOP 3 - AUG MARCH
A. Right gastric artery Fundus – short gastric artery 2015 MED 2016
E B. Left gastric artery
C. Right gastroomental artery

Source: Moore Clinically oriented anatomy 4th ed
BOARDS;
TOPNOTCH
D. Left gastroomental artery Chapter 2 (Ebook) MD FROM LA
E. Short gastric artery SALLE)

15 A surgeon is about to perform cholecystectomy. The Source: Moore Clinically oriented anatomy 4th ed KRISTEL DIAGNOSTIC
gallbladder can be localized in its fossa between which 2 Chapter 2 (Ebook) TANHUI EXAM -
anatomical lobes? (TOP 3 - AUG MARCH
A. Quadrate and left lobes 2015 MED 2016
C B. Quadrate and caudate lobes BOARDS;
C. Right and quadrate lobes TOPNOTCH
D. Caudate and right lobes MD FROM LA
E. Caudate and left lobes SALLE)

16 The right pulmonary artery: There are 2 pulmonary arteries – 1 right and 1 left. KRISTEL DIAGNOSTIC
A. Are 2 in number There are 4 pulmonary veins. TANHUI EXAM -
B. Are 4 in number (TOP 3 - AUG MARCH
C C. Courses posterior to the ascending aorta and the SVC
D. B and C
The right pulmonary artery courses posterior to
the ascending aorta and SVC!
2015 MED
BOARDS;
2016

E. None of the above TOPNOTCH


Source: Moore Clinically oriented anatomy 4th ed MD FROM LA
Chapter 1 (Ebook) SALLE)

17 What happens in May-Thurner syndrome? This condition predisposes to Deep vein KRISTEL DIAGNOSTIC
A. The aorta compresses the IVC thrombosis. TANHUI EXAM -
B. The left common iliac vein is compressed by the left (TOP 3 - AUG MARCH
D common iliac artery
C. The right common iliac vein is compressed by the right
2015 MED
BOARDS;
2016

common iliac artery TOPNOTCH


D. The left common iliac vein is compressed by the right MD FROM LA
common iliac artery SALLE)
E. The right common iliac vein is compressed by the left
common iliac artery

18 Which of the following is a syndesmosis? Source: Topnotch handout on anatomy KRISTEL DIAGNOSTIC
E A. Interosseus membrane
B. Gomphosis
TANHUI
(TOP 3 - AUG
EXAM -
MARCH
C. Distal tibiofibular joint 2015 MED 2016
D. A and B BOARDS;
E. All of the above TOPNOTCH
MD FROM LA
SALLE)

TOPNOTCH MEDICAL BOARD PREP ANATOMY SUPEREXAM Page 3 of 94


For inquiries visit www.topnotchboardprep.com.ph or email us at topnotchmedicalboardprep@gmail.com
TOPNOTCH MEDICAL BOARD PREP ANATOMY SUPEREXAM
For inquiries visit www.topnotchboardprep.com.ph or email us at topnotchmedicalboardprep@gmail.com
Item QUESTION EXPLANATION AUTHOR TOPNOTCH
# EXAM
19 Kiesselbach’s plexus is vascular anastomosis on the anterior Arterial supply KRISTEL DIAGNOSTIC
part of the nose. Juan dela Cruz damaged this plexus while Sphenopalatine artery (Maxillary artery) TANHUI EXAM -
picking his nose. Which of the following is least likely to Anterior and posterior ethmoidal artery (TOP 3 - AUG MARCH
contribute to the epistaxis? (ophthalmic artery branch) 2015 MED 2016
C A. Ophthalmic artery Greater palatine artery (Maxillary artery) BOARDS;
B. Maxillary artery Superior labial artery and lateral nasal branches of TOPNOTCH
C. Mandibular artery the facial artery MD FROM LA
D. Facial artery SALLE)
E. Ethmoidal artery Source: Moore Clinically oriented anatomy 4th ed
Chapter 7 (Ebook)

20 Which of the following is has a brush border lining its lumen? The brush border differentiates the proximal from KRISTEL DIAGNOSTIC
A. Small intestine the distal convoluted tubule. TANHUI EXAM -
D B. Proximal convoluted tubule (TOP 3 - AUG MARCH
C. Distal convoluted tubule Source: Topnotch handout on anatomy 2015 MED 2016
D. A and B BOARDS;
E. All of the above TOPNOTCH
MD FROM LA
SALLE)

21 The inner ear is housed within which bone? SIMILAR TO PREVIOUS BOARD EXAM LESTER MIDTERM 1
A. Temporal CONCEPT/PRINCIPLE. A few ENT questions were BRYAN CO EXAM -
B. Sphenoid present in our Anatomy exam, and were generally (TOP 10 - MARCH
C. Ethmoid tougher than the anatomy questions. AUG 2015 2016
A D. Maxillary MED
E. Zygomatic BOARDS;
TOPNOTCH
MD FROM
UST)
22 Which type of pelvis presents with a larger AP diameter SIMILAR TO PREVIOUS BOARD EXAM LESTER MIDTERM 1
compared to the transverse diameter? CONCEPT/PRINCIPLE. Anthropoid=AP BRYAN CO EXAM -
C A. Gynecoid diameter>transverse; Platypelloid (TOP 10 - MARCH
B. Android ("flatypelloid")=transverse diameter>AP diameter. AUG 2015 2016
C. Anthropoid Both came out in 2 seaparate questions during our MED
D. Platypelloid Anatomy exam. BOARDS;
E. Arthropod TOPNOTCH
MD FROM
UST)
23 Correct boundaries of the greater sciatic foramen except: SIMILAR TO PREVIOUS BOARD EXAM LESTER MIDTERM 1
A. anterolaterally by the greater sciatic notch CONCEPT/PRINCIPLE. The piriformis muscle BRYAN CO EXAM -
B. posteromedially by the sacrotuberous ligament divides the greater sciatic foramen into a (TOP 10 - MARCH
C. inferiorly by the sacrospinous ligament and ischial spine suprapiriform and infrapiriform foramen. There AUG 2015 2016
D. inferolaterally by the piriformis muscle were about 3 questions about the greater sciatic MED
D E. superiorly by the anterior sacroilliac ligament notch in our Anatomy exam. Expect a few difficult BOARDS;
questions about pelvic anatamy TOPNOTCH
MD FROM
UST)
24 Secretes surfactant in the lung: SIMILAR TO PREVIOUS BOARD EXAM LESTER MIDTERM 1
A. Type I alveolar CONCEPT/PRINCIPLE. The few histology BRYAN CO EXAM -
B. Type II alveolar cells questions asked during our anatomy exam were (TOP 10 - MARCH
C. Kulchitsky cells generally easy. AUG 2015 2016
D. Clara cells MED
B E. B, C, D are correct. BOARDS;
TOPNOTCH
MD FROM
UST)
25 A 63 year-old male smoker develops an apical sulcus tumor SIMILAR TO PREVIOUS BOARD EXAM LESTER MIDTERM 1
that compresses the brachial plexus and cervical stellate CONCEPT/PRINCIPLE. Thoracic outlet and inlet BRYAN CO EXAM -
ganglion. This may lead to: syndrome generally pertain to the same thing. (TOP 10 - MARCH
A. Pancoast syndrome Horner syndrome involves the cervical AUG 2015 2016
A B. Superior vena cava syndrome sympathetic trunk. 2 questions about this topic in MED
C. Horner's syndrome our Anatomy exam. BOARDS;
D. Thoracic outlet syndrome TOPNOTCH
E. Thoracic inlet syndrome MD FROM
UST)

26 To pass a needle into the pleural space in the midaxillary LESTER MIDTERM 1
line, the following structures will have to be pierced except? BRYAN CO EXAM -
A. Internal intercostals (TOP 10 - MARCH
B. Levatores costarum AUG 2015 2016
B C. External intercostals MED
D. Parietal pleura BOARDS;
E. Innermost intercostals TOPNOTCH
MD FROM
UST)
27 On percussing the anterior chest of a patient, you find the The right border of the heart is formed by the LESTER MIDTERM 1
right margin of the heart to lie 5 cm to right of the edge of the right atrium. Inferior border is mostly the right BRYAN CO EXAM -
sternum. Which chamber of the heart is likely to be ventricle with the apex of the left ventricle (TOP 10 - MARCH
enlarged? A. left ventricle B. left atrium C. right ventricle D. AUG 2015 2016
D right atrium E. both ventricles MED
BOARDS;
TOPNOTCH
MD FROM
UST)

TOPNOTCH MEDICAL BOARD PREP ANATOMY SUPEREXAM Page 4 of 94


For inquiries visit www.topnotchboardprep.com.ph or email us at topnotchmedicalboardprep@gmail.com
TOPNOTCH MEDICAL BOARD PREP ANATOMY SUPEREXAM
For inquiries visit www.topnotchboardprep.com.ph or email us at topnotchmedicalboardprep@gmail.com
Item QUESTION EXPLANATION AUTHOR TOPNOTCH
# EXAM
28 The following structures form the walls of the inguinal canal LESTER MIDTERM 1
except? BRYAN CO EXAM -
A. Conjoint tendon (TOP 10 - MARCH
D B. Aponeurosis of external obliique muscle AUG 2015 2016
C. Internal oblique muscle MED
D. Lacunar ligament BOARDS;
E. Fascia transversalis TOPNOTCH
MD FROM
UST)
29 To pass a needle into the cavity of the tunica vaginalis in the SIMILAR TO PREVIOUS BOARD EXAM LESTER MIDTERM 1
scrotum, the following structures have to be pierced except? CONCEPT/PRINCIPLE. The tunica albuginea is the BRYAN CO EXAM -
C A. Dartos muscle fibrous covering of the testis. There were also (TOP 10 - MARCH
B. Colles' fascia other questions about Colles's and Scarpa's fascia. AUG 2015 2016
C. Tunica albuginea MED
D. Internal spermatic fascia BOARDS;
E. Cremasteric fascia TOPNOTCH
MD FROM
UST)
30 The hilum of the right kidney contains the following The right suprarenal adrenal gland does not LESTER MIDTERM 1
important structures except? extend downward to the hilum of the right kidney. BRYAN CO EXAM -
D A. Renal pelvis
B. Tributaries of right renal vein
(TOP 10 -
AUG 2015
MARCH
2016
C. Sympathetic nerve fibers MED
D. Part of the right suprarenal gland BOARDS;
E. Branches of the right renal artery TOPNOTCH
MD FROM
UST)
31 A 53-year-old man has difficulty with breathing through his The posterior ethmoidal sinus opens into the LESTER MIDTERM 1
nose. On examination, his physician finds that he has superior nasal meatus. The maxillary, frontal, and BRYAN CO EXAM -
swelling of the mucous membranes of the superior nasal anterior and middle ethmoidal sinuses drain into (TOP 10 - MARCH
C meatus. Which opening of the paranasal sinuses is most the middle nasal meatus. AUG 2015 2016
likely plugged? MED
A. Middle ethmoidal sinus BOARDS;
B. Maxillary sinus TOPNOTCH
C. Posterior ethmoidal sinus MD FROM
D. Anterior ethmoidal sinus UST)
E. Frontal sinus

32 An oncologist is reviewing a CT scan of a 74-year-old man The quadrate lobe of the liver drains bile into the LESTER MIDTERM 1
with newly diagnosed hepatocellular carcinoma. He locates left hepatic duct and receives blood from the left BRYAN CO EXAM -
the affected quadrate lobe of the liver that: hepatic artery. It lies between the gallbladder (TOP 10 - MARCH
C A. Lies between the IVC and ligamentum venosum
B. Receives blood from the right hepatic artery
fossa and the ligamentum teres hepatic, is a medial
inferior segment, and is a part of the left lobe.
AUG 2015
MED
2016

C. Drains bile into the left hepatic duct BOARDS;


D. Is a medial superior segment TOPNOTCH
E. Is functionally a part of the right lobe MD FROM
UST)

33 In a patient with portal hypertension, which of the following The right colic vein belongs to the portal venous LESTER MIDTERM 1
veins is most likely to be dilated? system and empties into the superior mesenteric BRYAN CO EXAM -
A. Right colic vein vein, which joins the splenic vein to form the (TOP 10 - MARCH
B B. Inferior epigastric vein portal vein. The inferior epigastric, AUG 2015 2016
C. Inferior phrenic vein inferior phrenic, suprarenal, and ovarian veins MED
D. Suprarenal vein belong to the systemic (or caval) venous system BOARDS;
E. Ovarian vein and drain directly or indirectly into the IVC. TOPNOTCH
MD FROM
UST)
34 A 2-year-old boy presents with pain in his groin that has The deep inguinal ring lies in the transversalis LESTER MIDTERM 1
been increasing over the past few weeks. He is found to have fascia, just lateral to the inferior epigastric vessels. BRYAN CO EXAM -
a The superficial inguinal ring is in the aponeurosis (TOP 10 - MARCH
degenerative malformation of the transversalis fascia during of the external oblique muscle. The inguinal AUG 2015 2016
development. Which of the following structures on the ligament and the anterior wall of the inguinal MED
B anterior abdominal wall is likely defective? canal are formed by the aponeurosis of the BOARDS;
A. Superficial inguinal ring external oblique muscle. The sac of a direct TOPNOTCH
B. Deep inguinal ring inguinal hernia is formed by the peritoneum. MD FROM
C. Inguinal ligament UST)
D. Sac of a direct inguinal hernia
E. Anterior wall of the inguinal canal

35 A pediatric surgeon is resecting a possible malignant mass The left umbilical vein becomes the round LESTER MIDTERM 1
from the liver of a neonate with cerebral palsy. The surgeon ligament of the liver after birth. The right BRYAN CO EXAM -
divides the round ligament of the liver during surgery. A umbilical vein does not leave a fibrous remnant (TOP 10 - MARCH
fibrous remnant of which of the following fetal vessels is because it degenerates during the early embryonic AUG 2015 2016
C severed?
A. Ductus venosus
period. The ductus venosus forms the ligamentum
venosum; the ductus arteriosus forms the
MED
BOARDS;
B. Ductus arteriosus ligamentum arteriosum; the umbilical artery TOPNOTCH
C. Left umbilical vein forms the medial umbilical MD FROM
D. Right umbilical vein ligament. UST)
E. Umbilical artery

TOPNOTCH MEDICAL BOARD PREP ANATOMY SUPEREXAM Page 5 of 94


For inquiries visit www.topnotchboardprep.com.ph or email us at topnotchmedicalboardprep@gmail.com
TOPNOTCH MEDICAL BOARD PREP ANATOMY SUPEREXAM
For inquiries visit www.topnotchboardprep.com.ph or email us at topnotchmedicalboardprep@gmail.com
Item QUESTION EXPLANATION AUTHOR TOPNOTCH
# EXAM
36 A 33-year-old patient is suffering from a sudden occlusion at The first two posterior intercostal arteries are LESTER MIDTERM 1
the origin of the descending (thoracic) aorta. This condition branches of the highest (superior) intercostal BRYAN CO EXAM -
would most likely decrease blood flow in which of the artery of the costocervical trunk; the remaining (TOP 10 - MARCH
following intercostal arteries? nine branches are from the thoracic aorta. The AUG 2015 2016
A. Upper six anterior internal thoracic artery gives off the upper six MED
B. All of the posterior
anterior intercostal arteries and is divided into the BOARDS;
E C. Upper two posterior superior epigastric and musculophrenic arteries, TOPNOTCH
D. Lower anterior which gives off anterior intercostal arteries in the MD FROM
E. Lower six posterior 7th, 8th, and 9th intercostal spaces and ends in the UST)
10th intercostal space where it anastomoses with
the deep circumflex iliac artery.
37 The following statements concerning the left suprarenal The medulla is innervated by preganglionic LESTER MIDTERM 1
gland are correct except? sympathetic fibers. BRYAN CO EXAM -
A. The gland extends along the medial border of the left (TOP 10 - MARCH
kidney from the upper pole to the hilus. AUG 2015 2016
E B. The left suprarenal vein drains into the left renal vein. MED
C. The gland is separated from the left kidney by perirenal BOARDS;
fat. TOPNOTCH
D. The gland lies behind the lesser sac. MD FROM
E. The medulla is innervated by post-ganglionic UST)
sympathetic nerve fibers

38 A thoracic surgeon removed the right middle lobar bronchus The right middle lobar (secondary) bronchus LESTER MIDTERM 1
along with lung tissue from a 57-year-old heavy smoker with leads to the medial and lateral bronchopulmonary BRYAN CO EXAM -
lung cancer. Which of the following bronchopulmonary segments. The right superior lobar bronchus (TOP 10 - MARCH
segments must contain cancerous tissues? divides into the superior, posterior, and anterior AUG 2015 2016
A. Medial and lateral segmental (tertiary) bronchi. The right inferior MED
A B. Anterior and posterior lobar bronchus has the anterior, lateral, posterior, BOARDS;
C. Anterior basal and medial basal and anterior segmental bronchi. TOPNOTCH
D. Anterior basal and posterior basal MD FROM
E. Lateral basal and posterior basal UST)

39 A 19-year-old man came to the emergency department, and The middle cardiac vein ascends in the posterior LESTER MIDTERM 1
his angiogram exhibited that he was bleeding from the vein interventricular groove, accompanied by the BRYAN CO EXAM -
that is accompanied by the posterior interventricular artery. posterior interventricular branch of the right (TOP 10 - MARCH
Which of the following veins is most likely to be ruptured? coronary artery. The great cardiac vein is AUG 2015 2016
A. Great cardiac vein accompanied by the anterior interventricular MED
B. Middle cardiac vein artery, the anterior cardiac vein drains directly BOARDS;
B C. Anterior cardiac vein
D. Small cardiac vein
into the right atrium, and the small cardiac vein is
accompanied by the marginal artery.
TOPNOTCH
MD FROM
E. Oblique veins of the left atrium UST)

40 After an automobile accident, a back muscle that forms the The latissimus dorsi forms boundaries of the LESTER MIDTERM 1
boundaries of the triangle of auscultation and the lumbar auscultation and lumbar triangles and receives BRYAN CO EXAM -
triangle receives no blood. Which of the following muscles blood from the thoracodorsal artery. The levator (TOP 10 - MARCH
might be ischemic? scapulae, rhomboid minor, and splenius capitis AUG 2015 2016
A. Levator scapulae muscles do not form boundaries of these two MED
B. Rhomboid minor triangles. The trapezius muscle forms a boundary BOARDS;
C C. Latissimus dorsi
D. Trapezius Lat- inferior
of the auscultation triangle but not the lumbar
triangle. The levator scapulae, rhomboid minor,
TOPNOTCH
MD FROM
E. Splenius capitis and trapezius muscles receive blood from the UST)
Superior- trapezius transverse cervical artery. The splenius capitis
muscle receives blood from the occipital and
Med- infraspinarus, ters major transverse cervical arteries.
41 The isthmus of the thyroid gland lies anterior to the GEORGE MIDTERM 2
A. Cricoid cartilage MICHAEL EXAM -
C B. Thyroid cartilage SOSUAN MARCH
C. 2-3 tracheal rings (TOP 5 - AUG 2016
D. 4-5 tracheal rings
E. NOTA
Rhomboid major, serratus anterior, 2015 MED
BOARDS;
efector spinae+ floor TOPNOTCH
MD FROM
UST)
42 The nasal septum is comprised of the following, except: GEORGE MIDTERM 2
A. Septal cartilage MICHAEL EXAM -
B. Cribiform plate of ethmoid SOSUAN MARCH
C. Vomer (TOP 5 - AUG 2016
B D. Perpendicular plate of ethmoid 2015 MED
E. NOTA BOARDS;
TOPNOTCH
MD FROM
UST)
43 The structure that drains into the inferior nasal meatus all other choices drain to the middle nasal meatus GEORGE MIDTERM 2
A. Nasolacrimal duct MICHAEL EXAM -
B. Anterior ethmoid sinus SOSUAN MARCH
C. Frontal sinus (TOP 5 - AUG 2016
D. Maxillary sinus 2015 MED
A E. NOTA BOARDS;
TOPNOTCH
MD FROM
UST)
TOPNOTCH MEDICAL BOARD PREP ANATOMY SUPEREXAM Page 6 of 94
For inquiries visit www.topnotchboardprep.com.ph or email us at topnotchmedicalboardprep@gmail.com
TOPNOTCH MEDICAL BOARD PREP ANATOMY SUPEREXAM
For inquiries visit www.topnotchboardprep.com.ph or email us at topnotchmedicalboardprep@gmail.com
Item QUESTION EXPLANATION AUTHOR TOPNOTCH
# EXAM
44 Aqueous humor is produced by the The aqueous humor is produced by the ciliary GEORGE MIDTERM 2
A. Choroid process of the uveal tract and drained into the MICHAEL EXAM -
B. Ciliary process anterior chamber angle. SOSUAN MARCH
C. Cornea (TOP 5 - AUG 2016
D. Retina 2015 MED
B E. Iris BOARDS;
TOPNOTCH
MD FROM
UST)
45 Origins of the arterial supply to the rectum, except Superior rectal artery from the inferior GEORGE MIDTERM 2
A. Superior mesenteric artery mesenteric; Middle rectal artery from the internal MICHAEL EXAM -
B. Inferior mesenteric artery iliac artery; inferior rectal artery from the internal SOSUAN MARCH
A C. Internal iliac artery pudendal artery from the internal iliac artery (TOP 5 - AUG 2016
D. Internal pudendal artery 2015 MED
E. NOTA BOARDS;
TOPNOTCH
MD FROM
UST)
46 Blood supply ot the liver includes 75% from the portal vein and 25%from the GEORGE MIDTERM 2
A. 25% from portal vein hepatoc artery MICHAEL EXAM -
B. 75% from hepatic artery SOSUAN MARCH
C. 75% from portal vein (TOP 5 - AUG 2016
C D. 20% from the celiac artery 2015 MED
E. Both A and B BOARDS;
TOPNOTCH
MD FROM
UST)
47 Most common used vein in the antecubittal fossa for GEORGE MIDTERM 2
phlebotomy MICHAEL EXAM -
A. Median cubittal vein SOSUAN MARCH
B. Brachial vein (TOP 5 - AUG 2016
A C. Basilic vein
D. Cephalic vein
2015 MED
BOARDS;
E. AOTA TOPNOTCH
MD FROM
UST)
48 Divides the axillary artery into three parts GEORGE MIDTERM 2
A. Teres major MICHAEL EXAM -
B. Teres minor SOSUAN MARCH
C. Scalene medius (TOP 5 - AUG 2016
D. Pectoralis major 2015 MED
E E. Pectoralis minor BOARDS;
TOPNOTCH
MD FROM
UST)
49 Support of the uterus is provided by the following, except: The other choices comprise the endopelvic fascia GEORGE MIDTERM 2
A. Uterosacral ligament MICHAEL EXAM -
B. Cardinal ligament SOSUAN MARCH
C. Ileococcygeus (TOP 5 - AUG 2016
D. Puborectalis 2015 MED
E E. Broad ligament BOARDS;
TOPNOTCH
MD FROM
UST)
50 The left primary bronchus GEORGE MIDTERM 2
A. Has one eparterial and one hyparterial bronchi MICHAEL EXAM -
B. Is narrower and shorter than the right SOSUAN MARCH
C. Has the aorta arching over it (TOP 5 - AUG 2016
D. Has a more vertical direction than the right 2015 MED
E. NOTA BOARDS;
C TOPNOTCH
MD FROM
UST)
51 A fluid found in the membranous labyrinth Endolymph is the fluid contained in the GEORGE MIDTERM 2
A. Perilymph membranous labyrinth of the inner ear and rich in MICHAEL EXAM -
B. Corticolymph potassium. SOSUAN MARCH
C. Blood (TOP 5 - AUG 2016
D D. Endolymph
E. NOTA
2015 MED
BOARDS;
TOPNOTCH
MD FROM
UST)
52 A pair of opening on the lateral wall of the oral cavity The Stensen's duct or parotid duct opens opposite GEORGE MIDTERM 2
A. Ducts of Rivinius the 2nd upper molar. MICHAEL EXAM -
B. Wharton's duct SOSUAN MARCH
C. Stensen's duct (TOP 5 - AUG 2016
D. Boath A and B 2015 MED
C E. AOTA BOARDS;
TOPNOTCH
MD FROM
UST)

TOPNOTCH MEDICAL BOARD PREP ANATOMY SUPEREXAM Page 7 of 94


For inquiries visit www.topnotchboardprep.com.ph or email us at topnotchmedicalboardprep@gmail.com
TOPNOTCH MEDICAL BOARD PREP ANATOMY SUPEREXAM
For inquiries visit www.topnotchboardprep.com.ph or email us at topnotchmedicalboardprep@gmail.com
Item QUESTION EXPLANATION AUTHOR TOPNOTCH
# EXAM
53 The bulk of muscles that make up the floor of the oral cavity The mylohyoid forms the floor of the oral cavity GEORGE MIDTERM 2
A. Genioglossus and the roof of the submandibular triangle. MICHAEL EXAM -
B. Stylohyoid SOSUAN MARCH
C. Mylohyoid (TOP 5 - AUG 2016
C D. Digastric 2015 MED
E. NOTA BOARDS;
TOPNOTCH
MD FROM
UST)
54 The Stensen's duct pierces this structure to open into the The Stensen's duct or parotid duct crosses the GEORGE MIDTERM 2
oral cavity masseter to pierce the buccinator to open into the MICHAEL EXAM -
A A. Buccinator
B. Masster
oral cavity. SOSUAN
(TOP 5 - AUG
MARCH
2016
C. Mentalis 2015 MED
D. Zygoma BOARDS;
E. NOTA TOPNOTCH
MD FROM
UST)
55 Spongiocytes are located in this layer A spongiocyte is a cell in the zona fasciculata of the GEORGE MIDTERM 2
A. Zona glomerulosa adrenal cortex containing lipid droplets that show MICHAEL EXAM -
B. Zona fasciculata pronounced vacuolization. SOSUAN MARCH
C. Zona reticularis (TOP 5 - AUG 2016
D. Adrenal medulla 2015 MED
B E. AOTA BOARDS;
TOPNOTCH
MD FROM
UST)
56 Origin of blood supply of the suprarenal glands The suprarenal glands has three origins of blood GEORGE MIDTERM 2
A. Inferior phrenic artery supply. MICHAEL EXAM -
B. Renal artery SOSUAN MARCH
C. Abdominal aorta (TOP 5 - AUG 2016
D. Both B and C 2015 MED
E E. AOTA BOARDS;
TOPNOTCH
MD FROM
UST)
57 Most common direction of hip dislocation 90% of the hip dislocation occurs posteriorly. GEORGE MIDTERM 2
A. Anterior MICHAEL EXAM -
B. Posterior SOSUAN MARCH
C. Superior (TOP 5 - AUG 2016
D. Inferior 2015 MED
E. Antero-inferior BOARDS;
B TOPNOTCH
MD FROM
UST)
58 Crutch paralysis injures this nerve Crutch paralysis is a form of paralysis which can GEORGE MIDTERM 2
A. Axillary nerve occur when the radial nerve, is under constant MICHAEL EXAM -
B. Radial nerve pressure, such as by the use of a crutch. SOSUAN MARCH
C. Median nerve (TOP 5 - AUG 2016
D. Musculocutaneous nerve 2015 MED
E. Ulnar nerve BOARDS;
B TOPNOTCH
MD FROM
UST)
59 Forms the hood of the clitoris GEORGE MIDTERM 2
A. Frenulum MICHAEL EXAM -
B. Labia majora SOSUAN MARCH
C. Prepuce (TOP 5 - AUG 2016
D. Labia minora 2015 MED
E. NOTA BOARDS;
TOPNOTCH
C MD FROM
UST)
60 The ovarian vessels are enclosed by this ligament The suspensory ligament of the ovary, also GEORGE MIDTERM 2
A. True ovarian ligament infundibulopelvic ligament, is a fold of peritoneum MICHAEL EXAM -
B. Suspensory ligament that extends out from the ovary to the wall of the SOSUAN MARCH
B C. Round ligament
D. Cardinal ligament
pelvis and encloses the ovarian vessels. (TOP 5 - AUG
2015 MED
2016

E. NOTA BOARDS;
TOPNOTCH
MD FROM
UST)
61 The lens of the eye is made up of what epithelial layer? Other parts of the eye: The corneal epithelium is JAN MIDTERM 3
A. Simple squamous made up of stratified squaomous. The corneal CHRISTIAN EXAM -
B. Simple columnar endothelium is made up of simple squamous. FELICIANO MARCH
C.Simple cuboidal (TOP 2 - AUG 2016
C D. Stratified squamous non keratinizing 2015 MED
E. Pseudostratified columnar BOARDS;
TOPNOTCH
MD FROM
UST)

TOPNOTCH MEDICAL BOARD PREP ANATOMY SUPEREXAM Page 8 of 94


For inquiries visit www.topnotchboardprep.com.ph or email us at topnotchmedicalboardprep@gmail.com
TOPNOTCH MEDICAL BOARD PREP ANATOMY SUPEREXAM
For inquiries visit www.topnotchboardprep.com.ph or email us at topnotchmedicalboardprep@gmail.com
Item QUESTION EXPLANATION AUTHOR TOPNOTCH
# EXAM
62 Frey's syndrome commonly termed as crocodile tears is a The Auriculotemporal branch of the trigeminal JAN MIDTERM 3
condition wherein beads of perspiration appear on the skin nerve carries parasympathetic fibers to the sweat CHRISTIAN EXAM -
when the patient eats due to penetrating injury to the glands of the scalp and the parotid salivary gland. FELICIANO MARCH
parotid glands. This is due to faulty regeneration of the As a result of severance and inappropriate (TOP 2 - AUG 2016
auriculotemporal nerve which is a branch of what regeneration, the parasympathetic nerve fibers 2015 MED
nerve/ganglion? may switch course, resulting in "gustatory BOARDS;
A. Trigeminal nerve sweating" or sweating in the anticipation of eating, TOPNOTCH
B. Facial nerve instead of the normal salivatory response. MD FROM
A C. Glossopharyngeal nerve UST)
D. Vagus nerve
E. Sueprior cervical ganglion

63 90% of epistaxis occurs in the Keiseelbach's plexus or Little's SIMILAR TO PREVIOUS BOARD EXAM JAN MIDTERM 3
triangle. 4 arteries contribute to this plexus. Which artery CONCEPT/PRINCIPLE. The dorsal nasal artery is CHRISTIAN EXAM -
does NOT contribute? the termminal branch of the opthalmic artery and FELICIANO MARCH
A. Anterior ethmoidal artery of ophthalmic artery supplies the dorsum and root of the nose. All the (TOP 2 - AUG 2016
B. Sphenopalatine artery of maxillary artery other 4 arteries make up the Keisselbach's plexus 2015 MED
D C. Greater palatine artery of maxillary artery BOARDS;
D. Dorsal nasal artery of ophthalmic artery TOPNOTCH
E. Superior labial artery of facial artery MD FROM
UST)

64 When thyroid follicular cells are stimulated by TSH and fiiled SIMILAR TO PREVIOUS BOARD EXAM JAN MIDTERM 3
with colloid they assume what configuration? CONCEPT/PRINCIPLE. When inactive thyroid CHRISTIAN EXAM -
A. Transitional follicles are simple cuboidal but assume a tall FELICIANO MARCH
B. Tall columnar columnar configuration when stimulated by TSH. (TOP 2 - AUG 2016
B C. Simple cuboidal
D. Stratified squamous non keratinizing
2015 MED
BOARDS;
E. No specific configuration TOPNOTCH
MD FROM
UST)
65 All of the ff nerves arises from the posterior cord of the For branches of the posterior cord. Remember the JAN MIDTERM 3
brachial plexus EXCEPT? mnemonic STAR. Subscapular nerve, CHRISTIAN EXAM -
A. Subscapular Thoracodorsal, Axillary and Radial. The long FELICIANO MARCH
B. Long thoracic throacic nerve arises from the C5-C7 roots of the (TOP 2 - AUG 2016
C. Thoracodorsal plexus not the cord. 2015 MED
B D. Axillary BOARDS;
E. Radial TOPNOTCH
MD FROM
UST)
66 The dorsalis pedis pulse can be located on what landmark? Choice C should have been: Lateral to the extensor JAN MIDTERM 3
A. Lateral to the flexor hallucis longus and medial to flexor hallucis longus and medial to extensor digitorum CHRISTIAN EXAM -
hallucis longus longus . SIMILAR TO PREVIOUS BOARD EXAM FELICIANO MARCH
B .Lateral to the extensor hallucis brevis and medial to CONCEPT/PRINCIPLE. The dorsalis pedis artery (TOP 2 - AUG 2016
extnesor hallucis brevis pulse can be palpated readily lateral to the 2015 MED
Bonus C. Lateral to the extensor hallucis longus and medial to
extnesor hallucis longus
extensor hallucis longus tendon and medially to
the extensor digitorum longus tendon on the
BOARDS;
TOPNOTCH
D. Lateral to the flexor hallucis brevis and medial to flexor dorsal surface of the foot. Choice E refers to the MD FROM
hallucis brevis femoral artery UST)
E. Behind the inguinal ligament between the ASIS and
symphysis pubis

67 What cell prodcues the hormone that activates bone SIMILAR TO PREVIOUS BOARD EXAM JAN MIDTERM 3
resorption and increases blood calcium level? CONCEPT/PRINCIPLE. The questions refes to CHRISTIAN EXAM -
A. Follicular cells Parathormone or PTH secreted by the chief cells of FELICIANO MARCH
B. Parafolllicular cells the parathyroid gland. Parafollicular cellsof the (TOP 2 - AUG 2016
C. Oxyntic cells thyroid gland secrete calcitonin. Oxyphil cells 2015 MED
E D. Oxyphil cells although part of the parathyroids have unknown BOARDS;
E. Chief cells functions. Oxyntic cells is in the stomach. TOPNOTCH
MD FROM
UST)
68 All of the ff are contents of the spermatic cord EXCEPT? SIMILAR TO PREVIOUS BOARD EXAM JAN MIDTERM 3
A. Testicular artery CONCEPT/PRINCIPLE The ilioinguinal nerve psses CHRISTIAN EXAM -
B. Processus vaginalis through the inguinal canal but lies outside the FELICIANO MARCH
D C. Deferential artery
D. Ilioinguinal nerve
spermatic cord. All the other structures plus the
vas deferens, cremasteric artery and genital
(TOP 2 - AUG
2015 MED
2016

E. Pampiniform plexus branch of the genitofemoral nerve are contents. BOARDS;


TOPNOTCH
MD FROM
UST)
69 What structure is termed as the false vocal cords? SIMILAR TO PREVIOUS BOARD EXAM JAN MIDTERM 3
A. Quadrangular membrane CONCEPT/PRINCIPLE. The vestibular fold is a CHRISTIAN EXAM -
B B. Vestibular folds fixed fold on each side of the larynx and is termed FELICIANO MARCH
C. Aryepiglottic folds as the false vocal cords. While the mobile vocal (TOP 2 - AUG 2016
D. Vocal folds folds immediately below it are the true vocal 2015 MED
E. Rima glottidis cords. The gap b/w the true vocal cord is termed BOARDS;
as the rima glottidis or glottis. The quadrangular TOPNOTCH
membrane extends between the epiglottis and MD FROM
arytenoid cartilage and its inferior margin forms UST)
the vestibular folds.

TOPNOTCH MEDICAL BOARD PREP ANATOMY SUPEREXAM Page 9 of 94


For inquiries visit www.topnotchboardprep.com.ph or email us at topnotchmedicalboardprep@gmail.com
TOPNOTCH MEDICAL BOARD PREP ANATOMY SUPEREXAM
For inquiries visit www.topnotchboardprep.com.ph or email us at topnotchmedicalboardprep@gmail.com
Item QUESTION EXPLANATION AUTHOR TOPNOTCH
# EXAM
70 The medial wall of the bony orbit is formed by the ff bones SIMILAR TO PREVIOUS BOARD EXAM JAN MIDTERM 3
EXCEPT? CONCEPT/PRINCIPLE. The orbital plate of the CHRISTIAN EXAM -
A. Frontal process of the maxilla maxilla forms the floor of the orbit. The 4 other FELICIANO MARCH
B. Lacrimal bone choices forms the medial wall of the orbit. The (TOP 2 - AUG 2016
C C. Orbital plate of the maxilla orbital plate of the frontal bone forms the roof of 2015 MED
D. Body of sphenoid the orbit while the zygomatic and the greater wing BOARDS;
E. Orbital plate of the ethmoid of the sphenoid forms the lateral wall. TOPNOTCH
MD FROM
UST)
71 What rotator cuff muscle is a lateral rotator of the shoulder There are 2 lateral rotators of the shoulder: JAN MIDTERM 3
and supplied by a trunk of the brachial plexus? infraspinatus supplied by suprascapular nerve CHRISTIAN EXAM -
A. Supraspinatus from the trunk of the brachial plexus and the teres FELICIANO MARCH
B B. Infraspinatus minor supplied by the axillary nerve from the (TOP 2 - AUG 2016
C. Teres Minor posteriro cord. 2015 MED
D.Teres Major BOARDS;
E. Subscapularis TOPNOTCH
MD FROM
UST)
72 Which spinal nerves innervates the main muscle of The question is refers to the phrenic nerve. JAN MIDTERM 3
respiration? Remember: C3 4 5 keeps the diaphragm alive. CHRISTIAN EXAM -
A. C2 C3 C4 FELICIANO MARCH
B. C3 C4 C5 (TOP 2 - AUG 2016
C. C4 C5 C6 2015 MED
B D. C5 C6 C7 BOARDS;
E. C6 C7 T1 TOPNOTCH
MD FROM
UST)
73 What is the correct sequence of the CSF pathway? Knowthe CSF pathway by heart.. Lateral ventricle- JAN MIDTERM 3
A .Lateral ventricle- Foramen of Magendie- 3rd ventricle- Foramen of Monroe- 3rd ventricle- Cerebral CHRISTIAN EXAM -
Cerebral aqueduct- 4th ventricle- Foramen of Monroe - aqueduct- 4th ventricle- Foramen of Lushka and FELICIANO MARCH
Arachnoid villi Magendie -Subarachnoid space-Arachnoid villi (TOP 2 - AUG 2016
D B. 3rd ventricle- Foramen of Magendie- Latreral ventricle- 2015 MED
Cerebral aqueduct- 4th ventricle- Foramen of Monroe - BOARDS;
Arachnoid villi TOPNOTCH
C. 3rd ventricle- Foramen of Monroe- Lateral ventricle- MD FROM
Cerebral aqueduct- 4th ventricle- Foramen of Magendie - UST)
Arachnoid villi
D. Lateral ventricle- Foramen of Monroe- 3rd ventricle-
Cerebral aqueduct- 4th ventricle- Foramen of Magendie -
Arachnoid villi
E. Lateral ventricle- Cerebral aqueduct- 3rd ventricle-
Foramen of Monroe- 4th ventricle- Foramen of Magendie -
Arachnoid villi
74 The outer hair cells of the organ of Corti are attached SIMILAR TO PREVIOUS BOARD EXAM JAN MIDTERM 3
inferiorly to basilar membrane and superiorly to what CONCEPT/PRINCIPLE. The Reissner's aka the CHRISTIAN EXAM -
membrane? Vestibualr membrane is the roof of the scala media FELICIANO MARCH
A. Reissner's membrane while the floor is the basilar membrane. The (TOP 2 - AUG 2016
B. Shrapnel's membrane tectorial membrane overlies the outer hair cells of 2015 MED
E C. Vestibular membrane the organ of Corti and during acoustic stimulation BOARDS;
D. Tympanic membrane stimulates the inner hair cells through fluid TOPNOTCH
E. Tectorial membrane coupling. MD FROM
UST)

75 What cranial foramina and structure passing through it is Ophthalmic artery passes through the optic canal. JAN MIDTERM 3
correctly paired? The maxillary nerve passes through the formaen CHRISTIAN EXAM -
A. Superior orbital fissure- opthalmic artery rotundum while the Mandibular nerve passes FELICIANO MARCH
B. Foramen spinosum- Middle meningeal artery through the foramen ovale. GP nerve passes (TOP 2 - AUG 2016
B C. Foramen rotundum- Mandibular nerve through jugular foramen while the facial nerve 2015 MED
D. Internal accoustic meatus- Glossopharyngeal nerve passes through the internal accoustic meatus BOARDS;
E. Jugular foramen- Facial nerve TOPNOTCH
MD FROM
UST)
76 Which statement regarding the lungs is correct? The right lung is divided by the oblique and JAN MIDTERM 3
A.The left lung is divided by the oblique and horizontal horizontal fissure into three lobes. The apex is 1 CHRISTIAN EXAM -
fissure into three lobes inch or 2.5 cm above the clavicle. The FELICIANO MARCH
B. Each lung has an apex which projects upward into the bornchopulmonary segment and not the lobe is (TOP 2 - AUG 2016
neck 2.5 inches above the clavicle the anatomical and functional unit of the lung. 2015 MED
D C. The pulmonary lobe is the anatomical and functional unit BOARDS;
of the lung TOPNOTCH
D. All the lymph from all parts of the lungs leaves the hilum MD FROM
and drains into the tracheobronchial nodes and into the UST)
bronchomediastinal trunks
E. None of the above

77 What lobe of the prostate is primarily affected in prostate Lateral and middle lobe (periurethral zone) is JAN MIDTERM 3
adenocarcinoma? affected in BPH while posterior lobe (peripheral CHRISTIAN EXAM -
C A. Anterior lobe
B. Lateral lobe
zone) is affected in prostate adenoCA FELICIANO
(TOP 2 - AUG
MARCH
2016
C. Posterior lobe 2015 MED
D. Middle lobe BOARDS;
E. Inferior lobe TOPNOTCH
MD FROM
UST)

TOPNOTCH MEDICAL BOARD PREP ANATOMY SUPEREXAM Page 10 of 94


For inquiries visit www.topnotchboardprep.com.ph or email us at topnotchmedicalboardprep@gmail.com
TOPNOTCH MEDICAL BOARD PREP ANATOMY SUPEREXAM
For inquiries visit www.topnotchboardprep.com.ph or email us at topnotchmedicalboardprep@gmail.com
Item QUESTION EXPLANATION AUTHOR TOPNOTCH
# EXAM
78 What statement regarding the liver is true? Experiments have shown that the quadrate and JAN MIDTERM 3
A. The quadrate and caudate lobes are a functional part of caudate lobes are a functional part of the left lobe CHRISTIAN EXAM -
A the left lobe of the liver
B. Divided into a large right lobe and small left lobe by the
of the liver. It is divided into a large right lobe and
small left lobe by the falciform ligament. The first
FELICIANO
(TOP 2 - AUG
MARCH
2016
coronary ligament liver segment is regarded as the caudate lobe and 2015 MED
C. The first liver segment is regarded as the quadrate lobe 4th segment is the quadrate lobe. Blood supply is BOARDS;
and 4th segment is the caudate lobe 70% portal vein and 30% hepatic vein. TOPNOTCH
D. Half of blood supply is from the portal vein and the other MD FROM
half is from the hepatic artery UST)
E. None of the above

79 Which of the ff nerve does not pass through the greater SIMILAR TO PREVIOUS BOARD EXAM JAN MIDTERM 3
sciatic foramen? CONCEPT/PRINCIPLE. The ff structures passes CHRISTIAN EXAM -
A. Superior gluteal nerve thru the greater sciatic foramen: Sciatic Nerve, FELICIANO MARCH
D B. Inferior gluteal nerve Superior Gluteal Nerve, Inferior Gluteal Nerve, (TOP 2 - AUG 2016
C. Sciatic nerve Pudendal Nerve, Posterior, Femoral Cutaneous 2015 MED
D. Obturator nerve Nerve, Nerve to Quadratus Femoris, Nerve to BOARDS;
E. Pudendal nerve Obturator Internus, Superior Gluteal Artery & TOPNOTCH
Vein, Inferior Gluteal Artery & vein, Internal MD FROM
Pudendal Artery & vein and the Piriformis muscle. UST)
Please review the contents of the lesser sciatic
foramen as well.
80 Which among the ff parts of the ear is part of the bony SIMILAR TO PREVIOUS BOARD EXAM JAN MIDTERM 3
labyrynth? CONCEPT/PRINCIPLE. The inner ear is divided CHRISTIAN EXAM -
A. Utricle into the bony labyrynth (external) and FELICIANO MARCH
C B. Saccule membranous labyrnth (internal). Bony: Vestibule, (TOP 2 - AUG 2016
C. Semicircular canal Semicircular canal, cochlea. Membranous: Utricle, 2015 MED
D. Cohclear duct saccule, cochlear duct, semicircular duct BOARDS;
E. Auditory ossicles TOPNOTCH
MD FROM
UST)
81 1. Which of the following does NOT play a role in parasitic ANDREW FINAL
A infections?
a. IL - 6
TIU (TOP 1 -
AUG 2015
EXAM -
MARCH
b. IL - 4 MED 2016
c. IgE BOARDS;
d. IL - 5 TOPNOTCH
e. none of the above MD FROM
CIM)
82 2. Which of the following structures does not pass through obturator nerve arises from the lumbar plexus (L2 ANDREW FINAL
both greater and lesser sciatic foramen? – L4) and emerges on the medial border of the TIU (TOP 1 - EXAM -
a. Internal pudendal artery psoas muscle within the abdomen. It runs forward AUG 2015 MARCH
b. pudendal nerve on the lateral wall of the pelvis to reach the upper MED 2016
c. obturator internus nerve part of the obturator foramen where it divides into BOARDS;
D d. obturator nerve anterior and posterior divisions. TOPNOTCH
e. none of the above Snells’ Clinical anatomy by regions 9th edition p. MD FROM
465 CIM)
83 3. If the aryepiglottic fold forms the superior border, which The thickened inferior margin forms the ANDREW FINAL
of the following is the inferior border of the quadrangular vestibular ligament and the vestibular ligaments TIU (TOP 1 - EXAM -
membrane? form the interior of the vestibular folds. AUG 2015 MARCH
a. Vocal ligament Snells’ Clinical anatomy by regions 9th edition p. MED 2016
b. Vestibular ligament 647 BOARDS;
B c. Median thyrohyoid ligament TOPNOTCH
d. Cricotracheal ligament MD FROM
e. Cricoid cartilage CIM)
84 4. Which of the following cartilages is found in the These two small rod shaped cartilages are found in ANDREW FINAL
quadrangular membrane? the aryepiglottic folds and serve to strengthen TIU (TOP 1 - EXAM -
a. Corniculate cartilage them. AUG 2015 MARCH
b. Cuneiform cartilage Snells’ Clinical anatomy by regions 9th edition p. MED 2016
B c. Epiglottis 646 BOARDS;
d. Thyroid cartilage TOPNOTCH
e. Arytenoid cartilage MD FROM
CIM)
85 5. Which of the following is found superior to the arytenoid Snells’ Clinical anatomy by regions 9th edition p. ANDREW FINAL
cartilages? 646 TIU (TOP 1 - EXAM -
a. Corniculate cartilage AUG 2015 MARCH
A b. Cuneiform cartilage MED 2016
c. Epiglottis BOARDS;
d. Thyroid cartilage TOPNOTCH
e. Arytenoid cartilage MD FROM
CIM)
86 6. Which of the following is a membranous labyrinth? The membranous labyrinth is lodged within the ANDREW FINAL
a. Vestibule bony labyrinth. It is filed with endolymph and TIU (TOP 1 - EXAM -
b. Semicircular canals surrounded by perilymph. It consists of utricle and AUG 2015 MARCH
D c. Cochlea saccule which are lodged in the bony vestibule; MED 2016
d. Utricle three semicircular ducts, which lie within the bony BOARDS;
e. None of the above semicircular canals; and the duct of the cochlea, TOPNOTCH
which lies within the bony cochlea. All these MD FROM
structures freely communicate with each other. CIM)
Snells’ Clinical anatomy by regions 9th edition p.
569

TOPNOTCH MEDICAL BOARD PREP ANATOMY SUPEREXAM Page 11 of 94


For inquiries visit www.topnotchboardprep.com.ph or email us at topnotchmedicalboardprep@gmail.com
TOPNOTCH MEDICAL BOARD PREP ANATOMY SUPEREXAM
For inquiries visit www.topnotchboardprep.com.ph or email us at topnotchmedicalboardprep@gmail.com
Item QUESTION EXPLANATION AUTHOR TOPNOTCH
# EXAM
87 7. Which of the following is responsible for depression of the Contraction of the lateral pterygoids pulls forward ANDREW FINAL
mandible? the neck of the mandible and articular disc so that TIU (TOP 1 - EXAM -
a. Temporalis the latter moves onto the articular tubercle. The AUG 2015 MARCH
b. Lateral pterygoid forward movement of the disc is limited by the MED 2016
B c. Medial pterygoid tension of the fibroelastic tissue, which tethers the BOARDS;
d. Masseter disc to the temporal bone posteriorly. TOPNOTCH
e. None of the above Snells’ Clinical anatomy by regions 9th edition p. MD FROM
572 CIM)
88 8. Which of the following divides the cochlear canal into scala Snells’ Clinical anatomy by regions 9th edition p. ANDREW FINAL
vestibule and scala tympani? 569 TIU (TOP 1 - EXAM -
a. Basilar membrane AUG 2015 MARCH
b. Reissner’s membrane MED 2016
A c. Tectorial membrane BOARDS;
d. Vestibular membrane TOPNOTCH
e. Spiral ligament MD FROM
CIM)
89 9. Which of the following membranes vibrates to code Hair cells are attached through the phalangeal ANDREW FINAL
acoustic information into nerve impulses and is responsible cells to the basilar membrane, which vibrates TIU (TOP 1 - EXAM -
for amplitude discrimination? during sound reception. The stereocilia of these AUG 2015 MARCH
a. Basilar membrane hair cells are in turn attached to the tectorial MED 2016
b. Reissner’s membrane membrane which also vibrates. However, the BOARDS;
A c. Tectorial membrane tectorial membrane and the basilar membrane are TOPNOTCH
d. Vestibular membrane hinged at different points. Thus, a shearing effect MD FROM
e. Spiral ligament occurs between the basilar membrane ( and the CIM)
cells attached to it) and the tectorial membrane
when sound vibrations impinge on the ear.
Histology 7th edition. Pawlina. P. 952
90 10. Which of the following landmarks signify the intersection Rhinion refers to the lower end of the suture ANDREW FINAL
between the frontal bone and 2 nasal bones? between the nasal bones. TIU (TOP 1 - EXAM -
a. Nasion Wikipedia (cant find it in snells) AUG 2015 MARCH
b. Rhinion MED 2016
c. Pterion BOARDS;
d. Glabella TOPNOTCH
A e. Asterion MD FROM
CIM)
91 11. Which of the following lies above the pelvic inlet? SNell's CLinical anatomy for Medical students 5th ANDREW FINAL
a. true pelvis edition p. 277 TIU (TOP 1 - EXAM -
b. false pelvis AUG 2015 MARCH
c. pelvic cavity MED 2016
d. lesser pelvis BOARDS;
B e. none of the above TOPNOTCH
MD FROM
CIM)
92 12. Which of the following types of pelvis is described with a A - heart shaped with triangular anterior segment ANDREW FINAL
greater anteroposterior diameter, straight walls, small C - normal female pelvis TIU (TOP 1 - EXAM -
subpubic arch, and large sacrosciatic notches? D - transversely wide with flattened AUG 2015 MARCH
a. android anteroposterior diameter MED 2016
b. anthropoid SNell's CLinical anatomy for Medical students 5th BOARDS;
B c. gynecoid edition p. 299 TOPNOTCH
d. platypelloid MD FROM
e. none of the above CIM)
93 13. Which of the following is a branch from the aorta? this confusing question came out. Best answer is ANDREW FINAL
a. inferior rectal artery still superior rectal artery, a branch of the inferior TIU (TOP 1 - EXAM -
b. superior vesical artery mesenteric artery. AUG 2015 MARCH
c. superior rectal artery SNell's CLinical anatomy for Medical students 5th MED 2016
C d. vaginal artery
e. all of the above
edition p. 291 BOARDS;
TOPNOTCH
MD FROM
CIM)
94 14. A 5 year old male comes to you for epistaxis. Which of the The following contributes: ANDREW FINAL
following does NOT contribute to the blood supply in the anterior ethmoidal artery from ophthalmic artery TIU (TOP 1 - EXAM -
Kiesselbach area? sphenopalatine artery from terminal branch of AUG 2015 MARCH
a. ophthalmic artery maxillary artery MED 2016
b. sphenopalatine artery greater palatine artery from maxillary artery BOARDS;
c. greater palatine artery septal branch of superior labial artery from facial TOPNOTCH
D d. superior nasal artery artery MD FROM
e. superior labial artery CIM)
95 15. Which of the following is NOT a globulin? Junqueira's Basic Histology 12th edition p. 203 ANDREW FINAL
a. transferrin TIU (TOP 1 - EXAM -
b. fibrinogen AUG 2015 MARCH
c. Beta2 - macroglobulin MED 2016
d. fibronectin BOARDS;
e. immunoglobulin TOPNOTCH
B MD FROM
CIM)
96 16. Which of the following develops from a reticular cell? Junqueira's Basic Histology 12th edition p. 209 ANDREW FINAL
a. eosinophil TIU (TOP 1 - EXAM -
C b. platelet AUG 2015 MARCH
c. RBC MED 2016
d. neutrophil BOARDS;
e. basophil TOPNOTCH
MD FROM
CIM)

TOPNOTCH MEDICAL BOARD PREP ANATOMY SUPEREXAM Page 12 of 94


For inquiries visit www.topnotchboardprep.com.ph or email us at topnotchmedicalboardprep@gmail.com
TOPNOTCH MEDICAL BOARD PREP ANATOMY SUPEREXAM
For inquiries visit www.topnotchboardprep.com.ph or email us at topnotchmedicalboardprep@gmail.com
Item QUESTION EXPLANATION AUTHOR TOPNOTCH
# EXAM
97 17. Which of the following zones of the adrenal cortex has The cells are most densely filled with cytoplasmic ANDREW FINAL
long cords of large polyhedral cells separated by fenestrated lipid droplets and as a result of lipid dissolution TIU (TOP 1 - EXAM -
sinusoidal capillaries? during tissue preparation, often appear vacuolated AUG 2015 MARCH
a. zona glomerulosa or spongy. MED 2016
B b. zona fasciculata Junqueira's Basic Histology 12th edition p. 356 BOARDS;
c. zona reticularis TOPNOTCH
d. zona pellucida MD FROM
e. zona medulla CIM)
98 18. The skin is not the largest organ of the body as it is Junqueira's Basic Histology 12th edition p. 320 ANDREW FINAL
considered an organ system. Part of its functions is TIU (TOP 1 - EXAM -
protection of the body by triggering an immune response. AUG 2015 MARCH
One of the cells that play a role in alerting the immune MED 2016
system is bone marrow derived and capable of binding, BOARDS;
processing, and presenting antigens to T lymphocytes. Which TOPNOTCH
of the following layers of the skin is this cell predominantly MD FROM
E located?
a. stratum corneum
CIM)

b. stratum lucidum
c. stratum germinativum
d. stratum granulosum
e. stratum spinosum
99 19. Which of following supporting cells of the CNS is derived oligodendrocyte - neural tube - CNS - myelin ANDREW FINAL
from neural tube and functions in the repair processes? production TIU (TOP 1 - EXAM -
a. oligodendrocyte neurolemmocyte - neural crest - PNS - myelin AUG 2015 MARCH
b. neurolemmocyte production MED 2016
C c. astrocyte ependymal cell - neural tube - CNS - lines the BOARDS;
d. ependymal cell cavities TOPNOTCH
e. microglia microglia - bone marrow - CNS - immune related MD FROM
activity CIM)
Junqueira's Basic Histology 12th edition p. 151
100 20. At what level does the ureter turns forward and medially http://radiopaedia.org/articles/ureter ANDREW FINAL
to insert into the posterolateral wall of the urinary bladder? TIU (TOP 1 - EXAM -
a. pelvic brim AUG 2015 MARCH
b. coccyx MED 2016
C c. ischial spine
d. bifurcation of common iliac vessels
BOARDS;
TOPNOTCH
e. sacroiliac joint MD FROM
CIM)
101 A 30 y/o female patient is brought to the emergency room in Page 11 of Topnotch Handout. To establish an ANGELA DIAGNOSTIC
respiratory distress. On PE she has an anterior neck mass, emergency airway you do a cricothyroidotomy. PAULINE P. EXAM - AUG
which relatives recounted that it recently suddenly increased Through the cricothyroid membrane located CALIMAG- 2015
in size. You decided to create an emergency airway to retore inferior to the thyroid cartilage and superior to the LOYOLA
respiration. At what level could you rapidly create an airway cricoid cartilage, the true vocal cords lie superior (TOP 8 - FEB
with minimum danger of hemorrhage and damage to the to it. 2015 MED
D vocal cords? BOARDS;
A. Through the third tracheal ring TOPNOTCH
B. Just above the jugular notch MD FROM
C. Just above the thyroid cartilage UST)
D. Just below the thyroid cartilage
E. Between the 3rd and 4th tracheal ring

102 During thyroidectomy of the patient in the above scenario, Page 11 of Topnotch Handout. The External ANGELA DIAGNOSTIC
the surgeon blindly clamped the superior thyroid artery and laryngeal nerve may be injured during ligation of PAULINE P. EXAM - AUG
ligates it. After 24 hours post-op the patient was noted to the superior thyroid artery. The cricothyroid is CALIMAG- 2015
have a change in vocal quality. Which of the following supplied by the External laryngeal nerve. The LOYOLA
muscles may be affected due to an injury to its innervation other muscles of phonation are supplied by the (TOP 8 - FEB
during the ligation of the superior thyroid artery?
Recurrent laryngeal nerve. 2015 MED
D A. Thyroarytenoid BOARDS;
B. Lateral cricoarytenoid TOPNOTCH
C. Posterior cricoarytenoid MD FROM
D. Cricothyroid UST)
E. All of the above

103 A term newborn was noted to have grunting and retractions. SIMILAR TO PREVIOUS BOARD EXAM ANGELA DIAGNOSTIC
His chest xray showed a markedly elevated right CONCEPT/PRINCIPLE. Phrenic nerve palsy will be PAULINE P. EXAM - AUG
hemidiaphragm with essentially clear lung fields. PE showed seen as elevation of diagphram on chest xray. CALIMAG- 2015
a globular abdomen. The most likely diagnosis is: LOYOLA
A A. Phrenic nerve palsy (TOP 8 - FEB
B. Congenital diaphragmatic hernia 2015 MED
C. Pulmonary hypoplasia BOARDS;
D. TTN TOPNOTCH
E. PPHN MD FROM
UST)

TOPNOTCH MEDICAL BOARD PREP ANATOMY SUPEREXAM Page 13 of 94


For inquiries visit www.topnotchboardprep.com.ph or email us at topnotchmedicalboardprep@gmail.com
TOPNOTCH MEDICAL BOARD PREP ANATOMY SUPEREXAM
For inquiries visit www.topnotchboardprep.com.ph or email us at topnotchmedicalboardprep@gmail.com
Item QUESTION EXPLANATION AUTHOR TOPNOTCH
# EXAM
104 An 80 y/o male went to the OPD with complaints of low Page 1 and 3 of supplement Topnotch Handout. L5 ANGELA DIAGNOSTIC
back pain. Further tests showed a herniated disc root compression will manifest at the lateral PAULINE P. EXAM - AUG
compressing the L5 root. Where will the patient have surface of the leg and dorsum of the foot. CALIMAG- 2015
dermatomal pain? LOYOLA
C A. Groin
B. Posterior thigh
(TOP 8 - FEB
2015 MED
C. Lateral part of the lower leg BOARDS;
D. Medial aspect of the calf TOPNOTCH
E. Big toe MD FROM
UST)
105 A 29 y/o male presented at the ER due to abdominal pain. Page 10 and 11 of Topnotch Handout. The rule is ANGELA DIAGNOSTIC
History revealed that the pain started in the periumbilical that pain originating from the midgut is generally PAULINE P. EXAM - AUG
area. This was accompanied by anorexia. After 6 hours, the perceived in the periumbilical region. The midgut: CALIMAG- 2015
patient started vomiting and the pain became localized in the in the 5 mm embryo, it is seen beginning just LOYOLA
B right lower quadrant The organ involved is most likely a caudal to the entrance of the bile duct into the (TOP 8 - FEB
derivative of the:
duodenum and terminating at the beginning of the 2015 MED
A. Foregut last third of the transverse colon (from anterior to BOARDS;
B. Midgut posterior intestinal portals). It is suspended from TOPNOTCH
C. Hindgut the dorsal abdominal wall by a short mesentery MD FROM
D. All of the above and communicates with the yolk sac via the UST)
E. None of the above vitelline duct. Its derivatives consist of the small
intestines (except the first part of the duodenum
to the common bile duct entrance); the cecum, the
appendix, the ascending colon; and the right one-
half to two-thirds or proximal part of the
transverse colon. It is supplied by the superior
mesenteric artery and vagus nerve.
106 The patient was then brought to the OR for emergency Page 1-2 of Day 3 Topnotch Handout. McBurney's ANGELA DIAGNOSTIC
appendectomy, a McBurney's incision was done. This incision, is oblique beginning laterally from above PAULINE P. EXAM - AUG
incision will pass through the following except:
and ending medially and located at about one- CALIMAG- 2015
A. External oblique third the distance along a line from the ASIS to the LOYOLA
D B. Internal oblique umbilicus. The skin and subcutaneous tissue are (TOP 8 - FEB
C. Transversus abdominis incised down to the external oblique muscle, the 2015 MED
D. Rectus abdominis underlying internal oblique and transversus BOARDS;
E. None of the above abdominis are split and separated. TOPNOTCH
MD FROM
UST)
107 A 10 month old male was diagnosed with Congenital SIMILAR TO PREVIOUS BOARD EXAM ANGELA DIAGNOSTIC
megacolon. On barium studies which segment is considered CONCEPT/PRINCIPLE. Page 11 of Day 3 Topnotch PAULINE P. EXAM - AUG
normal: A. the dilated proximal colon Handout. In Hirscprungs disease/Congenital CALIMAG- 2015
A B. the contracted distal colon
C. the whole colon
megacolon there is absence of autonomic ganglion
cells in the myenteric plexus distal to the dilated
LOYOLA
(TOP 8 - FEB
D. Both A and B segment of colon. 2015 MED
E. None of the above BOARDS;
TOPNOTCH
MD FROM
UST)
108 In a bilateral recurrent layngeal nerve injury the vocal cords SIMILAR TO PREVIOUS BOARD EXAM ANGELA DIAGNOSTIC
assume what position? CONCEPT/PRINCIPLE. Bilateral recurrent PAULINE P. EXAM - AUG
A. Midway between full abduction and adduction laryngeal nerve palsy: In this condition both cords CALIMAG- 2015
B. Paramedian assume a paramedian position compromising the LOYOLA
C. Full Abduction airway. This commonly occurs following total (TOP 8 - FEB
B D. Full Adduction thyroidectomy or in thyroid malignancies. The 2015 MED
E. Either C or D patient will commonly manifest with stridor. The BOARDS;
voice will be near normal. TOPNOTCH
MD FROM
UST)
109 As a surgeon who will perform a laparoscopic Page 9 of Day 3 Topnotch Handout. Liver is ANGELA DIAGNOSTIC
cholecystectomy, you must know the boundaries of the superior, cystic duct is inferior, common hepatic PAULINE P. EXAM - AUG
triangle of Calot which consists of:
duct is medial. CALIMAG- 2015
A. Cystic duct, common hepatic duct, inferior border of the LOYOLA
A liver
B. Cystic duct, common hepatic duct, common bile duct
(TOP 8 - FEB
2015 MED
C. Common bile duct, common hepatic duct, inferior margin BOARDS;
of the liver TOPNOTCH
D. Cystic duct, common bile duct, inferior border of the MD FROM
liver UST)
E. Cystic duct, common hepatic artery, inferior border of
the liver

C110 A 9 month old male was noted to have an inguinal mass. A SIMILAR TO PREVIOUS BOARD EXAM ANGELA DIAGNOSTIC
diagnosis of cryptorchid testes was made. Which of the CONCEPT/PRINCIPLE. Page 22 of day 3 Topnotch PAULINE P. EXAM - AUG
following controls the descent of testes into the scrotum:
Handout. 26-28 weeks-the testes descended CALIMAG- 2015
A. Withdrawal of maternal estrogen retroperitoneally from the posterior abdominal LOYOLA
B. Enlargement of fetal pelvis wall to the deep inguinal rings; due to (TOP 8 - FEB
C. Androgens enlargement of fetal pelvis. 2-3 days descent into 2015 MED
D. All of the above the scrotum; controlled by androgens. BOARDS;
E. Both A and B TOPNOTCH
MD FROM
UST)

TOPNOTCH MEDICAL BOARD PREP ANATOMY SUPEREXAM Page 14 of 94


For inquiries visit www.topnotchboardprep.com.ph or email us at topnotchmedicalboardprep@gmail.com
TOPNOTCH MEDICAL BOARD PREP ANATOMY SUPEREXAM
For inquiries visit www.topnotchboardprep.com.ph or email us at topnotchmedicalboardprep@gmail.com
Item QUESTION EXPLANATION AUTHOR TOPNOTCH
# EXAM
111 A 23 y/o male arrived at the ER. He a suffered severe head Page 33 of supplement Topnotch Handout. The ANGELA DIAGNOSTIC
trauma in a motorcycle accident. Radiographic studies of the mandibular branch/V3 of the trigeminal exits thru PAULINE P. EXAM - AUG
head revealed a basilar skull fracture in the region of the the foramen ovale. It will manifest as Loss of CALIMAG- 2015
foramen ovale. Which of the following functional losses general sensation in skin over mandible, LOYOLA
would most likely be related to this injury? :
mandibular teeth, tongue, paralysis of muscles of (TOP 8 - FEB
A. Inability to abduct eye mastication; jaw deviation to injured side, 2015 MED
B. Loss of general sensation in skin over maxilla Hypoacusis due to paralysis of the tensor tympani BOARDS;
C. Ageusia muscle. B is related to maxillary branch of TOPNOTCH
D. Loss of afferent limb of corneal reflex trigeminal/V2 which exits from the foramen MD FROM
E. Hypoacusis rotundum, C is alteration or loss of taste realted to UST)
facial nerve injury which exits from the internal
E auditory meatus, D loss of afferent limb of corneal
reflex is due to injury to the Ophthalmic branch of
the trigeminal/V1 which exits from the superior
orbital fissure, Inability to abduct the eye is due to
injury of the abducens which also exits from the
superior orbital fissure.
112 The physician was listening for breath sounds over the right Page 20 of Day 3 Topnotch Handout. The parietal ANGELA DIAGNOSTIC
hemithorax and was concerned when no sounds were heard pleura reflection passes obliquely across the 8th PAULINE P. EXAM - AUG
on the front of the chest wall at the level of the tenth rib in rib in MCL not the 10th. CALIMAG- 2015
the MCL, the following statements are correct, except: LOYOLA
D A. In a healthy individual, the lower border of the right lung (TOP 8 - FEB
in the MCL in the midrespiratory position is at the level of 2015 MED
the 6th rib B. No breath sounds are heard because the BOARDS;
stethoscope was located over the liver TOPNOTCH
C. The costodiaphragmatic recess is situated between the MD FROM
lower border of the lung and the parietal pleura UST)
D. The parietal pleura in the MCL crosses the tenth rib
E. The lower margin of the lung crosses the tenth rib on the
sides of the vertebral column

113 A nexus is an area of low electrical resistance and is involved Page 5 of Histology Topnotch Handout. Gap ANGELA DIAGNOSTIC
in metabolic and electrical coupling of cells. These are found junctions or Nexus are found in nearly all tissues PAULINE P. EXAM - AUG
in nearly all tissues except:
except skeletal muscle. CALIMAG- 2015
A. Osteocytes LOYOLA
B B. Skeletal muscle
C. Neurons
(TOP 8 - FEB
2015 MED
D. Smooth muscle BOARDS;
E. Cardiac muscle TOPNOTCH
MD FROM
UST)
114 Type IIa muscle fibers are also known as:
Page 13 of Histology Topnotch Handout. Type IIa ANGELA DIAGNOSTIC
A. Red, slow, oxidative fibers are Red, fast, oxidative-glycolytic fibers. Type I are PAULINE P. EXAM - AUG
B. White, fast, glycolytic fibers Red, slow, oxidative fibers, while Type IIb are fast, CALIMAG- 2015
C. Red, fast, oxidative-glycolytic fibers glycolytic fibers. LOYOLA
C D. White, slow, oxidative-glycolytic fibers (TOP 8 - FEB
E. Red, slow, oxidative-glycolytic fibers 2015 MED
BOARDS;
TOPNOTCH
MD FROM
UST)
115 The liver has several functions including the storage of Page 24 of Histology Topnotch Handout. Ito ANGELA DIAGNOSTIC
vitamin A and other fat soluble vitamins. This is specifically a cells/Stellate Cells/Lipocytes stores vitamin A and PAULINE P. EXAM - AUG
function of the Ito Cells which are found in the:
other fat soluble vitamins and are found in the CALIMAG- 2015
A. Sinusoids space of disse or perisinusoidal space. LOYOLA
B. Bile canaliculi (TOP 8 - FEB
D C. Principal parenchyma cell 2015 MED
D. Space of disse BOARDS;
E. Rokitansky aschoff sinus TOPNOTCH
MD FROM
UST)

TOPNOTCH MEDICAL BOARD PREP ANATOMY SUPEREXAM Page 15 of 94


For inquiries visit www.topnotchboardprep.com.ph or email us at topnotchmedicalboardprep@gmail.com
TOPNOTCH MEDICAL BOARD PREP ANATOMY SUPEREXAM
For inquiries visit www.topnotchboardprep.com.ph or email us at topnotchmedicalboardprep@gmail.com
Item QUESTION EXPLANATION AUTHOR TOPNOTCH
# EXAM
116 The normal anatomical position of the epididymis is SIMILAR TO PREVIOUS BOARD EXAM ANGELA DIAGNOSTIC
________________ in relation to the testes: CONCEPT/PRINCIPLE. The testes are located PAULINE P. EXAM - AUG
A. Postero-lateral within the scrotum, with the epididymis situated CALIMAG- 2015
B. Inferior on the posterolateral aspect of each testicle. LOYOLA
C. Anterior Commonly, the left testicle lies lower than the (TOP 8 - FEB
D. Superior right. They are suspended from the abdomen by 2015 MED
E. Lateral the spermatic cord – collection of vessels, nerves BOARDS;
and ducts that supply the testes. The epididymis is TOPNOTCH
5–10 mm thick and extends from the upper to the MD FROM
A caudal pole of the testis. The epididymis can be UST)
divided in caput, corpus and cauda. A thin capsule
and the serosa of the tunica vaginalis cover the
epididymis.

8–10 efferent ducts transport the sperm from the
rete testis into the ductus epididymidis. The
epididymis consists mostly of the coiled
epididymal duct, at the cauda of the epididymis
the ductus deferens starts.
Reference:
http://www.ncbi.nlm.nih.gov/pubmed/16944482
J Clin Ultrasound. 2006 Oct;34(8):385-92.
Normal and variant appearances of the adult
epididymis and vas deferens on high-resolution
sonography.
Puttemans T1, Delvigne A, Murillo D.
117 The prostatic segment of the male urethra is lined with this Page 27 of Histology Topnotch Handout. ANGELA DIAGNOSTIC
type of epithelium: PAULINE P. EXAM - AUG
A. Stratified columnar CALIMAG- 2015
B. Urothelium LOYOLA
B C. Pseudostratified columnar epithelium (TOP 8 - FEB
D. Stratified squamous epithelium 2015 MED
E. Pseudostratified squamous epithelium BOARDS;
TOPNOTCH
MD FROM
UST)
118 The anal canal above the dentate line is sensitive to:
SIMILAR TO PREVIOUS BOARD EXAM ANGELA DIAGNOSTIC
A. Pain CONCEPT/PRINCIPLE. Page 11 of Day 3 Topnotch PAULINE P. EXAM - AUG
B. Temperature Handout. The anal canal segment above the CALIMAG- 2015
C. Touch dentate line is derived from the hindgut, supplied LOYOLA
D. Pressure and drained by the superior rectal, sensitive to (TOP 8 - FEB
E E. Stretch stretch. 2015 MED
BOARDS;
TOPNOTCH
MD FROM
UST)
119 Which of the following blood cells has the longest life span:
SIMILAR TO PREVIOUS BOARD EXAM ANGELA DIAGNOSTIC
A. RBC CONCEPT/PRINCIPLE. Page 10 of Histology PAULINE P. EXAM - AUG
B. Neutrophils Topnotch Handout. ADULT RBC- 120 days, Fetal CALIMAG- 2015
C. Platelets RBC-90 days, Platelets-10 days, Reticulocytes- 1-2 LOYOLA
D. Eosinophils days, Neutrophils- 1-4 days, Eosinophils-1-2 (TOP 8 - FEB
A E. Reticulocytes weeks, Basophils-Several months, Lymphocytes- 2015 MED
hours to years, Monocytes- hours to years. BOARDS;
TOPNOTCH
MD FROM
UST)
120 A 56 y/o male came to the cardiac catheterization SIMILAR TO PREVIOUS BOARD EXAM ANGELA DIAGNOSTIC
laboratory for suspected acute coronary syndrome. Coronary CONCEPT/PRINCIPLE. Page 17 of Day 2 Topnotch PAULINE P. EXAM - AUG
angiography showed a 90% occlusion in the distal left Handout. In approximately 70% of the population, CALIMAG- 2015
circumflex. The patient had arrived at the hospital 24 hours the PDA originates from the right coronary artery; LOYOLA
earlier with crescendo angina. He had no prior history of it is codominant in 20%, meaning both the right (TOP 8 - FEB
coronary artery disease. His ECG showed Sinus rhythm at coronary artery and LCx feed the PDA; and 10% 2015 MED
C 75/min with acute myocardial injury of the posterior are left dominant, meaning the LCx alone supplies BOARDS;
myocardium manifested as down-sloping ST-segment the PDA. TOPNOTCH
depression in leads V1 to V5. The coronary artery that MD FROM
supplies the PDA determines coronary “dominance.” In UST)
approximately 70% of the population, the PDA originates
from: A. Left circumflex artery
B. Left Coronary artery
C. Right Coronary artery
D. Marginal artery
E. Both A and C

121 The following layers of abdominal wall have scrotal Superficial fascia - Dartos muscle; External oblique LYNN DARYL MIDTERM 1
derivatives except: - External spermatic fascia; Internal Oblique - FELICIANO EXAM - AUG
A. External oblique Cremaster muscle; Transversalis fascia - Internal VILLAMATE 2015
C B. Peritoneum spermatic fascia; Peritoneum- Tunica vaginalis R, MD (TOP 5
C. Transversus abdominis - FEB 2015
D. Transversalis fascia MED
E. Superficial fascia BOARDS;
TOPNOTCH
MD FROM
EAC)
TOPNOTCH MEDICAL BOARD PREP ANATOMY SUPEREXAM Page 16 of 94
For inquiries visit www.topnotchboardprep.com.ph or email us at topnotchmedicalboardprep@gmail.com
TOPNOTCH MEDICAL BOARD PREP ANATOMY SUPEREXAM
For inquiries visit www.topnotchboardprep.com.ph or email us at topnotchmedicalboardprep@gmail.com
Item QUESTION EXPLANATION AUTHOR TOPNOTCH
# EXAM
122 The following are parts of the greater omentum EXCEPT? Gastrohepatic/hepatogastric (and LYNN DARYL MIDTERM 1
A. Gastrohepatic hepatoduodenal) are parts of the lesser omentum. FELICIANO EXAM - AUG
B. Gastrocolic VILLAMATE 2015
C. Gastrosplenic R, MD (TOP 5
D. Gastrophrenic - FEB 2015
E. No exception MED
A BOARDS;
TOPNOTCH
MD FROM
EAC)
123 The most common site of diverticulitis is the: LYNN DARYL MIDTERM 1
A. Cecum FELICIANO EXAM - AUG
B. Ascending colon VILLAMATE 2015
C. Transverse colon R, MD (TOP 5
E D. Descending colon - FEB 2015
E. Sigmoid MED
BOARDS;
TOPNOTCH
MD FROM
EAC)
124 A 28-year old male tennis player comes to the clinic and This is a case of tennis elbow or lateral LYNN DARYL MIDTERM 1
complained of pain when he opens the door or lift a glass. epicondylitis. Muscle attached to lateral FELICIANO EXAM - AUG
Which of the following muscles is least likely involved in this epicondyle include the following: Anconeus VILLAMATE 2015
E condition?
A. Extensor carpi radialis brevis
muscle, the supinator, extensor carpi radialis
brevis, extensor digitorum, extensor digiti, and
R, MD (TOP 5
- FEB 2015
B. Extensor digitorum extensor carpi ulnaris. ECRL is the exception. Its MED
C. Extensor carpi ulnaris origin is the lateral supracondylar ridge of the BOARDS;
D. Supinator humerus. This is a similar question asked in the TOPNOTCH
E. Extensor carpi radialis longus boards. MD FROM
EAC)

125 The radial artery is the smaller of the terminal branches of The radial artery is the smaller of the terminal LYNN DARYL MIDTERM 1
the brachial artery. It begins at the level of: branches of brachial artery. It begins in the cubital FELICIANO EXAM - AUG
A. Lateral epicondyle of the humerus fossa at the level of the neck of the radius and VILLAMATE 2015
B. Head of the radius passes downward and laterally, beneath the R, MD (TOP 5
C. Radial neck bracioradialis muscle. In the middle third of its - FEB 2015
C D. Ulnar head course, the superficial branch of the radial nerve MED
E. Styloid process of the radius lies on its lateral side. SIMILAR TO PREVIOUS BOARDS;
BOARD EXAM CONCEPT TOPNOTCH
MD FROM
EAC)
126 A 50 year old male patient was noted to have cyanosis and Dorsalis pedis artery can be palpated medially to LYNN DARYL MIDTERM 1
decreased sensation of left foot. You decided to assess the the extensor digitorum longus tendon or laterally FELICIANO EXAM - AUG
dorsalis pedis pulse. It is palpated: to the EHL tendon. Structures that pass in front of VILLAMATE 2015
A. Medially to the extensor hallucis longus tendon medial malleolus: Great saphenous vein and R, MD (TOP 5
B. Medially to the extensor digitorum longus tendon saphenous nerve. Behind the medial malleolus: - FEB 2015
B C. In front of the medial malleolus Posterior tibial artery, tibial nerve, tibialis MED
D. Behind the medial malleolus posterior tendon, FDL and FHL. Behind the lateral BOARDS;
E. Behind the lateral malleolus malleolus: Sural nerve and small saphenous vein. TOPNOTCH
SIMILAR TO PREVIOUS BOARD EXAM CONCEPT MD FROM
EAC)
127 A 35 y/o patient suffered from a vehicular crash affecting his Layers of the kidney from outer to inner: LYNN DARYL MIDTERM 1
right kidney. On exploration, the first layer of the kidney to Pararenal fat, renal fascia/Gerota's fascia, FELICIANO EXAM - AUG
be encountered during surgery is the: perirenal fat, fibrous/true capsule. Renal fascia VILLAMATE 2015
A. Pararenal fat encloses both the kidney and suprarenal glands. R, MD (TOP 5
B. Perirenal fat SIMILAR TO PREVIOUS BOARD EXAM CONCEPT - FEB 2015
C. Renal fascia MED
A D. Outer corte BOARDS;
E. Fibrous capsule TOPNOTCH
MD FROM
EAC)
128 The least dilatable portion of the male urethra: PROSTATIC URETHRA - widest and most dilatable LYNN DARYL MIDTERM 1
A. Prostatic urethra portion; MEMBRANOUS - shortest and least FELICIANO EXAM - AUG
B. Membranous urethra dilatable; PENILE URETHRA - longest, EXTERNAL VILLAMATE 2015
C. Penile urethra MEATUS - narrowest part; FOSSA R, MD (TOP 5
D. Sphincter urethra TERMINALIS/NAVICULAR FOSSA - lies within the - FEB 2015
E. None of the above glans penis. SIMILAR TO PREVIOUS BOARD EXAM MED
CONCEPT BOARDS;
B TOPNOTCH
MD FROM
EAC)
129 The most commonly injured portion of the ureter during The ureter runs vertically downward behind and LYNN DARYL MIDTERM 1
gynecologic surgery? enters the pelvis by crossing the bifurcation of the FELICIANO EXAM - AUG
A. Abdominal portion common iliac artery in front of the sacroiliac joint. VILLAMATE 2015
C B. Middle portion In vaginal hysterectomy, the ureter courses lateral R, MD (TOP 5
C. Pelvic portion to the uterine cervix. In oophorectomy, it lies - FEB 2015
D. Near the bifurcation of aorta medial to ovarian vessels. It also passes inferior to MED
E. As it crosses the pelvic brim the uterine vessels. SIMILAR TO PREVIOUS BOARDS;
BOARD EXAM CONCEPT TOPNOTCH
MD FROM
EAC)

TOPNOTCH MEDICAL BOARD PREP ANATOMY SUPEREXAM Page 17 of 94


For inquiries visit www.topnotchboardprep.com.ph or email us at topnotchmedicalboardprep@gmail.com
TOPNOTCH MEDICAL BOARD PREP ANATOMY SUPEREXAM
For inquiries visit www.topnotchboardprep.com.ph or email us at topnotchmedicalboardprep@gmail.com
Item QUESTION EXPLANATION AUTHOR TOPNOTCH
# EXAM
130 Which of the following is true regarding safe subclavian vein Supraclavicular approach - Patient in LYNN DARYL MIDTERM 1
catherization using the infraclavicular approach? Trendelenburg position with head turned to FELICIANO EXAM - AUG
A. Needle is inserted below the lower border of the clavicle opposite side, the posterior border of clavicular VILLAMATE 2015
at the junction of medial third and outer 2/3 of the clavicle origin of SCM is palpated. Needle is inserted and is R, MD (TOP 5
and is pointed downward and posteriorly toward the middle directed downward in the direction of opposite - FEB 2015
suprasternal notch nipple and enters the junction of IJV and MED
B. The needle is directed upward in the direction of the subclavian vein. Infraclavicular approach - BOARDS;
opposite nipple. needle is inserted through the skin just below the TOPNOTCH
D C. The needle is directed downward in the direction of the lower border of th clavicle at the junction of the MD FROM
opposite nipple. medial third and outer 2/3. Needle is pointed and EAC)
D. Needle is inserted below the lower border of the clavicle upward and posteriorly toward the middle of the
at the junction of medial third and outer 2/3 of the clavicle suprasternal notch. SIMILAR TO PREVIOUS
and is pointed upward and posteriorly toward the middle BOARD EXAM CONCEPT.
suprasternal notch
E. None of the above.

131 Hyaline cartilage is composed of what type of collagen? Type I - Fibrocartilage and elastic cartilage; Type II LYNN DARYL MIDTERM 1
A. Type I - Hyaline cartilage. SIMILAR TO PREVIOUS FELICIANO EXAM - AUG
B. Type II BOARD EXAM CONCEPT. VILLAMATE 2015
C. Type III R, MD (TOP 5
B D. Type IV - FEB 2015
E. Type V MED
BOARDS;
TOPNOTCH
MD FROM
EAC)
132 The main glycosaminoglycan content of the bone is Hyaluronic acid - synovial fluid, vitreous humor; LYNN DARYL MIDTERM 1
A. Chondroitin sulfate Chrondoitin sulfate - none, cornea, skin; FELICIANO EXAM - AUG
B. Hyaluronic acid Dermatan sulftage - skin, tendon; Heparan sulfate - VILLAMATE 2015
C. Dermatan sulfate basal laminae, Keratan sulfate - cornea, nucleus R, MD (TOP 5
A D. Heparan sulfate pulposus, annulus fibosus - FEB 2015
E. Keratan sulfate MED
BOARDS;
TOPNOTCH
MD FROM
EAC)
133 Patient presented with chronic cough and weight loss and The superficial (subpleural) plexus lies beneath LYNN DARYL MIDTERM 1
was diagnosed with lung mass at the hilar area. Lymph the visceral pleura and drains over the surface of FELICIANO EXAM - AUG
metastases will first affect nodes at the: ? the lungs toward the hilum, where the lymph VILLAMATE 2015
B A. Bronchomediastinal lymph trunks
B. Bronchopulmonary nodes
vessels enter the bronchopulmonary nodes. All
the lymph from the lung then leaves the hilum and
R, MD (TOP 5
- FEB 2015
C. Paratracheal drains into the tracheobronchial nodes and then MED
D. Tracheobronchial nodes into the bronchomediastinal lymph trunks. BOARDS;
E. Parasternal SIMILAR TO PREVIOUS BOARD EXAM CONCEPT. TOPNOTCH
MD FROM
EAC)
134 A 50 year old woman was noted to have 2 cm breast cancer Lymph from medial breast quadrants, drains to LYNN DARYL MIDTERM 1
located in the nipple areolar complex of the left breast. the parasternal lymph nodes or to the opposite FELICIANO EXAM - AUG
Which of the following lymph nodes is most likely to be breast while lymph from the inferior quadrants VILLAMATE 2015
affected? may pass deeply to the abdominal lymph nodes R, MD (TOP 5
A. Parasternal lymph nodes (subdiaphragmatic inferior phrenic lymph nodes). - FEB 2015
C B. Inferior phrenic lymph nodes More than 75% of the lymph, especially from the MED
C. Axillary lymph nodes lateral breast quadrants drains to the axillary BOARDS;
D. Subscapular lymph nodes lymph nodes, iniitially to the anterior or pectoral TOPNOTCH
E. Supraclavicular lymph nodes nodes for the most part. MD FROM
EAC)
135 A 15-year-old male suffered from a sprained ankle due to Ankle is the most frequently injured major joint in LYNN DARYL MIDTERM 1
excessive inversion of the foot. This would most likely the body. Ankle sprains (torn fibers of ligaments) FELICIANO EXAM - AUG
demenstrated a torn of? are most common. A sprained ankle is nearly VILLAMATE 2015
A. Anterior talofibular ligament always an inversion injury. Lateral ligament is R, MD (TOP 5
B. Posterior talofibular ligament more frequently injured that medial ligament - FEB 2015
C. Tendocalcaneus because it is much weaker. The anterior MED
D. Deltoid ligament talofibular ligament, a part of lateral ligament is BOARDS;
E. Long plantar ligament most vulnerable and most commonly torn during TOPNOTCH
ankle sprains because it is much weaker. Severe MD FROM
eversion - deltoid or medial ligament. (Moore) EAC)
A
136 Patient underwent splenectomy after he suffered from a Splenorenal ligament connects the spleen to the LYNN DARYL MIDTERM 1
vehicular crash severe injury to the spleen. Which of the left kidney. Between the layers of ths ligament, FELICIANO EXAM - AUG
following ligaments is the most vascular? the splenic artery divides into 5 or more branches VILLAMATE 2015
A. Gastrosplenic ligament that enter the hilum. Gastrosplenic ligament is R, MD (TOP 5
B. Splenorenal ligament closely related to to short gastricvessels. - FEB 2015
B C. Splenocolic MED
D. Splenophrenic BOARDS;
E. Spleno-omental TOPNOTCH
MD FROM
EAC)

TOPNOTCH MEDICAL BOARD PREP ANATOMY SUPEREXAM Page 18 of 94


For inquiries visit www.topnotchboardprep.com.ph or email us at topnotchmedicalboardprep@gmail.com
TOPNOTCH MEDICAL BOARD PREP ANATOMY SUPEREXAM
For inquiries visit www.topnotchboardprep.com.ph or email us at topnotchmedicalboardprep@gmail.com
Item QUESTION EXPLANATION AUTHOR TOPNOTCH
# EXAM
137 Which of the following blood vessel joins the splenic vein to The portal vein is about 5 cm long and is formed LYNN DARYL MIDTERM 1
form the portal vein? behind the neck of the pancreas by the union of FELICIANO EXAM - AUG
A. Inferior mesenteric vein superior mesenteric and splenic veins. It ascends VILLAMATE 2015
B. Right gastric vein to the right, behind the first part of duodenum. It R, MD (TOP 5
C. Inferior mesenteric vein then runs upward in front of the opening into the - FEB 2015
D D. Superior mesenteric vein lesser sac to the porta hepatis, where it divides MED
E. Hepatic vein into right and left terminal branches. BOARDS;
TOPNOTCH
MD FROM
EAC)
138 When oculomotor nerve is affected, patient may present with loss of corneal/blink reflex - CN VII; Extorsion of LYNN DARYL MIDTERM 1
A. Inability to blink the eye and weakness of downward gaze - CN IV; FELICIANO EXAM - AUG
B. Extorsion of the eye loss of afferent pupillary reflex, blindness - CN II; VILLAMATE 2015
C. Loss of afferent pupillary reflex Medial deviation - CN VI; Ptosis, dilated and fixed R, MD (TOP 5
D. Drooping of the eyelid pupil, eyes turned down and out - CN III - FEB 2015
D E. Medial deviation of the affected eye MED
BOARDS;
TOPNOTCH
MD FROM
EAC)
139 Which of the following segment of the colon is most LYNN DARYL MIDTERM 1
susceptible to ischemia? FELICIANO EXAM - AUG
A. Cecum VILLAMATE 2015
B. Ascending colon R, MD (TOP 5
C. Descending colon - FEB 2015
D. Splenic flexure MED
D E. Sigmoid BOARDS;
TOPNOTCH
MD FROM
EAC)
140 Which of the following statements is true regarding renal The kidney has a dark brown outer cortex and a LYNN DARYL MIDTERM 1
cortex? light brown inner medulla. The cortex closely FELICIANO EXAM - AUG
A. It is thicker and has a lighter brown in color thnan the adheres to renal capsule, not the renal fascia. The VILLAMATE 2015
medulla. renal cortex extends into the medulla between R, MD (TOP 5
B. It contains the upper expanded end of the ureter. adjacent pyramids as the renal columns, while the - FEB 2015
C. The apices of the renal pyramids extends into the renal medulla is composed of renal pyramids, each MED
E cortex. having its base oriented toward the cortex and itx BOARDS;
D. The renal fascia closely adheres to the renal cortex and is apex, the renal papilla, projecting medially. TOPNOTCH
continuously laterally with fascia transversalis. MD FROM
E. It extends into the medulla between adjacent pyramids. EAC)

141 A patient with leukemia needs to undergo bone marrow All the other options are the usual sites for bone EDWARD MIDTERM 2
biopsy. The following bones are ideal for bone marrow marrow biopsy, the Posterior iliac crest and HARRY EXAM - AUG
biopsy except: sternum are for adults, in children, the tibia VALLAJERA, 2015
B A. Sternum especially the anterior portion may be used and MD (TOP 8 -
B. Femur the vertebra FEB 2015
C. Posterior iliac crest MED
D. Tibia BOARDS;
E. None of the above TOPNOTCH
MD FROM
PERPETUAL
BINAN)
142 Articular cartilage is mostly made up of this proteoglycan Chondroitin sulfate is the major component of EDWARD MIDTERM 2
A. Chondroitin sulfate articular cartilage, in combination with some HARRY EXAM - AUG
B. Keratan sulfate keratan sulfate, it forms aggrecan which is the VALLAJERA, 2015
C. Heparan sulfate major component of articular cartilage. MD (TOP 8 -
D. Dermatan sulfate FEB 2015
E. Heparin MED
BOARDS;
A TOPNOTCH
MD FROM
PERPETUAL
BINAN)
143 Tennis elbow involves which among of the following muscle? The extensor muscles of the wrist is involved in EDWARD MIDTERM 2
A. Extensor carpi radialis tennis elbow HARRY EXAM - AUG
B. Biceps brachii VALLAJERA, 2015
C. Brachioradialis MD (TOP 8 -
D. Flexor carpi ulnaris FEB 2015
A E. Flexor carpi radialis MED
BOARDS;
TOPNOTCH
MD FROM
PERPETUAL
BINAN)
144 The following are muscles for dorsiflexion of the ankle All of the muscles above are used for dorsiflexion EDWARD MIDTERM 2
except: of the ankle HARRY EXAM - AUG
A. Extensor hallucis longus VALLAJERA, 2015
B. Extensor digitorum longus MD (TOP 8 -
E
C. Peroneus tertius FEB 2015
D. Tibialis anterior MED
E. None of the above BOARDS;
TOPNOTCH
MD FROM
PERPETUAL

TOPNOTCH MEDICAL BOARD PREP ANATOMY SUPEREXAM Page 19 of 94


For inquiries visit www.topnotchboardprep.com.ph or email us at topnotchmedicalboardprep@gmail.com
TOPNOTCH MEDICAL BOARD PREP ANATOMY SUPEREXAM
For inquiries visit www.topnotchboardprep.com.ph or email us at topnotchmedicalboardprep@gmail.com
Item QUESTION EXPLANATION AUTHOR TOPNOTCH
# EXAM
BINAN)

145 During subclavian vein catheterization, how would you The 18-gauge introducer needle is inserted 1 cm EDWARD MIDTERM 2
insert the needle as you puncture the inferior border of the inferior to the junction of the middle and proximal HARRY EXAM - AUG
clavicle in the infraclavicular approach? third of the clavicle while aiming slightly cephalad VALLAJERA, 2015
A. Upward and directed posterior to the middle of the SCM toward your index finger in the suprasternal MD (TOP 8 -
B. Upward and directed to the suprasternal notch notch. FEB 2015
B C. Downward
D. Straight
MED
BOARDS;
E. None of the above TOPNOTCH
MD FROM
PERPETUAL
BINAN)
146 Which of the following veins in the arm is used for large vein The basilic vein is most commonly used as a EDWARD MIDTERM 2
catheterization due to its increasing diameter as it goes venous access for venous catheterization because HARRY EXAM - AUG
proximally? it becomes larger as goes proximally VALLAJERA, 2015
A. Cephalic MD (TOP 8 -
B B. Basilic FEB 2015
C. Median cubital vein MED
D. Innominate vein BOARDS;
E. None of the above TOPNOTCH
MD FROM
PERPETUAL
BINAN)
147 A man sustained a stab wound located at the 4th ICS directed The needle is inserted at the subxiphoid area and EDWARD MIDTERM 2
inferiorly, you were able to appreciate Beck's triad. What is is directed superiorly and to the left aiming for the HARRY EXAM - AUG
the quickest way to perform pericardiocentesis? patient's left shoulder. VALLAJERA, 2015
A. 4th ICS parasternal line left MD (TOP 8 -
B. 4th ICS parasternal line right FEB 2015
D C. 5th ICS midclavicular line MED
D. Subxiphoid BOARDS;
E. 2nd ICS parasternal line left TOPNOTCH
MD FROM
PERPETUAL
BINAN)
148 At which tracheal cartilage would you perform The 2nd and 3rd tracheal rings are the best sites to EDWARD MIDTERM 2
tracheostomy? perform tracheostomy but care must be noted to HARRY EXAM - AUG
A. 1st and 2nd avoid hitting the isthmus of the thyroid gland VALLAJERA, 2015
B. 2nd and 3rd which is highly vascularized. If ever the thyroid MD (TOP 8 -
C. 3rd and 4th isthmus is encountered, it should be retracted FEB 2015
B D. 4th and 5th inferiorly. MED
E. cricothyroid membrane BOARDS;
TOPNOTCH
MD FROM
PERPETUAL
BINAN)
149 The dorsalis pedis pulse is best appreciated where? It is appreciated best between the 1st and 2nd EDWARD MIDTERM 2
A. Between 1st and 2nd metatarsal space metatarsal space of the foot. HARRY EXAM - AUG
B. Between 2nd and 3rd metatarsal space VALLAJERA, 2015
C. 3rd and 4th metatarsal space MD (TOP 8 -
D. Between 4th and 5th metatarsal space FEB 2015
A E. Behind medial malleolus MED
BOARDS;
TOPNOTCH
MD FROM
PERPETUAL
BINAN)
150 The dorsalis pedis pulse is related to which structure? It is closely related to the lateral border of the EDWARD MIDTERM 2
A. Lateral to the extensor hallucis longus extensor hallucis longus of the 1st toe. HARRY EXAM - AUG
B. Medial to the extensor hallucis longus VALLAJERA, 2015
C. Lateral to the flexor hallucis longus MD (TOP 8 -
D. Lateral to the medial malleolus FEB 2015
A E. None of the above MED
BOARDS;
TOPNOTCH
MD FROM
PERPETUAL
BINAN)
151 The spleen is closest to which of the following? The spleen is closest to the inferior border of the EDWARD MIDTERM 2
A. Kidney diaphragm HARRY EXAM - AUG
B. Stomach VALLAJERA, 2015
C. Splenic flexure MD (TOP 8 -
D. Tail of the pancreas FEB 2015
E. Inferior border of the diaphragm MED
E BOARDS;
TOPNOTCH
MD FROM
PERPETUAL
BINAN)

TOPNOTCH MEDICAL BOARD PREP ANATOMY SUPEREXAM Page 20 of 94


For inquiries visit www.topnotchboardprep.com.ph or email us at topnotchmedicalboardprep@gmail.com
TOPNOTCH MEDICAL BOARD PREP ANATOMY SUPEREXAM
For inquiries visit www.topnotchboardprep.com.ph or email us at topnotchmedicalboardprep@gmail.com
Item QUESTION EXPLANATION AUTHOR TOPNOTCH
# EXAM
152 Which of the following cells produces the fibers that form the These cells produce the fibers that form the EDWARD MIDTERM 2
structural framework of most lymphatic structure of lymphatic organs HARRY EXAM - AUG
organs? VALLAJERA, 2015
A. Mononuclear cells MD (TOP 8 -
B. Reticular cells FEB 2015
C. Macrophages MED
D. Bilobed cells BOARDS;
E. None of the above TOPNOTCH
B MD FROM
PERPETUAL
BINAN)
153 The spleen is easily ruptured in blunt abdominal trauma All of the above are possible causes of the spleen EDWARD MIDTERM 2
because of which of the following factors: ruputuring in blunt abdominal trauma. HARRY EXAM - AUG
A. Highly vascular organ VALLAJERA, 2015
B. Soft parenchyma MD (TOP 8 -
E C. Sudden deceleration and acceleration
D. Direct blow to the area of the spleen
FEB 2015
MED
E. All of the above BOARDS;
TOPNOTCH
MD FROM
PERPETUAL
BINAN)
154 A patient came in the ER due to blunt abdominal trauma The patient has probably a ruptured spleen, Kehr's EDWARD MIDTERM 2
secondary to a vehicular accident, the patient is awake and sign is the occurrence of acute pain in the tip of HARRY EXAM - AUG
conversant and just complains of a minimal vague the shoulder due to the presence of blood or other VALLAJERA, 2015
abdominal pain, vital signs are still stable yet you were able irritants in the peritoneal cavity when a person is MD (TOP 8 -
to elicit a positive Kehr's sign, what would you do next? lying down and the legs are elevated. Kehr's sign FEB 2015
A A. Perform emergency exploratory laparotomy in the left shoulder is considered a classical MED
B. Observe the patient symptom of a ruptured spleen. BOARDS;
C. GIve analgesics TOPNOTCH
D. Send the patient home MD FROM
E. None of the above PERPETUAL
BINAN)

155 A 1 year old boy is brought to you due to vomiting everytime The treatment of choice for intussusception in this EDWARD MIDTERM 2
after feeding and passage of jelly like stools, you were able to age group is barium enema or air enema HARRY EXAM - AUG
appreciate a small hard mass on the right lower quadrant, accompanied by GI decompression by inserting a VALLAJERA, 2015
what would you do next? NGT. MD (TOP 8 -
A. Perform exploratory laparotomy FEB 2015
B B. Perform a barium enema
C. Observe the patient
MED
BOARDS;
D. Give an anxiolytic TOPNOTCH
E. None of the above MD FROM
PERPETUAL
BINAN)
156 A 5 year old accidentally swallowed a 5 peso coin, on The orientation of the coin on both anterior and EDWARD MIDTERM 2
performing radiograph, the coin was located at the level of lateral views suggest that the coin is at the level of HARRY EXAM - AUG
T4, the coin on AP CXR looks like a slit while on lateral view the bifurcation of the trachea VALLAJERA, 2015
B it looks round, you know that the coin is most likely in the:
A. Esophagus
MD (TOP 8 -
FEB 2015
B. Bifurcation of the trachea MED
C. Right main bronchus BOARDS;
D. Left main bronchus TOPNOTCH
E. Inferior right bronchopulmonary segment MD FROM
PERPETUAL
BINAN)
157 All of the following organs actively fight pathogens except: The thymus is the site of maturation and selection EDWARD MIDTERM 2
A. Cervical lymph nodes of self-tolerant T lymphocytes HARRY EXAM - AUG
B. Thymus VALLAJERA, 2015
C. Spleen MD (TOP 8 -
D. Axillary lymph nodes FEB 2015
E. Tonsils MED
BOARDS;
B TOPNOTCH
MD FROM
PERPETUAL
BINAN)
158 Which of the following does not empty into the thoracic The right subclavian trunk drains into the right EDWARD MIDTERM 2
duct? lymphatic duct to enter the right subclavian vein HARRY EXAM - AUG
A. Right subclavian trunk VALLAJERA, 2015
B. Left jugular trunk MD (TOP 8 -
A C. Cisterna chyli FEB 2015
D. Left subclavian trunk MED
E. None of the above BOARDS;
TOPNOTCH
MD FROM
PERPETUAL
BINAN)

TOPNOTCH MEDICAL BOARD PREP ANATOMY SUPEREXAM Page 21 of 94


For inquiries visit www.topnotchboardprep.com.ph or email us at topnotchmedicalboardprep@gmail.com
TOPNOTCH MEDICAL BOARD PREP ANATOMY SUPEREXAM
For inquiries visit www.topnotchboardprep.com.ph or email us at topnotchmedicalboardprep@gmail.com
Item QUESTION EXPLANATION AUTHOR TOPNOTCH
# EXAM
159 As lymph flows through a lymph node, which of the following It flows from the afferent vessel then to the EDWARD MIDTERM 2
would it go through last subcapsular sinus then to the cortical sinus then to HARRY EXAM - AUG
A. Subcapsular sinus the medullary sinus and finally into the efferent VALLAJERA, 2015
B. Afferent lymphatic vessel vessel MD (TOP 8 -
C. Medullary sinus FEB 2015
C D. Cortical sinus MED
E. None of the above BOARDS;
TOPNOTCH
MD FROM
PERPETUAL
BINAN)
160 Most of the body's mucosa-associated lymphatic tissue can These 2 systems are constantly exposed to EDWARD MIDTERM 2
be found in which of the following pathogens and are therefore with the most HARRY EXAM - AUG
A. Respiratory and Reproductive tract number of mucosa associated lymphatic tissue. VALLAJERA, 2015
B. Gastrointestinal and urinary tract MD (TOP 8 -
C. Urinary and reproductive tract FEB 2015
D. Respiratory and gastrointestinal tract MED
D E. None of the above BOARDS;
TOPNOTCH
MD FROM
PERPETUAL
BINAN)
161 The dorsalis pedis pulse can be palpated immediately on PBEQ HAROLD JAY MIDTERM 3
which part of the foot? S. BAYTEC, EXAM - AUG
A. Medially to the external hallucis longus tendon MD (TOP 10 2015
B. Lateral to the external digitorum longus tendon - FEB 2015
D C. Medial to the tibialis anterior tendon
D. Lateral to the external hallucis longus tendon
MED
BOARDS;
E. lateral to the tibialis anterior tendon TOPNOTCH
MD FROM
FEU)
162 The dorsalis pedis artery courses its way distally and can be PBEQ HAROLD JAY MIDTERM 3
palpated between the following landmarks. S. BAYTEC, EXAM - AUG
A. 1st and 2nd metatarsals MD (TOP 10 2015
A B. 2nd and 3rd metatarsals - FEB 2015
C. 3rd an 4th metatarsals MED
D. 1st and 2nd proximal phalanges BOARDS;
E. 2nd and 3rd proximal phalanges TOPNOTCH
MD FROM
FEU)
163 Which of the following part is mostly affected in tennis tennis elbow is lateral epicondylitis HAROLD JAY MIDTERM 3
elbow? S. BAYTEC, EXAM - AUG
A. Medial part of the elbow MD (TOP 10 2015
B. lateral part of the elbow - FEB 2015
C. superior part of the elbow MED
B D. Inferior part of the elbow
E. Posterior part of the elbow
BOARDS;
TOPNOTCH
MD FROM
FEU)
164 25. SB, a 35 year old male, was diagnosed with hypertension hydrochlorothiazide inhibits sodium reabsorption HAROLD JAY MIDTERM 3
just last week. Her doctor decided to start him with in distal renal tubules resulting in increased S. BAYTEC, EXAM - AUG
hydrochlorothiazide. Where is the site of action of the drug? excretion of water and of Na, K, and hydrogen ions MD (TOP 10 2015
A. PCT - FEB 2015
B. DCT MED
C. CT BOARDS;
B D. LOH TOPNOTCH
E. Glomerulus MD FROM
FEU)

165 The breast receives its blood supply from which of the HAROLD JAY MIDTERM 3
following blood vessels? S. BAYTEC, EXAM - AUG
A. Perforating branches of internal mammary artery MD (TOP 10 2015
B. Lateral branches of the posterior intercostal arteries - FEB 2015
C. Branches from axillary artery MED
D. A and B BOARDS;
E. All of the above TOPNOTCH
E MD FROM
FEU)
166 By weight, which of the following is considered the largest ft if the question is largest organ, the answer HAROLD JAY MIDTERM 3
organ of the body? should be liver since the skin is not considered an S. BAYTEC, EXAM - AUG
Ab A. Skin
B. Liver
organ but an organ system MD (TOP 10
- FEB 2015
2015

C. Lung MED
D. Heart BOARDS;
E. Brain TOPNOTCH
MD FROM
FEU)
167 Which among the following muscle is considered as the gluteus maximus is generally the biggest. HAROLD JAY MIDTERM 3
longest muscle of the human body? Latissimus dorsi is the widest S. BAYTEC, EXAM - AUG
A. Gluteus maximus MD (TOP 10 2015
C B. Latissimus dorsi - FEB 2015
C. sartorius MED
D. Quadriceps femoris BOARDS;
E. Psoas major TOPNOTCH
MD FROM
FEU)
TOPNOTCH MEDICAL BOARD PREP ANATOMY SUPEREXAM Page 22 of 94
For inquiries visit www.topnotchboardprep.com.ph or email us at topnotchmedicalboardprep@gmail.com
TOPNOTCH MEDICAL BOARD PREP ANATOMY SUPEREXAM
For inquiries visit www.topnotchboardprep.com.ph or email us at topnotchmedicalboardprep@gmail.com
Item QUESTION EXPLANATION AUTHOR TOPNOTCH
# EXAM
168 All of the following statements regarding the vessels of the lymphatics from the middle third drain into the HAROLD JAY MIDTERM 3
vagina EXCEPT: internal iliac nodes. the table combined the S. BAYTEC, EXAM - AUG
A. Lymphatics from the middle third drain into the external lymphatic drainage of the upper and middle 3rd. MD (TOP 10 2015
iliac nodes OB Williams 23rd edition page 18. "Those from the - FEB 2015
B. The proximal portion is supplied by the cervical branch middle third drain into the internal iliac nodes, MED
of the uterine artery and those from the upper third drain into the BOARDS;
C. the middle rectal artery contributes to supply the external, internal, and common iliac nodes." TOPNOTCH
A posterior vaginal wall MD FROM
D. distal walls receive contributions from the internal FEU)
pudendal artery
E. extensive venous plexus immediately sorrounds the
vagina and follows the course of the arteries

169 This structure originated from the densest portion of the A B and C are all the same. HAROLD JAY MIDTERM 3
broad ligament and is considered as provider of the major S. BAYTEC, EXAM - AUG
support for the uterus and cervix MD (TOP 10 2015
E A. Cardinal ligament - FEB 2015
B. Mackenrodt ligament MED
C. Transverse cervical ligament BOARDS;
D. None of the above TOPNOTCH
E. all of the above MD FROM
FEU)

170 What is the most common type of leiomyoma of the uterus? SIMILAR TO PREVIOUS BOARD EXAM HAROLD JAY MIDTERM 3
A. subserous CONCEPT/PRINCIPLE S. BAYTEC, EXAM - AUG
B. Intramural MD (TOP 10 2015
C. submucous - FEB 2015
B D. cervical MED
E. None of the above BOARDS;
TOPNOTCH
MD FROM
FEU)
171 In a kidney surgery, when the surgeon penetrated the the choices are arranged from the outer to inner HAROLD JAY MIDTERM 3
peritoneum, the next layer that he will encounter is layers. almost all the pictures that i saw online and S. BAYTEC, EXAM - AUG
A. Pararenal fat on books have a fat in between the peritoneum MD (TOP 10 2015
B. Renal fascia and the gerotas fascia, and almost all sources - FEB 2015
C. Perinephric fat or perirenal fat describe this layer as the paranephric fat.. MED
A D. Renal capsule SIMILAR TO PREVIOUS BOARD EXAM BOARDS;
E. Renal cortex CONCEPT/PRINCIPLE TOPNOTCH
MD FROM
FEU)
172 Gerota's fascia is the structure that immediately encloses SIMILAR TO PREVIOUS BOARD EXAM HAROLD JAY MIDTERM 3
both the kidneys and suprarenal glands. This fascia is also CONCEPT/PRINCIPLE S. BAYTEC, EXAM - AUG
called as MD (TOP 10 2015
B A. Pararenal fat
B. Renal fascia
- FEB 2015
MED
C. Perinephric fat or perirenal fat BOARDS;
D. Renal capsule TOPNOTCH
E. Renal cortex MD FROM
FEU)
173 Which of the following structures is NOT derived from spleen is from mesoderm HAROLD JAY MIDTERM 3
endoderm? S. BAYTEC, EXAM - AUG
A. bronchus MD (TOP 10 2015
B. trachea - FEB 2015
C C. spleen MED
D. stomach BOARDS;
E. Duodenum TOPNOTCH
MD FROM
FEU)
174 Peyer's patches are seen in what part of the GIT? SIMILAR TO PREVIOUS BOARD EXAM HAROLD JAY MIDTERM 3
A. Large intestines CONCEPT/PRINCIPLE. Peyer's patches are only S. BAYTEC, EXAM - AUG
B. Ileum found in ileum MD (TOP 10 2015
C. jejunum - FEB 2015
D. duodenum MED
E. Stomach BOARDS;
B TOPNOTCH
MD FROM
FEU)
175 Brunner's glands are found in what part/s of the GIT Brunner's gland are found in duodenum and HAROLD JAY MIDTERM 3
A. Stomach peyer's patches are found in ileum. No peyer's S. BAYTEC, EXAM - AUG
B. jejunum patches nor brunner's glands are found in jejunum MD (TOP 10 2015
C. ileum - FEB 2015
D. duodenum MED
D E. B and D BOARDS;
TOPNOTCH
MD FROM
FEU)

TOPNOTCH MEDICAL BOARD PREP ANATOMY SUPEREXAM Page 23 of 94


For inquiries visit www.topnotchboardprep.com.ph or email us at topnotchmedicalboardprep@gmail.com
TOPNOTCH MEDICAL BOARD PREP ANATOMY SUPEREXAM
For inquiries visit www.topnotchboardprep.com.ph or email us at topnotchmedicalboardprep@gmail.com
Item QUESTION EXPLANATION AUTHOR TOPNOTCH
# EXAM
176 Which of the following muscles is LEAST likely to cause Thyrohyoid is attached to the inferior of the hyoid HAROLD JAY MIDTERM 3
elevation of the hyoid bone? bone so it depresses the latter. choice A is S. BAYTEC, EXAM - AUG
A. hypoglossus hypoglossus and not hyoglossus. i intentionally MD (TOP 10 2015
B. stylohyoid put the choice A which is hyPOglossus (CN XI) to - FEB 2015
C. Geniohyoid make a little trick to the question just like a typical MED
E D. Mylohyoid board exam question. some might have read the BOARDS;
E. Thyrohyoid choice A as hyoglossus which is a muscle which TOPNOTCH
does not elevate the hyoid bone. MD FROM
FEU)
177 The sternal angle of Louis is an important landmark of the IVD of T4 and T5 HAROLD JAY MIDTERM 3
chest because it lies on the following structures EXCEPT: S. BAYTEC, EXAM - AUG
A. Second costal cartilage MD (TOP 10 2015
B. Bifurcation of the trachea - FEB 2015
C. Intervertebral disc between T2 and T3 MED
C D. Junction of the superior and inferior mediastinum BOARDS;
E. junction of the ascending aorta and the aortic arch TOPNOTCH
MD FROM
FEU)
178 A patient came in at the ER due to gunshot wound on the SIMILAR TO PREVIOUS BOARD EXAM HAROLD JAY MIDTERM 3
abdomen which prompted exploratory laparotomy. Upon CONCEPT/PRINCIPLE. It is the part of the large S. BAYTEC, EXAM - AUG
opening, you saw that a portion of the abdominal aorta was intestines that is farthest from the SMA and IMA MD (TOP 10 2015
hit. Which of the following part of the large intestine is most - FEB 2015
D vulnerable for ischemia? MED
A. cecum BOARDS;
B. ascending colon TOPNOTCH
C. hepatic flexure MD FROM
D. splenic flexure FEU)
E. sigmoid

179 The total refractive power of the eye is mostly contributed 2/3 of the total refractive power is contributed by HAROLD JAY MIDTERM 3
by: cornea S. BAYTEC, EXAM - AUG
A. cornea MD (TOP 10 2015
B. pupil - FEB 2015
A C. lens
D. Vitreous humor
MED
BOARDS;
E. Acqueous humor TOPNOTCH
MD FROM
FEU)
180 Cricoid cartilage can be seen at what vertebral level? HAROLD JAY MIDTERM 3
A. C4 S. BAYTEC, EXAM - AUG
B. C5 MD (TOP 10 2015
C C. C6 - FEB 2015
D. C7 MED
E. T1 BOARDS;
TOPNOTCH
MD FROM
FEU)
181 You can palpate the pulsations of dorsalis pedis artery in SIMILAR TO PREVIOUS BOARD EXAM JEAN PAOLO FINAL EXAM
what location in the foot? CONCEPT/PRINCIPLE. M. DELFINO, - AUG 2015
A. Between 1st and 2nd metatarsals MD (TOP 10
B. Between 2nd and 3rd metatarsals - FEB 2015
C. Between 3rd and 4th metatarsals MED
D. In front of the ankle joint BOARDS;
A E. Medial to 1st metatarsal TOPNOTCH
MD FROM
FATIMA)
182 What is the lymph drainage of the lower 1/3 of esophagus? SIMILAR TO PREVIOUS BOARD EXAM JEAN PAOLO FINAL EXAM
A. Bronchomediastinal nodes CONCEPT/PRINCIPLE.. Lymph vessels from the M. DELFINO, - AUG 2015
B. Tracheobronchial nodes upper third of the esophagus drain into the deep MD (TOP 10
C. Paraaortic nodes cervical nodes, from the middle third into the - FEB 2015
D. Left gastric nodes superior and posterior mediastinal nodes, and MED
E. Posterior mediastinal nodes from the lower third into nodes along the left BOARDS;
D gastric blood vessels and the celiac nodes TOPNOTCH
MD FROM
FATIMA)
183 During TAHBSO of a 45 year old G3P3, what part of the SIMILAR TO PREVIOUS BOARD EXAM JEAN PAOLO FINAL EXAM
ureter is most commonly injured? CONCEPT/PRINCIPLE. M. DELFINO, - AUG 2015
A. Proximal 1/3 MD (TOP 10
B. Middle 1/3 - FEB 2015
C. Distal 1/3 MED
D. B and C BOARDS;
E. None of the above TOPNOTCH
C MD FROM
FATIMA)

TOPNOTCH MEDICAL BOARD PREP ANATOMY SUPEREXAM Page 24 of 94


For inquiries visit www.topnotchboardprep.com.ph or email us at topnotchmedicalboardprep@gmail.com
TOPNOTCH MEDICAL BOARD PREP ANATOMY SUPEREXAM
For inquiries visit www.topnotchboardprep.com.ph or email us at topnotchmedicalboardprep@gmail.com
Item QUESTION EXPLANATION AUTHOR TOPNOTCH
# EXAM
184 In doing subclavian vein catheterization, the needle should SIMILAR TO PREVIOUS BOARD EXAM JEAN PAOLO FINAL EXAM
be inserted through the skin just below the lower border of CONCEPT/PRINCIPLE.. The question describes an M. DELFINO, - AUG 2015
the clavicle at the junction of the medial third and outer two Infraclavicular approach: needle is inserted at MD (TOP 10
thirds. The needle is then pointed in what direction? lower border of clavicle, then it is pointed upward - FEB 2015
A. downward and medially toward the mediastinum and posteriorly toward the middle of the MED
B. downward and medially towards the opposite nipple suprasternal notch. Supraclavicular approach: The BOARDS;
C. downward and posteriorly towards the suprasternal needle is inserted through the skin at the site TOPNOTCH
D notch where the posterior border of the clavicular origin MD FROM
D. upward and posteriorly toward the suprasternal notch of sternocleidomastoid is attached to the upper FATIMA)
E. upward and medially towards the opposite nipple border of the clavicle. then it is pointed downward
and medially toward the mediastinum.
185 The first branch of the internal iliac artery supplies what JEAN PAOLO FINAL EXAM
organ? M. DELFINO, - AUG 2015
A. Middle portion of ureter MD (TOP 10
B B. Bladder
C. Uterus
- FEB 2015
MED
D. Rectum BOARDS;
E. Prostate gland TOPNOTCH
MD FROM
FATIMA)
186 Which of the following intrinsic muscles of the larynx can Posterior cricoarytenoid abducts the vocal cords JEAN PAOLO FINAL EXAM
abduct the vocal fold? by rotating arytenoid cartilage M. DELFINO, - AUG 2015
C A. Cricothyroid MD (TOP 10
B. Transverse arytenoid - FEB 2015
C. Posterior cricoarytenoid MED
D. Lateral cricoarytenoid BOARDS;
E. Oblique arytenoid TOPNOTCH
MD FROM
FATIMA)
187 During splenectomy, injury to what structure can result to SIMILAR TO PREVIOUS BOARD EXAM JEAN PAOLO FINAL EXAM
damage of pancreas? CONCEPT/PRINCIPLE.. The presence of the tail of M. DELFINO, - AUG 2015
A. Gastrosplenic ligament the pancreas in the splenicorenal ligament MD (TOP 10
B. Splenorenal ligament sometimes results in its damage during - FEB 2015
B C. Hepatosplenic ligament
D. Splenocolic ligament
splenectomy MED
BOARDS;
E. Pancreaticosplenic ligament TOPNOTCH
MD FROM
FATIMA)
188 What is the most common cause of aortic aneurysm? SIMILAR TO PREVIOUS BOARD EXAM JEAN PAOLO FINAL EXAM
A. Atherosclerosis CONCEPT/PRINCIPLE.. Atherosclerosis is the most M. DELFINO, - AUG 2015
B. Medial degeneration common cause of aortic aneurysm. MD (TOP 10
C. Congenital - FEB 2015
D. Infection MED
A E. None of the above BOARDS;
TOPNOTCH
MD FROM
FATIMA)
189 What is the location of epididymis in relation to the testis? SIMILAR TO PREVIOUS BOARD EXAM JEAN PAOLO FINAL EXAM
A. anteromedial CONCEPT/PRINCIPLE. M. DELFINO, - AUG 2015
B. medial MD (TOP 10
C. posterolateral - FEB 2015
D. posterior MED
C E. superolateral BOARDS;
TOPNOTCH
MD FROM
FATIMA)
190 What is the lining epithelium of seminal vesicles? seminal vesicles, prostate gland, epididymis and JEAN PAOLO FINAL EXAM
A. transitional vas deferens have pseudostratified columnar non- M. DELFINO, - AUG 2015
B. pseudostratified columnar ciliated lining epithelium. The latter 2 have MD (TOP 10
C. simple columnar stereocilia. - FEB 2015
D. simple cuboidal MED
E. stratified columnar BOARDS;
B TOPNOTCH
MD FROM
FATIMA)
191 What structure divides the subclavian artery into 3 Scalene anterior divides the subclavian artery into JEAN PAOLO FINAL EXAM
segments? 3 segments. 1st segment is medial to the muscle, M. DELFINO, - AUG 2015
A. 1st rib 2nd segment is posterior, and the 3rd segment is MD (TOP 10
E B. subclavius
C. pectoralis minor
lateral to the muscle. - FEB 2015
MED
D. teres major BOARDS;
E. scalene anterior TOPNOTCH
MD FROM
FATIMA)
192 The posterior belly of digastric is innervated by what nerve? Facial nerve innervates structures derived from JEAN PAOLO FINAL EXAM
A. CN X the 2nd pharyngeal arch: muscles of facial M. DELFINO, - AUG 2015
B. CN IX expression, stapedius, stylohyoid and posterior MD (TOP 10
C. CN VIII bellt of digastric. The anterior belly of digastric is - FEB 2015
D D. CN VII innervated by trigeminal nerve. MED
E. CN V BOARDS;
TOPNOTCH
MD FROM
FATIMA)

TOPNOTCH MEDICAL BOARD PREP ANATOMY SUPEREXAM Page 25 of 94


For inquiries visit www.topnotchboardprep.com.ph or email us at topnotchmedicalboardprep@gmail.com
TOPNOTCH MEDICAL BOARD PREP ANATOMY SUPEREXAM
For inquiries visit www.topnotchboardprep.com.ph or email us at topnotchmedicalboardprep@gmail.com
Item QUESTION EXPLANATION AUTHOR TOPNOTCH
# EXAM
193 All of the following drains into the right atrium except SIMILAR TO PREVIOUS BOARD EXAM JEAN PAOLO FINAL EXAM
A. Superior vena cava CONCEPT/PRINCIPLE.. SVC, IVC, coronary sinus, M. DELFINO, - AUG 2015
B. Coronary sinus anterior cardiac veins and least cardiac vein (aka MD (TOP 10
C. Small cardiac vein Thebesian vein) drain into the right atrium. The - FEB 2015
C D. Anterior cardiac vein following drains into the coronary sinus: great MED
E. Thebesian vein cardiac vein, middle cardiac vein, small cardiac BOARDS;
vein, posterior vein of left ventricle. TOPNOTCH
MD FROM
FATIMA)
194 During a difficult dental extraction, patient accidentally Because the right bronchus is the wider and more JEAN PAOLO FINAL EXAM
aspirated a tooth. He was immediately brought to you at the direct continuation of the trachea, foreign bodies M. DELFINO, - AUG 2015
ER. Upon PE, you noted that breath sounds are decreased on tend to enter the right instead of the left bronchus. MD (TOP 10
an area of the chest. There was noted intermittent cough. - FEB 2015
Where is the foreign body probably located? MED
B A. trachea BOARDS;
B. R primary bronchus TOPNOTCH
C. L secondary bronchus MD FROM
D. Larynx FATIMA)
E. carina

195 What is the reason for testis descent during infancy? In the male, the testis descends through the pelvis JEAN PAOLO FINAL EXAM
A. Maternal estrogen and inguinal canal during the seventh and eighth M. DELFINO, - AUG 2015
B. Testosterone months of fetal life. The normal stimulus for the MD (TOP 10
B C. Temperature descent of the testis is testosterone, which is - FEB 2015
D. pH secreted by the fetal testes.SIMILAR TO PREVIOUS MED
E. Gravity BOARD EXAM CONCEPT/PRINCIPLE. BOARDS;
TOPNOTCH
MD FROM
FATIMA)
196 Lesion in CN III will result to which eye position? CN III supplies all extraocular muscles except JEAN PAOLO FINAL EXAM
A. Abducted and depressed lateral rectus and superior oblique. The eye M. DELFINO, - AUG 2015
B. Adducted and extorted position after injury to CN III is abducted and MD (TOP 10
C. Abducted and extorted depressed. Lesion in CN IV would result to - FEB 2015
A D. Adducted and depressed extorted eyeball while lesion in CN VI would result MED
E. A and C to adducted position. BOARDS;
TOPNOTCH
MD FROM
FATIMA)
197 Thiazide diuretic acts on this portion of the nephron? giveaway question last boards JEAN PAOLO FINAL EXAM
A. PCT M. DELFINO, - AUG 2015
B. DCT MD (TOP 10
C. Ascending limb of LOH - FEB 2015
B D. Collecting duct
E. Collecting tubule
MED
BOARDS;
TOPNOTCH
MD FROM
FATIMA)
198 Which structure is not traversed during pericardiocentesis? Layers traversed during pericardiocentesis: JEAN PAOLO FINAL EXAM
A. Rectus sheath subxiphoid approach- skin, fascia, rectus sheath, M. DELFINO, - AUG 2015
B. Rectus abdominis rectus abdominis, fibrous layer of pericardium, MD (TOP 10
C. Serous pericardium parietal layer of visceral pericardium - FEB 2015
D. Endothoracic fascia MED
D E. Fibrous layer of pericardium BOARDS;
TOPNOTCH
MD FROM
FATIMA)
199 Goblet cells are present until what structure in the Respiratory epithelium (pseudostratified JEAN PAOLO FINAL EXAM
respiratory system? columnar with goblet cells) is present up to the M. DELFINO, - AUG 2015
A. Trachea level of bronchioles. MD (TOP 10
B. Bronchus - FEB 2015
C. Bronchiole MED
C D. Terminal bronchiole
E. Respiratory bronchiole
BOARDS;
TOPNOTCH
MD FROM
FATIMA)
200 The following are anterior relations of the right kidney Transversus abdominis is at the posterior of the JEAN PAOLO FINAL EXAM
except? kidney, not anterior M. DELFINO, - AUG 2015
A. adrenal MD (TOP 10
B. liver - FEB 2015
C. transversus abdominis MED
C D. R colic flexure BOARDS;
E. 2nd part of duodenum TOPNOTCH
MD FROM
FATIMA)
201 This shunt muscle contains the median nerve and the With the deltoid and long head of the triceps, the GRACE DIAGNOSTIC
brachial artery. coracobrachialis serves as a shunt muscle, ARVIOLA, EXAM - FEB
A. Biceps brachii resisting downward dislocation of th e head of the MD (TOP 3 - 2015
B. Brachialis humerus, as when carrying a heavy suitcase. AUG 2014
C. Coracobrachialis MED
C D. Deltoid BOARDS;
E. Triceps brachii TOPNOTCH
MD)

TOPNOTCH MEDICAL BOARD PREP ANATOMY SUPEREXAM Page 26 of 94


For inquiries visit www.topnotchboardprep.com.ph or email us at topnotchmedicalboardprep@gmail.com
TOPNOTCH MEDICAL BOARD PREP ANATOMY SUPEREXAM
For inquiries visit www.topnotchboardprep.com.ph or email us at topnotchmedicalboardprep@gmail.com
Item QUESTION EXPLANATION AUTHOR TOPNOTCH
# EXAM
202 The joint that is responsible for thumb opposition is: The CMC joint of the thumb is a saddle joint. Other GRACE DIAGNOSTIC
A. carpometacarpal joint CMC joints are of the plane type of synovial joint. ARVIOLA, EXAM - FEB
B. interphalangeal joint MD (TOP 3 - 2015
C. radiocarpal joint AUG 2014
D. metacarpophalangeal joint MED
A E. intercarpal joint BOARDS;
TOPNOTCH
MD)
203 During CABG, the surgeon inserted his finger into the In the embryology of the heart, as the heart tube GRACE DIAGNOSTIC
transverse pericardial sinus. What structures are folds, its venous end moves posterosuperiorly so ARVIOLA, EXAM - FEB
A demarcated?
A. SVC, pulmonary trunk, and aorta
that the venous end of the tube lies adjacent to the
arterial end, separated only by the transverse
MD (TOP 3 -
AUG 2014
2015

B. IVC, pulmonary veins pericardial sinus. MED


C. Right and left atria BOARDS;
D. Right and left ventricles TOPNOTCH
E. SVC and IVC MD)

204 The oblique vein of Marshall is a remnant of: The oblique vein of the left atrium (of Marshall) is GRACE DIAGNOSTIC
A. Right IVC a small vessel, relatively unimportant postnatally, ARVIOLA, EXAM - FEB
C B. Right SVC
C. Left SVC
that descends over the posterior wall of the left
atrium and merges with the great cardiac vein to
MD (TOP 3 -
AUG 2014
2015

D. Left IVC form the coronary sinus. This occasionally persists MED
E. Left brachiocephalic vein in adults, replacing or augmenting the right SVC. BOARDS;
**Expect at least one question in embryology.** TOPNOTCH
MD)
205 This space is involved in inguinal hernia repair. The space of Bogros is the anterolateral part of the GRACE DIAGNOSTIC
A. Space of Burns potential space between the transversalis fascia ARVIOLA, EXAM - FEB
B. Space of Bogros and the parietal peritoneum. This provides a plane MD (TOP 3 - 2015
C. Space of Retzius that can be opened without entering the AUG 2014
B D. Space of Douglas membranous peritoneal sac. Thus, the risk of MED
E. Space of Nuck contamination is minimized. This sac is used for BOARDS;
placing prostheses when repairing inguinal TOPNOTCH
hernias. MD)
206 What vertebra demarcates the junction of the sigmoid colon The sigmoid colon extends from the iliac fossa to GRACE DIAGNOSTIC
and the rectum? the S3 segment, where it joins the rectum. ARVIOLA, EXAM - FEB
A. L5 MD (TOP 3 - 2015
D B. S1
C. S2
AUG 2014
MED
D. S3 BOARDS;
E. S4 TOPNOTCH
MD)

207 During childbirth, which levator ani muscle is most often The pubococcygeus, the main and most medial GRACE DIAGNOSTIC
injured? part of the levator ani, is torn most often during ARVIOLA, EXAM - FEB
A. Puborectalis childbirth. MD (TOP 3 - 2015
B. Pubococcygeus AUG 2014
B C. Iliococcygeus MED
D. Coccygeus BOARDS;
E. Sphincter urethra TOPNOTCH
MD)

208 Which structure forms the divide between vagal and pelvic Orad to the left colic flexure, the parasympathetic GRACE DIAGNOSTIC
splanchnic parasympathetic nerves? innervation of the large intestine is vagally- ARVIOLA, EXAM - FEB
A. Hepatic flexure mediated. Aborad to this, it is innervated by pelvic MD (TOP 3 - 2015
B. Splenic flexure splanchnic nerves. AUG 2014
C. Junction between sigmoid colon and rectum MED
B D. Junction between rectum and anus BOARDS;
E. Dentate line TOPNOTCH
MD)

209 The Ligament of Treitz corresponds to what vertebra? The ligament of Treitz marks the duodenojejunal GRACE DIAGNOSTIC
A. L1 junction at the level of L2 vertebra, 2-3 cm to the ARVIOLA, EXAM - FEB
B B. L2
C. L3
left of the midline. MD (TOP 3 -
AUG 2014
2015

D. L4 MED
E. L5 BOARDS;
TOPNOTCH
MD)
210 What structures form the left sagittal fissure of the liver? The gallblader and IVC form the right sagittal GRACE DIAGNOSTIC
A. Ligamentum teres and ligamentum venosun fissure. ARVIOLA, EXAM - FEB
B. Right and left hepatic lobes MD (TOP 3 - 2015
C. Caudate lobe and quadrate lobe AUG 2014
D. Galbblader and IVC MED
E. Falciform ligament and coronary ligament BOARDS;
A TOPNOTCH
MD)
211 Trigeminal neuralgia most commonly affects what division of CN V1 is least frequently involved. GRACE DIAGNOSTIC
cranial nerve V? ARVIOLA, EXAM - FEB
A. V1 MD (TOP 3 - 2015
B. V2 AUG 2014
C. V3 MED
B D. V4
E. None; it has no preferences
BOARDS;
TOPNOTCH
MD)

TOPNOTCH MEDICAL BOARD PREP ANATOMY SUPEREXAM Page 27 of 94


For inquiries visit www.topnotchboardprep.com.ph or email us at topnotchmedicalboardprep@gmail.com
TOPNOTCH MEDICAL BOARD PREP ANATOMY SUPEREXAM
For inquiries visit www.topnotchboardprep.com.ph or email us at topnotchmedicalboardprep@gmail.com
Item QUESTION EXPLANATION AUTHOR TOPNOTCH
# EXAM
212 Nodose ganglion is associated with what cranial nerve? The nodose ganglion is the inferior ganglion of the GRACE DIAGNOSTIC
A. V vagus nerve that is concerned with the visceral ARVIOLA, EXAM - FEB
B. VII sensory components of the nerve. The superior MD (TOP 3 - 2015
C. IX ganglion, on the other hand, is concerned with the AUG 2014
D D. X general sensory component. MED
E. XI BOARDS;
TOPNOTCH
MD)
213 Serous demilunes are seen in: Serous demilunes are found in glands that produce GRACE DIAGNOSTIC
A. Parotid gland both watery and mucoid secretions, i.e. ARVIOLA, EXAM - FEB
B. Sublingual gland submandibular gland. They consist of elongated MD (TOP 3 - 2015
C. Submandibular gland mucus acini that have a crescentic cap of serous AUG 2014
D. Lacrimal gland cells over their ends. MED
C E. Sebaceous gland BOARDS;
TOPNOTCH
MD)
214 Synapsis during meiosis occurs in: Synapsis is the event wherein pairs of conjoined GRACE DIAGNOSTIC
A. Prophase chromatids derived from homologous ARVIOLA, EXAM - FEB
B. Metaphase chromosomes come together on the equatorial MD (TOP 3 - 2015
C. Anaphase plate to form groups of four chromatids called AUG 2014
D. Telophase tetrads MED
A E. Interphase BOARDS;
TOPNOTCH
MD)
215 Schmidt-Lantermann clefts are formed by: These clefts are formed when Schwann cells GRACE DIAGNOSTIC
A. Cardiac myocytes envelope neurons with myelin. These represent ARVIOLA, EXAM - FEB
B. Endothelial cells thin threads of Schwann cell cytoplasm that MD (TOP 3 - 2015
C. Goblet cells pursue a spiral course from the cell body nearly to AUG 2014
D. Schwann cells the axon. MED
D E. B-cells BOARDS;
TOPNOTCH
MD)
216 The acrosome reaction of the sperm involves what receptor The zona pellucida is composed of three GRACE DIAGNOSTIC
of the zona pellucida? glycoproteins: ZP1, ZP2, and ZP3. ARVIOLA, EXAM - FEB
A. ZP1 MD (TOP 3 - 2015
B. ZP2 AUG 2014
C. ZP3 MED
D. ZP4 BOARDS;
C E. ZP5 TOPNOTCH
MD)

217 Which epithelium has only a single layer of cells? The trachea is lined with pseudostratified GRACE DIAGNOSTIC
A. Skin columnar epithelium. Pseudostratified implies a ARVIOLA, EXAM - FEB
B. Trachea single layer of cells that only appear as multiple MD (TOP 3 - 2015
C. Urinary bladder layers when viewed under the light microscope. AUG 2014
D. Cornea MED
B E. Sweat glands BOARDS;
TOPNOTCH
MD)
218 Herring bodies contain: Herring bodies are found in the posterior GRACE DIAGNOSTIC
A. Oxytocin pituitary. They contain ADH and oxytocin. ARVIOLA, EXAM - FEB
B. Prolactin MD (TOP 3 - 2015
C. Growth Hormone AUG 2014
D. Melatonin MED
A E. Aldosterone BOARDS;
TOPNOTCH
MD)
219 Which cells undergo endomitosis? In endomitosis, the DNA undergoes multiple GRACE DIAGNOSTIC
A. Neurons replications without division of the cytoplasm. ARVIOLA, EXAM - FEB
B. Ovum This results in a giant polypoid cell. MD (TOP 3 - 2015
C. RBCs AUG 2014
D. Platelets MED
E. Keratinocytes BOARDS;
D TOPNOTCH
MD)
220 High endothelial venules are found in: HEVs are the portal of entry of new blood-borne GRACE DIAGNOSTIC
A. Thymus lymphocytes. ARVIOLA, EXAM - FEB
B. Lymph nodes MD (TOP 3 - 2015
B C. Bone marrow AUG 2014
D. Spleen MED
E. All of the above BOARDS;
TOPNOTCH
MD)

TOPNOTCH MEDICAL BOARD PREP ANATOMY SUPEREXAM Page 28 of 94


For inquiries visit www.topnotchboardprep.com.ph or email us at topnotchmedicalboardprep@gmail.com
TOPNOTCH MEDICAL BOARD PREP ANATOMY SUPEREXAM
For inquiries visit www.topnotchboardprep.com.ph or email us at topnotchmedicalboardprep@gmail.com
Item QUESTION EXPLANATION AUTHOR TOPNOTCH
# EXAM
221 Which of the following manifestation is consistent if the mass lesion on the ® sphenoid wing will directly LEAN MIDTERM
patient presents with a ® sphenoid wing meningioma? compress structures at the middle cranial fossa. ANGELO EXAM 1 -
A. Inability to protrude the tongue Skull base openings located at the sphenoid are as SILVERIO, FEB 2015
B. Inability to close the ® eye and wrinkle the ipsilateral follows: optic canal, Superior orbital fissure, MD (TOP 4 -
forehead rotundum, spinsoum, ovale. Hypoglossal nerve AUG 2014
C. Weakness in closing the jaw responsibe for tongue movement is not affected MED
D. increase risk for aspiration since it exits through the hypoglossal canal found BOARDS;
E. nystagmus at the occipital bone.facial nerve as well as the TOPNOTCH
C vestibular nerve passes through the internal
acoustic meatus located at the petrous part of the
MD), MD

temporal bone. therefore, facial movement and


balance perception is not affected. pt will not have
increase risk for aspiration since the vagus and
glossopharyngeal nerve passing through the
jugular foramen is not affected.
222 which of the following structure will strictly pass through the Abducent nerve is the only cranial nerve that will LEAN MIDTERM
cavernous sinus along with the internal carotid artery? pass through the fenestrations of the cavernous ANGELO EXAM 1 -
A. Abducent nerve sinuses. CN III,IV,V1,and V2 runs forward along SILVERIO, FEB 2015
A B. Opthalmic division of trigeminal nerve the lateral wall of the sinus and not through the MD (TOP 4 -
C. Trochlear nerve sinus AUG 2014
D. Oculomotor nerve MED
E. Maxillary nerve BOARDS;
TOPNOTCH
MD), MD
223 A 32 y/o male suffered a skull base fracture secondary MVA mandibular nerve provides motor to the muscles LEAN MIDTERM
causing transection of the (L) mandibular nerve. During of mastication, general sensation to the ant 2/3 of ANGELO EXAM 1 -
recovery, which of the following will the patient unable to the tongue and the lower jaw, and innervation to SILVERIO, FEB 2015
C perform?
A. Puff the left cheek
tensor veli palatini. Among the choices, patient
will not be able to tense the ipsilateral soft palate.
MD (TOP 4 -
AUG 2014
B. Taste sensation of the left anterior 2/3rd of the tongue buccinator and taste sensation of the anterior MED
C. tense the ipsilateral palate tongue is supplied by the facial nerve. levator veli BOARDS;
D. raises the soft palate during swallowing palatini which raises the soft palate is supplied by TOPNOTCH
E. none of the above the pharyngeal plexus. MD), MD

224 what is the characteristic feature of a 3rd lumbar vertebrae? typical lumbar vertebrae has the ff characteristics: LEAN MIDTERM
A. Short slender transverse process large kidney shaped body, strong cylindical ANGELO EXAM 1 -
B. Superior articular process face laterally pedicles, thick lamina, triangular vertebral SILVERIO, FEB 2015
C C. Thick cylindrical pedicles
D. Large round vertebral foramen
foramen, long slender transverse process, short,
flat, quadrangular spinous process. Superior
MD (TOP 4 -
AUG 2014
E. Long flat quadrangular spinous process articular process faces medially, and inferior MED
articular process faces laterally. BOARDS;
TOPNOTCH
MD), MD
225 A 56 y/o male underwent a colonoscopy secondary to recent at the distance of 1.5inches of 4cm the scope is LEAN MIDTERM
onset weightloss, change in the stool caliber, and already at the rectal ampulla. At the distance of ANGELO EXAM 1 -
hematochezia. During the procedure, a hemorrhagic 6.5in or 16cm, rectosigmoid junction is reach. SILVERIO, FEB 2015
polypoid mass noted on the anterior wall of the colon 14 cm Therefore, the mass is located at the rectum. MD (TOP 4 -
from the anal verge. where is the exact location of the mass? AUG 2014
A. anal canal MED
B B. rectum BOARDS;
C. rectosigmoid TOPNOTCH
D. sigmoid MD), MD
E. cannot be determine

226 Which of the following is true about the liver the centrilobular region of the liver acinus is the LEAN MIDTERM
A. Tha main suspensory structure of the liver is the hepatic most vulnerable to ischemic insult while the ANGELO EXAM 1 -
veins draining to the inferior vena cava periportal region is the most vulnerable to toxin SILVERIO, FEB 2015
B. Periportal regions of the liver acinus is highly vulnerable induced injury. The line of Cantlie functionally MD (TOP 4 -
A to ischemic insult
C. cantlie line separates the IV segment from V and VII
divides the liver into right and left in terms of
blood supply. falciform ligament only demarcates
AUG 2014
MED
segment the liver anatomically. the caudate lobe is the 1st BOARDS;
D. the falciform ligament divides the liver into left and right liver segment ( posterior segment). TOPNOTCH
functionally in terms of blood supply MD), MD
E. the caudate lobe belongs to the left inferior segment

227 aneurysmal dilatation and thrombosis of the proximal superior mesenteric artery lies behind the neck of LEAN MIDTERM
segment of the superior mesenteric artery will produce the pancreas. It traverses downward crossing the ANGELO EXAM 1 -
which of the following symptoms? 3rd part of the duodenum anteriorly. It provides SILVERIO, FEB 2015
A. gross hematochezia blood supply from the 2nd part of the duodenum MD (TOP 4 -
B. Jaundice secondary to necrosis and inflammation of the up to proximal 2/3 of the transverse colon. AUG 2014
A head of the pancreas
C. Upper GI obstruction presenting as nonbilous emesis
Referred pain is periumbilical because it supplies
the midgut derivative. patient will present with
MED
BOARDS;
D. compression of the head of the pancreas UGI obstruction however it would be a bilous type TOPNOTCH
E. severe continuous epigastric pain. of emesis since the biliary drainage is proximal to MD), MD
the external obstruction. the head of the pancreas
will least likely to necrosis since there extensive
anastomosis b/w SMA and gastroduodenal artery.

TOPNOTCH MEDICAL BOARD PREP ANATOMY SUPEREXAM Page 29 of 94


For inquiries visit www.topnotchboardprep.com.ph or email us at topnotchmedicalboardprep@gmail.com
TOPNOTCH MEDICAL BOARD PREP ANATOMY SUPEREXAM
For inquiries visit www.topnotchboardprep.com.ph or email us at topnotchmedicalboardprep@gmail.com
Item QUESTION EXPLANATION AUTHOR TOPNOTCH
# EXAM
228 which of the following is a derivative of the 5th pharyngeal thymus gland is derived from 3rd pharyngeal LEAN MIDTERM
pouch or the ultimobranchial body? pouch, same is true with the inferior parathyroid ANGELO EXAM 1 -
A. thymus gland. The superior parathyroid is derived from SILVERIO, FEB 2015
B. Palatine tonsils 4th pharyngeal pouch. The C cells or parafollicular MD (TOP 4 -
C C. C cells cells is the only derivative of the 5th pharyngeal AUG 2014
D. Superior parathyroid gland pouch. Palatine tonsils is related to 2nd MED
E. None of the above pharyngeal pouch. BOARDS;
TOPNOTCH
MD), MD
229 which of the following is a functional remnant of the at birth, the proximal portion of the umbilical LEAN MIDTERM
umbilical arteries? arteries remain as the superior vesical artery. ANGELO EXAM 1 -
A. Median umbilical ligament While the distal part will become vestigial SILVERIO, FEB 2015
C B. Distal part of the inferior epigastric artery remnant known as the medial umbilical ligament. MD (TOP 4 -
C. Superior vesical artery AUG 2014
D. Distal part of the common iliac artery MED
E. None of the above BOARDS;
TOPNOTCH
MD), MD
230 what layer of the epidermis contains eleidin? stratum lucidum contains refractile droplets of LEAN MIDTERM
A. Stratum spinosum eleidin in the cytoplasm which is responsible for ANGELO EXAM 1 -
B. Stratum germinativum its translucent appearance during microscopy. SILVERIO, FEB 2015
C. Stratum lucidum MD (TOP 4 -
D. Stratum granulosum AUG 2014
C E. All of the above MED
BOARDS;
TOPNOTCH
MD), MD
231 which of the following is not true regarding regarding the the arterial vasa vasorum is lesser and extends LEAN MIDTERM
histological difference between an artery and a vein ? only up to tunica media, while in the veins they are ANGELO EXAM 1 -
A. The thickest coat of the artery is the tunica media while more prominent and even extends up to tunica SILVERIO, FEB 2015
that of the vein is the tunica adventitia intima. Expect hard questions in the board exam. MD (TOP 4 -
B. Vasa vasorum is more abundant in the arteries than in AUG 2014
the veins MED
B C. the tunica intima of the arteries scalloped or corrugated BOARDS;
during cross section. TOPNOTCH
D. the internal elastic lamina is more prominent in the MD), MD
arteries than in the veins
E. none of the above

232 A 37 y/o male suffered a motor vehicular accident resulting radial nerve innervates the ECRL prior to LEAN MIDTERM
to transection of the deep branch of the radial nerve branching out to superfical and deep branch. The ANGELO EXAM 1 -
secondary to comminuted fracture of the right radius. Which rest of the choices are supplied by the deep branch SILVERIO, FEB 2015
of the following movements will the patient still be able to of the radial nerve. MD (TOP 4 -
perform? AUG 2014
A A. extends and abducts the hand at the wrist joint MED
B. extend the distal phalanx of the thumb BOARDS;
C. extends and adducts hand at the wrist joint TOPNOTCH
D. extends fingers and hands MD), MD
E. none of the above

233 a 37 y/o male had suppurative inflammation of the wrist the components of the carpal tunnel are as LEAN MIDTERM
joint secondary to previous open trauma. Wrist radiograph follows: FPL, FDP, FPS, median nerve. The FCR ANGELO EXAM 1 -
shows affectation of the carpal tunnel. Which of the following tendon traverses the wrist joint through a SILVERIO, FEB 2015
structures will not be affected? separate compartment lateral to the carpal tunnel. MD (TOP 4 -
A. Flexor digitorum superficialis tendon. AUG 2014
C B. Flexor pollicis longus tendon. C. Flexor carpi radialis MED
tendon D. Flexor digitorum profundus E. none of the above. BOARDS;
TOPNOTCH
MD), MD
234 A 47 y/o female is suffering from an suppurative ischial lesser sciatic foramen provides entrance into the LEAN MIDTERM
bursitis affecting the lesser sciatic foramen. Which of the perineum from the gluteal region. The follwing ANGELO EXAM 1 -
following structures will be affected? structures passes through this foramen: obturator SILVERIO, FEB 2015
A. Inferior gluteal nerve internus tendon, pudendal nerve, internal MD (TOP 4 -
B. Sciatic nerve pudendal artery and vein, obturator internus AUG 2014
C. Posterior cutaneous nerve of the thigh nerve. MED
D. tendon of the obturator internus BOARDS;
D E. none of the above TOPNOTCH
MD), MD

235 A 68 y/o female suffering from a chronic osteoarthritis of Quadriceps muscle is composed of Vastus lateralis, LEAN MIDTERM
the ® knee underwent total knee arthroplasty. Which of the intermedius, medialis and rectus femoris. During ANGELO EXAM 1 -
following muscle needs to be strengthen since it is the last to chronic knee disease, alteration of the knee joint SILVERIO, FEB 2015
recover among the quadriceps group? mobility and Limitation of motion causes MD (TOP 4 -
C A. Vastus lateralis weakness of the quadriceps. Among the AUG 2014
B. popliteus quadriceps, it I the vastus medialis that is first to MED
C. Vastus medialis atrophy and last to recovery. BOARDS;
D. rectus femoris TOPNOTCH
E. Vastus intermedius MD), MD

TOPNOTCH MEDICAL BOARD PREP ANATOMY SUPEREXAM Page 30 of 94


For inquiries visit www.topnotchboardprep.com.ph or email us at topnotchmedicalboardprep@gmail.com
TOPNOTCH MEDICAL BOARD PREP ANATOMY SUPEREXAM
For inquiries visit www.topnotchboardprep.com.ph or email us at topnotchmedicalboardprep@gmail.com
Item QUESTION EXPLANATION AUTHOR TOPNOTCH
# EXAM
236 A 21 y/o basketball player complains of pain and instability McMurray is performed with the patient is in LEAN MIDTERM
of the (L) knee. Upon your PE. It shows that it is positive for supine position and the test leg is in full flexion. ANGELO EXAM 1 -
Apley and McMurray test. Which structure is primarily The examiner laterally rotates the knee and then SILVERIO, FEB 2015
involved? extend it. If there would be click or pain upon MD (TOP 4 -
A. ACL extension. It is highly suggestive of meniscal tear. AUG 2014
B. PCL Apley test is done in prone with the test leg in 90 MED
C. Medial meniscus knee flexion. examiner applies compresion to the BOARDS;
D. Plica syndrome knee joint while doing rotation. if there is pain it is TOPNOTCH
E. Patella suggestive of meniscal tear. MD), MD

237 A 21 y/o male suffered a monteggia fracture after being hit Monteggia fracture is a fracture of the ulna with LEAN MIDTERM
during a basketball match. Upon your PE, you noted that he concomitant dislocation of the proximal ANGELO EXAM 1 -
cannot perform OK sign and there is extension of th index radioulnar joint. It is usually prone to anterior SILVERIO, FEB 2015
DIP and thumb IP joint. Which of the following is primarily interosseous nerve injury. This nerve supplies MD (TOP 4 -
affected. ? FPL, lateral half FDP, pronator quadratus, thenar AUG 2014
A. radial nerve eminence and lateral two lumbricals.patient will MED
B. Median nerve not be able to flex the DIP of the index and IP of BOARDS;
C. ulnar nerve the thumb. there will be no sensory deficit. TOPNOTCH
D. anterior interosseous nerve MD), MD
E. none of the above

238 Compression of the nerve that passess through the canal of the posterior interosseous branch of the radial LEAN MIDTERM
Frohse will cause which of the following deficit? nerve passes through the canal of frohse formed ANGELO EXAM 1 -
A. Weakness of wrist flexors by the two heads of the supinator muscle. It is a SILVERIO, FEB 2015
B. Numbness and paresthesia along dorsal aspect of the purely motor nerve supplying the extensor MD (TOP 4 -
lateral hand. muscles of the forearm. AUG 2014
C. Wrist drop MED
D. Weakness of elbow extension BOARDS;
E. none of the above TOPNOTCH
MD), MD

239 What structure is an upward extension of the posterior SIMILAR TO PREVIOUS BOARD EXAM LEAN MIDTERM
longitudinal ligament of the spine? CONCEPT/PRINCIPLE. ANGELO EXAM 1 -
A. Tectorial membrane SILVERIO, FEB 2015
B. Apical ligament MD (TOP 4 -
C. Alar ligament AUG 2014
D. Cruciate ligament MED
E. None of the above BOARDS;
TOPNOTCH
MD), MD
240 retrolisthesis or backward extension of T8 vertebra will use this guide in determining which spinal cord is LEAN MIDTERM
affect what level of spinal cord? contiguous with a given vertebra. Cervical ANGELO EXAM 1 -
A. T10 vertebra- add 1; upper thoracic- add 2, T7-T9 - SILVERIO, FEB 2015
B. T11 add 3, tenth thoracic- L1 and L2, 11th thoracic-L3 MD (TOP 4 -
C. T12 and L4, 12th thoracic- L5 segment, 1st lumbar - AUG 2014
D. L1 sacral and coccygeal cord segment. therefore T8 MED
E. None of the above vertebra will affect T11 spinal segment. BOARDS;
TOPNOTCH
MD), MD
241 The thyroid isthmus is known to lie in which area? SIMILAR TO PREVIOUS BOARD EXAM KEVIN MIDTERM 2
A. 2nd to 4th tracheal rings CONCEPT/PRINCIPLE. Isthmus is situated BRYAN LO, EXAM - FEB
B. 3rd to 5th tracheal rings between the 2nd to 4th tracheal rings MD (TOP 7 - 2015
C. midway from the thyroid and cricoid cartilages at C4 AUG 2014
D. C2 to C4 MED
E. C5 to C6 BOARDS;
TOPNOTCH
MD)
242 A 56 year old male patient suffered a motor vehicular SIMILAR TO PREVIOUS BOARD EXAM KEVIN MIDTERM 2
accident, he suffered multiple rib fractures and had CONCEPT/PRINCIPLE, 2nd ICS Mid clavicular for BRYAN LO, EXAM - FEB
ecchymoses along the right hemithorax, chest xray revealed emergency thoracentesis for pneumothorax, use MD (TOP 7 - 2015
blunting of the right costophrenic sulci. The surgeon plans to upper border of the rib to avoid neurovascular AUG 2014
insert a chest tube to drain the fluid at which site? bundle, be sure you're in the proper hemithorax MED
A. 2nd ICS right mid clavicular line upper border of the rib BOARDS;
B. 7th ICS right posterior axillary line upper border of the rib TOPNOTCH
C. 5th ICS right anterior axillary line upper border of the rib MD)
D. 5th ICS left midaxillary line lower border of the rib
E. 7th ICS left midaxillary line lower border of the rib

243 The ovaries are lined by this specific epithelium: ___________, SIMILAR TO PREVIOUS BOARD EXAM KEVIN MIDTERM 2
A. simple cuboidal CONCEPT/PRINCIPLE it is simple cuboidal BRYAN LO, EXAM - FEB
B. simple squamous MD (TOP 7 - 2015
C. simple columnar AUG 2014
D. stratified squamous MED
E. pseudostratified columnar BOARDS;
TOPNOTCH
MD)

TOPNOTCH MEDICAL BOARD PREP ANATOMY SUPEREXAM Page 31 of 94


For inquiries visit www.topnotchboardprep.com.ph or email us at topnotchmedicalboardprep@gmail.com
TOPNOTCH MEDICAL BOARD PREP ANATOMY SUPEREXAM
For inquiries visit www.topnotchboardprep.com.ph or email us at topnotchmedicalboardprep@gmail.com
Item QUESTION EXPLANATION AUTHOR TOPNOTCH
# EXAM
244 The SA and AV node are the primary and secondary SIMILAR TO PREVIOUS BOARD EXAM KEVIN MIDTERM 2
pacemakers of the heart respectively, from which of the CONCEPT/PRINCIPLE, the right coronary artery BRYAN LO, EXAM - FEB
following vessles does their predominant blood supply come supplies both SA and AV nodes MD (TOP 7 - 2015
from? AUG 2014
A. left circumflex MED
B. left anterior descending BOARDS;
C. right coronary artery TOPNOTCH
D. left coronary artery MD)
E. both B and C

245 A patient came in with a pea sized lump non tender on the SIMILAR TO PREVIOUS BOARD EXAM KEVIN MIDTERM 2
wrist, he was assessed to have a ganglion cyst, the surgeon CONCEPT/PRINCIPLE, usually ganglion cysts BRYAN LO, EXAM - FEB
knows that he must carefully dissect it out from this arises from tendon sheaths MD (TOP 7 - 2015
underlying tissue? AUG 2014
A. extensor muscles MED
B. superficial fasia BOARDS;
C. tendon sheath TOPNOTCH
D. synovial membrane MD)
E. deep fascia

246 Which of the following is considered as a direct branch from SIMILAR TO PREVIOUS BOARD EXAM KEVIN MIDTERM 2
the aorta? CONCEPT/PRINCIPLE, direct branches from the BRYAN LO, EXAM - FEB
A. right common carotid aorta are right brachiocephalic, left common MD (TOP 7 - 2015
B. internal jugular carotid, left subclavian, AUG 2014
C. right subclavian MED
D. left subclavian BOARDS;
E. left brachiocephalic TOPNOTCH
MD)

247 56 year old male chronic smoker had a history of weight loss, SIMILAR TO PREVIOUS BOARD EXAM KEVIN MIDTERM 2
body malaise comes for consult due to multiple swellings or CONCEPT/PRINCIPLE, Nasopharyngeal primaries BRYAN LO, EXAM - FEB
neck masses. Which set of lymph nodes should be palpated are the most common subtype to involve the MD (TOP 7 - 2015
and biopsied for suspicion of metastasis of nasopharyngeal posterior cervical lymph node chain AUG 2014
carcinoma MED
A. upper jugular BOARDS;
B. lower jugular TOPNOTCH
C. posterior cervical MD)
D. submandibular
E. middle jugular

248 A 50 year old female patient comes in for a mass near the SIMILAR TO PREVIOUS BOARD EXAM KEVIN MIDTERM 2
nipple areolar complex of the right breast, which of the CONCEPT/PRINCIPLE, the most common sentinel BRYAN LO, EXAM - FEB
following lymph nodes would most likely be affected node involved is still the axillary lymph nodes MD (TOP 7 - 2015
A. superficial pectoral AUG 2014
B. deep pectoral MED
C. internal mammary BOARDS;
D. axillary TOPNOTCH
E. all of the above MD)

249 A farmer swallowed some santol seeds, which area of the GI SIMILAR TO PREVIOUS BOARD EXAM KEVIN MIDTERM 2
tract will these seeds most likely cause obstruction CONCEPT/PRINCIPLE, narrowed area due to the BRYAN LO, EXAM - FEB
A. antral pyloric area ileocecal sphincter where the seeds may lodge MD (TOP 7 - 2015
B. ileocecal junction cause obstruction AUG 2014
C. colorectal junction MED
D. splenic flexure BOARDS;
E. duodenojejunal junction TOPNOTCH
MD)

250 Which of the following bones in the body undergoes SIMILAR TO PREVIOUS BOARD EXAM KEVIN MIDTERM 2
endochondral ossification? CONCEPT/PRINCIPLE, clavicle medial end is BRYAN LO, EXAM - FEB
A. mandible endochondral lateral end is intramembranous MD (TOP 7 - 2015
B. clavicle ossification Endochondral ossification - cartilage is AUG 2014
C. maxilla initially present. MED
D. frontal bone BOARDS;
E. parietal bone TOPNOTCH
MD)

251 Which is responsible for detecting linear (vertical) SIMILAR TO PREVIOUS BOARD EXAM KEVIN MIDTERM 2
deceleration? CONCEPT/PRINCIPLE. Slightly vague since both BRYAN LO, EXAM - FEB
A. utricle utricle and saccule are said to detect linear MD (TOP 7 - 2015
B. saccule acceleration and deceleration. Utricle detects AUG 2014
C. anterior semicircular canals linear acceleration and deceleration. Saccule more MED
D. posterior semicircular canals on the vertical acceleration as in riding an BOARDS;
E. lateral semicircular canals elevator, semicircular canals deal with angular TOPNOTCH
acceleration MD)

TOPNOTCH MEDICAL BOARD PREP ANATOMY SUPEREXAM Page 32 of 94


For inquiries visit www.topnotchboardprep.com.ph or email us at topnotchmedicalboardprep@gmail.com
TOPNOTCH MEDICAL BOARD PREP ANATOMY SUPEREXAM
For inquiries visit www.topnotchboardprep.com.ph or email us at topnotchmedicalboardprep@gmail.com
Item QUESTION EXPLANATION AUTHOR TOPNOTCH
# EXAM
252 A patient presenting with adenocarcinoma of the lower 1/3 SIMILAR TO PREVIOUS BOARD EXAM KEVIN MIDTERM 2
of the esophagus would most likely have metastasis to which CONCEPT/PRINCIPLE. Celiac nodes drain the BRYAN LO, EXAM - FEB
of the following lymph nodes? lower 1/3 of the esophagus MD (TOP 7 - 2015
A. bronchomediastinal AUG 2014
B. celiac MED
C. lower jugular BOARDS;
D. diaphragmatic TOPNOTCH
E. posterior cervical MD)

253 The most common anatomical location of the esophageal SIMILAR TO PREVIOUS BOARD EXAM KEVIN MIDTERM 2
tear in boerhaave's syndrome CONCEPT/PRINCIPLE, exact answer should be left BRYAN LO, EXAM - FEB
A. Right distal esophagus posterolateral distal 1/3 of the esophagus MD (TOP 7 - 2015
B. Right middle 1/3 of the esophagus AUG 2014
C. Left distal esophagus MED
D. Left middle 1/3 of the esophagus BOARDS;
E. A and B TOPNOTCH
MD)

254 A 26 year old male was rushed to the ER following a motor SIMILAR TO PREVIOUS BOARD EXAM KEVIN MIDTERM 2
vehicular accident, patient was noted to have clear to CONCEPT/PRINCIPLE, fracture of cribriform plate BRYAN LO, EXAM - FEB
serosanguinous fluid flowing from the nose positive for of the ethmoid may lead to anosmia, and csf leak MD (TOP 7 - 2015
glucose and with (+) halo sign on gauze pad, which of the through the nose. AUG 2014
following bone has most likely been fractured? MED
A. sphenoidal sinuses BOARDS;
B. petrous portion of the temporal bone TOPNOTCH
C. cribriform plate of the ethmoid MD)
D. frontal sinus of the frontal bone
E. pterygoid plate

255 a 25 year old male athlete was playing basketball when he SIMILAR TO PREVIOUS BOARD EXAM KEVIN MIDTERM 2
experienced sudden inversion of his right foot after landing CONCEPT/PRINCIPLE, in inversion of the foot, BRYAN LO, EXAM - FEB
badly from jumping, which of the following ligaments are most commonly damaged is the anterior MD (TOP 7 - 2015
most likely damaged ? talofibular ligament in the lateral side of the foot AUG 2014
A. anterior tibiotalar ligament MED
B. deltoid ligament BOARDS;
C. talocalcaneal ligament TOPNOTCH
D. anterior talofibular ligament MD)
E. calcaneal fibular ligament

256 A 50 year old male hypertensive patient has been SIMILAR TO PREVIOUS BOARD EXAM KEVIN MIDTERM 2
experiencing chest pains, syncopal attacks and exertional CONCEPT/PRINCIPLE, questions are indeed BRYAN LO, EXAM - FEB
dyspnea. Physical exam revealed a systolic murmur, which of becoming more clinical, triad of syncope chest MD (TOP 7 - 2015
the following valvular pathologies are most likely present? pain exertional dyspnea, systolic murmur is classic AUG 2014
A. mitral regurgitation of aortic stenosis, murmur may radiate to the MED
B. mitral stenosis carotids BOARDS;
C. pulmonic stenosis TOPNOTCH
D. aortic stenosis MD)
E. aortic regurgitation

257 A prostate examination via digital rectal examination was SIMILAR TO PREVIOUS BOARD EXAM KEVIN MIDTERM 2
done for a 50 year old male patient, which of the following CONCEPT/PRINCIPLE, DRE palpates the posterior BRYAN LO, EXAM - FEB
lobes of the prostate can be directly felt by this examination? lobe of the prostate area where most carcinomas MD (TOP 7 - 2015
A. middle lobe arise AUG 2014
B. anterior lobe MED
C. posterior lobe BOARDS;
D. lateral lobes TOPNOTCH
E. C and D MD)

258 The membranous urethra is lined by which lining SIMILAR TO PREVIOUS BOARD EXAM KEVIN MIDTERM 2
epithelium? CONCEPT/PRINCIPLE, what was asked was BRYAN LO, EXAM - FEB
A. transitional prostatic urethra lining transitional epithelium, MD (TOP 7 - 2015
B. pseudostratified columnar membranous urethra is a small (1 or 2 cm) AUG 2014
C. stratified squamous portion passing through the external urethral MED
D. simple columnar sphincter. This is the narrowest part of the urethra BOARDS;
E. both B and C lined by pseudostratified columnar epithelium, TOPNOTCH
spongy or penile urethra last part proximally is MD)
pseudostratified columnar and distally is stratified
squamous

TOPNOTCH MEDICAL BOARD PREP ANATOMY SUPEREXAM Page 33 of 94


For inquiries visit www.topnotchboardprep.com.ph or email us at topnotchmedicalboardprep@gmail.com
TOPNOTCH MEDICAL BOARD PREP ANATOMY SUPEREXAM
For inquiries visit www.topnotchboardprep.com.ph or email us at topnotchmedicalboardprep@gmail.com
Item QUESTION EXPLANATION AUTHOR TOPNOTCH
# EXAM
259 A 30 year old male patient had a motorcycle accident, SIMILAR TO PREVIOUS BOARD EXAM KEVIN MIDTERM 2
physical examination revealed absence of sensation at the CONCEPT/PRINCIPLE, know the dermatome and BRYAN LO, EXAM - FEB
lateral and posterior aspects of bilateral lower legs, this myotome levels MD (TOP 7 - 2015
corresponds to what dermatome level? AUG 2014
A. L2-L3 MED
B. L4-L5 BOARDS;
C. L5-S1 TOPNOTCH
D. S1-S2 MD)
E. both A and B

260 On reading the CT scan of a patient revealed a mass at the SIMILAR TO PREVIOUS BOARD EXAM KEVIN MIDTERM 2
level near the tracheal bifurcation, in determining the level of CONCEPT/PRINCIPLE, choose the best answer, BRYAN LO, EXAM - FEB
the lesion, the doctor is sure that at least part of the mass is near the tracheal bifurcation is at the level of T4- MD (TOP 7 - 2015
at level of T5 AUG 2014
A. T2-T3 MED
B. T3-T4 BOARDS;
C. T4-T5 TOPNOTCH
D. T5-T6 MD)
E. T6-T7

261 All of the following structures pass through the aortic hiatus RAYMUND MIDTERM 3
EXCEPT: MARTIN LI, EXAM - FEB
A. Thoracic duct MD (TOP 1 - 2015
B. Aorta AUG 2014
C. Azygos Vein MED
D. All of the above BOARDS;
E. None of the above TOPNOTCH
MD)

262 A left-sided dominant blood supply of the heart is best Right sided dominant - comes from the RCA; left- RAYMUND MIDTERM 3
characterized by: sided dominant - comes from the LMCA MARTIN LI, EXAM - FEB
A. The posterior 1/3 of the interventricular septum is MD (TOP 1 - 2015
supplied by a branch from the left coronary artery AUG 2014
B. The anterior interventricular artery is a branch of the right MED
coronary artery BOARDS;
C. The posterior interventricular artery comes from the left TOPNOTCH
main coronary artery MD)
D. A and C
E. A and B

263 A 7 year old child is brought to you for consult by his mother. A case of nursemaid's elbow cause by subluxation RAYMUND MIDTERM 3
Upon examination, the child is noted to keep his left forearm of radial head MARTIN LI, EXAM - FEB
pronated and partially flexed close to the body. Which is the MD (TOP 1 - 2015
most likely cause? AUG 2014
A. Supracondylar humeral fracture MED
B. Subluxation of the radial head BOARDS;
C. Surgical humeral neck fracture TOPNOTCH
D. Colles fracture MD)
E. None of the above

264 A patient comes to your clinic with complaints of hoarseness Recurrent laryngeal nerve may be injured during RAYMUND MIDTERM 3
of voice. History revealed that the patient recently ligation of the inferior thyroid artery or section of MARTIN LI, EXAM - FEB
underwent thyroid surgery. Injury most probably occurred the ligament of Berry MD (TOP 1 - 2015
during: AUG 2014
A. Ligation of the inferior thyroid artery MED
B. Ligation of the superior thyroid artery BOARDS;
C. Section of the ligament of Berry TOPNOTCH
D. A or C MD)
E. B or C

265 Which of the following statements regarding abdominal Duodenal papilla in the 2nd part of duodenum. RAYMUND MIDTERM 3
structural relationships is correct? Portal vein is formed behind the pancreas MARTIN LI, EXAM - FEB
A. The portal vein is formed posterior to the 3rd portion of MD (TOP 1 - 2015
the duodenum AUG 2014
B. The superior mesenteric artery passes anterior to the MED
horizontal portion of the duodenum BOARDS;
C. The major duodenal papilla is located in the first part of TOPNOTCH
the duodenum MD)
D. All are correct
E. None are correct

TOPNOTCH MEDICAL BOARD PREP ANATOMY SUPEREXAM Page 34 of 94


For inquiries visit www.topnotchboardprep.com.ph or email us at topnotchmedicalboardprep@gmail.com
TOPNOTCH MEDICAL BOARD PREP ANATOMY SUPEREXAM
For inquiries visit www.topnotchboardprep.com.ph or email us at topnotchmedicalboardprep@gmail.com
Item QUESTION EXPLANATION AUTHOR TOPNOTCH
# EXAM
266 Which of the following abdominal structures is NOT derived Speen is derived from mesoderm RAYMUND MIDTERM 3
from the embryologic endoderm? MARTIN LI, EXAM - FEB
A. Stomach MD (TOP 1 - 2015
B. Liver AUG 2014
C. Spleen MED
D. Pancreas BOARDS;
E. All are derived from the endoderm TOPNOTCH
MD)

267 Which of the following structures forms the majority of the Diaphragmatic surface formed by left ventricle. RAYMUND MIDTERM 3
diaphragmatic surface of the heart? Diaphragmatic border by right ventricle MARTIN LI, EXAM - FEB
A. Rigth ventricle MD (TOP 1 - 2015
B. Left ventricle AUG 2014
C. Right Atrium MED
D. Pulmonary vessels BOARDS;
E. Left atrium TOPNOTCH
MD)

268 A 62 year old woman presents to your clinic complaining of Long thoracic nerve injury usually occurs during RAYMUND MIDTERM 3
right arm weakness. Further examination reveals inability to MRM MARTIN LI, EXAM - FEB
abduct the right arm past horizontal position and protrusion MD (TOP 1 - 2015
of the right scapula in an abnormal position noted when AUG 2014
instructed to push against a wall. Which will most likely be a MED
significant finding in the patient’s history? BOARDS;
A. Right Rotator cuff injury TOPNOTCH
B. Right midshaft humeral fracture MD)
C. Poorly fitted crutches
D. Right modified radical mastectomy
E. History of humeral neck fracture

269 Which of the following structures is NOT transmitted CN III,IV,VI,V1 are transmitted through superior RAYMUND MIDTERM 3
through the superior orbital fissure? orbital fissure MARTIN LI, EXAM - FEB
A. Oculomotor nerve MD (TOP 1 - 2015
B. Mandibular branch of the mandibular nerve AUG 2014
C. Abducens nerve MED
D. Ophthalmic branch of the mandibular nerve BOARDS;
E. None of the above TOPNOTCH
MD)

270 A duodenal ulcer located in the lesser curvature of the Bleeding from left gastric artery due to ulcer in RAYMUND MIDTERM 3
stomach is most likely to cause which of the following lesser curvature. MARTIN LI, EXAM - FEB
complications: MD (TOP 1 - 2015
A. Bleeding from the gastroduodenal artery AUG 2014
B. Bleeding from a branch of the celiac artery MED
C. Bleeding from the left gastroepiploic artery BOARDS;
D. Pancreatitis secondary to perforation TOPNOTCH
E. None of the above MD)

271 A 57 year old man presents at the ER with complaints of Review dermatomal distribution RAYMUND MIDTERM 3
lower back pain. Physical examination findings reveals MARTIN LI, EXAM - FEB
weakened dorsiflexion of the foot and decreased sensation of MD (TOP 1 - 2015
the lateral surface of the leg and dorsum of the foot. Which of AUG 2014
the following nerve root is most likely compressed? MED
A. L2 BOARDS;
B. L5 TOPNOTCH
C. L4 MD)
D. S1
E. S2

272 A 5 year old male patient is noted to have a soft nontender Due to patent processus vaginalis RAYMUND MIDTERM 3
fullness of the left scrotum with associated positive MARTIN LI, EXAM - FEB
transillumination. The underlying predisposing factor for MD (TOP 1 - 2015
this condition is similar to that of: AUG 2014
A. Indirect inguinal hernia MED
B. Direct Inguinal hernia BOARDS;
C. Femoral hernia TOPNOTCH
D. Varicocele MD)
E. All of the above

273 A tracheostomy tube is best inserted in which of the Between 2nd and 3rd tracheal rings is preferred. RAYMUND MIDTERM 3
following locations? Above that there is danger of tracheal stenosis. MARTIN LI, EXAM - FEB
A. Between 1st and 2nd tracheal rings Below that there is danger of hitting isthmus and MD (TOP 1 - 2015
B. Between 2nd and 3rd tracheal rings vessels AUG 2014
C. Between 3rd and 4th tracheal rings MED
D. Between 4th and 5th tracheal rings BOARDS;
E. Between the cricoid and the 1st tracheal ring TOPNOTCH
MD)

TOPNOTCH MEDICAL BOARD PREP ANATOMY SUPEREXAM Page 35 of 94


For inquiries visit www.topnotchboardprep.com.ph or email us at topnotchmedicalboardprep@gmail.com
TOPNOTCH MEDICAL BOARD PREP ANATOMY SUPEREXAM
For inquiries visit www.topnotchboardprep.com.ph or email us at topnotchmedicalboardprep@gmail.com
Item QUESTION EXPLANATION AUTHOR TOPNOTCH
# EXAM
274 Which of the following structures is derived from the Portal vein derived from vitelline vein. Smooth RAYMUND MIDTERM 3
embryologic common cardinal veins? part of right atrium derived from sinus venosus. MARTIN LI, EXAM - FEB
A. superior vena cava MD (TOP 1 - 2015
B. pulmonary veins AUG 2014
C. right atrium MED
D. portal vein BOARDS;
E. None of the above TOPNOTCH
MD)

275 A sports player is noted to have an abnormal posterior Anterior drawer sign - displacement of tibia RAYMUND MIDTERM 3
location of the femur in relation to the tibia upon attempt at anterior to femur or femur located posterior in MARTIN LI, EXAM - FEB
passive displacement. Which structure is injured? relation to the tibia MD (TOP 1 - 2015
A. Anterior cruciate ligament AUG 2014
B. Posterior cruciate ligament MED
C. Medial collateral ligament BOARDS;
D. Lateral collateral ligament TOPNOTCH
E. Medial lemniscus MD)

276 When instructed to perform a pudendal nerve block, which RAYMUND MIDTERM 3
of the following landmarks is most important? MARTIN LI, EXAM - FEB
A. Tip of the ischial spine MD (TOP 1 - 2015
B. Ischeal tuberosity AUG 2014
C. Lesser sciatic foramen MED
D. Greater sciatic foramen BOARDS;
E. Perineal body TOPNOTCH
MD)

277 Your resident asked you to insert a chest tube thoracostomy RAYMUND MIDTERM 3
on a patient presenting with pleural effusion. Being a stellar MARTIN LI, EXAM - FEB
intern, you insert the chest tube on the: MD (TOP 1 - 2015
A. 5th ICS MCL upper border of the rib AUG 2014
B. 5th ICS MCL lower border of the rib MED
C. 8th ICS AAL upper border of the rib BOARDS;
D. 5th ICS AAL upper border of the rib TOPNOTCH
E. 5th ICS PAL lower border of the rib MD)

278 Failure of closure of the urethral folds will cause: Hypospadias - failure of urethral fold closure. RAYMUND MIDTERM 3
A. Epispadias Epispadias - extrophy MARTIN LI, EXAM - FEB
B. Phimosis MD (TOP 1 - 2015
C. Extrophy of the bladder AUG 2014
D. Hypospadias MED
E. None of the above BOARDS;
TOPNOTCH
MD)
279 A 43 year old man was brought to the hospital due to a Middle meningeal artery which is a branch of the RAYMUND MIDTERM 3
vehicular accident. Patient was noted to have loss of maxillary artery usual source of epidural MARTIN LI, EXAM - FEB
consciousness at the site of the accident with a lucid interval hematoma MD (TOP 1 - 2015
immediately after. Currently in the ER, the patient is being AUG 2014
evaluated for deterioration of sensorium. Careful history MED
reveals that the patient hit the part of his head near the BOARDS;
anatomic region called the pterion. CT scan revealed a TOPNOTCH
lentiform-shaped bleed. Which structure is most likely MD)
injured??
A. MCA
B. Bridging veins
C. Branch of the maxillary artery
D. Superior sagittal sinus
E. Branch of the internal carotid artery

280 A 23 y.o. male patient complaining of left testicular mass Testicular drained by paraaortic; prostate drained RAYMUND MIDTERM 3
underwent biopsy and was diagnosed to have testicular by internal iliac nodes; scrotum drained by MARTIN LI, EXAM - FEB
cancer. Which group of lymph nodes will primarily be inguinal nodes MD (TOP 1 - 2015
affected once the cancer spreads? AUG 2014
A. deep inguinal lymph nodes MED
B. paraaortic lymph nodes BOARDS;
C. superficial inguinal lymph nodes TOPNOTCH
D. internal iliac lymph nodes MD)
E. external iliac lymph nodes

281 Pain at the area of the anatomic "snuff box" following a fall The floor of the anatomic snuff box is formed by ERIC ROYD FINAL EXAM
on an outstretched hand may be caused by a fracture of the scaphoid and trapezium. Lateral bundary: TALAVERA, - FEB 2015
which of the following? tendons of EPB and AbdPL. Medial Boundary: MD (TOP 1 -
A. Scaphoid tendon of EPL AUG 2014
B. Capitate MED
C. Hamate BOARDS;
D. Pisiform TOPNOTCH
E. None of the above MD)

TOPNOTCH MEDICAL BOARD PREP ANATOMY SUPEREXAM Page 36 of 94


For inquiries visit www.topnotchboardprep.com.ph or email us at topnotchmedicalboardprep@gmail.com
TOPNOTCH MEDICAL BOARD PREP ANATOMY SUPEREXAM
For inquiries visit www.topnotchboardprep.com.ph or email us at topnotchmedicalboardprep@gmail.com
Item QUESTION EXPLANATION AUTHOR TOPNOTCH
# EXAM
282 Where is the fundus of the gallbladder located? The fundus of the gallbadder is located at the level ERIC ROYD FINAL EXAM
A. At the level of the 10th costal cartilage along the linea of the 9th costal cartilage along the linea TALAVERA, - FEB 2015
semilunaris semilunaris MD (TOP 1 -
B. At the transpyloric plane along the linea alba AUG 2014
C. At the level of the umbilicus MED
D. At the level of the 9th costal cartilage along the linea BOARDS;
semilunaris TOPNOTCH
E. None of the above MD)

283 An individual with tertiary syphilis would have damage Individuals with tertiary syphilis can present with ERIC ROYD FINAL EXAM
involving what structure in the spinal cord? tabes dorsalis which involves damage to the dorsal TALAVERA, - FEB 2015
A. Lateral Corticospinal Tract columns. Indiviuals present with loss of vibration, MD (TOP 1 -
B. Dorsal Column position sense and tactile discrimination. AUG 2014
C. Ventral Spinocerebellar Tract MED
D. Anterior Spinothalamic Tract BOARDS;
E. Lateral Spiothalamic Tract TOPNOTCH
MD)

284 Which of the following structures does not communicate The posterior ethmoidal sinus drain into the ERIC ROYD FINAL EXAM
with the middle meatus? superior meatus. The rest of the choices drain into TALAVERA, - FEB 2015
A. Maxillary Sinus the middle meatus. The nasolacrimal duct drains MD (TOP 1 -
B. Posterior Ethmoidal Sinus into the inferior meatus AUG 2014
C. Frontal Sinus MED
D. Anterior Ethmoidal Sinus BOARDS;
E. None of the above TOPNOTCH
MD)

285 Umbrella or Dome cells are part of the lining epithelium of Umbrella cells are multinucleated superficial cell ERIC ROYD FINAL EXAM
which organ ? of the bladder’s transitional epithelium, which has TALAVERA, - FEB 2015
A. Stomach vacuolated cytoplasm. MD (TOP 1 -
B. Small intestines AUG 2014
C. Uterus MED
D. Liver BOARDS;
E. Bladder TOPNOTCH
MD)

286 Which of the following statements is true regarding the the abdominal aorta enters the abdominal cavity ERIC ROYD FINAL EXAM
abdominal aorta? by piercing the aortic hiatus at the level of TV 12. TALAVERA, - FEB 2015
A. Enters the abdominal cavity at the level of TV 10 It is located to the left of the IVC. It has three MD (TOP 1 -
B. It is located to the right of the inferior vena cava unpaired visceral branches (celiac, superior AUG 2014
C. It terminates at the level of LV 4 mesenteric, inferior mesenteric). It terminates at MED
D. Has four (4) unparied visceral branches the level of LV4 into the common iliac and median BOARDS;
E. Terminal branches are the common iliac and inferior sacral artery TOPNOTCH
phrenic artery MD)

287 A 78 year old male presents with hematuria, urgency, The middle lobe is commonly involved in BPH ERIC ROYD FINAL EXAM
frequency and low back pain. Thorough evaulation resulting in obstruction of the prostatic urethra. TALAVERA, - FEB 2015
eventually lead to a diagnosis of prostate cancer. Which of While the posterior lobe (part of the peripheral MD (TOP 1 -
the following lobes of the prostate is most commonly zone) is commonly involved in carcinomatous AUG 2014
involved? transformation MED
A. Anterior BOARDS;
B. Middle TOPNOTCH
C. Posterior MD)
D. Right Lateral
E. Left Lateral

288 What muscle serves as guide in dividing the axillary artery The tendon of the pectoralis minor divides the ERIC ROYD FINAL EXAM
into three different parts? axillary artery into 3 major parts TALAVERA, - FEB 2015
A. Pectoralis Major MD (TOP 1 -
B. Pectoralis Minor AUG 2014
C. Serratus Anterior MED
D. Teres Major BOARDS;
E. Trapezius TOPNOTCH
MD)

289 Which of the following statements regarding the pancreas is The pancreas is a retroperitoneal organ. Blood ERIC ROYD FINAL EXAM
correct? supply is derived from the celiax and superior TALAVERA, - FEB 2015
A. It is an intraperitoneal organ mesenteric artery. It is located primarily in the MD (TOP 1 -
B. It derives its blood supply from the inferior mesenteric epigastric region. The portal vein is formed behind AUG 2014
artery the neck of the pancreas MED
C. It primarily located in the hypogastic and left BOARDS;
hypochondriac region TOPNOTCH
D. The portal vein is formed behind the tail of the pancreas MD)
E. The major pancreatic duct joins the common bile duct
which drains into the second part of the duodenum

TOPNOTCH MEDICAL BOARD PREP ANATOMY SUPEREXAM Page 37 of 94


For inquiries visit www.topnotchboardprep.com.ph or email us at topnotchmedicalboardprep@gmail.com
TOPNOTCH MEDICAL BOARD PREP ANATOMY SUPEREXAM
For inquiries visit www.topnotchboardprep.com.ph or email us at topnotchmedicalboardprep@gmail.com
Item QUESTION EXPLANATION AUTHOR TOPNOTCH
# EXAM
290 Which of the following structures is most likely to be The esophagus is related posterior to the left ERIC ROYD FINAL EXAM
compressed by an enlargement of the left atrium? atrium, enlargement of the left atrium can cause TALAVERA, - FEB 2015
A. Esophagus compression of the esophagus producing MD (TOP 1 -
B. Thymus symptoms such as dysphagia AUG 2014
C. Thyroid gland MED
D. Larynx BOARDS;
E. Superior vena cava TOPNOTCH
MD)

291 What is the only cranial nerve that exits from the dorsal The trochlear nerve is the only cranial nerve of the ERIC ROYD FINAL EXAM
surface of the brainstem? brainstem that exits from the dorsal surface TALAVERA, - FEB 2015
A. Abducens (CN VI) (particularly from the dorsal surface of the MD (TOP 1 -
B. Trigeminal (CN V) midbrain) AUG 2014
C. Hypoglossal (CN XII) MED
D. Facial (CN VII) BOARDS;
E. Trochlear (CN IV) TOPNOTCH
MD)

292 Brunner's glands are located in the submucosa of what Brunner's glands (or duodenal glands) are ERIC ROYD FINAL EXAM
segment of the gastrointestinal tract? compound tubular submucosal glands found in TALAVERA, - FEB 2015
A. Appendix that portion of the duodenum which is above the MD (TOP 1 -
B. Ileum hepatopancreatic sphincter (Sphincter of Oddi). AUG 2014
C. Duodenum The main function of these glands is to produce a MED
D. Stomach mucus-rich alkaline secretion (containing BOARDS;
E. Jejunum bicarbonate) TOPNOTCH
MD)

293 The fibers of the corticospinal tract passes through what Path of the corticopsinal tract: Motor Cortex ERIC ROYD FINAL EXAM
structure at the level of the midbrain? (Cerebrum) --> corona radiata --> posterior limb TALAVERA, - FEB 2015
A. Cerebral aqueduct of internal capsule --> cerebral peduncle MD (TOP 1 -
B. Superior Colliculus (midbrain)--> basis pontis (pons) --> pyramids AUG 2014
C. Red Nucleus (medulla) --> Decussate at level of lower medulla - MED
D. Medial Longitudinal Fasciculus -> lateral corticopinal tract (spinal cord) BOARDS;
E. Cerebral peduncles TOPNOTCH
MD)

294 A 65 year old chronic smoker presented with right arm pain This is a case of a Pancoast tumor. The said ERIC ROYD FINAL EXAM
and drooping of the right upper eyelid. Diagnostic work up symptoms arise from compression of the adjacent TALAVERA, - FEB 2015
revealed a mass over the apex of the right lung. Which of the structure, particularly the sympathetic chain MD (TOP 1 -
following muscles is affected to account for the finding of ganglia (Horner's Syndrome). The superior tarsal AUG 2014
ptosis over the right eye? muscle or the MULLER's muscle is innervated by MED
A. Lateral rectus sympathetic nerves. Compression of the said BOARDS;
B. Superior Oblique structures by the mass would produce weakness TOPNOTCH
C. Levator palpebrae superioris of the said muscle thus accounting for the ptosis. MD)
D. Superior tarsal LPS is innervated by CN III and is not involved in
E. Medial rectus Horner's syndrome

295 Which of the following structures is not located in the The heart and the pericardium are located in the ERIC ROYD FINAL EXAM
posterior mediastinum? middle mediastinum TALAVERA, - FEB 2015
A. Vagus nerve MD (TOP 1 -
B. Esophagus AUG 2014
C. Pericardium MED
D. Thoracic duct BOARDS;
E. Thoracic aorta TOPNOTCH
MD)

296 A 76 year old male was brought in the ER due to chest pain. A SIMILAR TO PREVIOUS BOARD EXAM ERIC ROYD FINAL EXAM
12L ECG was done which showed ST-segment elevation CONCEPT/PRINCIPLE: V1, V2: septal wall, I, avL: TALAVERA, - FEB 2015
involving leads II, III and avF. Involvement of the said leads high lateral wall, V3-V4: anterior wall, V5-V6: MD (TOP 1 -
signify infarction of which area of the heart? lateral wall, V3r-V4r: RV wall AUG 2014
A. Inferior wall MED
B. Septal wall BOARDS;
C. RV wall TOPNOTCH
D. Lateral Wall MD)
E. Anterior Wall

297 Which of the following statements concerning the larynx is All intrinsic muscles of the larynx are innercated ERIC ROYD FINAL EXAM
INCORRECT? by the recurrent laryngeal nerve except for the TALAVERA, - FEB 2015
A. Sensory innervation is provided by the recurrent cricothyroid which is innervated by the external MD (TOP 1 -
laryngeal and internal laryngeal nerve branch of the superior laryngeal nerve AUG 2014
B. Extends from the tip of the epiglottis to the lower border MED
of the cricoid cartilage BOARDS;
C. The cricothyroid muscle is innervated by the recurrent TOPNOTCH
laryngeal nerve MD)
D. The posterior cricoarytenoid muscle serves to abduct the
vocal folds
E. It can be divided into supraglottic, glottic and infraglottic
areas

TOPNOTCH MEDICAL BOARD PREP ANATOMY SUPEREXAM Page 38 of 94


For inquiries visit www.topnotchboardprep.com.ph or email us at topnotchmedicalboardprep@gmail.com
TOPNOTCH MEDICAL BOARD PREP ANATOMY SUPEREXAM
For inquiries visit www.topnotchboardprep.com.ph or email us at topnotchmedicalboardprep@gmail.com
Item QUESTION EXPLANATION AUTHOR TOPNOTCH
# EXAM
298 Lead V1 is placed in what location/position in the chest? SIMILAR TO PREVIOUS BOARD EXAM ERIC ROYD FINAL EXAM
A. 5th ICS at the left midclavicular line CONCEPT/PRINCIPLE: V1 4th ICS right sternal TALAVERA, - FEB 2015
B. 5th ICS at the left anterior axillary line border, V2 4th ICS left sternal border, V3 halfway MD (TOP 1 -
C. 4th ICS at the left sternal border between V2 and V4, V4 5th ICS left MCL, V5 5th AUG 2014
D. 4th ICS at the right sternal border ICS at left AAL, V6 5th ICS at left MAL, V3R halfway MED
E. 5th ICS at the right midclavicular line between V1 and V4r, V4r 5th ICS at right MCL BOARDS;
TOPNOTCH
MD)
299 Which of the following structures pass anterior to the medial Structures passing anterior to the medial ERIC ROYD FINAL EXAM
malleolus? malleolus: 1. Saphenous nerve 2. Great saphenous TALAVERA, - FEB 2015
A. Tibial nerve vein; Structures passing posterior to the medial MD (TOP 1 -
B. Saphenous nerve malleolus: 1. Tendon of the FDL, 2. Tendon of the AUG 2014
C. Tendon of the flexor digitorum longus FHL, 3. Tibialis posterior, 4. Tibial nerve, 5. MED
D. Tendon of the peroneus longus Posterior tibial artery BOARDS;
E. All of the above TOPNOTCH
MD)

300 Hassall's corpuscles are histologic features of which of the Hassall's corpuscles (or thymic corpuscles ERIC ROYD FINAL EXAM
following lymphoid organs? (bodies)) are structures found in the medulla of TALAVERA, - FEB 2015
A. Spleen the human thymus, formed from eosinophilic type MD (TOP 1 -
B. Lymph Nodes VI epithelial reticular cells arranged concentrically AUG 2014
C. Tonsils MED
D. Thymus BOARDS;
E. All of the above TOPNOTCH
MD)

301 A 57 y/o hypertensive, diabetic male was rushed to ER ipsilateral paralysis of the oculomotor nerve LEAN BACK-UP
secondary to deterioriation of consciousness. Pt was accompanied by contralateral ataxia and ANGELO MIDTERM
successfully stabilized at the ER. Upon secondary PE, Pt is intentional tremors is due to a vascular lesion at SILVERIO, EXAM - FEB
confused and oriented to place and person. his BP the central midbrain area known as Claude MD (TOP 4 - 2015
170/100mmHg, PR 76 bpm, RR 14 cpm. You noted that his syndrome. It affects the oculomotor nucleus, red AUG 2014
(L) eye is fixated in an inferolateral direction with dilated nucleus and the superior cerebellothalamic fibers. MED
nonresponsive pupils. while his (R) arm showed tremors and Weber syndrome is characterized by ipsilateral BOARDS;
incoordination when asked to reach for an object. no oculomotor paralysis and contralateral TOPNOTCH
paralysis was noted on both UE/LE. CT scan was requested hemiplegia. Combination of Weber and Claude MD), MD
however results were pending. what is the most probable syndrome constitute the Benedikt syndrome.
diagnosis? Foville syndrome is the vascular paramedian pons
A. Weber syndrome lesion represented by ipsilateral abducent nerve
B. Benedikt syndrome palsy and contralateral hemiparesis. Wallenberg
C. Claude syndrome syndrome is a lateral medullary syndrome
D. Foville syndrome secondary to occlusion of posterior inferior
E. Wallenberg syndrome cerebellar artery. it manifest with contralateral
hemianesthesia, ipsilateral loss of pain and
thermal sensation in the face, nystagmus, vertigo,
loss of taste from the ipsilateral side of th tonque
and hoarseness with dysphagia. Haines 3rd ed pp
175-190
302 Which of the following neck muscles is not supplied by the cervical plexus provides motor innervation to LEAN BACK-UP
ansa cervicalis ? most of the neck muscles. Ansa cervicalis is the ANGELO MIDTERM
A. geniohyoid union of C1-C3 fibers give innervation to SILVERIO, EXAM - FEB
B. Omohyoid sternohyoid, sternothyroid, and omohyoid. C1 MD (TOP 4 - 2015
C. sternohyoid fibers via hypoglossal nerve supplies the AUG 2014
D. Sternothyroid thyrohyoid and geniohyoid. other cervical plexus MED
E. None of the above branches include the phrenic nerve (C3-C5) BOARDS;
innervating the diaphragm and segmental nerves TOPNOTCH
supplying the prevertebral muscles including the MD), MD
levator scapulae. Snell 9th ed p 619.
303 A 76 y/o male went to your clinic secondary to productive this patient diagnosed with lung cancer on the left LEAN BACK-UP
cough, weight loss and voice changes. He was smoker with a upper lobe accompanied by mediastinal lymph ANGELO MIDTERM
50 pack years and an alcoholic beverage drinker. Xray was nodes can cause compression of the left recurrent SILVERIO, EXAM - FEB
done showing a suspicious mass at the left upper lobe of the laryngeal nerve as it hoops around the aortic arch MD (TOP 4 - 2015
lung. a follow up CT scan showed a probable lung carcinoma and left bronchial area. this will result to paralysis AUG 2014
with mediastinal Lymphadenopathies compressing the left of all the intrinsic laryngeal muscles except for MED
bronchial airway. which of the following laryngeal muscles cricothyroid. the latter is supplied by external BOARDS;
will be spared by this condition? laryngeal nerve. snell 9th ed p 648 TOPNOTCH
A. thyroarytenoid MD), MD
B. cricothyroid
C. posterior cricoarytenoid
D. oblique arytenoid
E. none of the above

TOPNOTCH MEDICAL BOARD PREP ANATOMY SUPEREXAM Page 39 of 94


For inquiries visit www.topnotchboardprep.com.ph or email us at topnotchmedicalboardprep@gmail.com
TOPNOTCH MEDICAL BOARD PREP ANATOMY SUPEREXAM
For inquiries visit www.topnotchboardprep.com.ph or email us at topnotchmedicalboardprep@gmail.com
Item QUESTION EXPLANATION AUTHOR TOPNOTCH
# EXAM
304 A 28 y/o male with a history of poorly controlled asthma and middle meatus is located between inferior and LEAN BACK-UP
Allergic Rhinitis presented to your clinic with a complain of middle nasal concha. It drains the frontal, anterior ANGELO MIDTERM
decrease sense of smell and taste. Upon doing ENT and middle ethmoid, maxillary sinuses. While SILVERIO, EXAM - FEB
examination, you noted a pale grayish polypoid mass located sphenoid sinus drains into the sphenoethmoidal MD (TOP 4 - 2015
between inferior and middle nasal concha. which of the recess superior to superior nasal concha. Posterior AUG 2014
following set of sinuses will most likely obstructed? ethmoidal sinus drains to the superior meatus MED
A. frontal, posterior ethmoid, maxillary sinuses above the middle concha. nasolacrimal duct drains BOARDS;
B. frontal, middle ethmoid, sphenoid, maxillary sinuses to the inferior meatus. Snell 9th ed pp 643-644 TOPNOTCH
C. frontal anterior, posterior and middle ethmoid, maxillary MD), MD
sinuses
D. frontal, anterior ethmoid, maxillary sinuses
E. frontal, ant and middle ethmoid, sphenoid, maxillary
sinuses

305 A 76 y/o female complaining of chronic vague abdominal aortic aneurysm with such length will most likely LEAN BACK-UP
pain was found to have aortic aneurysm extending from T11 affect structures that passes the aortic opening of ANGELO MIDTERM
-to L3 vertebra. What are the other structures will most the diaphragm ( T12 vertebrae). This include SILVERIO, EXAM - FEB
likely be affected by this condition? thoracic duct and azygous veins. Esophageal MD (TOP 4 - 2015
A. thoracic duct opening lies more anterior and it is at the level of AUG 2014
B. vagus nerve T10. it transmits vagus nerve, left gastric vessels MED
C. inferior vena cava and lymphatics. Caval opening lies at the 8th BOARDS;
D. left gastric artery thoracic vertebrae transmits IVC and right phrenic TOPNOTCH
E. righr phrenic nerve nerve. Snell 9th ed p 46. MD), MD

306 A 27 y/o male was sent to ER following an MVA. Pt was the patient is suffering from left tension LEAN BACK-UP
hypotensive, tachycardic and tachypneic along with neck pneumothorax. A needle thoracostomy is needed ANGELO MIDTERM
veins distention and (L) tracheal deviation. There were no inorder to alleviate the pressure in the left pleural SILVERIO, EXAM - FEB
breath sounds noted on the left. What landmark will you use area. by convention, it is best to insert a needle at MD (TOP 4 - 2015
inorder to perform needle thoracostomy? the level of the 2nd ICS either at midclavicular or AUG 2014
A. (L) 2nd ICS 0.5 cm from sternal edge anterior axillary line. Snell 9th ed p 45. MED
B. (L) 2nd ICS midclavicular line BOARDS;
C. (R) 3rd ICS anterior axillary line TOPNOTCH
D. (L) 5th ICS midaxillary line MD), MD
E. none of the above

307 the ductus arteriosus is a derivative of what aortic arch? Aortic arches: 1st - maxillary artery, 2nd- hyoid LEAN BACK-UP
A. (L) 3rd aortic arch and stapedial arteries, 3rd - common carotid ANGELO MIDTERM
B. ® 4th aortic arch artery and the 1st part of internal carotid arteries, SILVERIO, EXAM - FEB
C. (L) 5th aortic arch 4th - (L) arch of aorta between LCCA and LScA, ® - MD (TOP 4 - 2015
D. (L) 6th aortic arch proximal segment of subclavian artery. 5th- AUG 2014
E. ® 2nd aortic arch obliterated, no derivatives. 6th- (L) ductus MED
arteriosus, (R) proximal segment of (R) pulmonary BOARDS;
artery. Langmans embryology 9th ed 255-258. TOPNOTCH
MD), MD
308 Which of the following is not a tributary of the coronary most blood from the heart wall drains into the LEAN BACK-UP
sinus ? right atrium through the coronary sinus, its ANGELO MIDTERM
A. Great cardiac vein tributaries are the great, small, and middle cardiac SILVERIO, EXAM - FEB
B. Middle cardiac vein veins. The anterior cardiac vein drains blood MD (TOP 4 - 2015
C. Anterior cardiac vein directly to the right atrium bypassing the coronary AUG 2014
D. Small cardiac vein sinus. snell 9th ed p 89 MED
E. Both B and C. BOARDS;
TOPNOTCH
MD), MD
309 Which of the following is not a true statement regarding the the deep inguinal ring, a defect in the fascia LEAN BACK-UP
inguinal canal? transversalis, is located midway between ASIS and ANGELO MIDTERM
A. The superior wall is made up by the arching fibers of the pubic tubercle. Snell 9th ed p 127. SILVERIO, EXAM - FEB
Internal oblique and transversus abdominis MD (TOP 4 - 2015
B. The medial side of the posterior wall is reinforced by the AUG 2014
conjoint tendon MED
C. the deep inguinal ring is located at 1.3 cm medial to the BOARDS;
ASIS TOPNOTCH
D. the superficial inguinal ring gives attachment ot the MD), MD
external spermatic fascia.
E. none of the above

310 A 65 y/o male presenting weightloss, anemia and chronic the approximate length of each structures are as LEAN BACK-UP
diarrhea underwent a colonoscopy, a polypoid mass was follows. Anal canal- 1.5 in, rectum -5 in, sigmoid, ANGELO MIDTERM
located at the anterior wall of the colon 40 inches from the 10-15 inch, transverse 15 inch descending colon SILVERIO, EXAM - FEB
anal verge. What part of the colon where the mass is located? 10 in. therefore at 40 inches from the anal verge, MD (TOP 4 - 2015
A. sigmoid the mass is most likely located at the transverse AUG 2014
B. descending colon colon. MED
C. transverse colon BOARDS;
D. hepatic flexure TOPNOTCH
E. ascending colon MD), MD

TOPNOTCH MEDICAL BOARD PREP ANATOMY SUPEREXAM Page 40 of 94


For inquiries visit www.topnotchboardprep.com.ph or email us at topnotchmedicalboardprep@gmail.com
TOPNOTCH MEDICAL BOARD PREP ANATOMY SUPEREXAM
For inquiries visit www.topnotchboardprep.com.ph or email us at topnotchmedicalboardprep@gmail.com
Item QUESTION EXPLANATION AUTHOR TOPNOTCH
# EXAM
311 what is the most dilatable portion of the male urethra? the prostatic urethra is the widest and the most LEAN BACK-UP
A. Prostatic urethra dilatable portion of the entire male urethra. Snell ANGELO MIDTERM
B. Membranous urethra 9th ed p 278 SILVERIO, EXAM - FEB
C. Penile urethra MD (TOP 4 - 2015
D. Navicular fossa of the urethra AUG 2014
E. None of the above MED
BOARDS;
TOPNOTCH
MD), MD
312 the physiological umbilical herniation during embryonic ( SIMILAR TO PREVIOUS BOARD EXAM LEAN BACK-UP
development occur in what age of gestation? CONCEPT/PRINCIPLE). Development of the ANGELO MIDTERM
A. 5th week primary intestinal loop is characterized by rapid SILVERIO, EXAM - FEB
B. 6th week elongation particularly of the cephalic limb. As a MD (TOP 4 - 2015
C. 7th week result of the rapid growth and expansion of the AUG 2014
D. 8th week liver, the abdominal cavity temporarily becomes MED
E. 10th week too small to contain all the intestinal lipps and BOARDS;
they enter the extraembryonic cavity in the TOPNOTCH
umblical cord during 6th week of development. MD), MD
Langmans 9th ed p 307.
313 Which of the following is a branch of the anterior division of upon descent of the internal iliac artery at the LEAN BACK-UP
the internal iliac artery? level of the greater sciatic foramen, it divides into ANGELO MIDTERM
A. Lateral sacral artery anterior and posterior division. The branches of SILVERIO, EXAM - FEB
B. Superior gluteal artery the anterior division are the umbilical artery, MD (TOP 4 - 2015
C. Inferior gluteal artery obturator artery, inferior vesical artery, middle AUG 2014
D. Iliolumbar artery rectal arterym internal pudendal aftery, inferior MED
E. None of the above gluteal artery, uterine and vaginal arteries. while BOARDS;
the posterior division has only three branches: TOPNOTCH
iliolumbar, lateral sacral and superior gluteal MD), MD
artery. Snell 9th ed Regions p 256-257
314 A 27 y/o female was brought to ER secondary to saddle type the contents of the superficial perineal pouch are LEAN BACK-UP
injury due to MVA, pelvic CT scan was requested showing the following: root of the clitoris, bulbospongiosus, ANGELO MIDTERM
hematoma on the superficial perineal pouch. Which of the ischiocavernosus, superficial transverse perineal SILVERIO, EXAM - FEB
following structures will most likely be spared in this muscle, perioneal body and the perineal branch of MD (TOP 4 - 2015
condition? the pudendal nerve. The contents of the deep AUG 2014
A. sphincter urethrae perineal pouch includes the followin: sphincter MED
B. ischiocavernosus muscle urethrae, deep transverse perineal muscle, BOARDS;
C. bulbospongiosus muscle internal pudendal vessels, and dorsal nerve of the TOPNOTCH
D. perineal branch of the pudendal nerve clitoris. Snell 9th ed regions pp 322- 323 MD), MD
E. perineal body

315 A 34 y/o female was brought to ER secondary to a diffuse based on the history, the patient is most likely to LEAN BACK-UP
abdominal pain. she was hypotensive, tachycardic at have a perforated duodenal ulcer. The most ANGELO MIDTERM
presentation. PE showed diffuse rigidity of the abdomen with common site for duodenal ulcer is its 1st part SILVERIO, EXAM - FEB
both rebound and direct tenderness prominent on the which is intraperitoneal structure. CT scan finding MD (TOP 4 - 2015
epigastric area. CT scan was done showing blood collection of blood in the lesser sac indicates that the AUG 2014
in the lesser sac. she was a smoker and alcoholic beverage perforation is most likely at the posterior wall. MED
drinker. she always skip her meals because she want to loose gastroduodenal artery is immediately behind the BOARDS;
weight and that she always take mefenamic acid almost 1st part of the duodenum. affectation of this TOPNOTCH
everyday for her migraine.prior to onset, she claims to have structure can lead to bleeding in the lesser sac, MD), MD
recurrent history of vague epigastric pain whenever she skip and sometimes pancreatitis. Snell 9th ed p 176
meal and that there were occasions of nocturnal awakening
secondary to epigastric pain. what specific vascular structure
will most likely be affected in this setting?
A. superior pancreaticoduodenal artery
B. portal vein
C. gastroduodenal artery
D. (L) gastroepiploic artery
E. all of the above

316 which of the following is true about the schneiderian the schneiderian membrane refers to the LEAN BACK-UP
membrane of the nasal cavity? respiratory portion of the nasal cavity. It is ANGELO MIDTERM
A. Abundant venous plexus in the tunica propria characterized by pseudostratified columnar SILVERIO, EXAM - FEB
B. Tall pseudostratified columnar epithelium without epithelium with goblet cells lining the mucous MD (TOP 4 - 2015
goblet cells membrane. It has a relatively thick basal lamina; AUG 2014
C. No distinct basal lamina contains abundant venous plexuses, and serous/ MED
D. contains bundles of olfactor y nerves in the lamina mucous tubuloacinar glands in the tunica propria. BOARDS;
propria it also has characteristic lymphocytic infiltration in TOPNOTCH
E. all of the above the epithelial layer. other choices refers to the MD), MD
olfactory neuroepithelium. cabral histology p 178

TOPNOTCH MEDICAL BOARD PREP ANATOMY SUPEREXAM Page 41 of 94


For inquiries visit www.topnotchboardprep.com.ph or email us at topnotchmedicalboardprep@gmail.com
TOPNOTCH MEDICAL BOARD PREP ANATOMY SUPEREXAM
For inquiries visit www.topnotchboardprep.com.ph or email us at topnotchmedicalboardprep@gmail.com
Item QUESTION EXPLANATION AUTHOR TOPNOTCH
# EXAM
317 complete transection of the lateral cord of brachial plexus the lateral cord give rise to lateral pectoral nerve ( LEAN BACK-UP
will result to which of the following symptoms? pectoralis major), musculocutaneous nerve ( ANGELO MIDTERM
A. Decrease sensation in the lateral dorsal side of the the coracobrachialis, biceps brachii, brachialis, SILVERIO, EXAM - FEB
hand. sensory to the lateral border of the foreeam) and MD (TOP 4 - 2015
B. Decrease sensation in the lateral side of the forearm lateral root of the median nerve. A- is supplied by AUG 2014
C. inability to supinate the forearm the radial nerve which is a branch of the posterior MED
D. inability to flex the forearm cord. C and D- pt will only have weakness but not BOARDS;
E. all of the above. inability since the supinator muscle is supplied by TOPNOTCH
the radial nerve eventhough the biceps brachii is MD), MD
denervated. on the otherhand, there is still some
flexion of the forearm because of the reverse
action of other forearm muscle as well as the
partial innervation of brachialis by the radial
nerve. ( Snell 9th ed Regions p 354)
318 A 72 y/o male was brought to ER secondary to FOOSH injury the deep branch of radial nerve is a pure motor LEAN BACK-UP
( fall on a outstretched hand). X ray was done showing nerve supplying all the muscles of the posterior ANGELO MIDTERM
displaced compression fracture of the radial head. Physical compartment of the forearm except for the ECRL SILVERIO, EXAM - FEB
examination of the arm reveals findings that are consistent and supinator muscle. There would be no wrist MD (TOP 4 - 2015
with an injury to the deep branch of radial nerve. which of drop since the ECRL is powerful enough to AUG 2014
the following statement is correct. ? promote wrist extension. Snell 9th ed p 431. MED
A. MMT 3/5 in forearm supination BOARDS;
B. decrease sensation on the dorsal lateral side of the hand TOPNOTCH
C. presence of wristdrop MD), MD
D. all of the above
E. none of the above

319 Which of the following structures is not supplied by the the lateral plantar nerve supplies all of the LEAN BACK-UP
lateral plantar nerve? intrinsic muscles of the foot except for abductor ANGELO MIDTERM
A. Adductor hallucis hallucis, flexor digitorum brevis, flexor hallucis SILVERIO, EXAM - FEB
B. Flexor digitorum brevis brevis, and the 1st lumbrical muscle ( all of which MD (TOP 4 - 2015
C. Quadratus plantae are supplied by the medial plantar nerve) Snell p AUG 2014
D. Flexor digiti minimi 676. MED
E. None of the above BOARDS;
TOPNOTCH
MD), MD
320 Retrolisthesis of the 8th thoracic vertebra will affect what relationship of spinal cord segements to vertebral LEAN BACK-UP
corresponding spinal segment? numbers: cervical vertebrae: add 1, upper thoracic ANGELO MIDTERM
A. T8 vertebra (T1-T6): add 2, lower thoracic (T7-T9): SILVERIO, EXAM - FEB
B. T9 add 3, tenth thoracic: L1 and L2 cord segment, MD (TOP 4 - 2015
C. T10 11th thoracic: L3-L4 cord segment, 12 th thoracic: AUG 2014
D. T11 L5 cord segment; 1st lumbar: sacral and coccygeal MED
E. T12 segments. Snell 9th ed p 704 BOARDS;
TOPNOTCH
MD), MD
321 Epithelial lining of the ovaries: SIMILAR TO PREVIOUS BOARD EXAM DEBBIE BACK-UP
A. simple squamous CONCEPT/PRINCIPLE. ROSE MIDTERM
B. stratified squamous TANENGSY, EXAM - FEB
C. simple cuboidal MD (TOP 5 - 2015
D. simple columnar AUG 2014
E. pseudostratified columnar MED
BOARDS;
TOPNOTCH
MD)
322 Tongue papillae with no taste buds and thus serve a Fungiform - mushroom shaped; circumvallate - DEBBIE BACK-UP
mechanical function only: largest, located near the sulcus terminalis; foliate - ROSE MIDTERM
A. fungiform rudimentary in humans; filliform - most abundant TANENGSY, EXAM - FEB
B. circumvallate papillae MD (TOP 5 - 2015
C. foliate AUG 2014
D. filliform MED
BOARDS;
TOPNOTCH
MD)
323 Epithelial lining of lymphangiomas: SIMILAR TO PREVIOUS BOARD EXAM DEBBIE BACK-UP
A. simple squamous CONCEPT/PRINCIPLE. ROSE MIDTERM
B. stratified squamous TANENGSY, EXAM - FEB
C. simple cuboidal MD (TOP 5 - 2015
D. simple columnar AUG 2014
E. psuedostratified columnar MED
BOARDS;
TOPNOTCH
MD)
324 True of the jejunum but not of the ileum, except: DEBBIE BACK-UP
A. makes up the initial 3/5 of the small intestine ROSE MIDTERM
B. has longer vasa recta TANENGSY, EXAM - FEB
C. has 1-2 arterial arcades MD (TOP 5 - 2015
D. has a thicker wall AUG 2014
E. with prominent Peyer's patches MED
BOARDS;
TOPNOTCH
MD)

TOPNOTCH MEDICAL BOARD PREP ANATOMY SUPEREXAM Page 42 of 94


For inquiries visit www.topnotchboardprep.com.ph or email us at topnotchmedicalboardprep@gmail.com
TOPNOTCH MEDICAL BOARD PREP ANATOMY SUPEREXAM
For inquiries visit www.topnotchboardprep.com.ph or email us at topnotchmedicalboardprep@gmail.com
Item QUESTION EXPLANATION AUTHOR TOPNOTCH
# EXAM
325 The lower anal canal, as opposed to the upper anal canal: DEBBIE BACK-UP
A. is supplied by a branch of the inferior mesenteric artery ROSE MIDTERM
B. is drained by the deep inguinal nodes TANENGSY, EXAM - FEB
C. is derived from the ectoderm MD (TOP 5 - 2015
D. is lined by simple columnar epithelium AUG 2014
MED
BOARDS;
TOPNOTCH
MD)
326 The moderator band is located in which chamber of the The moderator band is located in the right DEBBIE BACK-UP
heart? ventricle. ROSE MIDTERM
A. right atrium TANENGSY, EXAM - FEB
B. left atrium MD (TOP 5 - 2015
C. right ventricle AUG 2014
D. left ventricle MED
BOARDS;
TOPNOTCH
MD)
327 True of the right atrium: The right atrium is larger than the left, has thicker DEBBIE BACK-UP
A. smaller than the left atrium walls, and has a sinus venarum, which is a smooth ROSE MIDTERM
B. has thicker walls portion developed from the embryonic sinus TANENGSY, EXAM - FEB
C. has a moderator band venosus. The moderator band is located in the MD (TOP 5 - 2015
D. with a sinus venarum, a coarse muscular portion of the right ventricle. AUG 2014
chamber MED
BOARDS;
TOPNOTCH
MD)
328 GIST is located in what layer of the gastrointestinal tract? DEBBIE BACK-UP
A. mucosa ROSE MIDTERM
B. submucosa TANENGSY, EXAM - FEB
C. muscularis MD (TOP 5 - 2015
D. adventitia AUG 2014
E. serosa MED
BOARDS;
TOPNOTCH
MD)
329 The accessory nerve crosses this triangle of the neck: DEBBIE BACK-UP
A. submental ROSE MIDTERM
B. submandibular TANENGSY, EXAM - FEB
C. carotid MD (TOP 5 - 2015
D. muscular AUG 2014
E. occipital MED
BOARDS;
TOPNOTCH
MD)
330 the lower ureter is supplied by the: Upper ureter - renal artery; middle ureter - DEBBIE BACK-UP
A. renal artery gonadal artery; lower ureter - superior vesical ROSE MIDTERM
B. superior vesical artery artery TANENGSY, EXAM - FEB
C. gonadal artery MD (TOP 5 - 2015
D. middle rectal AUG 2014
E. inferior vesical MED
BOARDS;
TOPNOTCH
MD)
331 Found in the superficial perineal space, except: The membranous urethra is found within the deep DEBBIE BACK-UP
A. bulbospongiosus perineal space. ROSE MIDTERM
B. ischiocavernosus TANENGSY, EXAM - FEB
C. spongy urethra MD (TOP 5 - 2015
D. perineal body AUG 2014
E. membranous urethra MED
BOARDS;
TOPNOTCH
MD)
332 A patient with an upper motor neuron lesion is expected to A person with an upper motor neuron lesion DEBBIE BACK-UP
exhibit: presents with spastic paralysis, increased muscle ROSE MIDTERM
A. flaccid paralysis tone, clonus, (+) Babinski. There is no atrophy or TANENGSY, EXAM - FEB
B. atrophy fasciculation in UMNL. MD (TOP 5 - 2015
C. clonus AUG 2014
D. fasciculations MED
E. reduced muscle tone BOARDS;
TOPNOTCH
MD)
333 Leaf shaped laryngeal cartilage: thyroid - largest laryneal cartilage; cricoid - signet DEBBIE BACK-UP
A. thyroid ring shaped; arytenoid - pyramidal; corniculate - ROSE MIDTERM
B. cricoid conical; cuneiform - club shaped TANENGSY, EXAM - FEB
C. epiglottis MD (TOP 5 - 2015
D. arytenoid AUG 2014
E. corniculate MED
BOARDS;
TOPNOTCH
MD)

TOPNOTCH MEDICAL BOARD PREP ANATOMY SUPEREXAM Page 43 of 94


For inquiries visit www.topnotchboardprep.com.ph or email us at topnotchmedicalboardprep@gmail.com
TOPNOTCH MEDICAL BOARD PREP ANATOMY SUPEREXAM
For inquiries visit www.topnotchboardprep.com.ph or email us at topnotchmedicalboardprep@gmail.com
Item QUESTION EXPLANATION AUTHOR TOPNOTCH
# EXAM
334 Example of a typical rib: Ribs 12 and 11 have a single facet of the heads, are DEBBIE BACK-UP
A. rib 12 short, with no neck or tubercle. Rib 10 articulates ROSE MIDTERM
B. rib 11 with the T10 vertebra only. Other atypical ribs: rib TANENGSY, EXAM - FEB
C. rib 10 1 - broadest, most curved, prominent scalene MD (TOP 5 - 2015
D. rib 9 tubercle; rib 2 - thinner, less curved, has AUG 2014
tuberosity for serratus anterior. MED
BOARDS;
TOPNOTCH
MD)
335 The triangle of Calot: The triangle of Calot contains the cystic artery, is DEBBIE BACK-UP
A. contains the cystic duct bounded by the following: superiorly by the liver, ROSE MIDTERM
B. is bounded by the liver laterally inferiorly by the cystic duct, and medially by the TANENGSY, EXAM - FEB
C. is bounded by the cystic artery inferiorly common hepatic duct. MD (TOP 5 - 2015
D. is bounded by the common hepatic duct medially AUG 2014
MED
BOARDS;
TOPNOTCH
MD)
336 The female pelvis, compared to the male pelvis, has a/n: Female pelvis: borader sacrum, shorter symphysis DEBBIE BACK-UP
A. narrower sacrum pubis, everted ischial tuberosity, shallower iliac ROSE MIDTERM
B. longer symphysis pubis fossa. TANENGSY, EXAM - FEB
C. oval shaped superior aperture MD (TOP 5 - 2015
D. inverted ischial tuberosity AUG 2014
E. deeper iliac fossa MED
BOARDS;
TOPNOTCH
MD)
337 Which part of the adrenals contain pale staining cells in Zona glomerulosa - basophilic arranged in cords of DEBBIE BACK-UP
cords with foamy appearances due to lipid droplet columnar or pyramidal cells. Zona reticularis - ROSE MIDTERM
accumulations? eosinophilic cells with large nucleus arranged in TANENGSY, EXAM - FEB
A. zona glomerulosa cords smaller and less spongy than the cells in MD (TOP 5 - 2015
B. zona fasciculata zona fasciculata. AUG 2014
C. zona reticularis MED
D. adrenal medulla BOARDS;
TOPNOTCH
MD)
338 Fibrocartilage is found in: DEBBIE BACK-UP
A. larynx ROSE MIDTERM
B. epiglottis TANENGSY, EXAM - FEB
C. auditory tube MD (TOP 5 - 2015
D. costal cartilage AUG 2014
E. intervertebral discs MED
BOARDS;
TOPNOTCH
MD)
339 Layer absent in thin skin: DEBBIE BACK-UP
A. corneum ROSE MIDTERM
B. lucidum TANENGSY, EXAM - FEB
C. granulosum MD (TOP 5 - 2015
D. spinosum AUG 2014
E. basale MED
BOARDS;
TOPNOTCH
MD)
340 Thickest layer in veins: DEBBIE BACK-UP
A. tunia intima ROSE MIDTERM
B. tunica media TANENGSY, EXAM - FEB
C. tunica adventitia MD (TOP 5 - 2015
D. none of the above AUG 2014
MED
BOARDS;
TOPNOTCH
MD)
341 The carotid sinus is found in the: The carotid sinus is a localized dilation at the JESSICA MAE BACK-UP
A. Anterior surface of the bifurcation of the carotid artery beginning of the internal carotid artery. SANCHEZ, MIDTERM
B. Posterior surface of the bifurcation of the common carotid The carotid body is a small structure that lies MD (TOP 4 - EXAM - FEB
artery posterior to the bifurcation of the common carotid AUG 2014 2015
C. Beginning of the internal carotid artery artery. MED
D. Beginning of the external carotid artery BOARDS;
Reference: Snell. Clinical Anatomy by Regions, 8th TOPNOTCH
ed. p. 747 MD)
342 The muscles that form the superior boundary of the carotid The boundaries of the carotid triangle are as JESSICA MAE BACK-UP
triangle: follows: SANCHEZ, MIDTERM
A. Posterior belly of digastric -Superiorly by the posterior belly of digastric MD (TOP 4 - EXAM - FEB
B. Superior belly of omohyoid -Posteriorly by the SCM AUG 2014 2015
C. Sternocleidomastoid -Anteriorly by the superior belly of the omohyoid. MED
D. Trapezius BOARDS;
TOPNOTCH
MD)
343 This nerve accompanies the superior thyroid artery: The superior thyroid artery, a branch of the ECA, is JESSICA MAE BACK-UP
A. Superior laryngeal accompanied by the external laryngeal nerve SANCHEZ, MIDTERM
B. External laryngeal which supplies the cricothyroid muscle. MD (TOP 4 - EXAM - FEB
C. Internal laryngeal AUG 2014 2015
D. Recurrent laryngeal Reference: Snell. Clinical Anatomy by Regions, 8th MED
ed. P. 749 BOARDS;
TOPNOTCH
TOPNOTCH MEDICAL BOARD PREP ANATOMY SUPEREXAM Page 44 of 94
For inquiries visit www.topnotchboardprep.com.ph or email us at topnotchmedicalboardprep@gmail.com
TOPNOTCH MEDICAL BOARD PREP ANATOMY SUPEREXAM
For inquiries visit www.topnotchboardprep.com.ph or email us at topnotchmedicalboardprep@gmail.com
Item QUESTION EXPLANATION AUTHOR TOPNOTCH
# EXAM
MD)

344 True of meiosis, except: Meiosis reduces the chromosomes to the haploid JESSICA MAE BACK-UP
A. Creates new set of chromosomes by random distribution number. SANCHEZ, MIDTERM
to daughter cells MD (TOP 4 - EXAM - FEB
B. Bestow each germ cell with the diploid number of AUG 2014 2015
chromosomes MED
C. Provides gamete variabilities BOARDS;
D. Produce daughter cells genetically different from the TOPNOTCH
parent and from each other MD)
345 Which of the following cartilages is characterized by the Reference: Topnotch Histology Handouts JESSICA MAE BACK-UP
absence of a perichondrium? SANCHEZ, MIDTERM
A. Adult hyaline cartilage MD (TOP 4 - EXAM - FEB
B. Fetal hyaline cartilage AUG 2014 2015
C. Fibrocartilage MED
D. Articular cartilage BOARDS;
TOPNOTCH
MD)
346 This structure divides the subclavian artery into 3 parts: Reference: Snell. Clinical Anatomy by Regions, 8th JESSICA MAE BACK-UP
A. First rib ed. P.751 SANCHEZ, MIDTERM
B. Pectoralis minor MD (TOP 4 - EXAM - FEB
C. Scalene anterior AUG 2014 2015
D. Teres major MED
BOARDS;
TOPNOTCH
MD)
347 The deep cervical lymph nodes lie along the: The deep cervical nodes form a vertical chain JESSICA MAE BACK-UP
A. Sternocleidomastoid along the course of the internal jugular vein within SANCHEZ, MIDTERM
B. Common carotid artery the carotid sheath. MD (TOP 4 - EXAM - FEB
C. External jugular vein The superficial cervical nodes lie along the course AUG 2014 2015
D. Internal jugular vein of the external jugular vein on the side of the neck. MED
BOARDS;
Reference: Snell. Clinical Anatomy by Regions, 8th TOPNOTCH
ed. p. 755-756 MD)
348 Tracheostomy tube insertion is usually done at the level of The preferred site for tracheostomy is through the JESSICA MAE BACK-UP
the: second to third ring, with the thyroid isthmus SANCHEZ, MIDTERM
A. Cricothyroid membrane retracted inferiorly. MD (TOP 4 - EXAM - FEB
B. 1st-2nd tracheal rings AUG 2014 2015
C. 2nd-3rd tracheal rings Reference: Snell. Clinical Anatomy by Regions, 8th MED
D. 4th-5th tracheal rings ed. P. 814 BOARDS;
TOPNOTCH
MD)
349 After sustaining a fracture of the mid-humerus following a Reference: Topnotch Anatomy Handouts JESSICA MAE BACK-UP
fall, the patient was unable to extend the wrist. This is SANCHEZ, MIDTERM
consistent with injury to the: MD (TOP 4 - EXAM - FEB
A. Axillary nerve AUG 2014 2015
B. Median nerve MED
C. Radial nerve BOARDS;
D. Ulnar nerve TOPNOTCH
MD)
350 Which of the following is not a characteristic of epithelium in Epithelium may be derived from any of the 3 germ JESSICA MAE BACK-UP
general? layers. SANCHEZ, MIDTERM
A. Avascular MD (TOP 4 - EXAM - FEB
B. Derived from the ectoderm only AUG 2014 2015
C. Capable of metaplasia MED
D. Rests on a basal lamina BOARDS;
TOPNOTCH
MD)
351 What vein accompanies the anterior interventricular artery? Great cardiac vein - anterior interventricular JESSICA MAE BACK-UP
A. Anterior cardiac artery SANCHEZ, MIDTERM
B. Great cardiac Middle cardiac vein - posterior interventricular MD (TOP 4 - EXAM - FEB
C. Middle cardiac artery AUG 2014 2015
D. Small cardiac Small cardiac vein - marginal branch of the RCA MED
BOARDS;
TOPNOTCH
MD)
352 The 6th aortic arch will become the: Aortic arches: JESSICA MAE BACK-UP
A. Common carotid artery I - Maxillary artery SANCHEZ, MIDTERM
B. Maxillary artery II - Hyoid and stapedial arteries MD (TOP 4 - EXAM - FEB
C. Pulmonary artery III - Common carotid, first part of the ICA, ECA AUG 2014 2015
D. Right subclavian artery IV - Left: arch of the aorta; Right: proximal MED
segment of the right subclavian artery BOARDS;
V - regresses TOPNOTCH
VI - proximal part of the pulmonary artery, and the MD)
ductus arteriosus on the left

Reference: Langman's Medical Embryology, 9th
ed. p. 255-258

TOPNOTCH MEDICAL BOARD PREP ANATOMY SUPEREXAM Page 45 of 94


For inquiries visit www.topnotchboardprep.com.ph or email us at topnotchmedicalboardprep@gmail.com
TOPNOTCH MEDICAL BOARD PREP ANATOMY SUPEREXAM
For inquiries visit www.topnotchboardprep.com.ph or email us at topnotchmedicalboardprep@gmail.com
Item QUESTION EXPLANATION AUTHOR TOPNOTCH
# EXAM
353 Melena results from gastrointestinal bleeding found: JESSICA MAE BACK-UP
A. Proximal to the ligament of Treitz SANCHEZ, MIDTERM
B. Distal to the ligament of Treitz MD (TOP 4 - EXAM - FEB
C. At the stomach only AUG 2014 2015
D. At the esophagogastric area only MED
BOARDS;
TOPNOTCH
MD)
354 A 40 year old man sustained a gunshot wound that traversed Reference: Topnotch Anatomy Handouts JESSICA MAE BACK-UP
segments 1 and 4 of the liver. This means that the parts of SANCHEZ, MIDTERM
the liver affected were: MD (TOP 4 - EXAM - FEB
A. Quadrate lobe AUG 2014 2015
B. Caudate lobe MED
C. Both BOARDS;
D. Neither TOPNOTCH
MD)
355 The extent to which neoplastic cells resemble comparable JESSICA MAE BACK-UP
normal cells is referred to as: SANCHEZ, MIDTERM
A. Differentiation MD (TOP 4 - EXAM - FEB
B. Pleomorphism AUG 2014 2015
C. Anaplasia MED
D. Dysplasia BOARDS;
TOPNOTCH
MD)
356 A patient complains of a boil located on her labia majora. Reference: Topnotch Anatomy Handouts JESSICA MAE BACK-UP
Lymphatic spread of the infection would most likely enlarge SANCHEZ, MIDTERM
which nodes? MD (TOP 4 - EXAM - FEB
A. Lumbar nodes AUG 2014 2015
B. Superficial inguinal nodes MED
C. External iliac nodes BOARDS;
D. Internal iliac nodes TOPNOTCH
MD)
357 A 63 year old female patient says that she has pain in her A femoral hernia is more common in women that JESSICA MAE BACK-UP
groin and upper thigh. Upon examination, you palpate a in men. SANCHEZ, MIDTERM
lump located below the inguinal ligament, lateral to its The neck of the sac always lies below and lateral to MD (TOP 4 - EXAM - FEB
attachment to the pubic tubercle. You suspect that this may the pubic tubercle, distinguishing it from an AUG 2014 2015
be a hernia passing through the: inguinal hernia, which lies above and medial to the MED
A. femoral canal pubic tubercle. BOARDS;
B. obturator canal TOPNOTCH
C. adductor hiatus Reference: Snell. Clinical Anatomy by Regions, 8th MD)
D. superficial inguinal ring ed. p. 580
358 The sensory receptor for hearing is located in the: JESSICA MAE BACK-UP
A. Macula SANCHEZ, MIDTERM
B. Ampulla MD (TOP 4 - EXAM - FEB
C. Organ of Corti AUG 2014 2015
D. Saccule MED
BOARDS;
TOPNOTCH
MD)
359 A forearm arterio-venous fistula for hemodialysis access is Radiocephalic AV fistula is the vascular access of JESSICA MAE BACK-UP
usually done by anastomosing the: choice for hemodialysis. SANCHEZ, MIDTERM
A. Radial artery and basilic vein MD (TOP 4 - EXAM - FEB
B. Radial artery and cephalic vein AUG 2014 2015
C. Radial artery and antecubital vein MED
D. Ulnar artery and basilic vein BOARDS;
TOPNOTCH
MD)
360 The aortic valve is best heard clinically at the: JESSICA MAE BACK-UP
A. Right 2nd ICS close to the sternal border SANCHEZ, MIDTERM
B. Left 2nd ICS close to the sternal border MD (TOP 4 - EXAM - FEB
C. Left 5th ICS midclavicular line AUG 2014 2015
D. Xiphisternal junction MED
BOARDS;
TOPNOTCH
MD)
361 Gab is an avid tennis player with a killer serve. In order to do palmaris brevis (Fig. 9.55) is a small muscle that MAIRRE BACK-UP
this, he must have a good grip around the racket. Which arises from the flexor retinaculum and palmar JAMES MIDTERM
muscle improves the grip of the palm especially when aponeurosis and is inserted into the skin of the GADDI, MD EXAM - FEB
holding rounded objects? palm. It is supplied by the superficial branch of the (TOP 4 - AUG 2015
A. Abductor pollicis brevis ulnar nerve. Its function is to corrugate the skin at 2013 MED
B. Flexor pollicis brevis the base of the hypothenar eminence and so BOARDS;
C. Palmaris brevis improve the grip of the palm in holding a rounded TOPNOTCH
D. Extensor pollicis brevis object. Snell 9th pg 397 MD)

APB – Abduction of the thumb
FPB - Flexes MCP joint of the thumb
EPB – Extends MCP joint of the thumb

TOPNOTCH MEDICAL BOARD PREP ANATOMY SUPEREXAM Page 46 of 94


For inquiries visit www.topnotchboardprep.com.ph or email us at topnotchmedicalboardprep@gmail.com
TOPNOTCH MEDICAL BOARD PREP ANATOMY SUPEREXAM
For inquiries visit www.topnotchboardprep.com.ph or email us at topnotchmedicalboardprep@gmail.com
Item QUESTION EXPLANATION AUTHOR TOPNOTCH
# EXAM
362 Marco Reus of Borussia Dortmund recently damaged his Lateral ligament sprains occur in running and MAIRRE BACK-UP
ankle ligaments following a tackle from the outside while on jumping sports. The lateral ligament is injured JAMES MIDTERM
international duty with Germany. Which ligament was most because it is much weaker than the medial liga- GADDI, MD EXAM - FEB
likely involved? ment, and is the ligament that resists inversion at (TOP 4 - AUG 2015
A. Anterior talofibular ligament the talocrural joint. The anterior talofibular 2013 MED
B. Posterior talofibular ligament ligament is most vulnerable and most commonly BOARDS;
C. Calcaneofibular ligament torn during ankle sprains, either partially or TOPNOTCH
D. Anterior tibiotalar ligament completely, resulting in instability of the ankle MD)
joint.
Moore 7th pg 666
363 In order to achieve anesthesia of an intercostal space, an In the chest wall, the neurovascular bundles are MAIRRE BACK-UP
intercostal nerve block can be done by inserting the needle sheltered by the inferior margins of the overlying JAMES MIDTERM
through: ribs. The needle is directed toward the rib near the GADDI, MD EXAM - FEB
A. the superior margin of the rib above the space lower border and the tip comes to rest near the (TOP 4 - AUG 2015
B. the inferior margin of the rib above the space subcostal groove, where the anesthetic is 2013 MED
C. the superior margin of the rib below the space infiltrated around the nerve. Remember that the BOARDS;
D. the inferior margin of the rib below the space order of structures from above downward is TOPNOTCH
E. Between the inferior margin of the rib above the space and intercostal vein, artery, and nerve (VAN) Snell 9th MD)
the superior margin of the rib below the space pg 43

364 A patient with a large fungating tongue mass is schedule to The distance between the incisors to the vocal MAIRRE BACK-UP
undergo operative intervention. The anesthesiologist plans cords measures 5.9in (15cm), between the JAMES MIDTERM
to do nasotracheal intubation. He knows that the distance incisors to the carina 7.9in (20cm), between the GADDI, MD EXAM - FEB
from the external nares to the carina measures: external nares to the carina 11.8in (30cm). Snell (TOP 4 - AUG 2015
A. 15 cm 9th pg 720 2013 MED
B. 20 cm BOARDS;
C. 30 cm TOPNOTCH
D. 40 cm MD)

365 During tracheostomy it is important to note that the thyroid The isthmus is usually anterior to the second and MAIRRE BACK-UP
isthmus is at the level of the: third tracheal rings. Moore 7th pg1018 JAMES MIDTERM
A. Cricoid and 1st tracheal ring GADDI, MD EXAM - FEB
B. 1st and 2nd tracheal rings (TOP 4 - AUG 2015
C. 2nd and 3rd tracheal rings 2013 MED
D. Cricoid and 2nd tracheal rings BOARDS;
TOPNOTCH
MD)
366 Dr. Wi, the resident on duty received a patient involved in a Le Fort I: horizontal fractures of the maxillae, MAIRRE BACK-UP
vehicular crash and upon seeing the patient noted extensive passing superior to the maxillary alveolar process JAMES MIDTERM
maxillofacial injuries. During the subsequent management Le Fort II: passes from the posterolateral parts of GADDI, MD EXAM - FEB
imaging was done which showed a fracture line involving the the maxillary sinuses superomedially through the (TOP 4 - AUG 2015
frontozygomatic sutures, the ethmoid and nasal bones, the infraorbital foramina, lacrimals, or ethmoids to the 2013 MED
superior orbital fissures and the greater wings of the bridge of the nose. BOARDS;
sphenoid. His diagnosis would be? Le Fort III: horizontal fracture that passes through TOPNOTCH
A. Le Fort I the superior orbital fissures and the ethmoid and MD)
B. Le Fort II nasal bones and extends laterally through the
C. Le Fort III greater wings of the sphenoid and the
D. Le Fort IV frontozygomatic sutures.
There is no Le Fort IV. Moore 7th pg 837

367 During left radical nephrectomy, the left kidney is removed The suprarenal glands are supplied by three MAIRRE BACK-UP
together with the left suprarenal gland. As part of the arteries, inferior phrenic artery (superior), JAMES MIDTERM
procedure, the renal artery is ligated. To prevent bleeding of abdominal aorta (middle) and renal artery GADDI, MD EXAM - FEB
the vessels supplying the suprarenal gland, arterial (inferior) Snell 9th pg 215 (TOP 4 - AUG 2015
branch/branches coming from which of the following should 2013 MED
also be ligated? BOARDS;
A. Inferior phrenic artery TOPNOTCH
B. Aorta MD)
C. Splenic artery
D. A and B
E. All of the above

368 True of the rectum: The rectum is primarily retroperitoneal but it is MAIRRE BACK-UP
A. The rectum is a purely retroperitoneal organ and begins as also subperitoneal. The peritoneum covers the JAMES MIDTERM
a continuation of the sigmoid colon anterior and lateral surfaces of the first third of GADDI, MD EXAM - FEB
B. It is covered by a part of peritoneum up to its distal third the rectum and only the anterior surface of the (TOP 4 - AUG 2015
C. The proximal third is covered by peritoneum at its middle third, leaving the lower third devoid of 2013 MED
anterior and lateral surfaces peritoneum. Moore 7th pgs 253, 370; Snell 9th pg BOARDS;
D. All of the above 265 TOPNOTCH
MD)

369 What structure/s pass/es through the diaphragmatic The caval opening lies at the level of T8 and MAIRRE BACK-UP
opening at the level of T12? transmits the inferior vena cava and the terminal JAMES MIDTERM
A. Aorta and Thoracic duct branches of the right phrenic nerve. The GADDI, MD EXAM - FEB
B. Vena cava and phrenic nerve esophageal opening lies at the level of T10 and (TOP 4 - AUG 2015
C. Azygos vein transmits the esophagus, the right and left vagus 2013 MED
D. A and C nerves, the esophageal branches of the left gastric BOARDS;
E. All of the above vessels and the lymphatics of the lower third of TOPNOTCH
the esophagus. The aortic opening lies at the level MD)
of T12 and transmits the aorta, the thoracic duct
TOPNOTCH MEDICAL BOARD PREP ANATOMY SUPEREXAM Page 47 of 94
For inquiries visit www.topnotchboardprep.com.ph or email us at topnotchmedicalboardprep@gmail.com
TOPNOTCH MEDICAL BOARD PREP ANATOMY SUPEREXAM
For inquiries visit www.topnotchboardprep.com.ph or email us at topnotchmedicalboardprep@gmail.com
Item QUESTION EXPLANATION AUTHOR TOPNOTCH
# EXAM
and the azygos vein. Snell 9th pg 46

370 A 56/F diagnosed with endometrial carcinoma confined to The ureter can be damaged during ligation of the MAIRRE BACK-UP
the uterus on TV-UTZ was about to undergo a radical uterine artery during hysterectomy as it passes JAMES MIDTERM
hysterectomy with bilateral salpingo-oophorectomy, the immediately inferior to the uterine artery near the GADDI, MD EXAM - FEB
attending OBGYN wanting to avoid post operative lateral part of the fornix of the vagina and as it (TOP 4 - AUG 2015
complications recalls that the ureter can sustain iatrogenic crosses the uterine artery approximately 2 cms 2013 MED
injury at which of the following locations? superior to the ischial spine. It can also be BOARDS;
A. As it passes immediately inferior to the uterine artery near damaged during oophorectomy when the ovarian TOPNOTCH
the lateral part of the fornix of the vagina artery is ligated since they are close to each other MD)
B. As it crosses the uterine artery approximately 2 cms as they cross the pelvic brim. Moore 7th pg 361
superior to the ischial spine
C. As it crosses the pelvic brim together with the ovarian
artery
D. All of the above

371 A 34/M was on his way home when he was suddenly The patient most likely suffered from an MAIRRE BACK-UP
mugged and was hit at the side of the head with a lead pipe extradural hematoma which usually presents as a JAMES MIDTERM
by unknown assailants. He was briefly unconscious and was brief concussion followed by a lucid interval for a GADDI, MD EXAM - FEB
awakened by concerned bystanders. He then reported the few hours. This then leads to drowsiness and (TOP 4 - AUG 2015
incident to the police and since there were no obvious coma. Extradural hematomas are arterial in origin 2013 MED
external injuries he decided to go home. The following and are usually caused by extravasation of blood BOARDS;
morning, he was found dead by his family. The vessel that from the torn branches of the middle meningeal TOPNOTCH
was most likely responsible for the death of the patient is a artery (specifically the anterior branch which lies MD)
branch of the? immediately on top of the pterion). The middle
A. Middle meningeal artery meningeal artery is a branch of the maxillary
B. Internal carotid artery artery which in itself is a branch of the external
C. External carotid artery carotid artery. Moore 7th pg 876
D. Superior sagittal sinus

372 A 60/M smoker, diabetic, suddenly experienced severe MAIRRE BACK-UP


crushing chest pain. He was brought to the nearest hospital JAMES MIDTERM
and an ECG was done which showed ST elevation of leads GADDI, MD EXAM - FEB
V4-V6. Which vessel was most likely occluded? (TOP 4 - AUG 2015
A. Left circumflex artery 2013 MED
B. Left anterior descending artery BOARDS;
C. Right marginal artery TOPNOTCH
D. Left main coronary artery MD)

Snell 9th pg 89
373 A 32/F underwent total thyroidectomy. Postoperatively the Near the inferior pole of the thyroid gland, the MAIRRE BACK-UP
patient complains of hoarseness. This suggests damage to right recurrent laryngeal nerve is intimately JAMES MIDTERM
the _________ which is closely related to the___________. related to the inferior thyroid artery. Because of GADDI, MD EXAM - FEB
A. Recurrent laryngeal nerve: Superior thyroid artery this the inferior thyroid artery is ligated some (TOP 4 - AUG 2015
B. Recurrent laryngeal nerve: Inferior thyroid artery distance lateral to the thyroid gland. Although the 2013 MED
C. External laryngeal nerve: Superior thyroid artery danger of injuring the left recurrent laryngeal BOARDS;
D. External laryngeal nerve: Inferior thyroid artery nerve during surgery is not as great, owing to its TOPNOTCH
more vertical ascent from the superior MD)
mediastinum, the artery and nerve are also closely
associated near the inferior pole of the thyroid
gland. Hoarseness is the usual sign of unilateral
recurrent nerve injury; however, temporary
aphonia or disturbance of phonation and laryngeal
spasm may occur. These signs usually result from
bruising the recurrent laryngeal nerves during
surgery or from the pressure of accumulated
blood and serous exudate after the operation.
Moore 7th pg 1043

TOPNOTCH MEDICAL BOARD PREP ANATOMY SUPEREXAM Page 48 of 94


For inquiries visit www.topnotchboardprep.com.ph or email us at topnotchmedicalboardprep@gmail.com
TOPNOTCH MEDICAL BOARD PREP ANATOMY SUPEREXAM
For inquiries visit www.topnotchboardprep.com.ph or email us at topnotchmedicalboardprep@gmail.com
Item QUESTION EXPLANATION AUTHOR TOPNOTCH
# EXAM
374 A 7/M fell while from a tree and landed on his outstretched The suprascapular nerve is vulnerable to injury in MAIRRE BACK-UP
hand. He was quickly brought to the emergency room and fractures of the middle third of the clavicle. Injury JAMES MIDTERM
the examining physician noted that the limb is medially to the suprascapular nerve results in loss of lateral GADDI, MD EXAM - FEB
rotated with loss of abduction. There was also a step off rotation of the humerus at the glenohumeral joint. (TOP 4 - AUG 2015
deformity palpable at the middle third of the clavicle. What Consequently the relaxed limb rotates medially 2013 MED
nerve was most likely injured? into the waiter’s tip position. The ability to initiate BOARDS;
A. Subscapular nerve abduction of the limb is also affected. TOPNOTCH
B. Median nerve Injury to the axillary nerve from fracture of the MD)
C. Axillary nerve surgical neck of the humerus or dislocation of the
D. Suprascapular nerve humeral head will produce the same set of deficits.
Moore 7th pg 710 & 1009

375 A 27/F has multiple closed comedones on her nose. She then Inside each cavernous sinus is the internal carotid MAIRRE BACK-UP
proceeded to squeeze them individually. After a few days she artery with its small branches, surrounded by the JAMES MIDTERM
had a severe headache which was followed by high grade carotid plexus of sympathetic nerve(s), and the GADDI, MD EXAM - FEB
fever, periorbital edema and diplopia. A diagnosis of abducent nerve. The oculomotor (CN III) and (TOP 4 - AUG 2015
cavernous sinus thrombosis was made. Which is/are found trochlear (CN IV) nerves, plus two of the three 2013 MED
inside the cavernous sinus? divisions of the trigeminal nerve (CN V1 and V2) BOARDS;
A. Internal carotid artery are embedded in the lateral wall of the sinus. TOPNOTCH
B. Trochlear nerve Moore 7th pg 869 MD)
C. Mandibular nerve
D. A and B
E. All of the above

376 Which of the following is/are TRUE of the inferior vena cava? The inferior vena cava has the following MAIRRE BACK-UP
A. There are two anterior visceral tributaries tributaries: 2 anterior visceral tributaries (hepatic JAMES MIDTERM
B. There are three veins of origin veins), 3 lateral visceral tributaries (right GADDI, MD EXAM - FEB
C. There are three visceral tributaries suprarenal, right renal and right gonadal vein), 5 (TOP 4 - AUG 2015
D. All of the above lateral abdominal wall tributaries (one inferior 2013 MED
E. A and C phrenic vein and four lumbar veins) and 3 veins of BOARDS;
origin (two common iliac veins and the median TOPNOTCH
sacral vein). Snell 9th pg 218 MD)
377 A 30/M has extensive metastatic testicular carcinoma. The preaortic lymph nodes lie around the origins MAIRRE BACK-UP
Metastatic foci would most likely be found where? of the celiac, superior mesenteric, and inferior JAMES MIDTERM
A. Superficial inguinal lymph nodes mesenteric arteries and are referred to as the GADDI, MD EXAM - FEB
B. Deep inguinal lymph nodes celiac, superior mesenteric, and inferior (TOP 4 - AUG 2015
C. Para-aortic lymph nodes mesenteric lymph nodes, respectively. They drain 2013 MED
D. Pre-aortic lymph nodes the lymph from the gastrointestinal tract, BOARDS;
E. All of the above extending from the lower one third of the TOPNOTCH
esophagus to halfway down the anal canal, and MD)
from the spleen, pancreas, gallbladder, and greater
part of the liver.
lateral aortic (para-aortic or lumbar) lymph nodes
drain lymph from the kidneys and suprarenals;
from the testes in the male and from the ovaries,
uterine tubes, and fundus of the uterus in the
female; from the deep lymph
vessels of the abdominal walls; and from the
common iliac nodes. Snell 9th pgs 132 & 220
378 Renal transplantation involves: The iliac fossa on the posterior abdominal wall is MAIRRE BACK-UP
A. End to side anastomosis of the donor renal artery to the the usual site chosen for transplantation of the JAMES MIDTERM
recipient external iliac artery kidney. The fossa is exposed through an incision in GADDI, MD EXAM - FEB
B. End to end anastomosis of the donor renal vein to the the anterior abdominal wall just above the (TOP 4 - AUG 2015
recipient internal iliac vein inguinal ligament. The iliac fossa in front of the 2013 MED
C. Anastomosis of the donor ureter to the recipient bladder iliacus muscle is approached retroperitoneally. BOARDS;
through a ureterocystostomy The renal artery is anastomosed end to end to the TOPNOTCH
D. All of the above internal iliac artery and the renal vein is MD)
E. A and B anastomosed end to side to the external iliac vein.
Ureterocystostomy is then performed by opening
the bladder and providing a wide entrance of the
ureter through the bladder wall. Snell 9th pg 210
379 The following are wholly derived from endoderm EXCEPT Ectoderm derivatives - central nervous system, MAIRRE BACK-UP
A. Greater vestibular glands cornea, retina, and lens, membranous labyrinth of JAMES MIDTERM
B. Prostate gland the inner ear, epidermis, epithelial cells of the GADDI, MD EXAM - FEB
C. Mammary glands sebaceous, sweat, and mammary glands; the (TOP 4 - AUG 2015
D. Bulbourethral glands mucous membrane lining the mouth, nasal 2013 MED
cavities, and paranasal sinuses; the pituitary gland BOARDS;
and the alveoli and ducts of the parotid salivary TOPNOTCH
glands; the mucous membrane of the lower half of MD)
the anal canal; and the terminal parts of the genital
tract and the male urinary tract
Entoderm: epithelial lining of the alimentary tract
from the mouth cavity down to halfway along the
anal canal, thyroid, parathyroid, thymus, liver, and
pancreas, linings of the respiratory tract,
pharyngotympanic tube and middle ear, urinary
bladder, parts of the female and male urethras,
greater vestibular glands, prostate gland,
bulbourethral glands, and vagina.

TOPNOTCH MEDICAL BOARD PREP ANATOMY SUPEREXAM Page 49 of 94


For inquiries visit www.topnotchboardprep.com.ph or email us at topnotchmedicalboardprep@gmail.com
TOPNOTCH MEDICAL BOARD PREP ANATOMY SUPEREXAM
For inquiries visit www.topnotchboardprep.com.ph or email us at topnotchmedicalboardprep@gmail.com
Item QUESTION EXPLANATION AUTHOR TOPNOTCH
# EXAM
380 Which of the following contains a submucosal layer? The wall of the gallbladder does not contain a MAIRRE BACK-UP
A. Gallbladder muscularis mucosae or submucosa. diFiore 12th JAMES MIDTERM
B. Trachea pg 384, the fallopian tubes lack a submucosal layer GADDI, MD EXAM - FEB
C. Fallopian tubes diFiore 12th pg 520, the lamina propria of the (TOP 4 - AUG 2015
D. Larynx laryngeal mucosa blends with the perichondrium 2013 MED
of the thyroid cartilage and there is no distinct BOARDS;
submucosa diFiore 12th pg 398. TOPNOTCH
MD)
381 Which of the following best describes the location of the SIMILAR TO PREVIOUS BOARD EXAM SCOTT BACK-UP
right adrenal gland? CONCEPT/PRINCIPLE. The right adrenal gland is RILEY ONG, MIDTERM
A. Inferior to the right lobe of the liver posterolateral to the inferior vena cava, lateral to MD (TOP 5 - EXAM - FEB
B. Posterolateral to the inferior vena cava the 1st lumbar vertebrae, posterior to the right AUG 2014 2015
C. Anterior to the 1st lumbar vertebra lobe of the liver and supeior to the right kidney. MED
D. Posterior to the right kidney BOARDS;
E. Medial to the spleen TOPNOTCH
MD)

382 Which of the following drains segment I of the liver? SCOTT BACK-UP
A. Middle hepatic vein RILEY ONG, MIDTERM
B. Right hepatic vein MD (TOP 5 - EXAM - FEB
C. Direct venous drainage into the inferior vena cava AUG 2014 2015
D. Direct venous drainage into the common hepatic vein MED
E. Left gastric vein BOARDS;
TOPNOTCH
MD)
383 A 65-year old female who has been bedridden for 3 months SCOTT BACK-UP
presented with symptoms of pneumonia. You suspect an RILEY ONG, MIDTERM
aspiration component for her disease. In which of the MD (TOP 5 - EXAM - FEB
following lung segments will you most likely see the AUG 2014 2015
infiltrates on her chest radiograph? MED
A. Left superior lingular segment BOARDS;
B. Left apicoposterior segment TOPNOTCH
C. Right superior segment MD)
D. Right posterobasal segment
E. Right medial segment

384 The embryonic right 4th aortic arch develops into which of It contributes to the proximal part of the right SCOTT BACK-UP
the following adult structures? subclavian artery. The left 4th aortic arch forms RILEY ONG, MIDTERM
A. Arch of the aorta the arch of the aorta. MD (TOP 5 - EXAM - FEB
B. Right internal carotid artery AUG 2014 2015
C. Right common carotid artery MED
D. Right subclavian artery BOARDS;
E. Right brachiocephalic artery TOPNOTCH
MD)

385 Which of the following is not true about the lesser sac of the The lesser sac is normally collapsed. It can become SCOTT BACK-UP
abdomen? huge and visible when it is filled with fluid (eg. RILEY ONG, MIDTERM
A. The stomach forms its anterior boundary ascites) during disease states. MD (TOP 5 - EXAM - FEB
B. It communicates with the greater sac through the foramen AUG 2014 2015
of Winslow MED
C. The gastrosplenic ligament forms its lateral boundary BOARDS;
D. The lesser omentum transmits the coronary veins which TOPNOTCH
can dilate as varices during portal hypertension MD)
E. The lesser sac is normally filled with fluid and is easily
visualized on CT imaging.

386 During a 12-lead ECG, where should lead V2 be normally SIMILAR TO PREVIOUS BOARD EXAM SCOTT BACK-UP
placed? CONCEPT/PRINCIPLE. RILEY ONG, MIDTERM
A. 4th intercostal space, left parasternal border MD (TOP 5 - EXAM - FEB
B. 5th intercostal space, left parasternal border AUG 2014 2015
C. 4th intercostal space, left midclavicular line MED
D. 5th intercostal space, left midclavicular line BOARDS;
E. 4th intercostal space, right parasternal border TOPNOTCH
MD)

387 The ureters enter the urinary bladder at which of its aspects? SIMILAR TO PREVIOUS BOARD EXAM SCOTT BACK-UP
A. Anterolateral CONCEPT/PRINCIPLE. RILEY ONG, MIDTERM
B. Lateral MD (TOP 5 - EXAM - FEB
C. Posteromedial AUG 2014 2015
D. Posterolateral MED
E. Superomedial BOARDS;
TOPNOTCH
MD)

TOPNOTCH MEDICAL BOARD PREP ANATOMY SUPEREXAM Page 50 of 94


For inquiries visit www.topnotchboardprep.com.ph or email us at topnotchmedicalboardprep@gmail.com
TOPNOTCH MEDICAL BOARD PREP ANATOMY SUPEREXAM
For inquiries visit www.topnotchboardprep.com.ph or email us at topnotchmedicalboardprep@gmail.com
Item QUESTION EXPLANATION AUTHOR TOPNOTCH
# EXAM
388 A patient suffered from myocardial infarction and showed SIMILAR TO PREVIOUS BOARD EXAM SCOTT BACK-UP
signs of hypotension and bradycardia. Which myocardial CONCEPT/PRINCIPLE. RILEY ONG, MIDTERM
wall is most likely affected given this clinical picture. MD (TOP 5 - EXAM - FEB
A. Anterior wall AUG 2014 2015
B. Anteroseptal wall MED
C. Lateral wall BOARDS;
D. Inferior wall TOPNOTCH
E. Posterior wall MD)

389 The thyroid isthmus most commonly lies over which tracheal SIMILAR TO PREVIOUS BOARD EXAM SCOTT BACK-UP
rings? CONCEPT/PRINCIPLE. RILEY ONG, MIDTERM
A. 1st and 2nd MD (TOP 5 - EXAM - FEB
B. 2nd and 3rd AUG 2014 2015
C. 3rd and 4th MED
D. 4th and 5th BOARDS;
E. 5th and 6th TOPNOTCH
MD)

390 Which of the following paranasal sinuses is the last to Present at birth: maxillary and ethmoid sinuses. 3 SCOTT BACK-UP
develop? years old: sphenoid sinus begins to appear. 6 years RILEY ONG, MIDTERM
A. Maxillary sinus old: frontal sinus begins to develop. MD (TOP 5 - EXAM - FEB
B. Ethmoid sinus AUG 2014 2015
C. Sphenoid sinus MED
D. Frontal sinus BOARDS;
E. Both C and D TOPNOTCH
MD)

391 The superior mesenteric vein drains directly into which of The portal vein is formed by the union of the SCOTT BACK-UP
the following structures? superior mesenteric vein and splennic vein. RILEY ONG, MIDTERM
A. Portal vein MD (TOP 5 - EXAM - FEB
B. Celiac vein AUG 2014 2015
C. Inferior vena cava MED
D. Splenic vein BOARDS;
E. Common hepatic vein TOPNOTCH
MD)

392 Which of the following forms the posterior border of the The right ventricle forms its anterior and inferior SCOTT BACK-UP
heart? border. The right atrium forms its right border. RILEY ONG, MIDTERM
A. Right atrium The left ventricle forms its left border. MD (TOP 5 - EXAM - FEB
B. Right ventricle AUG 2014 2015
C. Left atrium MED
D. Left ventricle BOARDS;
E. Apex TOPNOTCH
MD)

393 Which of the following structures is not contained within the The ilioinguinal nerve runs along with but outside SCOTT BACK-UP
spermatic cord? the spermatic cord. RILEY ONG, MIDTERM
A. Vas deferens MD (TOP 5 - EXAM - FEB
B. Ilioinguinal nerve AUG 2014 2015
C. Genital branch of genitofemoral nerve MED
D. Pampiniform plexus BOARDS;
E. Tunica vaginalis TOPNOTCH
MD)

394 Which of the following parts of the urinary bladder is most SIMILAR TO PREVIOUS BOARD EXAM SCOTT BACK-UP
sensitive to stretch? CONCEPT/PRINCIPLE. RILEY ONG, MIDTERM
A. Apex MD (TOP 5 - EXAM - FEB
B. Base AUG 2014 2015
C. Trigone MED
D. Ureteral insertion BOARDS;
E. Urethral opening TOPNOTCH
MD)

395 The stylohyoid muscle is innervated by which of the CN V3: masticator muscles, anterior belly of SCOTT BACK-UP
following nerves? digastric, mylohyoid. CN VII: facial muscles, RILEY ONG, MIDTERM
A. CN V3 stapedius, stylohyoid, posterior belly of digastric. MD (TOP 5 - EXAM - FEB
B. CN VII CN IX: stylopharyngeus. CN XII: intrinsic and AUG 2014 2015
C. CN IX extrinsic tongue muscles (except palatoglossus, MED
D. CN X which is innervated by CN X) BOARDS;
E. CN XII TOPNOTCH
MD)

396 An avulsion fracture at the base of the first proximal phalanx Gamekeeper's thumb: base of 1st proximal SCOTT BACK-UP
is known as: phalanx. Bennett's fracture: base of 1st RILEY ONG, MIDTERM
A. Gamekeeper's thumb metacarpal. Boxer's fracture: neck of 4th and 5th MD (TOP 5 - EXAM - FEB
B. Bennett's fracture metacarpal. Lisfranc fracture: metatarsal fracture AUG 2014 2015
C. Boxer's fracture MED
D. Colles fracture BOARDS;
E. Lisfranc fracture TOPNOTCH
MD)

TOPNOTCH MEDICAL BOARD PREP ANATOMY SUPEREXAM Page 51 of 94


For inquiries visit www.topnotchboardprep.com.ph or email us at topnotchmedicalboardprep@gmail.com
TOPNOTCH MEDICAL BOARD PREP ANATOMY SUPEREXAM
For inquiries visit www.topnotchboardprep.com.ph or email us at topnotchmedicalboardprep@gmail.com
Item QUESTION EXPLANATION AUTHOR TOPNOTCH
# EXAM
397 Which of the following inner ear structures is involved SIMILAR TO PREVIOUS BOARD EXAM SCOTT BACK-UP
during horizontal linear deceleration? CONCEPT/PRINCIPLE. Utricle: horizontal linear RILEY ONG, MIDTERM
A. Utricle acceleration. Saccule: vertical linear acceeleration. MD (TOP 5 - EXAM - FEB
B. Saccule Semicircular canals: angular acceleration. Scala AUG 2014 2015
C. Horizontal semicircular canal media and basilar membrane: involved in hearing MED
D. Scala media function. BOARDS;
E. Basilar membrane TOPNOTCH
MD)

398 Gastrointestinal stromal tumor (GIST), the most common SIMILAR TO PREVIOUS BOARD EXAM SCOTT BACK-UP
mesenchymal tumor of the stomach, most commonly arises CONCEPT/PRINCIPLE. RILEY ONG, MIDTERM
from which of its layers? MD (TOP 5 - EXAM - FEB
A. Mucosa AUG 2014 2015
B. Submucosa MED
C. Muscularis mucosae BOARDS;
D. Muscularis propria TOPNOTCH
E. Serosa MD)

399 What is the lining epithelium of the ovary? SIMILAR TO PREVIOUS BOARD EXAM SCOTT BACK-UP
A. Simple squamous CONCEPT/PRINCIPLE. RILEY ONG, MIDTERM
B. Simple cuboidal MD (TOP 5 - EXAM - FEB
C. Simple columnar AUG 2014 2015
D. Stratified squamous MED
E. Stratified cuboidal BOARDS;
TOPNOTCH
MD)
400 What is the lining epithelium of the prostatic urethra SIMILAR TO PREVIOUS BOARD EXAM SCOTT BACK-UP
A. Simple squamous CONCEPT/PRINCIPLE. RILEY ONG, MIDTERM
B. Stratified squamous MD (TOP 5 - EXAM - FEB
C. Transitional AUG 2014 2015
D. Simple cuboidal MED
E. Stratified cuboidal BOARDS;
TOPNOTCH
MD)
401 What is the most common type of Myoma Uteri? Yes, gyne questions can be seen in anatomy! JOSE CARLO DIAGNOSTIC
A. Submucous SIMILAR TO PREVIOUS BOARD EXAM MASANGKAY EXAM - AUG
B. Pedunculated CONCEPT/PRINCIPLE III, MD (TOP 2014
C. Subserosal 8 - FEB 2014
D. Intramural MED
E. Polypoid BOARDS;
TOPNOTCH
MD)
402 The Dosalis Pedis Artery is SIMILAR TO PREVIOUS BOARD EXAM JOSE CARLO DIAGNOSTIC
A. A continuation of the posterior tibial artery CONCEPT/PRINCIPLE MASANGKAY EXAM - AUG
B. Medial to the adductor longus tendon III, MD (TOP 2014
C. Medial to the medial malleolus 8 - FEB 2014
D. Lateral to the Extensor digitorum longus MED
E. Lateral to the Extensor hallucis longus BOARDS;
TOPNOTCH
MD)
403 Which of the following is derived from the 2nd pharyngeal Meckel's Cartilage is derived from the 1st JOSE CARLO DIAGNOSTIC
arch pharyngeal arch, Hyoid which is derived from the MASANGKAY EXAM - AUG
A. Reichter's Cartilage Reichter's Cartilage is derived from the 2nd III, MD (TOP 2014
B. Meckel's Cartilage pharyngeal arch 8 - FEB 2014
C. Hyoid bone MED
D. A and C BOARDS;
E. All of the above TOPNOTCH
MD)

404 The Cremaster Muscle is a continuation of which of the Ext spermatic fascia derived from Ext. oblique JOSE CARLO DIAGNOSTIC
following? muscle, Darto's from Superficial fascia, Internal MASANGKAY EXAM - AUG
A. External Oblique Muscle spermatic fascia from Transversalis fascia III, MD (TOP 2014
B. Rectus abdominis muscle 8 - FEB 2014
C. Internal Oblique muscle MED
D. Transversalis muscle BOARDS;
E. Transversalis fascia TOPNOTCH
MD)

405 Emphysema is a pulmonary disease where expiration is JOSE CARLO DIAGNOSTIC


active instead of its passive nature. In a patient with MASANGKAY EXAM - AUG
emphysema which of the following is not a muscle used in III, MD (TOP 2014
forced/active expiration? 8 - FEB 2014
A. Internal Intecostal MED
B. External Intercostal BOARDS;
C. Rectus abdominis TOPNOTCH
D. Serratus Posterior Inferior MD)
E. Transverse Thoracis

TOPNOTCH MEDICAL BOARD PREP ANATOMY SUPEREXAM Page 52 of 94


For inquiries visit www.topnotchboardprep.com.ph or email us at topnotchmedicalboardprep@gmail.com
TOPNOTCH MEDICAL BOARD PREP ANATOMY SUPEREXAM
For inquiries visit www.topnotchboardprep.com.ph or email us at topnotchmedicalboardprep@gmail.com
Item QUESTION EXPLANATION AUTHOR TOPNOTCH
# EXAM
406 Which of the following is the reason for the higher tendency SIMILAR TO PREVIOUS BOARD EXAM JOSE CARLO DIAGNOSTIC
of the Sigmoid Colon for Volvolus? CONCEPT/PRINCIPLE MASANGKAY EXAM - AUG
A. Because it is most redundant III, MD (TOP 2014
B. Because It is intraperitoneal 8 - FEB 2014
C. Because it is the narrowest MED
D. Because it is hypermotile BOARDS;
E. Because of its thin wall TOPNOTCH
MD)

407 Which of the following Sinuses is/are present at birth? JOSE CARLO DIAGNOSTIC
A. Frontal MASANGKAY EXAM - AUG
B. Maxillary III, MD (TOP 2014
C. Sphenoid 8 - FEB 2014
D. A and B MED
E. B and C BOARDS;
TOPNOTCH
MD)
408 A patient was rushed to the ED after an automobile accident, A Lucid interval was noted in this patient this is JOSE CARLO DIAGNOSTIC
patient came in with decreased sensorium and multiple frequently associated with the presence of an MASANGKAY EXAM - AUG
abrasions, after a few hours, the patient became conscious epidural hematoma due to a rupture of the middle III, MD (TOP 2014
and coherent which was eventually followed by a coma, on meningeal artery which passes in proximity to the 8 - FEB 2014
physical examination you noted crepitations on the Left pterion. The middle meningeal artery passes thru MED
Pterion area. As a brilliant doctor you knew that there was the Foramen Spinosum BOARDS;
an injured vessel which passes in which foramen of the skull TOPNOTCH
A. Foramen Magnun MD)
B. Foramen Lacerum
C. Foramen Ovale
D. Foramen Spinosum
E. Foramen Rotundum

409 Which of the following structures traverses the Aortic Hiatus The Aorta, Thoracic Duct and Azygous vein enters JOSE CARLO DIAGNOSTIC
of the diaphragm? the aortic hiatus. MASANGKAY EXAM - AUG
A. Thoracic Duct III, MD (TOP 2014
B. Hemiazygous vein 8 - FEB 2014
C. Right Phrenic Nerve MED
D. Vagus Nerve BOARDS;
E. Right Lymphatic Duct TOPNOTCH
MD)

410 A 1-month old female patient was noted to have Rib- Patient is a case of Turner's syndrome, these JOSE CARLO DIAGNOSTIC
notching on X-ray, a chromosomal study was done in this patients have a propensity to develop a Patent MASANGKAY EXAM - AUG
patient revealing a chromosomal count of 45 XO, if this Ductus Arteriosus which presents with rib III, MD (TOP 2014
patient would undergo repair of the said cardiac defect what notching on X-ray. The Left vagus nerve "recurrs" 8 - FEB 2014
structure may be injured? at the aortic arch to become the left recurrent MED
A. Right Phrenic Nerve laryngeal nerve, the Ductus Arteriosus is in BOARDS;
B. Right Vagus Nerve proximity to this structure. The right vagus nerve TOPNOTCH
C. Left Vagus Nerve "recurrs" to become the right recurrent at the MD)
D. Left Phrenic Nerve right subclavian artery.
E. Hemiazygous vein

411 A male patient was rushed to your trauma center after a Flexor carpi ulnaris tendon can be transected in a JOSE CARLO DIAGNOSTIC
massive bleeding secondary to a self-inflicted deep incised deep laceration of the wrist in the ulnar aspect. MASANGKAY EXAM - AUG
wound to the radial aspect of the left wrist, after a recent III, MD (TOP 2014
break-up with his girlfriend. Which of the following 8 - FEB 2014
structures is most likely preserved? MED
A. median nerve BOARDS;
B. flexor carpi radialis tendon TOPNOTCH
C. palmaris longus tendon MD)
D. flexor carpi ulnaris tendon
E. none of the above

412 A prison inmate was rushed to your emergency department from anterior to posterior: Renal vein, Renal JOSE CARLO DIAGNOSTIC
due to an apparent stab wound at the flank on the left artery, renal pelvis. MASANGKAY EXAM - AUG
midscapular line at the level of L2 which of the following III, MD (TOP 2014
structures may be least severed? 8 - FEB 2014
A. Renal vein MED
B. Renal artery BOARDS;
C. Renal pelvis TOPNOTCH
D. Psoas Muscle MD)
E. Quadratus Lumborum muscle

TOPNOTCH MEDICAL BOARD PREP ANATOMY SUPEREXAM Page 53 of 94


For inquiries visit www.topnotchboardprep.com.ph or email us at topnotchmedicalboardprep@gmail.com
TOPNOTCH MEDICAL BOARD PREP ANATOMY SUPEREXAM
For inquiries visit www.topnotchboardprep.com.ph or email us at topnotchmedicalboardprep@gmail.com
Item QUESTION EXPLANATION AUTHOR TOPNOTCH
# EXAM
413 An opera singer underwent a Near Total Thyroidectomy due Injury to the external branch of the superior JOSE CARLO DIAGNOSTIC
to the discovery of an apparent Papillary Thyroid Carcinoma, laryngeal nerve causes paralysis of the MASANGKAY EXAM - AUG
post-surgery, patient was unable to reach high notes, what cricothyroid muscle which tenses and stretches III, MD (TOP 2014
could have been the reason? the vocal cords enabling a person to reach high 8 - FEB 2014
A. Injury to the internal branch of the superior laryngeal notes. MED
nerve BOARDS;
B. Unilateral injury to the recurrent laryngeal nerve TOPNOTCH
C. Bilateral injury to the recurrent laryngeal nerve MD)
D. Injury to the external branch of the superior laryngeal
nerve
E. Injury to the rima glottidis

414 Which of the following is the major blood supply of the The Inferior Parathyroid artery supplies both the JOSE CARLO DIAGNOSTIC
parathyroid glands? superior and inferior parathyroid glands MASANGKAY EXAM - AUG
A. Superior Thyroid Artery III, MD (TOP 2014
B. Inferior Thyroid Artery 8 - FEB 2014
C. Superior Parathyroid artery MED
D. Inferior Parathyroid Artery BOARDS;
E. Middle Thyroid Artery TOPNOTCH
MD)

415 Which of the following cells is responsible for Osteoid SIMILAR TO PREVIOUS BOARD EXAM JOSE CARLO DIAGNOSTIC
formation? CONCEPT/PRINCIPLE MASANGKAY EXAM - AUG
A. Osteocyte III, MD (TOP 2014
B. Osteoblast 8 - FEB 2014
C. Osteoclast MED
D. Langhan's Cell BOARDS;
E. Langerhan's Cell TOPNOTCH
MD)

416 After a prolonged intubation of a COPD patient you decided Tracheostomy is performed at the level of the 2nd JOSE CARLO DIAGNOSTIC
to perform a tracheostomy to lessen the Dead Space and and 4th tracheal rings. MASANGKAY EXAM - AUG
improve ventilation, on what location are you going to III, MD (TOP 2014
perform the tracheostomy? 8 - FEB 2014
A. at the level of C4 vertebra MED
B. in between the thyroid and cricoid cartilage BOARDS;
C. above the thyoid cartilage TOPNOTCH
D. at the 1st to the 4th tracheal rings MD)
E. at the 2nd to the 4th tracheal rings

417 After an automobile accident you realized that the Severance of the Occulomotor nerve causes JOSE CARLO DIAGNOSTIC
occulomotor nerve has been severed due to what diplopia, loss of parallel gaze, fixed and dilated MASANGKAY EXAM - AUG
manifestation of the patient? pupil, loss of light reflex, loss of accomodation, III, MD (TOP 2014
A. Pinpoint pupils Ptosis (Droopy eyelids) (SIMILAR TO PREVIOUS 8 - FEB 2014
B. Droopy eyelids BOARD EXAM CONCEPT/PRINCIPLE) MED
C. Weakness looking down BOARDS;
D. Loss of Corneal reflex TOPNOTCH
E. Horizontal diplopia MD)

418 The melanocytes are seen in which layer of the epidermis? Melanocytes are seen in the basal layer of the JOSE CARLO DIAGNOSTIC
A. S. Corneum epidermis. (SIMILAR TO PREVIOUS BOARD EXAM MASANGKAY EXAM - AUG
B. S. Lucidum CONCEPT/PRINCIPLE) III, MD (TOP 2014
C. S. Granulosum 8 - FEB 2014
D. S. Spinosum MED
E. S. Basale BOARDS;
TOPNOTCH
MD)
419 The oxyphil cells are found in which human structure? The oxyphil cells are seen in the parathyroid JOSE CARLO DIAGNOSTIC
A. Pineal Gland glands still with unknown function. (SIMILAR TO MASANGKAY EXAM - AUG
B. Parathyroid Gland PREVIOUS BOARD EXAM CONCEPT/PRINCIPLE) III, MD (TOP 2014
C. Anterior Pituitary Gland 8 - FEB 2014
D. Posterior Pituitary Gland MED
E. Thyroid Gland BOARDS;
TOPNOTCH
MD)
420 A 35 year-old female mountain climber went under a The thoracodorsal nerve innervates the Latissimus JOSE CARLO DIAGNOSTIC
Modified Radical Mastectomy after being diagnosed with dorsi muscle which elevates the trunk (as if MASANGKAY EXAM - AUG
Breast Cancer. Post-op, patient can move all her limbs, can attempting to climb) this may be a result of a III, MD (TOP 2014
protract her scapula, can laterally and medially rotate her surgical procedure involving the axilla. 8 - FEB 2014
arm, can extend MP joints of all digits and can flex the wrist MED
BUT she can not elevate her trunk. Which nerve may be BOARDS;
injured intra-operatively? TOPNOTCH
A. Long Thoracic nerve MD)
B. Thoracodorsal Nerve
C. Axillary Nerve
D. Lateral Pectoral Nerve
E. Muculocutaneous Nerve

TOPNOTCH MEDICAL BOARD PREP ANATOMY SUPEREXAM Page 54 of 94


For inquiries visit www.topnotchboardprep.com.ph or email us at topnotchmedicalboardprep@gmail.com
TOPNOTCH MEDICAL BOARD PREP ANATOMY SUPEREXAM
For inquiries visit www.topnotchboardprep.com.ph or email us at topnotchmedicalboardprep@gmail.com
Item QUESTION EXPLANATION AUTHOR TOPNOTCH
# EXAM
421 The major blood supply to the parathyroid glands comes There is much controversy to this question. But WEBSTER MIDTERM 1
from: we have to choose, again, the BEST possible ALINDOG, EXAM - AUG
A. Superior thyroid artery answer. Most anatomy books will say that the MD (TOP 3 - 2014
B. Inferior thyroid artery inferior thyroid artery supplies the parathyroids, FEB 2014
C. Superior parathyroid artery but according to Snell 7th edition, the blood comes MED
D. Inferior parathyroid artery from both superior and inferior thyroid arteries. BOARDS;
However, we deem that the inferior thyroid artery TOPNOTCH
is more consistent, and hence the better answer. MD)
Nonetheless, of note, in a study done by Nobori, et
al (PubMed) they have found that 45% of their
subjects had a distinct anastomosing branch
between the superior and inferior thyroid arteries,
supplying also the superior parathyroids.
422 A 25-year old male patient was observed to have recurrent The cavernous sinuses are situated in the middle WEBSTER MIDTERM 1
pyogenic infections of the nasal sinuses. He later on cranial fossa on each side of the body of sphenoid. ALINDOG, EXAM - AUG
developed cavernous sinus thrombosis, a potentially serious They extend from the superior orbital fissure in MD (TOP 3 - 2014
condition because of the many delicate structures that can be front, to the apex of the petrous part of the FEB 2014
affected. Which of the following is least likely to be involved? temporal bone behind. Found within these sinuses MED
A. CN II are the internal carotid artery, CN III and IV, and BOARDS;
B. CN III the ophthalmic and maxillary divisions of CN V. TOPNOTCH
C. CN IV They drain posteriorly into the superior and MD)
D. Internal carotid artery inferior petrosal sinuses and inferiorly into the
E. None of the above. pterygoid venous plexus. They communicate with
the facial vein through the superior ophthalmic
vein.

423 Sounds produced by the aortic valve can be best heard at: Study also some surface anatomy. Pulmonic valve WEBSTER MIDTERM 1
A. 5th ICS, left midclavicular line is best heard at 2nd ICS, left parasternal; mitral ALINDOG, EXAM - AUG
B. 2nd ICS, right parasternal valve at 5th ICS, left MCL; and tricuspid valve at MD (TOP 3 - 2014
C. 2nd ICS, left parasternal the right half of the lower end of the body of FEB 2014
D. Right half of the lower end of the body of sternum sternum. MED
BOARDS;
TOPNOTCH
MD)
424 A perforating ulcer located at the lesser curvature of the Gastric ulcers usually occur within the body of the WEBSTER MIDTERM 1
stomach will most likely erode which branch of the celiac stomach along the lesser curvature above the ALINDOG, EXAM - AUG
artery? incisura angularis. If it perforates, it can erode the MD (TOP 3 - 2014
A. Splenic artery left gastric artery (reviewing the blood supply of FEB 2014
B. Gastroduodenal artery the stomach: Lesser curvature --- right and left MED
C. Left gastric artery gastric arteries; greater curvature --- right and left BOARDS;
D. Left gastroepiploic artery gastroepiploic arteries; fundus --- short gastric TOPNOTCH
artery). Both the left gastric artery and the splenic MD)
artery are branches of the celiac artery.
Gastroduodenal artery comes from the hepatic
artery whereas the left gastroepiploic artery
originates from the gastroduodenal artery.
425 The cell organelle containing oxidative enzymes, WEBSTER MIDTERM 1
synthesizing H2O2 and is involved in beta-oxidation of long ALINDOG, EXAM - AUG
chain fatty acids: MD (TOP 3 - 2014
A. Lysosome FEB 2014
B. Smooth ER MED
C. Mitochondria BOARDS;
D. Peroxisome TOPNOTCH
MD)

426 The most common site of aortic aneurysm: Aneurysms are dilatations of the aorta usually WEBSTER MIDTERM 1
A. Distal to the portion of aorta passing through the resulting from atherosclerosis, which causes ALINDOG, EXAM - AUG
diaphragmatic opening arterial wall weakening. MD (TOP 3 - 2014
B. Below the origin of the renal arteries FEB 2014
C. Along the portion contained within the posterior MED
mediastinum BOARDS;
D. Just distal to the bifurcation of the common iliacs TOPNOTCH
MD)

427 All of the following muscles are important in plantar flexing Gastrocnemius, soleus, and plantaris together WEBSTER MIDTERM 1
the foot at the ankle joint except for: serve as a powerful plantar flexor of the ankle ALINDOG, EXAM - AUG
A. Gastrocnemius joint. They provide the main propulsive force in MD (TOP 3 - 2014
B. Popliteus walking and running. They are supplied by the FEB 2014
C. Soleus tibial nerve, with nerve root coming from S1 and MED
D. Plantaris S2. BOARDS;
TOPNOTCH
MD)

TOPNOTCH MEDICAL BOARD PREP ANATOMY SUPEREXAM Page 55 of 94


For inquiries visit www.topnotchboardprep.com.ph or email us at topnotchmedicalboardprep@gmail.com
TOPNOTCH MEDICAL BOARD PREP ANATOMY SUPEREXAM
For inquiries visit www.topnotchboardprep.com.ph or email us at topnotchmedicalboardprep@gmail.com
Item QUESTION EXPLANATION AUTHOR TOPNOTCH
# EXAM
428 A 22-year old frisbee player was brought to the hospital after The anterior cruciate ligament prevents the WEBSTER MIDTERM 1
sustaining a knee injury. On physical exam, he was noted to posterior displacement of femur on tibia, or ALINDOG, EXAM - AUG
demonstrate a positive anterior drawer sign. This suggests conversely, prevents the anterior displacement of MD (TOP 3 - 2014
that: tibia on femur (anterior drawer test). Option A FEB 2014
A. There is injury to the knee ligament that is attached to refers to the lateral collateral ligament, which is MED
the lateral condyle of the femur above and to the head of the also more flexible and hence less susceptible to BOARDS;
fibula below. injury compared with the medial collateral TOPNOTCH
B. There is injury to the knee ligament that is attached to ligament (option B). Option C simply refers to the MD)
the medial condyle of the femur above and to the medial posterior cruciate ligament, whose stability is
aspect of the tibial shaft below. tested by the posterior drawer test.
C. There is injury to the knee ligament that prevents
anterior displacement of the femur on the tibia.
D. There is injury to the knee ligament that prevents
posterior displacement of the femur on the tibia.

429 Which of the following is not true about the mucous Anal columns are found only in the mucous WEBSTER MIDTERM 1
membrane of the lower half of the anal canal? membrane of the UPPER half of the anal canal. The ALINDOG, EXAM - AUG
A. It is sensitive to pain, temperature, touch and pressure rest of the options are true about the lower anal MD (TOP 3 - 2014
(somatic sensory innervation). canal. It is the pectinate line that indicates the FEB 2014
B. It is thrown into vertical mucous folds called anal level where the two halves join together. MED
columns. BOARDS;
C. The lymph drains downward to the medial group of TOPNOTCH
superficial inguinal nodes. MD)
D. Its blood supply comes directly from the inferior rectal
artery, a branch of the internal pudendal artery.
E. None of these.

430 A normal individual will usually have how many parathyroid It is a "mortal sin" to get wrong in "give-away" WEBSTER MIDTERM 1
glands? questions like this (chances are, everybody will ALINDOG, EXAM - AUG
A. 2 answer them correctly so please always read MD (TOP 3 - 2014
B. 4 CAREFULLY even those questions with obvious FEB 2014
C. 6 answers). Parathyroid glands can be as numerous MED
D. 7 as 7 in a normal individual. BOARDS;
TOPNOTCH
MD)

431 Venous drainage of the posterior ventricular walls including WEBSTER MIDTERM 1
the posterior interventricular septum drain directly to: ALINDOG, EXAM - AUG
A. Great cardiac vein MD (TOP 3 - 2014
B. Smallest cardiac vein FEB 2014
C. Coronary sinus MED
D. Middle cardiac vein BOARDS;
TOPNOTCH
MD)

432 A 46-year old male was rushed to the ER after getting Focused assessment with sonography in trauma WEBSTER MIDTERM 1
involved in a vehicular accident. An anterior abdominal blunt (FAST) is a quick, reliable, non-invasive procedure ALINDOG, EXAM - AUG
injury was suspected. FAST was performed with focus on 4 that can facilitate a timely diagnosis for patients MD (TOP 3 - 2014
areas including the so-called Morrison's pouch. As member with blunt abdominal trauma. It includes views of FEB 2014
of the trauma team you know that this area corresponds to (1) the hepatorenal recess (Morison pouch), (2) MED
the: the perisplenic view, (3) the subxiphoid BOARDS;
A. Splenorenal space pericardial window, and (4) the suprapubic TOPNOTCH
B. Suprapubic space window (Douglas pouch). MD)
C. Hepatorenal space
D. Subxiphoid area

433 This is one of the five terminal nerves of the brachial plexus Must master the brachial (and lumbar) plexus. It is WEBSTER MIDTERM 1
which is also the motor innervation of the anterior formed in the posterior triangle of the neck by the ALINDOG, EXAM - AUG
compartment of the arm, important in flexing the elbow joint union of the anterior rami of C5 to C8 and T1. It MD (TOP 3 - 2014
and supinating the forearm: can be divided into roots, trunks, divisions and FEB 2014
A. Musculocutaneous nerve cords. It has 5 terminal branches namely, the MED
B. Median nerve musculocutaneous nerve (innervating the BOARDS;
C. Radial nerve anterior/flexor compartment of the arm); the TOPNOTCH
D. Ulnar nerve axillary nerve (deltoid and teres minor); the radial MD)
nerve (posterior/extensor compartment of arm
and forearm); the median nerve (anterior/flexor
compartment of the forearm except flexor carpi
ulnaris and flexor digitorum profundus, medial
half); and the ulnar nerve (intrinsic muscles of the
hand).
434 True of utricle and saccule except: The utricle and saccule constitute the static WEBSTER MIDTERM 1
A. Housed by a central cavity known as vestibule labyrinth which functions during the linear ALINDOG, EXAM - AUG
B. Kinetic labyrinth acceleration of the head and with the effects of the MD (TOP 3 - 2014
C. Important for linear acceleration pull of gravity. On the other hand, the semicircular FEB 2014
D. Contain otoliths canals make up the kinetic labyrinth which is MED
E. None of the above important in angular deceleration and BOARDS;
acceleration; it has no otoliths. TOPNOTCH
MD)

TOPNOTCH MEDICAL BOARD PREP ANATOMY SUPEREXAM Page 56 of 94


For inquiries visit www.topnotchboardprep.com.ph or email us at topnotchmedicalboardprep@gmail.com
TOPNOTCH MEDICAL BOARD PREP ANATOMY SUPEREXAM
For inquiries visit www.topnotchboardprep.com.ph or email us at topnotchmedicalboardprep@gmail.com
Item QUESTION EXPLANATION AUTHOR TOPNOTCH
# EXAM
435 Which of the following statements about seminal vesicle is It is the epididymis that can store mature WEBSTER MIDTERM 1
not true? spermatozoa. Structures that can be palpated ALINDOG, EXAM - AUG
A. They are lobulated structures that can store during DRE: rectovesical pouch, full bladder, MD (TOP 3 - 2014
spermatozoa. seminal vesicles, displaced or enlarged ductus FEB 2014
B. They produce secretions added to the seminal fluid and deferentes, membranous part of urethra when MED
important in sperm nourishment. catheterized, and bulbo-urethral glands; ischial BOARDS;
C. Their blood supply comes from the superior vesicle and tuberosity and spine and sacrotuberous ligament; TOPNOTCH
superior rectal arteries. pelvic surface of sacrum and coccyx. In females, MD)
D. They can be palpated during digital rectal exam. vagina, cervix, ostium uteri, body of uterus when
E. All of the above. retroverted, recto-uterine fossa, and,
pathologically, broad ligaments, uterine tubes, and
ovaries. Blood supply of the seminal vesicle comes
from the inferior vesicle and middle rectal
arteries.
436 A 15-year old skateboarder lost balance and fell on his This condition corresponds to a distal lesion of the WEBSTER MIDTERM 1
outstretched right hand causing hyperextension of the wrist median nerve (true also in carpal tunnel ALINDOG, EXAM - AUG
joint . He then manifest numbness and pain over the palmar syndrome) in which there is an altered cutaneous MD (TOP 3 - 2014
aspects of his right thumb, index and middle fingers. Which sensation on the lateral 3 and 1/2 digits of the FEB 2014
of the following mechanisms of injury will most likely hand; a patient may also exhibit weakness in MED
explain the symptoms? opposition of the thumb which remains adducted BOARDS;
A. A fracture of the base of the metacarpal bones and extended as a result, so called "ape" hand. A TOPNOTCH
B. A supracondylar fracture of the humerus with proximal lesion of the median hand meanwhile, MD)
compression of the pronator teres muscle results from supracondylar fracture of the
C. An oblique fracture of the trapezium bone humerus or from compression between the
D. A dislocated lunate bone pronator teres leading to the the condition "hand
of benediction" - with the index and middle fingers
remained extended when attempting to flex.

437 The inferior thyroid vein which receives tributaries from the WEBSTER MIDTERM 1
lower poles of the gland and the isthmus drains directly to: ALINDOG, EXAM - AUG
A. Internal jugular vein MD (TOP 3 - 2014
B. Brachiocephalic vein FEB 2014
C. External jugular vein MED
D. Superior vena cava BOARDS;
TOPNOTCH
MD)

438 Melanocytes, the pigment-producing cells, are most Buzz words: stratum corneum - dead keratinized WEBSTER MIDTERM 1
numerous in this layer of the skin: cells, protection; stratum lucidum - only found in ALINDOG, EXAM - AUG
A. Stratum lucidum thick skin (palm and soles); stratum granulosum - MD (TOP 3 - 2014
B. Stratum granulosum keratohyaline granules; stratum spinosum - FEB 2014
C. Stratum corneum Langerhan cells, SCCA; stratum basale - MED
D. Stratum basale melanocytes, Merkel cells, basal cell CA. BOARDS;
TOPNOTCH
MD)

439 Which of the following anatomic relationships will best WEBSTER MIDTERM 1
locate the gallbladder? ALINDOG, EXAM - AUG
A. It is medial to the ligamentum teres within the falciform MD (TOP 3 - 2014
ligament. FEB 2014
B. It lies superior to the porta hepatis MED
C. It is lateral to the quadrate lobe of the liver. BOARDS;
D. 2/3 of which lies in the left lobe of the liver. TOPNOTCH
MD)

440 In performing the ECG, the V5 lead should be placed over Must know: V1 - 4th ICS, right parasternal area; V2 WEBSTER MIDTERM 1
the: - 4th ICS, left para sternal area; V3 - space between ALINDOG, EXAM - AUG
A. 2nd right ICS, parasternal area V2 and V4; V4 - 5th ICS, left MCL; V5 - 5th ICS, left MD (TOP 3 - 2014
B. 4th left ICS, parasternal area AAL; V6 - 5th ICS, left MAL. FEB 2014
C. 4th left ICS, midclavicular area MED
D. 5th left ICS, lateral to the midclavicular area BOARDS;
TOPNOTCH
MD)
441 A 45 year-old male painter went up to a ladder to paint the Scarpa's or the membranous layer of JULIET MIDTERM 2
roof, however, he slips and fell suffering a straddle injury. superficial fascia forms a tubular sheath over KRISTINE EXAM - AUG
Which of the following structure prevents the spread of the penis and scrotum in males which prevents EVANGELIST 2014
urine inferiorly to the thigh in case of ruptured penile extravasation of urine inferiorly in cases of A, MD (TOP 9
urethra: urethral trauma. In the male, Camper’s fascia is - FEB 2014
A. rectus sheath continued over the penis and outer surface of the MED
B. camper's fascia spermatic cord to the scrotum, where it helps to BOARDS;
C. scarpa's fascia form the dartos. This layer is sufficiently complete TOPNOTCH
D. denonvilier's fascia that fluids escaping from a ruptured vessel or MD)
E. conjoint tendon urethra (blood and/or urine) may accumulate
deep to it.
442 Which is not included in the drainage of the thyroid: The main drainage of the thyroid gland are JULIET MIDTERM 2
A. Superior thyroid vein superior thyroid vein and middle thyroid vein KRISTINE EXAM - AUG
B. Middle thyroid vein draining into internal jugular vein. The inferior EVANGELIST 2014
C. Inferior thyroid vein thyroid vein which drains isthmus and lower poles A, MD (TOP 9
D. Brachiocephalic of the gland which drains into left brachiocephalic - FEB 2014
E. Thyroidea ima vein in the thorax. Thyroidea ima may arise MED
from the brachiocephalic artery or the arch of BOARDS;
the aorta. TOPNOTCH
MD)
TOPNOTCH MEDICAL BOARD PREP ANATOMY SUPEREXAM Page 57 of 94
For inquiries visit www.topnotchboardprep.com.ph or email us at topnotchmedicalboardprep@gmail.com
TOPNOTCH MEDICAL BOARD PREP ANATOMY SUPEREXAM
For inquiries visit www.topnotchboardprep.com.ph or email us at topnotchmedicalboardprep@gmail.com
Item QUESTION EXPLANATION AUTHOR TOPNOTCH
# EXAM
443 The non-ciliated secretory cells which randomly interrupt Clara cells are nonciliated, secretory bronchiolar JULIET MIDTERM 2
the ciliated epithelial lining of the bronchial mucosa are epithelial cells which function as stem cells for KRISTINE EXAM - AUG
called: repair in the bronchioles and can divide into EVANGELIST 2014
A. Kulchitsky cells ciliated or nonciliated bronchiolar cells. A, MD (TOP 9
B. Clara cells Enterochromaffin cells, or Kulchitsky cells, are a - FEB 2014
C. Neuroendocrine cells type of enteroendocrine and neuroendocrine cell MED
D. Type 2 pneumocytes occurring in the epithelial lining the lumen of the BOARDS;
E. Dust cells digestive tract and the respiratory tract. Type 2 TOPNOTCH
pneumocytes are cells secreting surfactant in the MD)
alveoli. Dust cells or alveolar macrophage are in
the lungs that reside on respiratory surfaces and
clean off particles such as dust or microorganisms.
444 A 52 year-old man was rushed to the ER after a car accident. The porous, fragile nature of the ethmoid bone JULIET MIDTERM 2
He was noted to have clear fluid draining from his nose makes it particularly susceptible to fractures. The KRISTINE EXAM - AUG
which is apparently the CSF. The bone which is most likely ethmoid is usually fractured from an upward force EVANGELIST 2014
fractured is the: to the nose. This could occur by hitting the A, MD (TOP 9
A. nasal dashboard in a car crash or landing on the ground - FEB 2014
B. ethmoid after a fall. The ethmoid fracture can produce bone MED
C. frontal fragments that penetrate the cribriform plate. This BOARDS;
D. lacrimal trauma can lead to a leak of cerebral spinal fluid TOPNOTCH
E. zygomatic into the nasal cavity. MD)

445 Which chamber of the heart is most likely enlarged when Left Atrial Enlargement pushes the esophagus JULIET MIDTERM 2
there is narrowing of the thoracic esophagus on Barium toward the spine (as seen on barium swallow) and KRISTINE EXAM - AUG
swallow: is the most sensitive indicator of LAE. EVANGELIST 2014
A. right atrium A, MD (TOP 9
B. right ventricle - FEB 2014
C. left atrium MED
D. left ventricle BOARDS;
E. left auricle TOPNOTCH
MD)

446 A 23 year-old male patient seen at a clinic cannot focus on The ciliary ganglion is a parasympathetic ganglion JULIET MIDTERM 2
near objects but he can move his eyeball normally and see located in the posterior orbit on the lateral side of KRISTINE EXAM - AUG
distant objects clearly. This condition may indicate damage the optic nerve. The postganglionic axons run in EVANGELIST 2014
to: the short ciliary nerves and innervate two eye A, MD (TOP 9
A. Short ciliary nerve and ciliary ganglion muscles including the sphincter pupillae which - FEB 2014
B. Ciliary ganglion and oculomotor nerve constricts the pupil, and the ciliary muscle which MED
C. Oculomotor nerve and long ciliary nerve contracts, releasing tension on the Zonular Fibers, BOARDS;
D. Short and long ciliary nerves and making the lens more convex, resulting to TOPNOTCH
E. Long ciliary nerve and superior cervical ganglion accommodation. MD)

447 A 48 year-old woman was diagnosed of Phyllodes tumor. She Long thoracic nerve supplies serratus anterior. JULIET MIDTERM 2
underwent mastectomy. However, postoperatively, the The nerve is most commonly injured as it courses KRISTINE EXAM - AUG
woman experiences weakness in the ability to protract the superficial to the serratus anterior causing EVANGELIST 2014
scapula with difficulty raising the arm above her head. weakness in the ability to protract the scapula and A, MD (TOP 9
Damage to which of the following nerves is suggested by this difficulty in raising the arm above the head. It also - FEB 2014
finding: causes winging of the scapula. MED
A. Axillary BOARDS;
B. Supraclavicular TOPNOTCH
C. Spinal accessory MD)
D. Long Thoracic
E. Thoracodorsal

448 A 55 year-old male sustained from a motor vehicular Although protected under the bony ribcage, the JULIET MIDTERM 2
accident resulting to fracture of the 9th to 11th ribs left spleen remains the most commonly affected organ KRISTINE EXAM - AUG
posterior. BP is 80/50mmHg, PR=125bpm, RR=22cpm. The in blunt injury to the abdomen in all age groups. EVANGELIST 2014
most likely injured organ is the While some references occasionally document A, MD (TOP 9
A. stomach liver injuries as being more common, blunt - FEB 2014
B. pancreas injuries to the spleen are documented more MED
C. liver frequently as the primary solid organ injury in the BOARDS;
D. spleen abdomen. These injuries are common as a result TOPNOTCH
E. small intestine from motor vehicle crashes, domestic violence, MD)
sporting events, and accidents involving bicycle
handlebars.
449 Massive hemopericardium compromises cardiac and Cardiac tamponade is a life-threatening condition JULIET MIDTERM 2
systemic circulation. In emergency evacuation of the blood, that requires prompt diagnosis and management. KRISTINE EXAM - AUG
the best and safest way to make an incision en route to the Anatomically, pericardiocentesis is carried out EVANGELIST 2014
pericardial activity is via: immediately under the xiphoid process A, MD (TOP 9
A. 5th left ICS lateral to sternum (infrasternal), up and leftwards. - FEB 2014
B. 7th left ICS immediately lateral to sternum MED
C. 6th left ICS 8cm from misdternal line BOARDS;
D. 7th left ICS 8cm from midsternal line TOPNOTCH
E. immediately subxiphoidal MD)

TOPNOTCH MEDICAL BOARD PREP ANATOMY SUPEREXAM Page 58 of 94


For inquiries visit www.topnotchboardprep.com.ph or email us at topnotchmedicalboardprep@gmail.com
TOPNOTCH MEDICAL BOARD PREP ANATOMY SUPEREXAM
For inquiries visit www.topnotchboardprep.com.ph or email us at topnotchmedicalboardprep@gmail.com
Item QUESTION EXPLANATION AUTHOR TOPNOTCH
# EXAM
450 A 65 year-old female, hypertensive, diabetic suddenly The lesion is located in Wernicke's area, which is JULIET MIDTERM 2
developed dizziness and subsequently loss consciousness. the posterior region of the left superior temporal KRISTINE EXAM - AUG
She was rushed to the emergency room, she regained gyrus or the first gyrus of the temporal lobe. EVANGELIST 2014
consciousness. She cannot understand or obey commands, Brodmann's areas 21 and 42 correspond to A, MD (TOP 9
she talks and answers to questions irrelevantly. If an infarct Wernicke's area. With Wernicke's aphasia there is - FEB 2014
is suspected, what Brodmann Area is affected: usually a severe impairment in auditory MED
A. Brodmann Area 4 comprehension. Speech, while fluent, is BOARDS;
B. Brodmann Area 44 and 45 semantically inappropriate and paraphasic. TOPNOTCH
C. Brodmann Area 8 Comprehension and expression tend to be equally MD)
D. Brodmann Area 21, 42 impaired.
E. Brodmann Area 3,1,2

451 In portal hypertension, which of the following veins will not The tributaries of portal vein are the splenic vein, JULIET MIDTERM 2
engorge and is not a collateral circulation: inferior mesenteric vein, superior mesenteric vein, KRISTINE EXAM - AUG
A. Superior rectal left gastric vein, right gastric vein, and cystic veins. EVANGELIST 2014
B. Median sacral Median sacral vein directly drains to left common A, MD (TOP 9
C. Esophageal iliac vein then to inferior vena cava. - FEB 2014
D. Paraumbilical MED
E. Splenic BOARDS;
TOPNOTCH
MD)
452 During a dilatation and curettage in a 23 year-old G1P0 The body of the uterus is related anteriorly to the JULIET MIDTERM 2
patient after an incomplete abortion at 5 weeks AOG, the uterovesical pouch and the superior surface of the KRISTINE EXAM - AUG
instrument accidentally punctured the uterus anteriorly bladder. Perforation is usually caused by a surgical EVANGELIST 2014
hitting this structure: instrument used for scraping and removing A, MD (TOP 9
A. Urinary bladder material from the uterus. The instrument - FEB 2014
B. Uterine artery penetrates through the uterine wall, and rarely, MED
C. Sigmoid colon may migrate into the abdominal cavity where the BOARDS;
D. Ureter bladder may also be perforated. TOPNOTCH
E. Broad ligament MD)

453 A 19 year-old male suffered from a stab wound at the The segment that is seen protruding the patient's JULIET MIDTERM 2
anterior abdomen and was rushed to the ER. A segment of abdomen is the ileum which is in the lower part of KRISTINE EXAM - AUG
the intestine protruded and partially opened. The segment's the cavity and in the pelvis. The ileum receives EVANGELIST 2014
mucosa had few circular folds. It's mesentery was fatty with numerous short terminal vessels that form many A, MD (TOP 9
many vascular arcades. The segment that is seen protruding vascular arcades. At the ileal end of the mesentery, - FEB 2014
the patient's abdomen is: the fat is deposited throughout. MED
A. duodenal bulb BOARDS;
B. distal part of duodenum TOPNOTCH
C. proximal jejunum MD)
D. distal jejunum
E. distal ileum

454 A 12 year-old boy was brought to the ER because of acute During an asthmatic event the muscles JULIET MIDTERM 2
attack of Bronchial Asthma. Symptoms primarily are caused surrounding the bronchioles constrict. The KRISTINE EXAM - AUG
by mucosal inflammation and airway hyperresponsiveness. wheezing sound is caused by the contraction of EVANGELIST 2014
The specific part of the airway involved in the pathogenesis the bronchioles as the air passes through tubes A, MD (TOP 9
of Asthma is: that are almost completely blocked. - FEB 2014
A. trachea MED
B. bronchi BOARDS;
C. bronchioles TOPNOTCH
D. alveolar ducts MD)
E. alveolar sacs

455 A 48 year-old fish vendor was stabbed by an unknown Macula densa cells sense changes in sodium JULIET MIDTERM 2
assailant at the back. He was rushed to the ER with BP 60 chloride level, and will trigger an autoregulatory KRISTINE EXAM - AUG
palpatory, PR=130s, RR=20s. Which particular structure of response to increase or decrease reabsorption of EVANGELIST 2014
the kidney will be stimulated as a result of the above ions and water to the blood in order to alter blood A, MD (TOP 9
findings: volume and return blood pressure to normal. - FEB 2014
A. juxtaglomerular cells Decreased BP is detected initially by Macula MED
B. cells of PCT Densa which results to increased renin release BOARDS;
C. cells of loop of Henle from JG cells. TOPNOTCH
D. macula densa MD)
E. mesangial cells

456 Direct inguinal hernias are found within the Hesselbach's Direct inguinal hernia protrudes through the JULIET MIDTERM 2
triangle. The posterior wall of this triangle where hernias inguinal triangle of Hesselbach that lies between KRISTINE EXAM - AUG
protrude is: the inferior epigastric artery superolaterally, the EVANGELIST 2014
A. Inguinal ligament rectus abdominis medially and the inguinal A, MD (TOP 9
B. External oblique aponeurosis ligament inferiorly. The posterior wall of this - FEB 2014
C. Transversus abdominis aponeurosis triangle where hernias protrude is formed by the MED
D. Internal oblique aponeurosis transversalis fascia. The transversalis fascia forms BOARDS;
E. Transversalis fascia an investing fascial envelope of the abdominal TOPNOTCH
cavity. MD)

TOPNOTCH MEDICAL BOARD PREP ANATOMY SUPEREXAM Page 59 of 94


For inquiries visit www.topnotchboardprep.com.ph or email us at topnotchmedicalboardprep@gmail.com
TOPNOTCH MEDICAL BOARD PREP ANATOMY SUPEREXAM
For inquiries visit www.topnotchboardprep.com.ph or email us at topnotchmedicalboardprep@gmail.com
Item QUESTION EXPLANATION AUTHOR TOPNOTCH
# EXAM
457 A 35 year-old woman has an infected right big toe with A patient may present with an enlarged, painful JULIET MIDTERM 2
swollen lymph nodes. The group of node that is most likely superficial inguinal lymph node that is due to KRISTINE EXAM - AUG
affected is: lymphatic spread of pathogenic organisms that EVANGELIST 2014
A. Femoral entered the body through the legs and big toes. A, MD (TOP 9
B. Superficial inguinal Femoral lymph nodes are ocated in the upper - FEB 2014
C. Deep inguinal thigh portion along the femoral veins, immediately MED
D. Internal iliac below the inguinal lymph nodes which drain from BOARDS;
E. Paravertebral some of the genital parts, buttock, thigh and the TOPNOTCH
medial side of the leg. MD)

458 A 46 year-old woman had crow's feet at lateral angles of her Contraction of the orbicularis oculi muscle is JULIET MIDTERM 2
eyelids. These are due to contraction of: primarily responsible for the clinically observed KRISTINE EXAM - AUG
A. Procerus periorbital crow's feet. The orbicularis oculi EVANGELIST 2014
B. Corrugator supercilli muscle is bordered superolaterally by fibers of the A, MD (TOP 9
C. Frontalis frontalis muscle and medially by the levator - FEB 2014
D. Orbicularis oculi palpebrae muscle. The frontalis muscle and the MED
E. Levator palpebrae superioris corrugator muscle are responsible for the BOARDS;
appearance of wrinkles and vertical frown lines TOPNOTCH
accordingly. The procerus muscle, which overlies MD)
the nasal root, is responsible for a snout-nose
appearance and horizontal frown lines.
459 A 35 year-old man complains of inability to flex the elbow Musculocutaneous nerve supplies the flexor of the JULIET MIDTERM 2
joint. It involves a nerve supplying the muscle which arm which branches from the brachial plexus as KRISTINE EXAM - AUG
branches from the brachial plexus as: C5-C7. C5- Dorsal scapular Nerve, C5-C6- EVANGELIST 2014
A. C5 suprascapular nerve, C6,C7,C8-thoracodorsal A, MD (TOP 9
B. C5-C6 nerve, C8-T1-Ulnar nerve and medial root of - FEB 2014
C. C5-C7 median. MED
D. C6, C7, C8 BOARDS;
E. C8-T1 TOPNOTCH
MD)

460 Pulsation from an artery which is a continuation of anterior Dorsalis pedis artery is a continuation of anterior JULIET MIDTERM 2
tibial artery on the dorsum of the foot can be easily felt. On tibial artery. Medial to this lies the tendon of KRISTINE EXAM - AUG
the medial side of this artery lies the tendon of: extensor hallucis longus. On the lateral side lies EVANGELIST 2014
A. Extensor digitorum brevis the terminal part of the deep peroneal nerve and A, MD (TOP 9
B. Extensor digitorum longus extensor digitorum longus tendons. - FEB 2014
C. Extensor hallusis brevis MED
D. Extensor hallucis longus BOARDS;
E. Inferior extensor retinaculum TOPNOTCH
MD)

461 What is the vein that is used in arterio-venous fistula that is cephalic vein is lateral to the biceps brachii, while LUISA BACK-UP
located lateral to the biceps brachii? basilic vein is medial to the biceps brachii. SARANILLO, MIDTERM
A. Basilic vein MD (TOP 6 - EXAM AUG
B. brachial vein FEB 2014 2014 - FOR
C. Median cubital vein MED INCLUSION
D. Cephalic vein BOARDS; IN THE
E. Axillary vein TOPNOTCH SAMPLEX
MD)

462 In the standard 12 lead ECG what is the anatomic placement 4th ICS Right parasternal border = V1: 4th ICS Left LUISA BACK-UP
of the chest electrode lead V2? parasternal border = V2; 5th ICS Left MCL = V4; SARANILLO, MIDTERM
A. 4th ICS Right parasternal border 5th ICS Right MCL= none MD (TOP 6 - EXAM AUG
B. 4th ICS Left parasternal border FEB 2014 2014 - FOR
C. 5th ICS Right MCL MED INCLUSION
D. 5th ICS Left MCL BOARDS; IN THE
E. None of the choices TOPNOTCH SAMPLEX
MD)

463 A 55 year old male presents with chest pain. ECG shows ST Leads II, III, AVF refers the inferior wall, which is LUISA BACK-UP
segment elevation in lead II, III, AVF. What is the most likely supplied by the Right coronary artery. Septal, SARANILLO, MIDTERM
affected blood vessel supplying the affected part of the anterior, and lateral wall are supplied by the left MD (TOP 6 - EXAM AUG
heart? coronary artery. FEB 2014 2014 - FOR
A. Right coronary artery MED INCLUSION
B. Left marginal artery BOARDS; IN THE
C. left anterior descending artery TOPNOTCH SAMPLEX
D. Left circumflex artery MD)
E. C and D

464 What is the most common type of myoma? Intramural is the most common type, while LUISA BACK-UP
A. Submucosal myoma submucosal is the one frequently associated with SARANILLO, MIDTERM
B. Pedunculated subserosal myoma heavy, prolonged bleeding. MD (TOP 6 - EXAM AUG
C. Subserosal myoma FEB 2014 2014 - FOR
D. Intramural myoma MED INCLUSION
E. Pedunculated submucosal myoma BOARDS; IN THE
TOPNOTCH SAMPLEX
MD)
465 The following are muscles of inspiration except: Internal intercostal depresses the ribs during LUISA BACK-UP
A. Serratus posterior superior exhalation. Other choices elevate the ribs during SARANILLO, MIDTERM
B. External intercostal inspiration. MD (TOP 6 - EXAM AUG
C. Internal intercostal FEB 2014 2014 - FOR
D. Innermost intercostal MED INCLUSION
E. Subcostal BOARDS; IN THE
TOPNOTCH SAMPLEX
MD)
TOPNOTCH MEDICAL BOARD PREP ANATOMY SUPEREXAM Page 60 of 94
For inquiries visit www.topnotchboardprep.com.ph or email us at topnotchmedicalboardprep@gmail.com
TOPNOTCH MEDICAL BOARD PREP ANATOMY SUPEREXAM
For inquiries visit www.topnotchboardprep.com.ph or email us at topnotchmedicalboardprep@gmail.com
Item QUESTION EXPLANATION AUTHOR TOPNOTCH
# EXAM
466 A 25 year old male came in due a stab wound in his The muscular tube from the kidney to the LUISA BACK-UP
abdomen. Upon exploration you noticed a severed blood posterior surface of the bladder is the ureter. It SARANILLO, MIDTERM
vessel supplying the lower part of the the muscular tube has 3 blood supplies: upper part is supplied by MD (TOP 6 - EXAM AUG
from the kidney to the posterior surface of the bladder. What renal artery, the middle part is supplied by the FEB 2014 2014 - FOR
blood vessel is most likely involved? gonadal artery, the lower part is supplied by the MED INCLUSION
A. renal artery superior vesical artery. BOARDS; IN THE
B. testicular artery TOPNOTCH SAMPLEX
C. superior vesical artery MD)
D. iliolumbar artery
E. inferior vesical artery

467 A 65 year old female fall from a height of approximately 10 Fracture of the femoral neck will present with LUISA BACK-UP
step ladder sustaining injuries. Her right leg is shortened and shortened leg and laterally rotated thigh while SARANILLO, MIDTERM
the thigh is laterally rotated. What is the most likely dislocation of the femoral head will present with MD (TOP 6 - EXAM AUG
diagnosis. shortened leg and medially rotated thigh. FEB 2014 2014 - FOR
A. Fracture of the neck of the femur, right MED INCLUSION
B. fracture of the tibia, right BOARDS; IN THE
C. dislocation of the head of the femur, right TOPNOTCH SAMPLEX
D. fracture of the fibula, right MD)
E. none of the choices

468 A surgeon harvested part of the great saphenous vein from Saphenous nerve supplies the skin of the medial LUISA BACK-UP
the right lower extremity of a 30 year old patient. However, aspect of the leg and foot. Obturator nerve SARANILLO, MIDTERM
after the procedure, the patient complained of pain and supplies the skin of the medial thigh. Femoral MD (TOP 6 - EXAM AUG
paresthesia in the medial aspect of the leg and foot. What nerve supplies the antero-medial aspect of the FEB 2014 2014 - FOR
nerve is the most likely affected. thigh. Sciatic nerve supplies the posterior thigh, MED INCLUSION
A. saphenous nerve foot and leg. BOARDS; IN THE
B. obturator nerve TOPNOTCH SAMPLEX
C. femoral nerve MD)
D. sciatic nerve
E. A and B

469 A 6 month old infant has a long and narrow skull with frontal Premature closure of the sagittal suture called LUISA BACK-UP
and occipital expansion was brought in to your clinic by her scaphocephaly presents with long and narrow SARANILLO, MIDTERM
mother for consultation. You explained to the mother that skull with frontal and occipital expansion; MD (TOP 6 - EXAM AUG
the most likely reason for this is the premature closure of the involvement of the coronal suture forms a tower FEB 2014 2014 - FOR
cranial sutures. What suture is involved in this case? skull - a short high skull; premature closure of the MED INCLUSION
A. sagittal suture coronal and lambdoid suture on one side of the BOARDS; IN THE
B. coronal suture skull called plagiocephaly presents with TOPNOTCH SAMPLEX
C. lambdoid suture assymetric skull. MD)
D. coronal and lambdoid suture
E. coronal and sagittal suture

470 The following cranial nerves are both motor and sensory Mixed cranial nerves are vagus, glossopharyngeal, LUISA BACK-UP
except: facial, and trigeminal nerve. Vestibulocochlear SARANILLO, MIDTERM
A. Glossopharyngeal nerve is purely sensory nerve. MD (TOP 6 - EXAM AUG
B. Vagus FEB 2014 2014 - FOR
C. facial MED INCLUSION
D. vestibulocochlear BOARDS; IN THE
E. No exception TOPNOTCH SAMPLEX
MD)

471 A 35 year old male sustained facial laceration due to Facial lacerations tend to gape because the face LUISA BACK-UP
vehicular accident. The wound tend to gape because: has no deep fascia and the subcutaneous tissue is SARANILLO, MIDTERM
A. The face has no deep fascia loose. MD (TOP 6 - EXAM AUG
B. The subcutaneous tissue of the face is loose FEB 2014 2014 - FOR
C. The face has weak muscles MED INCLUSION
D. A and B only BOARDS; IN THE
E. All of the choices TOPNOTCH SAMPLEX
MD)

472 The anal canal is divided into 2 parts by the dentate line. LUISA BACK-UP
What is the lining epithelium of the anal canal above the SARANILLO, MIDTERM
dentate line? MD (TOP 6 - EXAM AUG
A. Simple columnar epithelium FEB 2014 2014 - FOR
B. Simple cuboidal epithelium MED INCLUSION
C. Stratified squamous epithelium BOARDS; IN THE
D. transitional epithelium TOPNOTCH SAMPLEX
E. simple squamous epithelium MD)

TOPNOTCH MEDICAL BOARD PREP ANATOMY SUPEREXAM Page 61 of 94


For inquiries visit www.topnotchboardprep.com.ph or email us at topnotchmedicalboardprep@gmail.com
TOPNOTCH MEDICAL BOARD PREP ANATOMY SUPEREXAM
For inquiries visit www.topnotchboardprep.com.ph or email us at topnotchmedicalboardprep@gmail.com
Item QUESTION EXPLANATION AUTHOR TOPNOTCH
# EXAM
473 A 35 year old female singer underwent a subtotal Superior laryngeal nerve supplies the cricothyroid LUISA BACK-UP
thyroidectomy due to a thyroid mass. After the surgery, she muscle which act as a tensor. Inferior laryngeal SARANILLO, MIDTERM
experienced mild hoarseness of voice and she can not reach nerve supplies the rest of the laryngeal muscles. MD (TOP 6 - EXAM AUG
high notes. What nerve is most likely injured? FEB 2014 2014 - FOR
A. lateral laryngeal nerve MED INCLUSION
B. superior laryngeal nerve BOARDS; IN THE
C. inferior laryngeal nerve TOPNOTCH SAMPLEX
D. phrenic nerve MD)
E. glossopharyngeal nerve

474 In pericardiocentesis, pericardial fluid is apirated between Accumulated pericardial fluid is aspirated LUISA BACK-UP
what layers of the heart? between the parietal layer of the serous SARANILLO, MIDTERM
A. Between fibrous and serous pericardium pericardium and the visceral layer of the MD (TOP 6 - EXAM AUG
B. Between the parietal layer of the serous pericardium and pericardium. FEB 2014 2014 - FOR
the fibrous pericardium MED INCLUSION
C. Between the myocardium and the epicardium BOARDS; IN THE
D. between the parietal layer of the serous pericardium and TOPNOTCH SAMPLEX
the visceral layer of the serous pericardium MD)
E. none of the choices

475 When inserting an NGT, the first esophageal narrowing From the external nares the following are the LUISA BACK-UP
encountered is between the esophagus and the cricoid approximated distance: up to the first esophageal SARANILLO, MIDTERM
cartilage. What is the distance from the external nares up to narrowing which is between the esophagus and MD (TOP 6 - EXAM AUG
this point. the cricoid cartilage is 18cm; up to the 2nd FEB 2014 2014 - FOR
A. 30cm esophageal narrowing between the esophagus and MED INCLUSION
B. 28cm the arch of aorta is 28cm; up to the cardiac orifice BOARDS; IN THE
C. 44 cm of the stomach is 44cm. An additional 12-14cm is TOPNOTCH SAMPLEX
D. 12-14cm added when pushed up to the pylorus. MD)
E. 18cm

476 The blood supply of the appendix comes directly from: LUISA BACK-UP
A. Posterior cecal artery SARANILLO, MIDTERM
B. Ileocolic artery MD (TOP 6 - EXAM AUG
C. Right middle colic artery FEB 2014 2014 - FOR
D. Left middle colic artery MED INCLUSION
E. Jejunoileal artery BOARDS; IN THE
TOPNOTCH SAMPLEX
MD)
477 Which of the following best characterized the parathyroid Usually, there are 4 parathyroid glands. Inferior LUISA BACK-UP
glands? thyroid artery supplies both the superior and SARANILLO, MIDTERM
A. The inferior parathyroid glands are more variable in inferior parathyroid glands. The superior glands MD (TOP 6 - EXAM AUG
location are constant in location, while the inferior glands FEB 2014 2014 - FOR
B. Inferior thyroid artery supplies the inferior parathyroid are more variable in location. MED INCLUSION
gland only BOARDS; IN THE
C. Usually, there are 4 parathyroid glands TOPNOTCH SAMPLEX
D. All of the above MD)
E. A and C only

478 An 18 year old boy had anterior dislocation of the shoulder axillary nerve is injured in anterior dislocation of LUISA BACK-UP
joint while playing basketball. An injury to this nerve is the shoulder joint. It will presents with inability to SARANILLO, MIDTERM
suspected which will present as: abduct the arm, altered sensation in the lateral MD (TOP 6 - EXAM AUG
A. Inability to abduct the arm arm, and weakness in lateral rotation of the arm. FEB 2014 2014 - FOR
B. Inability to adduct the arm MED INCLUSION
C. Weakness in medial rotation of the arm BOARDS; IN THE
D. altered sensation in the medial aspect of the arm TOPNOTCH SAMPLEX
E. all of the above MD)

479 The "yes" and "no" joints are synovial joints without The "yes" joint is the atlanto-occipital joint located LUISA BACK-UP
intervertebral disc. The "yes" joint is located between: between the atlas and occipital condyle, while the SARANILLO, MIDTERM
A. Atlas and occipital condyle "no" joint is the atlanto-axial joint which is MD (TOP 6 - EXAM AUG
B. Atlas and axis between atlas and axis. FEB 2014 2014 - FOR
C. Axis and occipital condyle MED INCLUSION
D. C3 and C4 BOARDS; IN THE
E. None of the choices TOPNOTCH SAMPLEX
MD)

480 What is the anatomic landmark of the cricoid cartilage? C4 =hyoid bone and Common carotid artery LUISA BACK-UP
A. C4 bifurcation. C5= thyroid cartilage. C6 = cricoid SARANILLO, MIDTERM
B. C5 cartilage, start of trachea and esophagus. T2 = MD (TOP 6 - EXAM AUG
C. C6 sternal notch. FEB 2014 2014 - FOR
D. T2 MED INCLUSION
E. C7 BOARDS; IN THE
TOPNOTCH SAMPLEX
MD)

TOPNOTCH MEDICAL BOARD PREP ANATOMY SUPEREXAM Page 62 of 94


For inquiries visit www.topnotchboardprep.com.ph or email us at topnotchmedicalboardprep@gmail.com
TOPNOTCH MEDICAL BOARD PREP ANATOMY SUPEREXAM
For inquiries visit www.topnotchboardprep.com.ph or email us at topnotchmedicalboardprep@gmail.com
Item QUESTION EXPLANATION AUTHOR TOPNOTCH
# EXAM
481 Which of the following cranial nerves will NOT be affected in CN III, IV, V1, V2 and VI and postganglionic fibers ANGELIS FINAL EXAM
cavernous sinus syndrome? en route to the orbit all pass through the ANDREA - AUG 2014
A. CN II cavernous sinus. COCOS, MD
B. CN III (TOP 1 - FEB
C. CN IV 2014 MED
D. CN VI BOARDS;
TOPNOTCH
MD)
482 This type of glia is important for the maintenance of the The astrocyte is important for physical support, ANGELIS FINAL EXAM
blood-brain barrier: repair, K metabolism, removal of excess ANDREA - AUG 2014
A. oligodendroglia neurotransmitters and maintenance of blood- COCOS, MD
B. astrocyte brain barrier. (TOP 1 - FEB
C. microglia 2014 MED
D. Schwann cell BOARDS;
TOPNOTCH
MD)
483 The V4 precordial lead is placed: Several questions on ECG lead placement were ANGELIS FINAL EXAM
A. On the 4th ICS to the left of the sternum asked during our board exam. ANDREA - AUG 2014
B. On the 5th ICS to the left of the sternum COCOS, MD
C. On the 5th ICS at the left midclavicular line (TOP 1 - FEB
D. On the 4th ICS at the left midclavicular line 2014 MED
BOARDS;
TOPNOTCH
MD)
484 This is the most common site of ureteral obstruction: The rest are common sites of obstruction as well. ANGELIS FINAL EXAM
A. Ureteropelvic junction ANDREA - AUG 2014
B. Ureterovesical junction COCOS, MD
C. Pelvic brim (TOP 1 - FEB
D. None of the above 2014 MED
BOARDS;
TOPNOTCH
MD)
485 Gastrulation happens within week: Gastrulation is the phase during which the single- ANGELIS FINAL EXAM
A. 1 layered blastula is reorganized into a trilaminar ANDREA - AUG 2014
B. 2 disc. It happens during week 3. COCOS, MD
C. 3 (TOP 1 - FEB
D. 4 2014 MED
BOARDS;
TOPNOTCH
MD)
486 The following are embryologic derivatives of ectoderm, Eustachian tube is an endodermal derivative. ANGELIS FINAL EXAM
EXCEPT: ANDREA - AUG 2014
A. Parotid gland COCOS, MD
B. Retina (TOP 1 - FEB
C. Melanocytes 2014 MED
D. Eustachian tube BOARDS;
TOPNOTCH
MD)
487 What is the function of Type 2 pneumocytes? Type 2 cells are also the ones which secrete the ANGELIS FINAL EXAM
A. For gas exchange pulmonary surfactant. ANDREA - AUG 2014
B. Degrade toxins COCOS, MD
C. Secrete IgA (TOP 1 - FEB
D. Precursors to type I pneumocytes 2014 MED
BOARDS;
TOPNOTCH
MD)
488 A young fetus of 2 months AOG would be expected to be Erythropoiesis happens in: yolksack for the first 2 ANGELIS FINAL EXAM
producing RBCs via the: months AOG, liver and spleen around 3 months, ANDREA - AUG 2014
A. Yolk sac and bone marrow predominantly from 5 months COCOS, MD
B. Liver up to birth. (TOP 1 - FEB
C. Bone marrow 2014 MED
D. Thymus BOARDS;
TOPNOTCH
MD)
489 The left circumflex coronary artery supplies the: SIMILAR TO PREVIOUS BOARD EXAM ANGELIS FINAL EXAM
A. Anterior 2/3 of interventricular septum CONCEPT/PRINCIPLE. LCX supplies the lateral ANDREA - AUG 2014
B. Posterior walls of the left ventricle and posterior walls of the left ventricle. COCOS, MD
C. Posterior 1/3 of interventricular septum (TOP 1 - FEB
D. Lateral wall of the right ventricle 2014 MED
BOARDS;
TOPNOTCH
MD)
490 Which of the following hormones is secreted by an acidophil? Acidophils secrete GH and prolactin. ANGELIS FINAL EXAM
A. FSH ANDREA - AUG 2014
B. GH COCOS, MD
C. TSH (TOP 1 - FEB
D. LH 2014 MED
BOARDS;
TOPNOTCH
MD)

TOPNOTCH MEDICAL BOARD PREP ANATOMY SUPEREXAM Page 63 of 94


For inquiries visit www.topnotchboardprep.com.ph or email us at topnotchmedicalboardprep@gmail.com
TOPNOTCH MEDICAL BOARD PREP ANATOMY SUPEREXAM
For inquiries visit www.topnotchboardprep.com.ph or email us at topnotchmedicalboardprep@gmail.com
Item QUESTION EXPLANATION AUTHOR TOPNOTCH
# EXAM
491 This ligament contains the portal triad: The hepatoduodenal ligament connects the liver to ANGELIS FINAL EXAM
A. gastrohepatic the duodenum. It contains the portal triad: hepatic ANDREA - AUG 2014
B. hepatosplenic artery, portal vein, and common bile duct. COCOS, MD
C. hepatoduodenal (TOP 1 - FEB
D. falciform 2014 MED
BOARDS;
TOPNOTCH
MD)
492 This/these segment/s of the small intestine contain/s the Brunner's glands are found in the submucosa of ANGELIS FINAL EXAM
Brunner's glands: the duodenum. Several histology questions were ANDREA - AUG 2014
A. duodenum asked during our board exam. COCOS, MD
B. jejunum (TOP 1 - FEB
C. ileum 2014 MED
D. Both A and B BOARDS;
TOPNOTCH
MD)
493 On peripheral blood smear, which cell presents with a large SIMILAR TO PREVIOUS BOARD EXAM ANGELIS FINAL EXAM
kidney-shaped nucleus? CONCEPT/PRINCIPLE. ANDREA - AUG 2014
A. Eosinophil COCOS, MD
B. Monocyte (TOP 1 - FEB
C. B cell 2014 MED
D. T cell BOARDS;
TOPNOTCH
MD)
494 Melanocytes are found in the: Stratum basale is sometimes referred to as ANGELIS FINAL EXAM
A. Stratum basale stratum germinativum, the deepest layer of the ANDREA - AUG 2014
B. Stratum germinativum epidermis. SIMILAR TO PREVIOUS BOARD EXAM COCOS, MD
C. Statum granulosum CONCEPT/PRINCIPLE. (TOP 1 - FEB
D. Both A and B 2014 MED
BOARDS;
TOPNOTCH
MD)
495 The most commonly injured rotator cuff muscle is: Teres major is not part of the rotator cuff muscles. ANGELIS FINAL EXAM
A. Teres minor ANDREA - AUG 2014
B. Teres major COCOS, MD
C. Supraspinatus (TOP 1 - FEB
D. Infraspinatus 2014 MED
BOARDS;
TOPNOTCH
MD)
496 A patient needs a femoral tap for ABG determination. What The acronym from lateral to medial is NAVEL: ANGELIS FINAL EXAM
compartment of the femoral sheath should you aspirate? nerve, artery, vein, empty space and lymphatics. ANDREA - AUG 2014
A. medial The nerve is not included in the femoral sheath. COCOS, MD
B. lateral (TOP 1 - FEB
C. posterior 2014 MED
D. anterior BOARDS;
TOPNOTCH
MD)
497 A woman suffered a traumatic injury to the upper limb which Clawing of the ring and little fingers is a keyphrase ANGELIS FINAL EXAM
results to an inability to spread and extend her fingers, with for lower segment brachial plexus injury. A ANDREA - AUG 2014
noted clawing of the ring and little fingers. Which segments question on brachial plexus was asked during our COCOS, MD
of the brachial plexus would have contributed to the nerve exam. (TOP 1 - FEB
that is damaged? 2014 MED
A. C5 and C6 BOARDS;
B. C6 and C7 TOPNOTCH
C. C7 and C8 MD)
D. C8 and T1

498 A patient suffers an Achilles tendon rupture. Which of the The popliteal tendon does not contribute to the ANGELIS FINAL EXAM
following muscles is NOT affected: formation of the calcaneal or Achilles tendon. ANDREA - AUG 2014
A. soleus COCOS, MD
B. plantaris (TOP 1 - FEB
C. popliteus 2014 MED
D. gastrocnemius BOARDS;
TOPNOTCH
MD)
499 During parotidectomy, the main trunk of the facial nerve was Buccinator is innervated by the facial nerve. The ANGELIS FINAL EXAM
accidentally lacerated. Which of the following muscles will be rest of the choices are innervated by the ANDREA - AUG 2014
affected? trigeminal nerve, specifically V3. COCOS, MD
A. masseter (TOP 1 - FEB
B. buccinator 2014 MED
C. temporalis BOARDS;
D. pterygoids TOPNOTCH
MD)

500 The nasolacrimal duct drains into which recess? SIMILAR TO PREVIOUS BOARD EXAM ANGELIS FINAL EXAM
A. Superior meatus CONCEPT/PRINCIPLE. There is no lateral meatus. ANDREA - AUG 2014
B. Middle meatus COCOS, MD
C. Inferior meatus (TOP 1 - FEB
D. Lateral meatus 2014 MED
BOARDS;
TOPNOTCH
MD)

TOPNOTCH MEDICAL BOARD PREP ANATOMY SUPEREXAM Page 64 of 94


For inquiries visit www.topnotchboardprep.com.ph or email us at topnotchmedicalboardprep@gmail.com
TOPNOTCH MEDICAL BOARD PREP ANATOMY SUPEREXAM
For inquiries visit www.topnotchboardprep.com.ph or email us at topnotchmedicalboardprep@gmail.com
Item QUESTION EXPLANATION AUTHOR TOPNOTCH
# EXAM
501 This nuclei is the most important source of norepinephrine SIMILAR TO PREVIOUS BOARD EXAM JAN BACK-UP
in the cerebral cortex: CONCEPT/PRINCIPLE.
CHARMAINE MIDTERM
A. Caudate nucleus Caudate nucleus-GABA PALOMAR, EXAM AUG
B. Substantia nigra pars compacta Substantia nigra pars compacta-dopamine MD (TOP 9 - 2014
C. Raphe nucleus Raphe nucleus-serotonin FEB 2014
D. Locus ceruleus Locus ceruleus-norepinephrine MED
E. Basal nucleus of Meynert Basal nucleus of Meynert-acetylcholine BOARDS;
TOPNOTCH
MD)
502 A 30-year-old man was allegedly stabbed in his right chest by SIMILAR TO PREVIOUS BOARD EXAM JAN BACK-UP
an unknown assailant as he was going to his car parked 2 CONCEPT/PRINCIPLE. The long thoracic nerve CHARMAINE MIDTERM
blocks away from the bar where he sings.On physical innervates the serratus anterior muscle which is PALOMAR, EXAM AUG
examination, patient is unable to raise his right arm above responsible for drawing the scapula forward MD (TOP 9 - 2014
the horizontal with winging of his right scapula. Which of the around the thoracic wall. A winged scapula is FEB 2014
following nerves is most likely affected? caused by paralysis of the serratus anterior MED
A. Axillary muscle.Snells 8th ed., 441, 434. BOARDS;
B. Thoracodorsal TOPNOTCH
C. Long thoracic MD)
D. Internal intercosta
E. Spinal accesory

503 This is also known as the morrison's pouch: SIMILAR TO PREVIOUS BOARD EXAM JAN BACK-UP
A. Hepatorenal recess CONCEPT/PRINCIPLE. Morrison's pouch- CHARMAINE MIDTERM
B. Splenorenal recess hepatorenal recess; the most posterior cavity in PALOMAR, EXAM AUG
C. Renocolic recess the peritoneal cavity. -Medical eponyms by MD (TOP 9 - 2014
D. Costodiagphragmatic recess Andrew J. Yee, MD, 2007 FEB 2014
MED
BOARDS;
TOPNOTCH
MD)
504 The following muscles are responsible for dorsiflexing the SIMILAR TO PREVIOUS BOARD EXAM JAN BACK-UP
foot, except: CONCEPT/PRINCIPLE. Anterior leg muscles- CHARMAINE MIDTERM
A. Extensor digitorum brevis action: dorsiflexion and extension, nerve: deep PALOMAR, EXAM AUG
B. Tibialis anterior peroneal nerve, muscles: tibialis anterior, extensor MD (TOP 9 - 2014
C. Peroneus longus digitorum longus, peroneus tertius, extensor FEB 2014
D. Extencsor digitorum longus hallucis longus, extensor digitorum brevis. - MED
E. None of the above Topnotch handouts. BOARDS;
Choice C, peroneous longus is found in the lateral TOPNOTCH
compartment of the leg and is responsible for MD)
plantar flexion and evertion
505 The gastric mucosa is lined by which epithelium: SIMILAR TO PREVIOUS BOARD EXAM JAN BACK-UP
A. Simple squamous CONCEPT/PRINCIPLE. -Junquiera and Carneiro, CHARMAINE MIDTERM
B. Simple cuboidal Basic Histology 11th ed., p. 290. PALOMAR, EXAM AUG
C. Simple columnar MD (TOP 9 - 2014
D. Stratified squamous FEB 2014
E. Stratified columnar MED
BOARDS;
TOPNOTCH
MD)
506 Melanocytes are found in which layer of the skin: SIMILAR TO PREVIOUS BOARD EXAM JAN BACK-UP
A. Stratum germinativum CONCEPT/PRINCIPLE. Eumelanin is a dark brown CHARMAINE MIDTERM
B. Stratum granulosum pigment produced by the melanocyte, a PALOMAR, EXAM AUG
C. Stratum corneum specialized cell of the epidermis found beneath or MD (TOP 9 - 2014
D. Stratum basale between the cells of the stratum basale and the FEB 2014
E. Stratum spinosum hair follicles. Melanocytes have rounded bodies MED
from which long irregular extensions branch into BOARDS;
the epidermis, running between the cells of the TOPNOTCH
strata basale and spinosum. -Junquiera and MD)
Carneiro, Basic Histology 11th ed., p. 363
507 The major mechanism of blindness in glaucoma is: SIMILAR TO PREVIOUS BOARD EXAM JAN BACK-UP
A. Optic nerve damage CONCEPT/PRINCIPLE. Vaughan and Ausbury's CHARMAINE MIDTERM
B. Increased intraocular pressure General Ophthalmology 17th ed., p. 214-215. PALOMAR, EXAM AUG
C. Optic disk enlargement MD (TOP 9 - 2014
D. Corneal inflammation FEB 2014
E. Lens opacification MED
BOARDS;
TOPNOTCH
MD)
508 Which of the folling statements regarding tube thoracostomy B. The skin incision is usually made over the JAN BACK-UP
is true? A. intercostal space below the space to be pierced. CHARMAINE MIDTERM
The site of insertion of the tube is at the 4th ICS at the C. The incision through the intercostal space is PALOMAR, EXAM AUG
anterior axillary line. kept close to the upper border of the rib to avoid MD (TOP 9 - 2014
B. The skin incision is usually made over the intercostal injuring the intercostal vessels and nerve. Clinical FEB 2014
space above the space to be pierced. Anatomy by Regions, Snell, 8th ed., 59. MED
C. The incision through the intercostal space is kept close to BOARDS;
the lower border of the rib to avoid injuring the intercostal TOPNOTCH
vessels and nerve. MD)
D. All of the statement are correct.
E. All of the statements are incorrect.

TOPNOTCH MEDICAL BOARD PREP ANATOMY SUPEREXAM Page 65 of 94


For inquiries visit www.topnotchboardprep.com.ph or email us at topnotchmedicalboardprep@gmail.com
TOPNOTCH MEDICAL BOARD PREP ANATOMY SUPEREXAM
For inquiries visit www.topnotchboardprep.com.ph or email us at topnotchmedicalboardprep@gmail.com
Item QUESTION EXPLANATION AUTHOR TOPNOTCH
# EXAM
509 C-shaped organ: SIMILAR TO PREVIOUS BOARD EXAM JAN BACK-UP
A. Esophagus CONCEPT/PRINCIPLE CHARMAINE MIDTERM
B. Duodenum PALOMAR, EXAM AUG
C. Spleen MD (TOP 9 - 2014
D. Pancreas FEB 2014
E. Rectum MED
BOARDS;
TOPNOTCH
MD)
510 Which of the following statements is true regarding the right SIMILAR TO PREVIOUS BOARD EXAM JAN BACK-UP
recurrent laryngeal nerve? CONCEPT/PRINCIPLE. Snell 8th ed., p. 127. The CHARMAINE MIDTERM
A. It hooks around the ligamentum arteriosum and ascends Right recurrent laryngeal nerve arises from the PALOMAR, EXAM AUG
in the groove between the trachea and the esophagus on the right vagus in the neck and hooks around the MD (TOP 9 - 2014
right side. subclavian artery and ascends between the FEB 2014
B. It supplies the cricothyroid muscle which is responsible trachea and esophagus. The left recurrent MED
for tensing the vocal cord. laryngeal nerve arises from the leftvagus trunk as BOARDS;
C. It branches to become the right vagus. the nerve crosses the arch of the aorta. it hooks TOPNOTCH
D. It loops around the right subclavian artery and ascends around the ligamentum arteriosum and ascends in MD)
between the trachea and esophagus. the groove between the trachea and the esophagus
E. All of the statements are true. on the on the left side. it suplies all the muscles
acting on the left vocal cord (except the
cricothyroid muscle, a tensor of the vocal cord,
which is supplied by the external laryngeal branch
of the vagus.
511 Dorsalis pedis pulse can be appreciated: Snell 8th ed., 658 JAN BACK-UP
A. Between the the tendons of flexor digitorum longus and CHARMAINE MIDTERM
flexor hallucis longus PALOMAR, EXAM AUG
B. Between the tendons of extensor hallucis longus and MD (TOP 9 - 2014
extensor digitorum longus FEB 2014
C. Between the anterosuperior iliac spine and the MED
symphysis pubis BOARDS;
D. Midway between the medial and lateral maleoli on the TOPNOTCH
front of the ankle MD)
E. Both B and D are correct

512 A 23 year-old-man was rushed to the hospital after Topnotch handout


JAN BACK-UP
sustaining multiple physical injuries after jumping from a Humeral fracture- and associated nerve injury
CHARMAINE MIDTERM
moving bus. X-ray of the humerus revealed midshaft 1. Surgical neck - axillary nerve
PALOMAR, EXAM AUG
fracture. Which of the following nerves would most likely be 2. midshaft / spiral groove fracture - radial nerve
MD (TOP 9 - 2014
injured? 3. Supracondylar - Median nerve
FEB 2014
A. Ulnar 4. Medial epicondyle - Ulnar nerve MED
B. Radial BOARDS;
C. Median TOPNOTCH
D. Musculocutaneous MD)
E. Axillary

513 SIMILAR TO PREVIOUS BOARD EXAM JAN BACK-UP


CONCEPT/PRINCIPLE CHARMAINE MIDTERM
PALOMAR, EXAM AUG
ECG of a 74 year old-male complaining of chest heaviness MD (TOP 9 - 2014
revealed ST-segment elevation in leads II, III and aVF. This FEB 2014
signifies? MED
A. Inferior wall myocaridal infarction BOARDS;
B. Inferior wall myocaridal ischemia TOPNOTCH
C. Anterolateral wall infarction MD)
D. Anterolateral wall ischemia
E. None of the above
514 An orogastric tube was inserted all the way to the 24-inch Snell 8th ed., p. 130 JAN BACK-UP
mark to a 57 year old male who had an episode of CHARMAINE MIDTERM
hematemesis. Where is the tip of the tube located? PALOMAR, EXAM AUG
A. esophagus MD (TOP 9 - 2014
B. cardia of stomach FEB 2014
C. pylorus MED
D. duodenum BOARDS;
E. Ileum TOPNOTCH
MD)

TOPNOTCH MEDICAL BOARD PREP ANATOMY SUPEREXAM Page 66 of 94


For inquiries visit www.topnotchboardprep.com.ph or email us at topnotchmedicalboardprep@gmail.com
TOPNOTCH MEDICAL BOARD PREP ANATOMY SUPEREXAM
For inquiries visit www.topnotchboardprep.com.ph or email us at topnotchmedicalboardprep@gmail.com
Item QUESTION EXPLANATION AUTHOR TOPNOTCH
# EXAM
515 Which of the following is true of jejunum but not of ileum? In the living, the jejunum can be distinguished JAN BACK-UP
A. Jejunum is longer compared to ileum. from the ileum by the following features: CHARMAINE MIDTERM
B. Jejunum is wider bored, thicker walled and redder than 1. The jejunum lies coiled in the upper part of the PALOMAR, EXAM AUG
the ileum. peritoneal cavity below the left side of the MD (TOP 9 - 2014
C. Aggregations of lymphoid tissue (Peyer's patches) are transverse mesocolon; the ileum is in the lower FEB 2014
present in the mucous membrane of the jejunum. part of the cavity and in the pelvis. MED
D. There are no plicae circulares in the jejunum. 2. The jejunum is wider bored, thicker walled, and BOARDS;
E. Jejunal mesenteric vessels receives numerous short redder than the ileum. The jejunal wall feels TOPNOTCH
terminal vessels that arise from more than 4 arcades. thicker because the permanent infoldings of the MD)
mucous membrane, the plicae circulares, are
larger, more numerous, and closely set in the
jejunum, whereas in the upper part of the ileum
they are smaller and more widely separated and in
the lower part they are absent.
3. The jejunal mesentery is attached to the
posterior abdominal wall above and to the left of
the aorta, whereas the ileal mesentery is attached
below and to the right of the aorta.
4. The jejunal mesenteric vessels form only one or
two arcades, with long and infrequent branches
passing to the intestinal wall. The ileum receives
numerous short terminal vessels that arise from a
series of three or four or even more arcades.
5. At the jejunal end of the mesentery, the fat is
deposited near the root and is scanty near the
intestinal wall. At the ileal end of the mesentery
the fat is deposited throughout so that it extends
from the root to the intestinal wall.
6. Aggregations of lymphoid tissue (Peyer's
patches) are present in the mucous membrane of
the lower ileum along the antimesenteric border.
In the living these may be visible through the wall
of the ileum from the outside.

516 The following are boundaries of the site where breath Triangle of auscultation: lateral-medial border of JAN BACK-UP
sounds are best heard except: scapula, medial- trapezius, inferior- latissimus CHARMAINE MIDTERM
A. Latissimus dorsi dorsi -topnotch handout PALOMAR, EXAM AUG
B. Trapezius MD (TOP 9 - 2014
C. Medial border of scapula FEB 2014
D. External oblique muscle MED
E. None of the above BOARDS;
TOPNOTCH
MD)
517 The following muscles elevate the ribs and increase the Muscles of inspiration (elevate the ribs, Increase JAN BACK-UP
anteroposterior, transverse and vertical diameters of the the AP, transverse and vertical diameters) - CHARMAINE MIDTERM
thoracic cage except: Serratus posterior superior, Levator costarum, PALOMAR, EXAM AUG
A. Serratus posterior inferior External intercostal, Innermost intercostal, MD (TOP 9 - 2014
B. External intercostal Subcostal; Muscles of expiration (depress the ribs) FEB 2014
C. Innermost intercostal - Serratus posterior inferior, Internal intercostal, MED
D. Subcostal And Transversus thoracis - Topnotch handout BOARDS;
E. None of the above TOPNOTCH
MD)

518 The following statements concerning the dermatomes are C6- thumb; C8-small finger JAN BACK-UP
true except: CHARMAINE MIDTERM
A. The C5 dermatome includes the tip of the shoulder on PALOMAR, EXAM AUG
the same side. MD (TOP 9 - 2014
B. The C8 dermatome includes the thumb on the same side. FEB 2014
C. The T10 dermatome lies over the skin of the umbilicus. MED
D. The L4 and L5 dermatome runs along the medial side of BOARDS;
the foot and big toe. TOPNOTCH
E. The S1 dermatome includes the lateral side of the foot MD)
and the small toe.

519 The cremaster muscle raise the testis and the scrotum External spermatic fascia is dervived from the JAN BACK-UP
upward for warmth and for protection againsts injury. The aponeurosis of the external oblique muscle; the CHARMAINE MIDTERM
cremasteric fascia is a derivative of: cremasteric fascia is derived from the internal PALOMAR, EXAM AUG
A. Superficial fascia oblique muscle, and the internal spermatic fascia MD (TOP 9 - 2014
B. External oblique muscle is derived from the fascia transversalis. Snell 8th FEB 2014
C. Transversalis fascia ed., 167. MED
D. Internal oblique muscle BOARDS;
E. Peritoneum TOPNOTCH
MD)

TOPNOTCH MEDICAL BOARD PREP ANATOMY SUPEREXAM Page 67 of 94


For inquiries visit www.topnotchboardprep.com.ph or email us at topnotchmedicalboardprep@gmail.com
TOPNOTCH MEDICAL BOARD PREP ANATOMY SUPEREXAM
For inquiries visit www.topnotchboardprep.com.ph or email us at topnotchmedicalboardprep@gmail.com
Item QUESTION EXPLANATION AUTHOR TOPNOTCH
# EXAM
520 A 30 year old man involved in a car accident was rushed to There were approximately 3 questions on bladder JAN BACK-UP
the ER with severe lower abdominal pain and gross injury and site of blood accumulation in our board CHARMAINE MIDTERM
hematuria. Radiologic evaluation revealed pelvic fracture. exam. ** Urine and PALOMAR, EXAM AUG
Which of the following statements regarding bladder rupture blood escape freely into the peritoneal cavity in MD (TOP 9 - 2014
is not true? intraperitoneal bladder rupture which usually FEB 2014
A. The bladder may rupture intraperitoneally if the occur with injury to the superior wall of a full MED
superior wall of a full bladder was injured. bladder (that has extended up into the abdomen). BOARDS;
B. The bladder may rupture intraperitoneally or -Snell 8th ed., 351. TOPNOTCH
extraperitoneally. MD)
C. Urine and blood escape freely into the peritoneal cavity
in extraperitoneal bladder rupture.
D. Extraperitoneal bladder rupture occurs when bony
fragments pierce the anterior part of the bladder wall below
the level of peritoneal reflection.
E. All of the statements are correct.

521 The foremost example of a saddle joint MIGUEL MIDTERM 1


A) 2nd carpometacarpal RAFAEL EXAM - FEB
B) 2nd metatarsophalangeal RAMOS, MD 2013
C) 1st metatarsophalangeal (TOP 3 - FEB
D) 1st carpometacarpal 2012 MED
BOARDS;
TOPNOTCH
MD)
522 A surgeon tells a medical student to tap the side of the face of MIGUEL MIDTERM 1
a patient who just had thyroid surgery. The surgeon is most RAFAEL EXAM - FEB
worried about damage to which of the following vessels? RAMOS, MD 2013
A) Common carotid artery (TOP 3 - FEB
B) External carotid artery 2012 MED
C) Anterior jugular vein BOARDS;
D) Superior and inferior thyroid artery TOPNOTCH
MD)

523 A 45-year-old woman comes into the ER with abdominal MIGUEL MIDTERM 1
pain that has progressively worsened since the previous RAFAEL EXAM - FEB
night. Imaging reveals that a loop of small intesting has RAMOS, MD 2013
passed through the epiploic foramen into the omental bursa (TOP 3 - FEB
and is constricted by the margins of the foramen. This should 2012 MED
not be surgically relieved because this procedure would risk BOARDS;
cutting the TOPNOTCH
A) Abdominal aorta MD)
B) Hepatic artery
C) Hepatic vein
D) Pancreatic duct

524 The 2nd rib is atypical because of this feature? MIGUEL MIDTERM 1
A) groove for subclavian vessels RAFAEL EXAM - FEB
B) only 1 facet that articulates with a single vertebrae RAMOS, MD 2013
C) tuberosity for serratus anterior (TOP 3 - FEB
D) contains the scalene tubercle 2012 MED
BOARDS;
TOPNOTCH
MD)
525 A 16-year-old teenage boy tried to commit suicide by MIGUEL MIDTERM 1
slashing his wrist after his girlfriend of one week broke up RAFAEL EXAM - FEB
with him. He sustained “suicide cuts” on the lateral side of RAMOS, MD 2013
his wrist, most likely sparing the: (TOP 3 - FEB
A) Radial artery 2012 MED
B) Median nerve BOARDS;
C) Flexor carpi radialis tendon TOPNOTCH
D) Ulnar artery MD)

526 An IVP is performed on a patient to evaluate the function and MIGUEL MIDTERM 1
structure of her kidneys. Examination of the resulting RAFAEL EXAM - FEB
radiographs reveal that the left kidney is normal but that RAMOS, MD 2013
there is a duplication of the ureter and renal pelvis on the (TOP 3 - FEB
right side. Further testing reveals that kidney function is 2012 MED
normal. This variation is a result of abnormal development of BOARDS;
which of the following structures? TOPNOTCH
A) Ureteric bud MD)
B) Metanephric blastema
C) Mesonephric duct
D) Mesonephric tubules

TOPNOTCH MEDICAL BOARD PREP ANATOMY SUPEREXAM Page 68 of 94


For inquiries visit www.topnotchboardprep.com.ph or email us at topnotchmedicalboardprep@gmail.com
TOPNOTCH MEDICAL BOARD PREP ANATOMY SUPEREXAM
For inquiries visit www.topnotchboardprep.com.ph or email us at topnotchmedicalboardprep@gmail.com
Item QUESTION EXPLANATION AUTHOR TOPNOTCH
# EXAM
527 A 12-year-old boy falls from a tree he is climbing, but catches MIGUEL MIDTERM 1
himself on a branch with his right hand. He swings by his RAFAEL EXAM - FEB
right arm and jumps to the ground. Several hours later he RAMOS, MD 2013
presents to the ER with right hand clumsiness. Which of the (TOP 3 - FEB
following structures has he most likely injured? 2012 MED
A) Lower trunk, brachial plexus BOARDS;
B) Axillary nerve TOPNOTCH
C) Musculocutaneous nerve MD)
D) Radial nerve

528 A gastric biopsy is performed on a 42-year-old male. The MIGUEL MIDTERM 1


specimen is observed to have numerous cells with apical RAFAEL EXAM - FEB
membrane-bound secretion granules in the gastric glands. RAMOS, MD 2013
From which area of the stomach was the biopsy taken (TOP 3 - FEB
A) Cardiac region 2012 MED
B) Columns of Morgagni BOARDS;
C) Fundic region TOPNOTCH
D) Pyloric region MD)

529 A 52 y/o male has had a chronic cough with occasional low- MIGUEL MIDTERM 1
volume hemoptysis for the past three weeks. He is a two RAFAEL EXAM - FEB
pack-per-day cigarette smoker, and drinks three to four cans RAMOS, MD 2013
of beer on weekends. PE reveals right-sided face and right (TOP 3 - FEB
arm swelling and engorgement of subcutaneous veins on the 2012 MED
right side of the neck. Which of the following veins is most BOARDS;
likely obstructed in this patient? TOPNOTCH
A) External jugular MD)
B) Subclavian
C) Brachiocephalic
D) Superior vena cava

530 Most of the muscles of the buttocks insert in the MIGUEL MIDTERM 1
A) Quadrate tubercle RAFAEL EXAM - FEB
B) Greater tuberosity RAMOS, MD 2013
C) Lesser trochanter (TOP 3 - FEB
D) Greater trochanter 2012 MED
BOARDS;
TOPNOTCH
MD)
531 After a motor vehicle accident, a patient is brought to the MIGUEL MIDTERM 1
emergency room. Xrays reveal that she has fractures of her RAFAEL EXAM - FEB
left ninth and tenth ribs. She has a rapid heart rate and low RAMOS, MD 2013
blood pressure. Peritoneal lavage reveals free blood in the (TOP 3 - FEB
peritoneal cavity. A surgeon is able to stop the bleeding by 2012 MED
placing a clamp across which of the following structures? BOARDS;
A) Falciform ligament TOPNOTCH
B) Gastrosplenic ligament MD)
C) Splenorenal ligament
D) Hepatoduodenal ligament

532 The glossopharyngeal nerve is transected accidentally MIGUEL MIDTERM 1


during a surgical procedure in a 45 y/o male. Which of the RAFAEL EXAM - FEB
following is most likely lost in this patient? RAMOS, MD 2013
A) Taste sensation from the anterior 2/3 of the tongue (TOP 3 - FEB
B) General sensation from the tonsillar lining 2012 MED
C) Salivary secretion from the submandibular gland BOARDS;
D) Protrusion of the tongue TOPNOTCH
MD)

533 Laceration of the male urethra just inferior to the urogenital MIGUEL MIDTERM 1
diaphragm would likely result in extravasation of urine into RAFAEL EXAM - FEB
all of the following regions except RAMOS, MD 2013
A) The abdominal wall between Scarpa's fascia and deep (TOP 3 - FEB
fascia 2012 MED
B) The anal triangle between superficial fat and deep fascia BOARDS;
C) The penis between the superficial fascia and deep (Buck's) TOPNOTCH
fascia MD)
D) The urogenital triangle between the deep layer of
superficial (Colle's)

TOPNOTCH MEDICAL BOARD PREP ANATOMY SUPEREXAM Page 69 of 94


For inquiries visit www.topnotchboardprep.com.ph or email us at topnotchmedicalboardprep@gmail.com
TOPNOTCH MEDICAL BOARD PREP ANATOMY SUPEREXAM
For inquiries visit www.topnotchboardprep.com.ph or email us at topnotchmedicalboardprep@gmail.com
Item QUESTION EXPLANATION AUTHOR TOPNOTCH
# EXAM
534 A 5-year-old male with a bounding pulse has a thrill best MIGUEL MIDTERM 1
palpated over the upper left sternal edge. A continuous RAFAEL EXAM - FEB
murmur is heard over the area on cardiac auscultation. If RAMOS, MD 2013
surgery is planned, the surgeon should intervene on a (TOP 3 - FEB
derivative of which of the following embryologic structures? 2012 MED
A) Sinus venosus BOARDS;
B) Bulbus cordis TOPNOTCH
C) 4th aortic arch MD)
D) 6th aortic arch

535 In portal hypertension, which of the following veins will not MIGUEL MIDTERM 1
engorge and is not a collateral of the circulation? RAFAEL EXAM - FEB
A) Inferior rectal RAMOS, MD 2013
B) Middle rectal (TOP 3 - FEB
C) Esophageal 2012 MED
D) Hepatic BOARDS;
TOPNOTCH
MD)

536 In performing a cricothyrotomy, an incision should be made MIGUEL MIDTERM 1


at which of the following locations? RAFAEL EXAM - FEB
A) Cricothyroid membrane, at the junction of clavicle and RAMOS, MD 2013
sternum (TOP 3 - FEB
B) Cricothyroid membrane, between the thyroid cartilage 2012 MED
and cricoid cartilage below BOARDS;
C) Thyrohyoid membrane, between the thyroid cartilage and TOPNOTCH
hyoid bone MD)
D) Sternal notch, at the junction of the clavicle and sternum

537 The structure that serves as a landmark in doing pudendal MIGUEL MIDTERM 1
block anesthesia through a transvaginal approach is the RAFAEL EXAM - FEB
A) Ischial spine RAMOS, MD 2013
B) Iliac crest (TOP 3 - FEB
C) Ischial tuberosity 2012 MED
D) Sacral promontory BOARDS;
TOPNOTCH
MD)

538 A neurologic exam of a 34-year-old man reveals a direct and MIGUEL MIDTERM 1
consensual light reflex in his left eye; but neither a direct nor RAFAEL EXAM - FEB
consensual reflex in his right eye. The nerve involved is the RAMOS, MD 2013
A) Right optic nerve (TOP 3 - FEB
B) Left optic nerve 2012 MED
C) Right oculomotor nerve BOARDS;
D) Left oculomotor nerve TOPNOTCH
MD)

539 A wall-climber's primary back muscle is in the following MIGUEL MIDTERM 1


group of muscles as which of the following RAFAEL EXAM - FEB
A) Serratus posterior RAMOS, MD 2013
B) Trapezius (TOP 3 - FEB
C) Splenus cervicis 2012 MED
D) Erector spinae BOARDS;
TOPNOTCH
MD)

540 A 17-year-old football player comes to the emergency MIGUEL MIDTERM 1


department hopping on his right foot. An X-ray of the left leg RAFAEL EXAM - FEB
shows a fracture of the neck of the fibula. Which of the RAMOS, MD 2013
following findings do you most expect on physical (TOP 3 - FEB
examination? 2012 MED
A) Loss of sensation on the sole of the left foot BOARDS;
B) weakened inversion of left foot TOPNOTCH
C) loss of plantarflexion of the left foot MD)
D) Loss of sensation on the dorsum ofthe left foot

TOPNOTCH MEDICAL BOARD PREP ANATOMY SUPEREXAM Page 70 of 94


For inquiries visit www.topnotchboardprep.com.ph or email us at topnotchmedicalboardprep@gmail.com
TOPNOTCH MEDICAL BOARD PREP ANATOMY SUPEREXAM
For inquiries visit www.topnotchboardprep.com.ph or email us at topnotchmedicalboardprep@gmail.com
Item QUESTION EXPLANATION AUTHOR TOPNOTCH
# EXAM
541 A 24/M, gang member, is hit over the right side of his head A classic case of epidural hematoma (head injury + ABDELSIMA FINAL EXAM
with a baseball bat during a brawl. He loses consciousness lucid interval + lens-shaped hemorrhage on CT). R OMAR II, - FEB 2014
for a few minutes, but recovers promptly. One hour later, he The most common artery damaged in epidural MD (TOP 2 -
is found unconscious at his home. The right pupil is fixed and hematoma is the anterior division of the middle AUG 2013
dilated. When he was rushed to the ER, stat CT done showed meningeal artery, a branch of the maxillary artery. MED
lenticular hyperdensity in the temporoparietal area. The BOARDS;
blood vessel which most likely bled in this case, is a direct TOPNOTCH
branch of the: MD - 200
A. Ophthalmic artery QUESTIONS)
B. Maxillary artery AND MARC
C. Internal carotid artery DENVER
D. Middle cerebral artery TIONGSON,
E. Anterior choroidal artery MD (40
QUESTIONS)

542 A 40/F with known tooth infection comes in due to a large, Patient has Ludwig's angina due to an acute ABDELSIMA FINAL EXAM
tender, fluctuant mass occupying the left lower side of her infection of the submandibular fascial space, R OMAR II, - FEB 2014
face and upper neck including the underside of the mouth. commonly secondary to a dental infection. MD (TOP 2 -
On examination, the mass is seen pushing up the floor of the AUG 2013
mouth on the left side. This patient has an acute infection of MED
this fascial space which is usually due to dental infection. BOARDS;
A. Visceral TOPNOTCH
B. Retropharyngeal MD - 200
C. Submandibular QUESTIONS)
D. Masticatory AND MARC
E. Sublingual DENVER
TIONGSON,
MD (40
QUESTIONS)
543 You are performing the neurologic examination on a 45/M, When the patient asked to put out the tongue, the ABDELSIMA FINAL EXAM
diagnosed with a skull base tumor. When you asked him to tongue will deviate toward the paralyzed side. The R OMAR II, - FEB 2014
protrude the tongue, you noted that the tongue deviated to normal genioglossus muscle will pull the MD (TOP 2 -
the left; indicating involvement of the: unaffected side of the tongue forward leaving the AUG 2013
A. Left glossopharyngeal nerve paralyzed side of the tongue stationary MED
B. Right glossopharyngeal nerve BOARDS;
C. Left hypoglossal nerve TOPNOTCH
D. Right hypoglossal nerve MD - 200
E. Right vagus nerve QUESTIONS)
AND MARC
DENVER
TIONGSON,
MD (40
QUESTIONS)
544 Which of the following would necessitate cricothyroidotomy Section of the ELN causes weakness of the voice. ABDELSIMA FINAL EXAM
or tracheostomy? Bilateral complete section of the RLN causes loss R OMAR II, - FEB 2014
A. Section of the external laryngeal nerve of speech and impairment of breathing because MD (TOP 2 -
B. Unilateral partial section of the recurrent laryngeal nerve the glottis is partially closed (vocal fold assumed AUG 2013
C. Bilateral partial section of the recurrent laryngeal nerve position midway between abduction and MED
D. Unilateral complete section of the recurrent laryngeal adduction.) A bilateral partial section of the RLN is BOARDS;
nerve the worst, causing bilateral paralysis of abductors. TOPNOTCH
E. Bilateral complete section of the recurrent laryngeal nerve The vocal cord is drawn together, leading to acute MD - 200
breathlessness and stridor; cricothyroidotomy or QUESTIONS)
tracheostomy would be necessary. AND MARC
DENVER
TIONGSON,
MD (40
QUESTIONS)
545 Knowledge of important airway distances would help in the Incisor teeth to vocal cords: 15 cm; incisor teeth to ABDELSIMA FINAL EXAM
correct performance of procedures such as endotracheal carina: 20 cm; external nares to carina: 30 cm R OMAR II, - FEB 2014
intubation. What is the average distance between the incisor MD (TOP 2 -
teeth and the carina? AUG 2013
A. 15 cm MED
B. 20 cm BOARDS;
C. 25 cm TOPNOTCH
D. 30 cm MD - 200
E. 44 cm QUESTIONS)
AND MARC
DENVER
TIONGSON,
MD (40
QUESTIONS)
546 A 12/M came in due to epistaxis. Most cases of epistaxis ABDELSIMA FINAL EXAM
occur on the anterioinferior septum and involve septal R OMAR II, - FEB 2014
branches of the: MD (TOP 2 -
A. Posterior ethmoidal artery AUG 2013
B. Anterior ethmoidal artery MED
C. Facial artery BOARDS;
D. Sphenopalatine artery TOPNOTCH
E. Greater palatine artery MD - 200
QUESTIONS)
AND MARC
DENVER
TIONGSON,
MD (40

TOPNOTCH MEDICAL BOARD PREP ANATOMY SUPEREXAM Page 71 of 94


For inquiries visit www.topnotchboardprep.com.ph or email us at topnotchmedicalboardprep@gmail.com
TOPNOTCH MEDICAL BOARD PREP ANATOMY SUPEREXAM
For inquiries visit www.topnotchboardprep.com.ph or email us at topnotchmedicalboardprep@gmail.com
Item QUESTION EXPLANATION AUTHOR TOPNOTCH
# EXAM
QUESTIONS)

547 A 40/M, a known case of coronary artery disease, Intercostobrachial nerve communicates with the ABDELSIMA FINAL EXAM
experiences severe crushing pain in the chest referred to the medial cutaneous nerve of the arm. R OMAR II, - FEB 2014
medial side of the arm and armpit. Which nerve is MD (TOP 2 -
responsible for the referred pain to the medial side of the AUG 2013
arm? MED
A. Anterior intercostal nerve BOARDS;
B. Posterior intercostal nerve TOPNOTCH
C. Lateral intercostal nerve MD - 200
D. Intercostobrachial nerve QUESTIONS)
E. Lateral cutaneous nerve AND MARC
DENVER
TIONGSON,
MD (40
QUESTIONS)
548 A patient with carcinoma of the lower third of the esophagus The upper third of the esophagus drains into the ABDELSIMA FINAL EXAM
who underwent surgical removal of the lesion also had the deep cervical; the middle third into the R OMAR II, - FEB 2014
stomach, upper half of duodenum, spleen and omenta mediastinal. MD (TOP 2 -
removed, because all drain commonly into this group of AUG 2013
lymph nodes: MED
A. Deep cervical BOARDS;
B. Mediastinal TOPNOTCH
C. Celiac MD - 200
D. Superior mesenteric QUESTIONS)
E. Inferior mesenteric AND MARC
DENVER
TIONGSON,
MD (40
QUESTIONS)
549 A barium swallow can also be used to assess the size of this If you think about it carefully, the question simply ABDELSIMA FINAL EXAM
chamber of the heart: asks the posteriormost of the chambers. R OMAR II, - FEB 2014
A. Left atrium Remember that the anterior wall of the esophagus MD (TOP 2 -
B. Left ventricle and the posterior wall of the left atrium are in AUG 2013
C. Right atrium close proximity; hence, a barium swallow can be MED
D. Right ventricle used to assess the size of the LA. BOARDS;
TOPNOTCH
MD - 200
QUESTIONS)
AND MARC
DENVER
TIONGSON,
MD (40
QUESTIONS)
550 How many intervertebral discs are there? No IV disc between C1 and C2, sacrum and coccyx. ABDELSIMA FINAL EXAM
A. 21 R OMAR II, - FEB 2014
B. 23 MD (TOP 2 -
C. 25 AUG 2013
D. 27 MED
E. 29 BOARDS;
TOPNOTCH
MD - 200
QUESTIONS)
AND MARC
DENVER
TIONGSON,
MD (40
QUESTIONS)
551 A 24/M, a member of the college varsity swimming team, ABDELSIMA FINAL EXAM
comes in to you complaining of shoulder pain. On R OMAR II, - FEB 2014
examination, you note pain on the shoulder while in the mid- MD (TOP 2 -
range of abduction. You are considering rotator cuff AUG 2013
tendinitis, due to this muscle rubbing on the acromion MED
because of a degenerated subacromial bursa. BOARDS;
A. Supraspinatus TOPNOTCH
B. Infraspinatus MD - 200
C. Teres minor QUESTIONS)
D. Teres major AND MARC
E. Subscapularis DENVER
TIONGSON,
MD (40
QUESTIONS)

TOPNOTCH MEDICAL BOARD PREP ANATOMY SUPEREXAM Page 72 of 94


For inquiries visit www.topnotchboardprep.com.ph or email us at topnotchmedicalboardprep@gmail.com
TOPNOTCH MEDICAL BOARD PREP ANATOMY SUPEREXAM
For inquiries visit www.topnotchboardprep.com.ph or email us at topnotchmedicalboardprep@gmail.com
Item QUESTION EXPLANATION AUTHOR TOPNOTCH
# EXAM
552 An infant was diagnosed to have Erb-Duchenne palsy as a Erb-Duchenne palsy usually affects the (1) ABDELSIMA FINAL EXAM
complication of difficult delivery. The right limb was seen suprascapular nerve causing palsy of the R OMAR II, - FEB 2014
hanging limp, rotated medially and pronated. Which of the supraspinatus (responsible for shoulder MD (TOP 2 -
following muscles is unaffected in Erb-Duchenne palsy? abduction) and infraspinatus (lateral rotation AUG 2013
A. Supraspinatus shoulder); (2) nerve to subclavius (depression of MED
B. Infraspinatus clavicle); (3) musculocutaneous nerve causing BOARDS;
C. Biceps brachii palsy of the biceps, brachialis and coracobrachialis TOPNOTCH
D. Subclavius (flex and supinate elbow); and (4) axillary nerve, MD - 200
E. None of the above supplying the deltoid (shoulder abudciton). QUESTIONS)
AND MARC
DENVER
TIONGSON,
MD (40
QUESTIONS)
553 A 55/F obese, diagnosed case of bilateral knee osteoarthritis, Poorly adjusted crutches may result in axillary ABDELSIMA FINAL EXAM
has been using crutches for a year to help her ambulate. She nerve palsy presenting as weak abduction of the R OMAR II, - FEB 2014
comes in to you complaining of difficulty in abducting her shoulder and lateral rotation of the arm (actions of MD (TOP 2 -
right shoulder. On examination, you note loss of the rounded the deltoid and teres minor, respectively); loss of AUG 2013
contour of the right shoulder and loss of sensation on the sensation on the lower half of the deltoid; and MED
lower half of the right deltoid. Which nerve must have been wasting of the deltoid. BOARDS;
affected? TOPNOTCH
A. Medial antebrachial cutaneous nerve MD - 200
B. Medial cutaneous nerve QUESTIONS)
C. Axillary nerve AND MARC
D. Musculocutaneous nerve DENVER
E. Median nerve TIONGSON,
MD (40
QUESTIONS)

554 Ureteral stones are most likely to lodge at the narrowest part SIMILAR TO PREVIOUS BOARD EXAM ABDELSIMA FINAL EXAM
of the ureter which is at the: CONCEPT/PRINCIPLE. R OMAR II, - FEB 2014
A. Ureteropelvic junction MD (TOP 2 -
B. Ureterovesical junction AUG 2013
C. Point at which the ureter crosses over the iliac vessels MED
D. Pelvic brim BOARDS;
TOPNOTCH
MD - 200
QUESTIONS)
AND MARC
DENVER
TIONGSON,
MD (40
QUESTIONS)
555 You were tasked to perform urethral catheterization on a Things to remember. The external orifice of the ABDELSIMA FINAL EXAM
patient prior to surgery. You note resistance in passing the glans is the narrowest part; the prostatic urethra R OMAR II, - FEB 2014
catheter at the narrowest part of the urethra which is the: is the widest and most dilatable; the part within MD (TOP 2 -
A. External orifice of the glans penis the bulb is subject to chronic inflammation and AUG 2013
B. Membranous urethra stricture formation and is the most common site of MED
C. Bulbar urethra rupture after a perineal blow. BOARDS;
D. Prostatic urethra TOPNOTCH
MD - 200
QUESTIONS)
AND MARC
DENVER
TIONGSON,
MD (40
QUESTIONS)
556 In performing lumbar puncture in an infant, it is important to Spinal cord ends at the lower border of L1 in the ABDELSIMA FINAL EXAM
remember that the spinal cord terminates as low as this adult; and at L3 in the infant. The subarachnoid R OMAR II, - FEB 2014
vertebral level. space extends down as far as the lower border of MD (TOP 2 -
A. L1 S2. AUG 2013
B. L2 MED
C. L3 BOARDS;
D. L4 TOPNOTCH
E. L5 MD - 200
QUESTIONS)
AND MARC
DENVER
TIONGSON,
MD (40
QUESTIONS)
557 In the adult, the notochord gives rise to the: Expect a few (1-2) embryology questions. ABDELSIMA FINAL EXAM
A. Spinal cord R OMAR II, - FEB 2014
B. Vertebral body MD (TOP 2 -
C. Annulus fibrosis AUG 2013
D. Nucleus pulposus MED
E. It has no adult derivatives BOARDS;
TOPNOTCH
MD - 200
QUESTIONS)
AND MARC
DENVER
TIONGSON,
MD (40
QUESTIONS)

TOPNOTCH MEDICAL BOARD PREP ANATOMY SUPEREXAM Page 73 of 94


For inquiries visit www.topnotchboardprep.com.ph or email us at topnotchmedicalboardprep@gmail.com
TOPNOTCH MEDICAL BOARD PREP ANATOMY SUPEREXAM
For inquiries visit www.topnotchboardprep.com.ph or email us at topnotchmedicalboardprep@gmail.com
Item QUESTION EXPLANATION AUTHOR TOPNOTCH
# EXAM
558 A 24/M basketball player lands awkwardly from a rebound, ACL tear. ABDELSIMA FINAL EXAM
and feels that his right knee has hyperextended, bent inward R OMAR II, - FEB 2014
and popped. The knee becomes swollen over the next few MD (TOP 2 -
hours. On PE, you note a positive Lachman test. The patient AUG 2013
has suffered a tear in the ligament which prevents: MED
A. Anterior displacement of the tiba on the femur BOARDS;
B. Posterior displacement of the tibia on the femur TOPNOTCH
C. Adduction of the leg at the knee MD - 200
D. Abduction fo the leg at the knee QUESTIONS)
AND MARC
DENVER
TIONGSON,
MD (40
QUESTIONS)
559 A 35/M, known case of Pott's disease, developed a psoas ABDELSIMA FINAL EXAM
abscess.. On PE, you noted that he had 1+ patellar tendon R OMAR II, - FEB 2014
reflex on the same side and he had difficulty walking. What MD (TOP 2 -
nerve is involved? AUG 2013
A. Femoral nerve MED
B. Obturator nerve BOARDS;
C. Sciatic nerve TOPNOTCH
D. Genitofemoral nerve MD - 200
E. Pudendal nerve QUESTIONS)
AND MARC
DENVER
TIONGSON,
MD (40
QUESTIONS)
560 The germinal epithelium of the ovary is an example of: Other simple cuboidal epithelium include the ABDELSIMA FINAL EXAM
A. Simple squamous epithelium LENS of the eye, the respiratory bronchioles, R OMAR II, - FEB 2014
B. Stratified squamous epithelium thyroid cells, and the pigment layer of the retina. MD (TOP 2 -
C. Simple columnar epithelium AUG 2013
D. Stratified columnar epithelium MED
E. Simple cuboidal epithelium BOARDS;
TOPNOTCH
MD - 200
QUESTIONS)
AND MARC
DENVER
TIONGSON,
MD (40
QUESTIONS)
561 the structure which passes along the borders of the Simons Answer: Recurrent LN can usually be seen within BLAKE MIDTERM 2
triangle is the: the Simons triangle bounded by esophagus, ITA, WARREN EXAM - FEB
a. recurrent laryngeal nerve and common carotid. ANG, MD 2014
b. median nerve (TOP 1 - AUG
c. superficial peroneal nerve 2013 MED
d. greater auricular nerve BOARDS;
TOPNOTCH
MD)

562 the weight of the thyroid gland in a normal adult is: 20g is the average weight of the thyroid gland BLAKE MIDTERM 2
a. 20g WARREN EXAM - FEB
b. 30g ANG, MD 2014
c. 40g (TOP 1 - AUG
d. 45g 2013 MED
BOARDS;
TOPNOTCH
MD)
563 the esophagus is lined by this epithelium BLAKE MIDTERM 2
a. stratified squamous WARREN EXAM - FEB
b. simple columnar ANG, MD 2014
c. stratified cuboidal (TOP 1 - AUG
d. pseudostratified columnar 2013 MED
BOARDS;
TOPNOTCH
MD)
564 the smooth portion of the ventricles of the heart is derived BLAKE MIDTERM 2
from WARREN EXAM - FEB
a. primitive ventricle ANG, MD 2014
b. bulbos cordis (TOP 1 - AUG
c. sinus venosus 2013 MED
d. none of the above BOARDS;
TOPNOTCH
MD)

TOPNOTCH MEDICAL BOARD PREP ANATOMY SUPEREXAM Page 74 of 94


For inquiries visit www.topnotchboardprep.com.ph or email us at topnotchmedicalboardprep@gmail.com
TOPNOTCH MEDICAL BOARD PREP ANATOMY SUPEREXAM
For inquiries visit www.topnotchboardprep.com.ph or email us at topnotchmedicalboardprep@gmail.com
Item QUESTION EXPLANATION AUTHOR TOPNOTCH
# EXAM
565 a patient presents to the trauma center after a vehicular Left 2nd ICS MCL. Although technically anywhere BLAKE MIDTERM 2
crash with tachypnea and hypotension. the trachea is within the chest can be accessed, it is typically WARREN EXAM - FEB
deviated to the right with decrease pulses on both arms and within this area to avoid any major structures. ANG, MD 2014
legs. as an initial/immediate life saving procedure, the (TOP 1 - AUG
physician should access the thorax through 2013 MED
a. right 2nd ICS MCl BOARDS;
b. right 5th ICS MAL TOPNOTCH
c. left 2nd ICS MCL MD)
d. left 2nd ICS MAL

566 blood cell component with the longest half life Adult RBC has half life of 90-120 days BLAKE MIDTERM 2
a. basophil WARREN EXAM - FEB
b. rbc ANG, MD 2014
c. platelet (TOP 1 - AUG
d. neutrophil 2013 MED
BOARDS;
TOPNOTCH
MD)
567 osteoid is predominantly produced by: BLAKE MIDTERM 2
a. osteoblast WARREN EXAM - FEB
b. osteoclast ANG, MD 2014
c. osteocyte (TOP 1 - AUG
d. all of the above 2013 MED
BOARDS;
TOPNOTCH
MD)
568 branchial arch derivative of the stylopharyngeus is: BLAKE MIDTERM 2
a. branchial arch 1 WARREN EXAM - FEB
b. branchial arch 2 ANG, MD 2014
c. branchial arch 3 (TOP 1 - AUG
d. branchial arch 5 2013 MED
BOARDS;
TOPNOTCH
MD)
569 oxyphil cell is found in: BLAKE MIDTERM 2
a. parathyroid gland WARREN EXAM - FEB
b. pituitary gland ANG, MD 2014
c. adrenal gland (TOP 1 - AUG
d. thyroid gland 2013 MED
BOARDS;
TOPNOTCH
MD)
570 the vein that joins the subcostal vein to form the azygos vein BLAKE MIDTERM 2
is: WARREN EXAM - FEB
a. ascending lumbar ANG, MD 2014
b. musculophrenic (TOP 1 - AUG
c. ascending phrenic 2013 MED
d. superior epigastric BOARDS;
TOPNOTCH
MD)

571 fracture of the humeral midshaft produces this abnormality: Radial nerve injury (know the difference of BLAKE MIDTERM 2
a. inability to flex the elbow joint neuropraxia, axonotmesis, etc.) causes inability to WARREN EXAM - FEB
b. numbness of the medial portion of the palm extend the wrist joint. ANG, MD 2014
c. inability to extend wrist joint (TOP 1 - AUG
d. inability to abduct the upper arm 2013 MED
BOARDS;
TOPNOTCH
MD)
572 the epithelial covering of the lens of the eye is: BLAKE MIDTERM 2
a. simple squamous WARREN EXAM - FEB
b. simple cuboidal ANG, MD 2014
c. simple columnar (TOP 1 - AUG
d. pseudostratified cuboidal 2013 MED
BOARDS;
TOPNOTCH
MD)
573 the sternal angle of Louis coincides with this vertebral level BLAKE MIDTERM 2
a. C4 WARREN EXAM - FEB
b. T4 ANG, MD 2014
c. T8 (TOP 1 - AUG
d. C6 2013 MED
BOARDS;
TOPNOTCH
MD)
574 the left gastroepiploic vein drains to the BLAKE MIDTERM 2
a. splenic vein WARREN EXAM - FEB
b. SMV ANG, MD 2014
c. IMV (TOP 1 - AUG
d. Poral vein 2013 MED
BOARDS;
TOPNOTCH
MD)

TOPNOTCH MEDICAL BOARD PREP ANATOMY SUPEREXAM Page 75 of 94


For inquiries visit www.topnotchboardprep.com.ph or email us at topnotchmedicalboardprep@gmail.com
TOPNOTCH MEDICAL BOARD PREP ANATOMY SUPEREXAM
For inquiries visit www.topnotchboardprep.com.ph or email us at topnotchmedicalboardprep@gmail.com
Item QUESTION EXPLANATION AUTHOR TOPNOTCH
# EXAM
575 the major blood supply to the femoral head is BLAKE MIDTERM 2
a. medial circumflex artery WARREN EXAM - FEB
b. lateral circumflex artery ANG, MD 2014
c. feeding artery to the femoral head (TOP 1 - AUG
d. deep femoral artery 2013 MED
BOARDS;
TOPNOTCH
MD)
576 drainage of the nasolacrimal duct is towards the: BLAKE MIDTERM 2
a. superior meatus WARREN EXAM - FEB
b. middle meatus ANG, MD 2014
c. inferior meatus (TOP 1 - AUG
d. oropharynx 2013 MED
BOARDS;
TOPNOTCH
MD)
577 a person was stabbed at the transpyloric area through the BLAKE MIDTERM 2
neck of the pancreas. which of the following structures will WARREN EXAM - FEB
be least affected: ANG, MD 2014
a. SMA (TOP 1 - AUG
b. IMA 2013 MED
c. Splenic vein BOARDS;
d. Portal vein TOPNOTCH
MD)

578 most common site Of ureteral obstruction: Answer: Uteropelvic junction , but the narrowest BLAKE MIDTERM 2
a. ureteropelvic junction portion is ureterovesical junction WARREN EXAM - FEB
b. pelvic brim level ANG, MD 2014
c. ureterovesical junction (TOP 1 - AUG
2013 MED
BOARDS;
TOPNOTCH
MD)
579 the most common hernia in females Indirect hernia is the most common type of hernia BLAKE MIDTERM 2
a. direct in females; males are more involved than females. WARREN EXAM - FEB
b. indirect ANG, MD 2014
c. femoral (TOP 1 - AUG
d. obturator 2013 MED
BOARDS;
TOPNOTCH
MD)
580 craniophayngioma would present with what visual field Bilateral temporal hemianopsia BLAKE MIDTERM 2
defect WARREN EXAM - FEB
a. binasal hemianopsia ANG, MD 2014
b. bitempora hemianopsia (TOP 1 - AUG
c. right/left homonymous hemianopsia 2013 MED
d. right superior quantantanopsia BOARDS;
TOPNOTCH
MD)
581 In endocrine histology, the presence of oxyphil cells will tell Oxyphil cells are found in the parathyoid gland. TIMOTHY MIDTERM 1
us that the tissue specimen came from what organ? There function is still unknown. TANG LEE EXAM - FEB
A. Thyroid gland SAY, MD 2014
B. Parathyroid gland (TOP 4 - AUG
C. Pituitary gland 2013 MED
D. Adrenal gland BOARDS;
E. Gonads TOPNOTCH
MD)

582 Which is the most proximal part of the respiratory The respiratory conducting system in proximal to TIMOTHY MIDTERM 1
conducting system that has absent goblet cells and distal order: trachea, main bronchi, lobar bronchi, TANG LEE EXAM - FEB
submucous glands? terminal bronchioles, respiratory bronchioles, SAY, MD 2014
A. Lobar bronchi alveolar ducts, alveolar sacs, alveoli. Goblet cells (TOP 4 - AUG
B. Segmental bronchi and submucous glands are present up to the 2013 MED
C. Terminal bronchioles segmental bronchi. Smooth muscles are present BOARDS;
D. Respiratory bronchioles up to alveolar ducts. Elastic fibers are present up TOPNOTCH
E. Alveolar ducts to alveoli sacs. MD)

583 A 32 yr old biker fell and sustain lacerations in the right arm Brachioradialis is the only flexor muscle that is TIMOTHY MIDTERM 1
with suspicious fracture. In the ER, X-rays were requested also innervated by the radial nerve. Midshaft TANG LEE EXAM - FEB
which showed fracture in the midshaft of the humerus. fractures of the humerus usually affect the radial SAY, MD 2014
Which of the following muscles would exhibit weakness? nerve. Coracobrachialis and brachialis are (TOP 4 - AUG
A. Coracobrachialis innervated by the musculocutaneous nerve. 2013 MED
B. Brachialis Pronator teres and flexor digitorum profundus are BOARDS;
C. Brachioradialis innervated by the median nerve. TOPNOTCH
D. Pronator teres MD)
E. Flexor digitorum profundus

TOPNOTCH MEDICAL BOARD PREP ANATOMY SUPEREXAM Page 76 of 94


For inquiries visit www.topnotchboardprep.com.ph or email us at topnotchmedicalboardprep@gmail.com
TOPNOTCH MEDICAL BOARD PREP ANATOMY SUPEREXAM
For inquiries visit www.topnotchboardprep.com.ph or email us at topnotchmedicalboardprep@gmail.com
Item QUESTION EXPLANATION AUTHOR TOPNOTCH
# EXAM
584 A patient sustain an injury in the posterior cord of the The posterior cord gives rise to the upper and TIMOTHY MIDTERM 1
brachial plexus. The following actions are compromised lower subscapular nerves, thoracodorsal, axillary TANG LEE EXAM - FEB
EXCEPT? and radial nerves. Supination of the forearm is the SAY, MD 2014
A. Medial rotation of the shoulder main action of biceps brachii which is innervated (TOP 4 - AUG
B. Supination of the forearm by the musculocutaneous nerve. Elevation of the 2013 MED
C. Elevation of the trunk trunk is an action of the latissimus dorsi BOARDS;
D. Extension of the forearm innervated by the thoracodorsal nerve. Radial TOPNOTCH
E. Lateral rotation of the shoulder nerve innervates the posterior compartment of MD)
the arm and forearm (extensors). The muscles of
the shoulder are innervated by the other nerves
mentioned.
585 Which of the following muscles abducts, flexes and laterally Actions: TIMOTHY MIDTERM 1
rotates the thigh at the knee joint? Tensor fascia lata: abducts, medially rotates and TANG LEE EXAM - FEB
A. Sartorius flexes thigh SAY, MD 2014
B. Tensor fascia lata Rectus femoris: flexes the thigh and extends the (TOP 4 - AUG
C. Rectus femoris leg 2013 MED
D. Gracilis Gracilis: adducts thigh, flexes and medially rotate BOARDS;
E. Pectineus the leg TOPNOTCH
Pectineus: adducts, flexes and medially rotate the MD)
thigh
586 A varsity basketball player had an audible pop in his right Anterior drawer sign: Forward sliding of the tibia TIMOTHY MIDTERM 1
knee upon landing from a jump shot. This was accompanied on the femur due to rupture of the anterior TANG LEE EXAM - FEB
by excruciating pain and swelling of the knee joint. To check cruciate ligament SAY, MD 2014
for possible ligamental tear, the rehabilitation medicine Posterior drawer sign: Backward sliding of the (TOP 4 - AUG
specialist asked the patient to lie down and perform the tibia on the femur due to rupture of the posterior 2013 MED
anterior drawer test by doing this maneuver? cruciate ligament BOARDS;
A. Forward sliding of the femur on the tibia due to rupture TOPNOTCH
of either the anterior or posterior cruciate ligament MD)
B. Forward sliding of the femur on the tibia due to rupture
of the posterior cruciate ligament
C. Forward sliding of the tibia on the femur due to rupture
of the posterior cruciate ligament
D. Forward sliding of the femur on the tibia due to rupture
of the anterior cruciate ligament
E. Forward sliding of the tibia on the femur due to rupture
of the anterior cruciate ligament

587 Which is the final draining vessel of the conventional The pathway for aqueous humor outflow are the TIMOTHY MIDTERM 1
aqueous humor outflow pathway? following: trabecular meshwork → Schlemm's TANG LEE EXAM - FEB
A. Orbital veins canal → Collector channels → Aqueous veins → SAY, MD 2014
B. Trabecular meshwork Episcleral veins → Orbital veins → Intracranial (TOP 4 - AUG
C. Schlemm's canal cavernous sinus 2013 MED
D. Episcleral veins BOARDS;
E. Aqueous veins TOPNOTCH
MD)

588 A 60 yr old patient with known atherosclerotic heart disease Review the visual pathway. Vascular and TIMOTHY MIDTERM 1
suddenly complain of an acute onset blurring of vision, in neoplastic (malignant or benign tumours) lesions TANG LEE EXAM - FEB
which stroke was entertained. MRI shows that the lesion from the optic tract, to visual cortex can cause a SAY, MD 2014
involved the cerebral cortex supplied by the left posterior contralateral homonymous hemianopsia. The (TOP 4 - AUG
cerebral artery, what visual field defect is expected? difference is that cortical lesions tend to have 2013 MED
A. Right homonymous hemianopsia without macular macular sparing since the optic tract send signals BOARDS;
sparing coming from the macula to both cortices and they TOPNOTCH
B. Left homonymous hemianopsia without macular sparing have dual blood supply from the MCA (aside from MD)
C. Bitemporal hemianopsia without macular sparing the PCA).
D. Right homonymous hemianopsia with macular sparing
E. Left homonymous hemianopsia with macular sparing

589 An otorhinolaryngologist is examing the tympanic The adult external canal is divided into an outer TIMOTHY MIDTERM 1
membrane of a 20 yr old patient with chronic otitis media. cartilagenous portion in its outer 1/3 and bony TANG LEE EXAM - FEB
He knows that the external auditory canal is divided into a? portion in its inner 2/3. It measures about 2.5 cms SAY, MD 2014
A. 1/2 outer bony portion and 1/2 inner cartilaginous on the whole. (TOP 4 - AUG
portion 2013 MED
B. 1/3 outer cartilaginous portion and 2/3 inner bony BOARDS;
portion TOPNOTCH
C. 1/3 outer bony portion and 2/3 inner cartilaginous MD)
portion
D. 2/3 outer cartilaginous portion and 1/3 inner bony
portion
E. 2/3 outer bony portion and 1/3 inner cartilaginous
portion

TOPNOTCH MEDICAL BOARD PREP ANATOMY SUPEREXAM Page 77 of 94


For inquiries visit www.topnotchboardprep.com.ph or email us at topnotchmedicalboardprep@gmail.com
TOPNOTCH MEDICAL BOARD PREP ANATOMY SUPEREXAM
For inquiries visit www.topnotchboardprep.com.ph or email us at topnotchmedicalboardprep@gmail.com
Item QUESTION EXPLANATION AUTHOR TOPNOTCH
# EXAM
590 A patient had been in a car accident wherein he suffered The pathology of a cape-like loss of sensory TIMOTHY MIDTERM 1
superfical lacerations but the neurologic exam was normal. 6 functions is indicative of syringomyelia or a TANG LEE EXAM - FEB
months later, he complains of loss of pain and temperature central cord syndrome. Anterior cord will result to SAY, MD 2014
sensations in the hands and forearm. This progressed to sensory deficits from a dermatome level down up (TOP 4 - AUG
include mild weakness of hand grip and extension of the to the lower extremities. Brown-Sequard is a 2013 MED
sensory loss to the arm and shoulders. An MRI of the spinal hemisection syndrome. Posterior cord syndrome BOARDS;
cord shows a cyst in a segment of the spinal cord. This will show loss of position sense and vibration. TOPNOTCH
pathology is consistent with? Tabes dorsalis is also a posterior cord syndrome. MD)
A. Anterior cord syndrome
B. Central cord syndrome
C. Brown-Sequard's syndrome
D. Posterior cord syndrome
E. Tabes dorsalis

591 During thyroidectomy, the surgeon accidently ligated the All muscles of phonation are innervated by the TIMOTHY MIDTERM 1
right recurrent laryngeal nerve. Which of the following recurrent larngeal nerve except for the TANG LEE EXAM - FEB
actions on the vocal cord by the muscles of phonation is cricothyroid which is responsible for tensing the SAY, MD 2014
SPARED? vocal cords (TOP 4 - AUG
A. Tenses 2013 MED
B. Abducts BOARDS;
C. Adducts TOPNOTCH
D. Relaxes MD)
E. Opens

592 A athlete had just finsihed running a marathon. He has All muscles of the thoracic wall are responsible for TIMOTHY MIDTERM 1
labored breathing. The following muscles are responsible for inspiration except serratus post inf, internal TANG LEE EXAM - FEB
inspiration EXCEPT? intercostal and tranverse thoracis muscles which SAY, MD 2014
A. External intercostal are responsible for labored expiration. (TOP 4 - AUG
B. Innermost intercostal 2013 MED
C. Serratus posterior superior BOARDS;
D. Serratus posterior inferior TOPNOTCH
E. Subcostal MD)

593 In the embryology of the primitive heart, which of the Review the fate of the fetal heart. Bulbus cordis TIMOTHY MIDTERM 1
following is INCORRECTLY paired? becomes the right ventricle. Sinus venosus (right TANG LEE EXAM - FEB
A. Truncus arteriosus = Pulmonary trunk horn) becomes the right atrium. SAY, MD 2014
B. Bulbus cordis = Right atrium (TOP 4 - AUG
C. Sinus venosus = Coronary sinus 2013 MED
D. Primitive atrium = Left atrium BOARDS;
E. Primitive ventricle = Left Ventricle TOPNOTCH
MD)

594 A 12-yr old boy was eating hurriedly when the school bell In sitting and standing positions, foreign bodies TIMOTHY MIDTERM 1
rang which signaled the end of recess period. He suddenly tend to lodge in the posterior basal BPS of the TANG LEE EXAM - FEB
choked while swallowing the siomai whole without chewing. right lower lobe. Please review the usual location SAY, MD 2014
Heimlick maneuver was done but was unsuccessful to of foreign body aspiration with different positions (TOP 4 - AUG
dislodge the food from the airways. Rigid bronchoscopy was of the body. 2013 MED
done. The siomai is most likely lodge in the? BOARDS;
A. Posterior bronchopulmonary segment of the right TOPNOTCH
superior lobe MD)
B. Posterior bronchopulmonary segment of the left
superior lobe
C. Superior bronchopulmonary segment of the right
inferior lobe
D. Inferior lingular bronchopulmonary segment of the left
superior lobe
E. Posterior basal bronchopulmonary segment of the right
inferior lobe

595 The abdominal cavity is lined by the peritoneum. Which fo For A, the parietal peritonuem should line the TIMOTHY MIDTERM 1
the following is TRUE regarding the peritoneum? internal surface of the wall and the visceral TANG LEE EXAM - FEB
A. The peritoneum is divided into two continuous layers, peritoneum should invest the organs. Parietal is SAY, MD 2014
the parietal peritoneum investing the organs and the visceral sensitive to pressure, pain and heat, with pain (TOP 4 - AUG
peritoneum lining the internal surface of the the abdomino- generally localized. Visceral is sensitive to 2013 MED
pelvic wall. stretching and pain is poorly localized. The BOARDS;
B. The pain fibers subserving the parietal peritoneum is ascending and descending colons are TOPNOTCH
primarily sensitive to stretching. retroperitoneal not the transverse colon. MD)
C. The duodenum, pancreas, and transverse colon are
retroperitoneal organs.
D. Pain in the visceral peritoneum is generally localized.
E. The peritoneal cavity is the space between the parietal
and visceral peritoneum that contains 50 mL of serous fluid
to allow sliding between the layers.

TOPNOTCH MEDICAL BOARD PREP ANATOMY SUPEREXAM Page 78 of 94


For inquiries visit www.topnotchboardprep.com.ph or email us at topnotchmedicalboardprep@gmail.com
TOPNOTCH MEDICAL BOARD PREP ANATOMY SUPEREXAM
For inquiries visit www.topnotchboardprep.com.ph or email us at topnotchmedicalboardprep@gmail.com
Item QUESTION EXPLANATION AUTHOR TOPNOTCH
# EXAM
596 A patient undergone upper GI endoscopy for recurrent The esophagus is 25 cm long but from the incisor, TIMOTHY MIDTERM 1
peptic ulcer disease. At what level of the gastrointestinal 15 cm - UES, 18 cm - thoracic inlet, 25 cm - arch of TANG LEE EXAM - FEB
tract is the scope in at 40 cm from the central incisors? the aorta, 40 cm - LES SAY, MD 2014
A. Lower esophageal sphincter (TOP 4 - AUG
B. Cardia of the stomach 2013 MED
C. Body of the stomach BOARDS;
D. Antrum of the stomach TOPNOTCH
E. Pylorus MD)

597 A trauma surgeon operated on a patient who had a gunshot Jejunum has deeper red color, thick and heavy TIMOTHY MIDTERM 1
wound. In running the bowel, the surgeon differentiates the wall, 2-4 cm in caliber, greater vascularity, long TANG LEE EXAM - FEB
small intestines because he knows that the jejunum has the vasa recta, few and large arcades, less fat, tall and SAY, MD 2014
following characteristics EXCEPT? large plica, and few lymphoid nodules.\ (TOP 4 - AUG
A. Deeper red color 2013 MED
B. Thinner walls BOARDS;
C. Fewer arcades TOPNOTCH
D. Longer vasa recta MD)
E. Larger plica circularis

598 What is TRUE regarding the ligametum teres? The ligamentum teres is a remnant of the left TIMOTHY MIDTERM 1
A. It forms a border of between the quadrate lobe and the umbilical vein. The right umbilical vein obliterates TANG LEE EXAM - FEB
left lobe of the liver. during the fetal period. It is empty and does not SAY, MD 2014
B. It is a remnant of the right umbilical vein. conduct any vessels. The true division of the left (TOP 4 - AUG
C. It contains the portal triad that enters the liver at the and right lobes of the liver is the imaginary line 2013 MED
porta hepatis. passing through the gallbladder and IVC. BOARDS;
D. It divides the liver into a left and right lobe. TOPNOTCH
E. It is a remnant of the ductus venosus MD)

599 In doing pelvic surgery, you noticed that there is a nerve Lateral border of the psoas - iliohypogastric, TIMOTHY MIDTERM 1
found anterior to the psoas muscle, this is probably the? ilioinguinal and femoral nerve TANG LEE EXAM - FEB
A. Iliohypogastric nerve Anterior - genitofemoral SAY, MD 2014
B. Ilioinguinal nerve Medial - obturator (TOP 4 - AUG
C. Genitofemoral nerve 2013 MED
D. Femoral nerve BOARDS;
E. Obturator nerve TOPNOTCH
MD)

600 Which of the following is NOT located in the deep perineal Root of the penis is found in the superficial TIMOTHY MIDTERM 1
space in males? perineal space. TANG LEE EXAM - FEB
A. Root of the penis SAY, MD 2014
B. Bulbourethral gland (TOP 4 - AUG
C. Dorsal nerve of the penis 2013 MED
D. Sphincter urethra BOARDS;
E. Deep transverse perineal muscle TOPNOTCH
MD)

601 Adult venous system is derived from the following Three Pairs of Veins are present in embryo: RACHELLE FINAL EXAM
embryonic structures, EXCEPT: vitelline veins, umbilical veins, common cardinals MENDOZA, - FEB 2013
A. Vitelline vein veins. Common Cardinal Veins collect the MD (TOP 9 -
B. Umbilical vein deoxygenated blood from the body wall. Umbilical AUG 2012
C. Cardinal vein Veins carrying the oxygenated blood from the MED
D. Sinus venosus Placenta. Vitelline veins bring the deoxygenated BOARDS;
E. None of the above blood from the Yolk Sac & gut TOPNOTCH
MD)

602 EG, 47-year old female, known hypertensive and diabetic, Bell's palsy is a peripheral facial nerve palsy, RACHELLE FINAL EXAM
went to your clinic due to sudden onset of left-sided facial usually presenting with inability to wrinkle MENDOZA, - FEB 2013
weakness. On examination, you noted that there was forehead, inability to close eyelid and lagging of MD (TOP 9 -
asymmetry in facial expression with lagging noted on the left facial expression on the side affected. Sensation is AUG 2012
side, blunting of left nasolabial fold and inability to close the typically intact, since facial sensation comes from MED
left eyelid. If you are considering Bell's palsy, which of the CN V (trigeminal). BOARDS;
following is consistent with this finding? TOPNOTCH
A. Loss of sensation on left side of the face MD)
B. Loss of sensation on right side of the face
C. Unable to wrinkle left side of the forehead
D. Able to wrinkle both side of the forehead
E. A and C

603 A 25-year old male presents with severe colicky lumbar pain Since the pain is felt on the lumbar area, the RACHELLE FINAL EXAM
on the right. KUB was done, revealing a radioluscent stone, highest area of anatomic ureteral constriction MENDOZA, - FEB 2013
that is most likely lodged where? should be considered in this patient. MD (TOP 9 -
A. Mid-ureter AUG 2012
B. Point where ureter enters the pelvic brim MED
C. Ureteropelvic junction BOARDS;
D. Point where ureter enters the bladder TOPNOTCH
E. Any of the above MD)

TOPNOTCH MEDICAL BOARD PREP ANATOMY SUPEREXAM Page 79 of 94


For inquiries visit www.topnotchboardprep.com.ph or email us at topnotchmedicalboardprep@gmail.com
TOPNOTCH MEDICAL BOARD PREP ANATOMY SUPEREXAM
For inquiries visit www.topnotchboardprep.com.ph or email us at topnotchmedicalboardprep@gmail.com
Item QUESTION EXPLANATION AUTHOR TOPNOTCH
# EXAM
604 Pringle maneuver involves compression of which of the Pringle manuever is used to control bleeding RACHELLE FINAL EXAM
following? during a liver procedure without compromising MENDOZA, - FEB 2013
A. Portal vein too much of its blood supply. It involves MD (TOP 9 -
B. Bile duct compression of the heaptic arty, as most of the AUG 2012
C. Hepatic artery liver's blood supply would come from the portal MED
D. All of the above vein. BOARDS;
TOPNOTCH
MD)
605 Femoral sheath encloses the following, EXCEPT: The femoral sheath does not enclose the femoral RACHELLE FINAL EXAM
A. Femoral nerve nerve. MENDOZA, - FEB 2013
B. Femoral artery MD (TOP 9 -
C. Femoral vein AUG 2012
D. Lymph vessels MED
E. None of the above BOARDS;
TOPNOTCH
MD)
606 The following structures pass through the esophageal All of these pass through the esophageal aperture, RACHELLE FINAL EXAM
perture of the diaphragm, EXCEPT: alongside esophagus (T10 level) MENDOZA, - FEB 2013
A. Right vagus MD (TOP 9 -
B. Left gastric vesses AUG 2012
C. Esophagus MED
D. Lymphatics from lower 3rd of esophagus BOARDS;
E. None of the above TOPNOTCH
MD)

607 A 26-year old male came from a fist fight and was brought to All EOMs have a common origin at the annulus of RACHELLE FINAL EXAM
your clinic due to difficulty moving the eyeball. MRI revealed zinn except for 2: superior oblique (roof) and MENDOZA, - FEB 2013
isolated rupture of annulus of zinn. Which of the following inferior oblique (floor) MD (TOP 9 -
eye muscles whose origin would remain unaffected? AUG 2012
A. Lateral rectus MED
B. Superior rectus BOARDS;
C. Inferior rectus TOPNOTCH
D. Superior oblique MD)
E. None of the above

608 During thyroidectomy, the first major blood vessel to be The anterior jugular vein descends through the RACHELLE FINAL EXAM
encountered is: midline and joins the opposite vein via the jugular MENDOZA, - FEB 2013
A. Anterior jugular vein notch just above the sternum. This will then join MD (TOP 9 -
B. External jugular vein the external jugular vein deep to the SCM. AUG 2012
C. Internal jjugular vein MED
D. Subclavian vein BOARDS;
E. A and B TOPNOTCH
MD)

609 Most common blood supply of the SA node comes from the: 90% of the time, the SA node is supplied with a RACHELLE FINAL EXAM
A. Left coronary artery nodal branch coming from the right coronary MENDOZA, - FEB 2013
B. Right coronary artery artery. MD (TOP 9 -
C. Left circumflex artery AUG 2012
D. Right marginal artery MED
E. A and C BOARDS;
TOPNOTCH
MD)
610 If the 3rd branchial arch fails to form or mature, which of the The 3rd branchial arch gives rise to the greater RACHELLE FINAL EXAM
following clinical manifestation would be observed? horn of the hyoid, stylopharyngeus muscle and CN MENDOZA, - FEB 2013
A. No facial expression would be appreciated IX. Therefore, if it fails to form or mature to adult MD (TOP 9 -
B. Deafness structures, patient would be expected to present AUG 2012
C. Hoarseness with deficiency of the greater horn of the hyoid MED
D. Decreased secretion of parotid gland and lack of stylopharynegus muscle. And since CN BOARDS;
E. All of the above IX would be absent, there would be also be poor TOPNOTCH
gag reflex and decreased parotid gland secretion. MD)

611 The following layers of the anterior abdominal wall give rise External oblique gives rise to external spermatic RACHELLE FINAL EXAM
to an equivalent covering of scrotal and spermatic fascia, fascia. Internal oblique gives rise to cremasteric MENDOZA, - FEB 2013
EXCEPT: muscle and fascia. Transversalis fascia gives rise to MD (TOP 9 -
A. Subcutaneous fascia internal spermatic fascia. Transversus abdominis AUG 2012
B. External oblique does not contributw to any portion of the scrotum. MED
C. Internal oblique BOARDS;
D. Transversus abdominis TOPNOTCH
E. A and C MD)

612 A 35-year old male presented with enlarged scrotum on the Varicocole is the engorgement of the venous RACHELLE FINAL EXAM
left. Examination of the scrotal content revealed appearance drainage of the testis, the pampiniform plexus. It MENDOZA, - FEB 2013
of "bag of worms." Which of the following clinical usually presents with enlargement of the scrotum MD (TOP 9 -
condition/s is/are considered? and examination would reveal "bag of worms" AUG 2012
A. Varicocoele appearance. Varicocoel is more commonly MED
B. Orchitis observed in the left scrotum, which drains to the BOARDS;
C. Renal carcinoma left renal vein and is subjected to higher pressure TOPNOTCH
D. B and C than the right. Hence, varicocoele is usually MD)
E. A and C associated with renal tumors or other pathologies
that may increase renal vein pressure.

TOPNOTCH MEDICAL BOARD PREP ANATOMY SUPEREXAM Page 80 of 94


For inquiries visit www.topnotchboardprep.com.ph or email us at topnotchmedicalboardprep@gmail.com
TOPNOTCH MEDICAL BOARD PREP ANATOMY SUPEREXAM
For inquiries visit www.topnotchboardprep.com.ph or email us at topnotchmedicalboardprep@gmail.com
Item QUESTION EXPLANATION AUTHOR TOPNOTCH
# EXAM
613 Which enteroendocrine cells produce a compound that I cells produce cholecystokinin, S cells produce RACHELLE FINAL EXAM
promotes insulin secretion and inhibits glucagon secretin, K cells produce gastrin inhinitory peptide MENDOZA, - FEB 2013
production? and L cells produce glucagon-like peptide 1 (GLP MD (TOP 9 -
A. L cells 1). The latter stimulate insulin production and AUG 2012
B. S cells inhibits glucagon secretion. MED
C. K cells BOARDS;
D. I cells TOPNOTCH
E. None of the above MD)

614 An athlete suffered from fracture of his left medial malleolus, Structures running behind the medial malleolus: RACHELLE FINAL EXAM
transecting all structures behind it. Which of the following tibialis poetrior, flexor digitorum longus, posterior MENDOZA, - FEB 2013
will NOT be affected? tibial artery, posterior tibial nerve and flexor MD (TOP 9 -
A. Tibialis posterior hallocis longus. AUG 2012
B. Posterior tibial artery MED
C. Posterior tibial nerve BOARDS;
D. Flexor hallucis longus TOPNOTCH
E. Deep peroneal nerve MD)

615 Tranverse fracture of the humeral mid-shaft would result to: Tranverse fracture of the mid-shaft of the RACHELLE FINAL EXAM
A. Loss of sensation on anterior forearm humerus would injure the deep brachial artery MENDOZA, - FEB 2013
B. Claw hand and the radial nerve. The radial nerve typically MD (TOP 9 -
C. Ape hand supplies posterior compartments of arm and AUG 2012
D. Inability to extend wrist forearm and thereby resposible for allowing MED
E. A and C extension of elbow and wrist. BOARDS;
TOPNOTCH
MD)
616 A stroke patient was found to have a necrotic focus on the Brodmann areas 44 and 45 correspond to Broca's RACHELLE FINAL EXAM
brain, specifically identified as Brodmann areas 44 and 45. speech area. MENDOZA, - FEB 2013
The patient is then expected to: MD (TOP 9 -
A. have difficulty in speaking but can comprehend AUG 2012
B. have difficulty understanding but can speak MED
C. not be able to hear and interpret sounds BOARDS;
D. not be able to associate visual input to other information TOPNOTCH
E. none of the above MD)

617 A trauma patient was found to have total obliteration of CN V-2 (maxillary branch of the trigeminal nerve) RACHELLE FINAL EXAM
foramen rotundum. Which of the following clinical condition passes through foramen rotundum. The maxillary MENDOZA, - FEB 2013
would be a finding in this patient? branch is resposnible for somatic sensory supply MD (TOP 9 -
A. Loss of sensation along the maxillary area of the face of the maxillary area of the face and mouth. CN V- AUG 2012
B. Loss of sensation along the mandibular area of the face 3 passes through foramen ovale, while the middle MED
C. Epidural hemorrhage meningeak artery passes through foramen BOARDS;
D. A and C spinosum. TOPNOTCH
E. B and C MD)

618 The widest and most dilatable portion of the male urethra: Prostatic urethra is the widest and most dilatable. RACHELLE FINAL EXAM
A. Membranous Membranous part is the shortest and least MENDOZA, - FEB 2013
B. Spongy dilatable. The penile/spongy urethra is the MD (TOP 9 -
C. Penile narrowest part of the entire urethre. AUG 2012
D. Prostatic MED
E. B and C BOARDS;
TOPNOTCH
MD)
619 Arthrocentesis is most difficult to carry out in which joint? Arthrocentesis of the ankle is more difficult than RACHELLE FINAL EXAM
A. Knee that of the other joints. The knee is the most MENDOZA, - FEB 2013
B. Elbow common and the easiest joint for the physician to MD (TOP 9 -
C. Ankle aspirate AUG 2012
D. Wrist MED
E. A and C BOARDS;
TOPNOTCH
MD)
620 Histopath cross-section of the gallbladder shows the The gallbladder is lined with tall columnar cells RACHELLE FINAL EXAM
following features, EXCEPT: with microvilli, except the terminal ducts MENDOZA, - FEB 2013
A. Tall columnar epithelium with microvilli (cuboidal epithelium). No muscularis mucosae MD (TOP 9 -
B. Muscularis mucosae can be found. AUG 2012
C. Muscularis MED
D. Adventitia BOARDS;
E. A and C TOPNOTCH
MD)

621 A part of anterior pituitary gland which has a small collar VON ANDRE DIAGNOSTIC
cells around infundibular stalk that mainly contains the MEDINA, MD EXAM - AUG
portal venules. (TOP 4 - FEB 2012
A. Pars Distalis 2012 MED
B. Pars Intermedia BOARDS;
C. Pars Tuberalis TOPNOTCH
D. Pars nervosa MD)
E. None of the above

TOPNOTCH MEDICAL BOARD PREP ANATOMY SUPEREXAM Page 81 of 94


For inquiries visit www.topnotchboardprep.com.ph or email us at topnotchmedicalboardprep@gmail.com
TOPNOTCH MEDICAL BOARD PREP ANATOMY SUPEREXAM
For inquiries visit www.topnotchboardprep.com.ph or email us at topnotchmedicalboardprep@gmail.com
Item QUESTION EXPLANATION AUTHOR TOPNOTCH
# EXAM
622 Which portion or zone of the liver acinus is exposed to blood periportal= exposed to blood high in O2, involved VON ANDRE DIAGNOSTIC
high in oxygen content and nutrients , and is primarily in eclampsia Midzonal= intermediate, involved MEDINA, MD EXAM - AUG
involved in the pathology of preeclampsia /eclampsia? in yellow fever centrilobular= (TOP 4 - FEB 2012
A. Centrilobular exposed to blood low in O2, involved in ischemic 2012 MED
B. Periportal injury, right sided heart failure BOARDS;
C. Midzonal TOPNOTCH
D. Space of Disse MD)
E. Sinusoids

623 The inferior parathyroid glands are derived from which In humans, the superior parathyroid glands are VON ANDRE DIAGNOSTIC
branchial pouch? derived from the 4th branchial pouch while the MEDINA, MD EXAM - AUG
A. 1st inferior parathyroid glands come from 3rd (TOP 4 - FEB 2012
B. 2nd branchial pouch. 2012 MED
C. 3rd BOARDS;
D. 4th TOPNOTCH
E. A and B MD)

624 Diaphragm is the most important muscle of inspiration. embryonic origin


VON ANDRE DIAGNOSTIC
Which embryonic structure gives rise to the central tendon Septum transversum= central tendon MEDINA, MD EXAM - AUG
of the diaphragm which is fused partially with the fibrous Pleuroperitoneal membrane= bulk of muscular (TOP 4 - FEB 2012
pericardium of the heart? part of diaphragm Body wall mesenchyme= 2012 MED
A. Pleuroperitoneal membrane peripheral muscular part of the diaphragm
BOARDS;
B. Dorsal mesentery of the esophagus dorsal mesentery= diphragmatic crura TOPNOTCH
C. Septum transversum MD)
D. Body wall mesenchyme
E. All of the above

625 This structure encloses the kidney and suprarenal gland and renal fascia is also known as gerota's fascia VON ANDRE DIAGNOSTIC
blends with the fascia of IVC and aorta. MEDINA, MD EXAM - AUG
A. Fibrous capsule (TOP 4 - FEB 2012
B. Perirenal fat 2012 MED
C. Gerota's fascia BOARDS;
D. Renal fascia TOPNOTCH
E. C and D MD)

626 A 46 year old patient came to emergency department due to sternal border= right ventricle VON ANDRE DIAGNOSTIC
gunshot wound. During physical examination, it was found MEDINA, MD EXAM - AUG
out that the bullet entered the sternal area. Which chamber (TOP 4 - FEB 2012
of the heart will most probably be injured in this case? 2012 MED
A. Right atrium BOARDS;
B. Right ventricle TOPNOTCH
C. Left ventricle MD)
D. Left atrium
E. Diaphragm

627 An aneurysm of the abdominal aorta at the aortic hiatus of the aortic hiatus of the diaphragm transmits the VON ANDRE DIAGNOSTIC
the diaphragm is most likely to result in the compression of azygous vein and thoracic duct. MEDINA, MD EXAM - AUG
which of the following pairs of structures? (TOP 4 - FEB 2012
A. Vagus nerve and azygous vein 2012 MED
B. Esophagus and vagus nerve BOARDS;
C. Azygous vein and thoracic duct TOPNOTCH
D. Thoracic duct and vagus nerve MD)
E. Inferior vena cava and phrenic nerve

628 During appendectomy performed at McBurney's point, Iliohypogastric nerve runs medially and inferiorly VON ANDRE DIAGNOSTIC
which of the following structures is most likely to be injured? between the internal oblique and transverse MEDINA, MD EXAM - AUG
A. Deep circumflex femoral artery abdominal muscles at Mcburney's point. (TOP 4 - FEB 2012
B. Inferior epigastric artery 2012 MED
C. Iliohypogastric nerve BOARDS;
D. Genitofemoral nerve TOPNOTCH
E. Spermatic cord MD)

629 As the uterine artery passes from the internal iliac artery to the ureter runs under the uterine artery near the VON ANDRE DIAGNOSTIC
the uterus, it crosses which of the following structures that is cervix; thus, the ureter is sometimes mistakenly MEDINA, MD EXAM - AUG
sometimes mistakenly ligated during pelvic surgery? ligated during pelvic surgery (TOP 4 - FEB 2012
A. Ovarian artery 2012 MED
B. Ovarian ligament BOARDS;
C. Uterine tube TOPNOTCH
D. Ureter MD)
E. Round ligament of the uterus

TOPNOTCH MEDICAL BOARD PREP ANATOMY SUPEREXAM Page 82 of 94


For inquiries visit www.topnotchboardprep.com.ph or email us at topnotchmedicalboardprep@gmail.com
TOPNOTCH MEDICAL BOARD PREP ANATOMY SUPEREXAM
For inquiries visit www.topnotchboardprep.com.ph or email us at topnotchmedicalboardprep@gmail.com
Item QUESTION EXPLANATION AUTHOR TOPNOTCH
# EXAM
630 During a game, a 25 year old baseball player receives a VON ANDRE DIAGNOSTIC
severe blow to the head that fractures the optic canal. Which MEDINA, MD EXAM - AUG
of the following pairs of structures is most likely to be (TOP 4 - FEB 2012
damaged? 2012 MED
A. Optic nerve and ophthalmic vein BOARDS;
B. Ophthalmic vein and ophthalmic nerve TOPNOTCH
C. Ophthalmic artery and and optic nerve MD)
D. Ophthalmic nerve and optic nerve
E. Ophthalmic artery and ophthalmic vein

631 What is the lining epithelium of the endocervical canal? the cervix has a simple columnar epithelium, VON ANDRE DIAGNOSTIC
A. Simple columnar except for the external portion which is continous MEDINA, MD EXAM - AUG
B. Simple squamous with the vagina which has a stritified squamous (TOP 4 - FEB 2012
C. Stratified squamous epithelium 2012 MED
D. Cuboidal BOARDS;
E. Pseudostratified columnar TOPNOTCH
MD)

632 Communication of the scala vestibuli and scala tympani the scala vestibuli and the scala tympani are VON ANDRE DIAGNOSTIC
occurs at the: actually one perilymphatic space separated by the MEDINA, MD EXAM - AUG
A. Oval window cochlear duct (scala media). The scala vestibuli (TOP 4 - FEB 2012
B. Round window and tympani communicate with each other at 2012 MED
C. Helicotrema helicotrema. BOARDS;
D. Endolymphatic sac TOPNOTCH
E. None of the above MD)

633 Which of the following is a characteristic of the cornea? the cornea is the transparent anterior portion of VON ANDRE DIAGNOSTIC
A. It represents the anterior portion of the tunica vasculosa the tunica fibrosa, the outer covering of the eye; MEDINA, MD EXAM - AUG
B. It is the anterior transparent portion of the tunica fibrosa thus it forms the anterior wall of the anterior (TOP 4 - FEB 2012
C. It forms the anterior boundary of the posterior chamber chamber of the eye. 2012 MED
of the eye BOARDS;
D. It is devoid of nerve endings TOPNOTCH
E. All of the above MD)

634 In a surgical procedure in which you are to remove the Cystic artery arises mainly from right hepatic VON ANDRE DIAGNOSTIC
gallbladder and therefore you need to ligate the cystic artery. artery MEDINA, MD EXAM - AUG
This artery arises from which of the following? (TOP 4 - FEB 2012
A. Proper hepatic artery 2012 MED
B. Right hepatic artery BOARDS;
C. Left hepatic artery TOPNOTCH
D. Right gastric artery MD)
E. Common hepatic artery

635 Mrs. Succor suffered from an embolic infarct. She came in to VON ANDRE DIAGNOSTIC
the emergency department complaining of difficulty in MEDINA, MD EXAM - AUG
speaking and left arm weakness. Which of the following (TOP 4 - FEB 2012
arteries is most likely affected? 2012 MED
A. Middle cerebral artery BOARDS;
B. Anterior cerebral artery TOPNOTCH
C. Vertebral artery MD)
D. Posterior cerebal artery
E. Basilar artery

636 The following veins drain into the coronary sinus EXCEPT: great cardiac vein, small cardiac vein and middle VON ANDRE DIAGNOSTIC
A. Great cardiac vein cardiac vein drain into the coronary sinus. MEDINA, MD EXAM - AUG
B. Small cardiac vein (TOP 4 - FEB 2012
C. Middle cardiac vein 2012 MED
D. Anterior cardiac vein BOARDS;
E. None of the above TOPNOTCH
MD)

637 Mrs. Jennifer Marikina is suffering from an ovarian VON ANDRE DIAGNOSTIC
malignancy with hepatic metastasis. After 3 days, you MEDINA, MD EXAM - AUG
noticed that her abdomen is getting bigger. On physical (TOP 4 - FEB 2012
examination, there is a positive fluid wave which is highly 2012 MED
suggestive that the patient has ascites. Her attending BOARDS;
physician decided to do a paracentesis. The PREFERRED TOPNOTCH
entry site for paracentesis is? MD)
A. Halfway between the umbilicus and the pubic symphysis
B. Between umbilicus and ASIS, lateral to rectus abdominis
muscle, Left
C. Between umbilicus and ASIS, lateral to the restus
abdominis muscle, Right
D. Between umbilicus and ASIS, medial to the Rectus
abdominis
E. Any of the above

TOPNOTCH MEDICAL BOARD PREP ANATOMY SUPEREXAM Page 83 of 94


For inquiries visit www.topnotchboardprep.com.ph or email us at topnotchmedicalboardprep@gmail.com
TOPNOTCH MEDICAL BOARD PREP ANATOMY SUPEREXAM
For inquiries visit www.topnotchboardprep.com.ph or email us at topnotchmedicalboardprep@gmail.com
Item QUESTION EXPLANATION AUTHOR TOPNOTCH
# EXAM
638 A 58 year old G8P8 underwent TAHBSO with bilateral lymph VON ANDRE DIAGNOSTIC
node dissection for the high grade cervical cancer. Few hours MEDINA, MD EXAM - AUG
post-surgery, she complained that she cannot feel her right (TOP 4 - FEB 2012
inner leg. On your physical examination, impaired right thigh 2012 MED
adduction was observed. The iatrogenically damaged BOARDS;
structure is most likely the: TOPNOTCH
A. Femoral nerve MD)
B. Sciatic nerve
C. Obturator nerve
D. Pudendal nerve
E. Superior gluteal nerve

639 The following structures are parts of the osteon, EXCEPT: VON ANDRE DIAGNOSTIC
A. Osteocytes MEDINA, MD EXAM - AUG
B. Concentric Lamellae (TOP 4 - FEB 2012
C. Canaliculi 2012 MED
D. Volkmann's canal BOARDS;
E. A and B TOPNOTCH
MD)

640 The structure which has a head and a base that are united by SIMILAR TO PREVIOUS BOARD EXAM VON ANDRE DIAGNOSTIC
two limbs and is connected to the oval window is the? CONCEPT/PRINCIPLE MEDINA, MD EXAM - AUG
A. Incus (TOP 4 - FEB 2012
B. Malleus 2012 MED
C. Stapes BOARDS;
D. Stapedius TOPNOTCH
E. Tensor Tympani MD)

641 All of the following are contained in the vertebral canal, Spinal Nerve is located OUTSIDE the vertebral LITO JAY DIAGNOSTIC
EXCEPT? canal (exit throught intervertebral foramen) MACARAIG, EXAM - AUG
A. Spinal cord MD (TOP 8 - 2013
B. Spinal nerve FEB 2013
C. Dorsal nerve roots MED
D. Ventral nerve roots BOARDS;
E. Meninges TOPNOTCH
MD)

642 Which of the following is/are true regarding the "yes" joint? The atlanto-occipital joint is found between the C1 LITO JAY DIAGNOSTIC
A. Atlanto-occipital joint and occipital condyles. It is also called the "yes MACARAIG, EXAM - AUG
B. Atlanto-axial joint joint" because it facilitates extension and flexion of MD (TOP 8 - 2013
C. Joint between C1 and occipital condyles the neck. FEB 2013
D. Joint between C1 and C2 MED
E. choices A and C are both correct BOARDS;
F. choices B and D are both correct TOPNOTCH
MD)

643 The vertebral levels are used as reference points for locating At the level of C4 vertebra, hyoid bone and LITO JAY DIAGNOSTIC
anatomical landmarks. Which among the following choices bifurcation of common carotid artery are found. MACARAIG, EXAM - AUG
is/are found at the level of C4 vertebra? The start of trachea and Cricoid cartilage are MD (TOP 8 - 2013
A. Hyoid bone found at the level of C6 vertebra. FEB 2013
B. Start of trachea MED
C. Cricoid cartilage BOARDS;
D. choices A and B are both correct TOPNOTCH
E. choices B and C are both correct MD)

644 This is a condition which resulted from chronic stress Spondylolisthesis involves degeneration of LITO JAY DIAGNOSTIC
fracture of Pars interarticularis seen as "collar around the pedicles of lumbar vertebra. Osteomyelitis MACARAIG, EXAM - AUG
neck of Scottie dog" on Xray. Usually at the L5 vertebra of involves infection within vertebral bodies. AS MD (TOP 8 - 2013
adolescent athletes. involves the bamboo spine deformity. HNP is the FEB 2013
A. Spondylosis protrusion of Nucleus Pulposus due to a break on MED
B. Spondylolisthesis the annulus fibrosus. BOARDS;
C. Ankylosing spondylitis TOPNOTCH
D. Osteoyelitis MD)
E. Herniation of Nucleus Pulposus

645 During an ER duty, a 24 y/o male patient was brought in due The ideal site for lumbar puncture is 1 level above LITO JAY DIAGNOSTIC
to seizure. History revealed severe ear pain 2 weeks prior to or below the spinous process of L4 vertebra MACARAIG, EXAM - AUG
consult, followed by occasional headaches. Four days PTC, (between L3-L4 or between L4-L5). MD (TOP 8 - 2013
patient complained of fever 38-39 degrees Celsius. Upon PE, FEB 2013
patient had (+) meningeal signs. You are entertaining MED
bacterial meningitis. What vertebral level is ideal to insert BOARDS;
the needle for Lumbar Tap? TOPNOTCH
A. between L1 and L2 MD)
B. between L2 and L3
C. between L3 and L4
D. choices A and B are both correct
E. choices B and C are both correct

TOPNOTCH MEDICAL BOARD PREP ANATOMY SUPEREXAM Page 84 of 94


For inquiries visit www.topnotchboardprep.com.ph or email us at topnotchmedicalboardprep@gmail.com
TOPNOTCH MEDICAL BOARD PREP ANATOMY SUPEREXAM
For inquiries visit www.topnotchboardprep.com.ph or email us at topnotchmedicalboardprep@gmail.com
Item QUESTION EXPLANATION AUTHOR TOPNOTCH
# EXAM
646 A 32 year-old male patient was brought in the ER due to The Right Ventricle is the chamber of the heart LITO JAY DIAGNOSTIC
stabbing. Patient was hypotensive and bleeding profusely that lies directly behind the sternum. MACARAIG, EXAM - AUG
from the chest. Despite all efforts of resuscitation, patient MD (TOP 8 - 2013
subsequently expired. If the wound was located between the FEB 2013
4th and 5th rib Left parasternal line, the chamber of the MED
heart that is most likely affected was? BOARDS;
A. Left Atrium TOPNOTCH
B. Left Ventricle MD)
C. Right Atrium
D. Right Ventricle

647 Volksmann Ischemic Contracture is characterized by severe In a Volksmann ischemic contracture, the fracture LITO JAY DIAGNOSTIC
flexion of the affected arm, wrist and fingers. The following is found at the supracondylar area of humerus and MACARAIG, EXAM - AUG
structures are affected EXCEPT? NOT at the midshaft. MD (TOP 8 - 2013
A. Midshaft of humerus FEB 2013
B. Brachial artery MED
C. Median nerve BOARDS;
D. A and B TOPNOTCH
E. B and C MD)

648 The following are the boundaries of the Anatomical Snuffbox, The first three choices are the ONLY boundaries of LITO JAY DIAGNOSTIC
EXCEPT? the Anatomical Snuffbox. MACARAIG, EXAM - AUG
A. Extensor Pollicis Longus MD (TOP 8 - 2013
B. Extensor Pollicis Brevis FEB 2013
C. Abductor Pollicis Longus MED
D. Abductor Pollicis Brevis BOARDS;
E. There is NO exception TOPNOTCH
MD)

649 These granulocytes are described with "bilobulated nucleus" Eosinophils are characterized by their bilobulated LITO JAY DIAGNOSTIC
and plays roles in fighting viral infections, as proved by the nucleus with numerous pink-orange granules. MACARAIG, EXAM - AUG
RNAses they contain. They are highly involved in Asthma MD (TOP 8 - 2013
A. Basophil pathophysiology and Helminthic infections. They FEB 2013
B. Eosinophil contain RNAses which proves their effectivity MED
C. Neutrophil against viral infections. BOARDS;
D. NK cells TOPNOTCH
E. Lymphocytes MD)

650 While jogging, a 40 y/o male was bumped by a tricycle on the The common peroneal nerve is the root of LITO JAY DIAGNOSTIC
lateral side of his left leg. Patient was unable to evert his left superficial and deep peroneal nerve. If it was MACARAIG, EXAM - AUG
foot, with weakness in plantar flexion but normal affected, signs of deep peroneal nerve lesion must MD (TOP 8 - 2013
dorsiflexion. What nerve was affected? also be present (like weak or no dorsiflexion). The FEB 2013
A. Common Peroneal Nerve superficial peroneal nerve is responsible for MED
B. Superficial Peroneal Nerve eversion and plantar flexion. Tibial nerve is for BOARDS;
C. Deep Peroneal Nerve plantar flexion only. Common fibular nerve is the TOPNOTCH
D. Tibial Nerve other term for common peroneal nerve. MD)
E. Common Fibular Nerve

651 You are assigned in the Neuro-OPD and saw a patient who There are 3 anatomical lobes of the cerebellum. LITO JAY DIAGNOSTIC
presented with gait instability. Patient was NOT able to The anterior lobe (paleo-cerebellum), MACARAIG, EXAM - AUG
perform the "heel-to-shin" movement. You know that Flocculonodular lobe (archi-cerebellum), and the MD (TOP 8 - 2013
Cerebellum is involved. But what lobe? posterior lobe (neo-cerebellum). Anterior lobe FEB 2013
A. Anterior lobe lesions will result to gait instability and patient MED
B. Flocculo-nodular lobe will be unable to perform the heel-to-shin BOARDS;
C. Posterior lobe movement. TOPNOTCH
D. Archi-cerebellum MD)
E. Neo-cerebellum

652 Which among the following spinal cord lesions will present In ALS and Brown-Sequard (hemisection), both LITO JAY DIAGNOSTIC
as both upper and lower motor neuron lesion? the cortico-spinal tract (Upper MN) and the MACARAIG, EXAM - AUG
A. Poliomyelitis anterior horn cells (Lower MN) are affected. MD (TOP 8 - 2013
B. Amyotrophic Lateral Sclerosis FEB 2013
C. Brown-sequard MED
D. Choices A and B are both correct BOARDS;
E. Choices B and C are both correct TOPNOTCH
MD)

653 The muscles of facial expression are derived from? Derivatives of pharyngeal Arch: 1. Muscles of LITO JAY DIAGNOSTIC
A. 1st Pharyngeal Pouch Mastication, 2. Muscles pf Facial expression, 3. MACARAIG, EXAM - AUG
B. 1st Pharyngeal Arch Muscles of deglutition, 4-6. Muscles for Phonation MD (TOP 8 - 2013
C. 2nd Pharyngeal Pouch FEB 2013
D. 2nd Pharyngeal Arch MED
E. None of the above BOARDS;
TOPNOTCH
MD)

TOPNOTCH MEDICAL BOARD PREP ANATOMY SUPEREXAM Page 85 of 94


For inquiries visit www.topnotchboardprep.com.ph or email us at topnotchmedicalboardprep@gmail.com
TOPNOTCH MEDICAL BOARD PREP ANATOMY SUPEREXAM
For inquiries visit www.topnotchboardprep.com.ph or email us at topnotchmedicalboardprep@gmail.com
Item QUESTION EXPLANATION AUTHOR TOPNOTCH
# EXAM
654 The internal carotid artery (ICA) supplies the anterior part of ICA is a derivative of the distal part of the 3rd LITO JAY DIAGNOSTIC
the brain through the Circle of Willis. The ICA is a derivative Aortic Arch. The proximal part will give rise to the MACARAIG, EXAM - AUG
of? Common carotid artey. MD (TOP 8 - 2013
A. Proximal Part of the 3rd Aortic arch FEB 2013
B. Distal Part of the 3rd Aortic Arch MED
C. Right part of the 4th Aortic Arch BOARDS;
D. Left part of the 4th Aortic Arch TOPNOTCH
E. None of the above MD)

655 The genioglossus is an extrinsic muscle of the tongue and is The genioglossus is innervated by CN XII and LITO JAY DIAGNOSTIC
responsible for what movement? protrudes the tongue. MACARAIG, EXAM - AUG
A. elevation MD (TOP 8 - 2013
B. depression FEB 2013
C. protrusion MED
D. retraction BOARDS;
E. None of the above TOPNOTCH
MD)

656 A patient with Mallory Weiss tear came to ER due to severe The Mallory Weiss tear is most commonly located LITO JAY DIAGNOSTIC
hematemesis, pulsating in character. Knowing the diagnosis, at the gastro-esophageal junction. The bleeding is MACARAIG, EXAM - AUG
you already know that the bleeding blood vessel is? described as "pulsating", hence arterial. And the MD (TOP 8 - 2013
A. Middle esophageal artery blood supply of the distal 3rd of esophagus is the FEB 2013
B. Middle esophageal vein Left gstric artery which is a branch of the celiac MED
C. Left gastric artery trunk. BOARDS;
D. Left gastric vein TOPNOTCH
E. Descending aorta MD)

657 Which among the following statements is/are true regarding Jejunum is shorter, thicker, more vascular and LITO JAY DIAGNOSTIC
the difference between Jejunum and Ileum? with long vasa recta. Ileum has more fat, arcades, MACARAIG, EXAM - AUG
A. Jejunum is shorter and lymph nodes. MD (TOP 8 - 2013
B. Ileum has more fat FEB 2013
C. Jejunum is thicker MED
D. B and C only BOARDS;
E. All of the above TOPNOTCH
MD)

658 Which among the following statements is/are true regarding The round ligament of the liver is AKA the LITO JAY DIAGNOSTIC
the round ligament of the liver? Ligamentum Teres. It is a derivative from the left MACARAIG, EXAM - AUG
A. AKA falciform ligament umbilical vein. The right umbilical vein obliterates MD (TOP 8 - 2013
B. AKA ligamentum teres without any derivative in the later stage of life. FEB 2013
C. From the Left umbilical vein MED
D. A and C only BOARDS;
E. B and C only TOPNOTCH
MD)

659 The rectum is divided into three (3) anatomical divisions. the superior rectal artery is from IMA, the middle LITO JAY DIAGNOSTIC
And its inferior part (distal 3rd) is being supplied by a direct rectal artery is from the Internal iliac artery, and MACARAIG, EXAM - AUG
branch of? the inferior rectal artery is from the the internal MD (TOP 8 - 2013
A. Inferior Mesenteric Artery pudendal artery which is also a branch of the FEB 2013
B. Sigmoidal artery internal iliac artery MED
C. Internal pudendal artery BOARDS;
D. A and C only TOPNOTCH
E. All of the above MD)

660 Which among the following statements is/are true regarding A normal adult kidney contains 12 pyramids. 2 LITO JAY DIAGNOSTIC
the anatomy of the kidney? major calyces and 3 minor calyces. MACARAIG, EXAM - AUG
A. 12 pyramids MD (TOP 8 - 2013
B. 2 major calyces FEB 2013
C. 3 minor calyces MED
D. A and C only BOARDS;
E. All of the above TOPNOTCH
MD)

661 Which of the following is bilobed? Basophils and Eosinophils have nuclei that are HAZEL MIDTERM 2
A. neutrophils bilobed. Neutrophils are multinucleated. KAREN RAZ, - AUG 2013
B. basophils MD (TOP 6 -
C. lymphocytes FEB 2013
D. monocytes MED
E. Platelets BOARDS;
TOPNOTCH
MD)

TOPNOTCH MEDICAL BOARD PREP ANATOMY SUPEREXAM Page 86 of 94


For inquiries visit www.topnotchboardprep.com.ph or email us at topnotchmedicalboardprep@gmail.com
TOPNOTCH MEDICAL BOARD PREP ANATOMY SUPEREXAM
For inquiries visit www.topnotchboardprep.com.ph or email us at topnotchmedicalboardprep@gmail.com
Item QUESTION EXPLANATION AUTHOR TOPNOTCH
# EXAM
662 Which of the following statements is true? Bronchioles have lesser cartilage and mucous HAZEL MIDTERM 2
A. Bronchioles have cartilage plates similar to those of glands than bronchi. However, smooth muscle KAREN RAZ, - AUG 2013
bronchi. cells are greater in the distal airways. Bronchioles MD (TOP 6 -
B. Bronchioles have greater amount of smooth muscle than are formed when the airways are reduced to < 1 FEB 2013
bronchi. mm. MED
C. Bronchioles are formed when the airways are reduced to BOARDS;
a diameter of < 1 cm. TOPNOTCH
D. Bronchioles have greater number of mucous glands than MD)
bronchi.
E. None of the above.

663 The following are found inside the lacunae, except? Osteoblast are found outside of lacunar cells in HAZEL MIDTERM 2
A. Osteocytes bone. They are responsible for bone KAREN RAZ, - AUG 2013
B. Osteoclast formation.Osteoblasts that become trapped in the MD (TOP 6 -
C. Osteoblast bone matrix and remain isolated in lacunae FEB 2013
D. Chrondrocyte become osteocytes. MED
E. None of the above BOARDS;
TOPNOTCH
MD)
664 Protein production for extracellular distribution are Protein produced from the RER are transported HAZEL MIDTERM 2
produced in the: out of the cell for ectracellular use. Free ribosomes KAREN RAZ, - AUG 2013
A. Smooth ER produce protein for intracellular use. Smooth ER MD (TOP 6 -
B. Rough ER functions in cholesterol synthesis and all other FEB 2013
C. Free ribosomes food products except complex carbohydrates. MED
D. Nucleus Lysozyme produces enzymes for self - BOARDS;
E. Lysozyme desctruction. TOPNOTCH
MD)

665 In the respiratory tract, what differentiates the terminal Respiratory bronchioles begin where alveoli start HAZEL MIDTERM 2
bronchioles from the respiratory bronchioles? to appear. There are lesser smooth muscle, cilia KAREN RAZ, - AUG 2013
A. The presence of alveoli in respiratory bronchioles and greater amount of elastic fiber. MD (TOP 6 -
B. Presence of greater number of smooth muscle in FEB 2013
respiratory bronchioles MED
C. Presence of greater number of cilia in respiratory BOARDS;
bronchioles TOPNOTCH
D. Lesser elastic fibers in respiratory bronchioles MD)
E. All of the above

666 Cancer of the esophagus presents with early metastasis due Esophagus lacks serosal layer, that is why HAZEL MIDTERM 2
to the absence of which layer? esophageal cancer when diagnosed is usually in KAREN RAZ, - AUG 2013
A. epithelium the advanced stages due to early dissemination MD (TOP 6 -
B. submucosa and metastasis. FEB 2013
C. Muscularis propria MED
D. Lamina propria BOARDS;
E. Serosa TOPNOTCH
MD)

667 A 15- year old triathlethe complains of low back pain which a - spondylolisthesis, b - spondylosis, c - HAZEL MIDTERM 2
is aggravated by activity. Physical examination was normal, ankylosing spondylitis KAREN RAZ, - AUG 2013
no neurological symptoms were noted. After x-ray of his MD (TOP 6 -
lumbar spine, he was diagnosed to have spondylolysis. What FEB 2013
could be the finding which led to this diagnosis? MED
A. Degeneration of pedicles of lumbar vertebrae BOARDS;
B. Formation of bone spurs and osteophytes TOPNOTCH
C. Ossification of annulus fibrosus MD)
D. Scottie dog collar lesion on L5
E. None of the above

668 A patient came to you presenting with fever, nuchal rigidity A lumbar puncture is a puncture into the HAZEL MIDTERM 2
and changes in sensorium. Meningitis was among the subarachnoid space of the spinal cord to obtain KAREN RAZ, - AUG 2013
differentials. Lumbar tap was contemplated. At what level is cerebrospinal fluid (CSF) for clinical MD (TOP 6 -
lumbar tap usually performed? investigation, to remove excess fluid FEB 2013
A. T12 - L1 or inject medication. Lumbar tap is performed MED
B. L1 - L2 between L3 - L4 or L4 and L-5. BOARDS;
C. L3 TOPNOTCH
D. L4 - L5 MD)
E. None of the above

669 A trauma patient presented with difficulty of breathing. CXR CTT insertion is done at the 5th - 6th anterior HAZEL MIDTERM 2
showed blunting of the right costophrenic angle. You decided axillary line and directed inferiorly to drain blood KAREN RAZ, - AUG 2013
to do a CTT insertion to drain the fliud. At what level would or fluid from the thoracic cavity. The tube is MD (TOP 6 -
you insert the CTT tube? directed superiorly to drain pneumothorax. FEB 2013
A. 4th - 5th ICS R Anterior Axillary Line directed superiorly MED
B. 4th -5th ICS R Mid - axillary line directed inferiorly. BOARDS;
C. 5th - 6th ICS R Mid- axillary line directed superiorly TOPNOTCH
D. 5th - 6th ICS R Anterior Axillary Line directed inferiorly MD)
E. None of the above

TOPNOTCH MEDICAL BOARD PREP ANATOMY SUPEREXAM Page 87 of 94


For inquiries visit www.topnotchboardprep.com.ph or email us at topnotchmedicalboardprep@gmail.com
TOPNOTCH MEDICAL BOARD PREP ANATOMY SUPEREXAM
For inquiries visit www.topnotchboardprep.com.ph or email us at topnotchmedicalboardprep@gmail.com
Item QUESTION EXPLANATION AUTHOR TOPNOTCH
# EXAM
670 Amphiarthrosis: Joints are classified based on their mobility and HAZEL MIDTERM 2
A. Symphysis pubis composition. They are classified as synarthrosis KAREN RAZ, - AUG 2013
B. Radio - ulnar joint (fibrous) - immovable, amphiarthrosis MD (TOP 6 -
C. Atlanto - axial joint (cartilagenous) - slightly movable, diarthrosis FEB 2013
D. wrist (synovial) - freely movable. Symphysis pubis is a MED
E. Distal interphalangeal joints fibrocartilagenous joint that is slightly movable. BOARDS;
TOPNOTCH
MD)
671 This muscle is considered the main flexor of the forearm and The muscles innervated by musculocutaneous HAZEL MIDTERM 2
its innervation. nerve includes the following: coracobrachialis KAREN RAZ, - AUG 2013
A. Coracobrachialis muscle, musculocutaneous nerve (flexes and adducts ARM), biceps brachii (flexes MD (TOP 6 -
B. Biceps brachii muscle, musculocuteneous nerve and main supinator of FOREARM), brachialis FEB 2013
C. Brachialis muscle, musculocutaneous nerve (main flexor of the FOREARM). Deltoid muscle is MED
D. Deltoid muscle, musculocutaneous nerve innervated by axillary nerve, a part of the rotator BOARDS;
E. None of the above cuff muscles. TOPNOTCH
MD)

672 Psoas sign is considered positive if there is_____________? psoas sign is a medical sign that indicates HAZEL MIDTERM 2
A. Increased tenderness at the RLQ with palpation of the LLQ irritation to the iliopsoas group of hip flexors in KAREN RAZ, - AUG 2013
B. Pain at the RLQ with hip flexion and internal rotation of the abdomen, and consequently indicates that the MD (TOP 6 -
the leg inflamed appendix is retrocaecal in orientation (as FEB 2013
C. Pain on passive extension of the right thigh the iliopsoas muscle is retroperitoneal). It is MED
D. Pain on slow compression of the abdominal wall with elicited by performing the psoas test by passively BOARDS;
rapid release extending the thigh of a patient lying on his side TOPNOTCH
E. All of the above with knees extended, or asking the patient to MD)
actively flex his thigh at the hip.

673 A complication of radical mastectomy which presents as Injury to the long thoracic nerve produces a HAZEL MIDTERM 2
difficulty in horizontal extension of the upper extermity winged scapula due to paralysis of serratus KAREN RAZ, - AUG 2013
involves injury to the? anterior. Intercostobrachial nerve involvement MD (TOP 6 -
A. Long thoracic nerve causes loss of sensation of the upper inner arm. FEB 2013
B. Intercostobrachial nerve Paralysis of the pectoralis major muscle with MED
C. Medial pectoral nerve weakness of abduction is due to damaged medial BOARDS;
D. Thoracodorsal nerve pectoral nerve. TOPNOTCH
E. Axillary nerve MD)

674 A 60 year old smoker presents with hemoptysis, weight loss, The signs and symptoms presented are due to the HAZEL MIDTERM 2
weakness of the upper limbs, fatigue and difficulty in paraneoplastic syndromes accompanying oat KAREN RAZ, - AUG 2013
swallowing. Electrolyte levels show hyponatremia. What cell/small cell carcinoma of the lung. Lambert - MD (TOP 6 -
could be the possible diagnosis for this patient? Eaton Syndrome presents with muscle weakness, FEB 2013
A. Oat cell CA of the lung fatigue and difficulty in swallowing. SIADH is MED
B. Adenocarcinoma of the lung manifested by hyponatremia (dilutional), and BOARDS;
C. Squamous cell CA ACTH production. Smoking is a major TOPNOTCH
D. PTB predisposing factor in small cell and squamous cell MD)
E. None of the above CA.

675 Lingula is the embryonic counterpart of which lobe of the Lingula is used to denote a projection of the upper HAZEL MIDTERM 2
lung? lobe of the left lung that serves as the homologue KAREN RAZ, - AUG 2013
A. Right lower lobe of the right middle lobe. MD (TOP 6 -
B. Right upper lobe FEB 2013
C. Right middle lobe MED
D. Left Lower lobe BOARDS;
E. Left Upper lobe TOPNOTCH
MD)

676 The inferior diaphragmatic surface of the heart is composed Base/Posterior surface - Left atrium; Apex: LV at HAZEL MIDTERM 2
mainly of the______? 5th ICS MCL; Sternal border: RV; Diaphragmatic KAREN RAZ, - AUG 2013
A. Left atrium surface: LV MD (TOP 6 -
B. Left ventricle FEB 2013
C. Right atrium MED
D. Right Ventricle BOARDS;
E. Base of the heart TOPNOTCH
MD)

677 Leaflets of the valves which separates the right atrium form leaflets of the tricuspid valve: anterior, posterior, HAZEL MIDTERM 2
the right ventricle except: septal. KAREN RAZ, - AUG 2013
A. anterior MD (TOP 6 -
B. septal FEB 2013
C. medial MED
D. posterior BOARDS;
E. None of the above TOPNOTCH
MD)

678 Paracentesis sites: 2 cms below the umbillicus, HAZEL MIDTERM 2


lateral border of the rectus abdominis muscle KAREN RAZ, - AUG 2013
superior and medial to the ASIS MD (TOP 6 -
FEB 2013
MED
BOARDS;
TOPNOTCH
MD)

Most ideal site for paracentesis?


TOPNOTCH MEDICAL BOARD PREP ANATOMY SUPEREXAM Page 88 of 94
For inquiries visit www.topnotchboardprep.com.ph or email us at topnotchmedicalboardprep@gmail.com
TOPNOTCH MEDICAL BOARD PREP ANATOMY SUPEREXAM
For inquiries visit www.topnotchboardprep.com.ph or email us at topnotchmedicalboardprep@gmail.com
Item QUESTION EXPLANATION AUTHOR TOPNOTCH
# EXAM
A. midway between symphysis and umbillicus
B. 1 cm to the left of the lateral border of the rectus
abdominis at the level of the umbillicus
C. Immediately above the symphysis pubis
D. Mc Burney's point
E. above the umbillicus

679 Which of the following associations is correct? HAZEL MIDTERM 2


A. Superficialis fascia : external spermatic fascia KAREN RAZ, - AUG 2013
B. Transversalis fascia : Dartos muscle MD (TOP 6 -
C. Internal Oblique muscle : Cremasteric muscle FEB 2013
D. Peritoneum : none MED
E. None of the above BOARDS;
TOPNOTCH
MD)

680 Which of the ff structures of the renal hilus is most Order from anterior to posterior: Vein, Artery, HAZEL MIDTERM 2
posterior? Pelvis KAREN RAZ, - AUG 2013
A. Renal artery MD (TOP 6 -
B. Renal vein FEB 2013
C. Gerotas fascia MED
D. Perirenal fat BOARDS;
E. Renal pelvis TOPNOTCH
MD)
681 A 15 yo boy complaining of pain in the lower right part of the Answer: B. it is supplied by a long small artery MICHELLE MIDTERM 1
anterior abdominal wall was seen by a physician. Upon that does not anastomose with other arteries JAY - AUG 2013
examination, he was found to have a temperature of 38.3OC. (pp. 234, Snell’s Clinical Anatomy By Regions, 8th FRANCISCO,
He had a furred tongue and was extremely tender in the edition.) Notes: All choices are factors that MD (TOP 9 -
lower right quadrant. The abdominal muscles in that area contribute to the appendix’s predilection to FEB 2013
were found to be firm on palpation and became more spastic infection except choice B which is a factor for MED
when increased pressure was applied. The organ most likely predisposition to perforation. BOARDS;
affected in this case has the following contributory factors TOPNOTCH
why it is predisposed to infection, except; MD)
A. It is a long, narrow, blind-ended tube, which encourages
stasis of large-bowel contents.
B. It is supplied by a long small artery that does not
anastomose with other arteries.
C. It has a large amount of lymphoid tissue in its wall.
D. The lumen has a tendency to become obstructed by
hardened intestinal contents, which leads to further
stagnation of its contents.

682 The following statements concerning the pancreas are Answer: B. The main pancreatic duct opens into MICHELLE MIDTERM 1
correct except which? the third part of the duodenum. JAY - AUG 2013
A. The pancreas receives part of the arterial supply from the Notes: The main pancreatic duct opens into the FRANCISCO,
splenic artery. second part of the duodenum, at about its middle, MD (TOP 9 -
B. The main pancreatic duct opens into the third part of the with the bile duct on the major duodenal papilla. FEB 2013
duodenum. Sometimes, the main duct drains separately into MED
C. The uncinate process of the pancreas projects from the the duodenum. BOARDS;
head of the pancreas. TOPNOTCH
D. The bile duct (CBD) lies posterior to the head of the MD)
pancreas.

683 What part of the ciliary body does the aqueous humor is Answer: D. Pars plicata (Robbins and Cotran MICHELLE MIDTERM 1
formed? Pathologic Basis of Disease, 8th ed.) JAY - AUG 2013
A. Pars optica FRANCISCO,
B. Pars nervosa MD (TOP 9 -
C. Pars pigmentosa FEB 2013
D. Pars plicata MED
BOARDS;
TOPNOTCH
MD)
684 The intercellular spaces in the stratum spinosum of the Answer: A. Membrane-coating granules MICHELLE MIDTERM 1
epidermis contain lipid-containing sheets that are Notes: Membrane-coating granules are present in JAY - AUG 2013
impermeable to water. This material is released from keratinocytes in the stratum spinosum (and FRANCISCO,
A. Membrane-coating granules stratum granulosum). The contents of these MD (TOP 9 -
B. Keratohyalin granules granules are released into the intercellular spaces FEB 2013
C. Langerhans cells to help waterproof the skin. Keratinocytes in the MED
D. Sebaceous glands stratum granulosum also possess keratohyalin BOARDS;
granules; these contain proteins that bind keratin TOPNOTCH
filaments together. MD)

TOPNOTCH MEDICAL BOARD PREP ANATOMY SUPEREXAM Page 89 of 94


For inquiries visit www.topnotchboardprep.com.ph or email us at topnotchmedicalboardprep@gmail.com
TOPNOTCH MEDICAL BOARD PREP ANATOMY SUPEREXAM
For inquiries visit www.topnotchboardprep.com.ph or email us at topnotchmedicalboardprep@gmail.com
Item QUESTION EXPLANATION AUTHOR TOPNOTCH
# EXAM
685 Which of the following is lined by an epithelium containing Answer: B. Terminal bronchiole MICHELLE MIDTERM 1
ciliated cells and Clara cells? Notes: Terminal bronchioles are lined by a simple JAY - AUG 2013
A. Trachea cuboidal epithelium containing ciliated cells and FRANCISCO,
B. Terminal bronchiole Clara cells. Clara cells can divide and regenerate MD (TOP 9 -
C. Intrapulmonary bronchi both cell types. FEB 2013
D. Alveolar duct MED
BOARDS;
TOPNOTCH
MD)
686 Which of the following cells in the inner ear are involved in Answer: A. Hair cells in the maculae MICHELLE MIDTERM 1
detecting movements of the head? Notes: Neuroepithelial hair cells in the maculae of JAY - AUG 2013
A. Hair cells in the maculae the saccule and the utricule detect linear FRANCISCO,
B. Cells of Hensen movement of the head. These cells are connected MD (TOP 9 -
C. Hair cells in the organ of Corti to the vestibular portion of the acoustic nerve. FEB 2013
D. Inner pillar cells MED
BOARDS;
TOPNOTCH
MD)
687 The following statements concerning the stomach are true; Answer: B. The lesser curvature of the stomach MICHELLE MIDTERM 1
except receives its blood supply from the right and JAY - AUG 2013
A. The splenic artery runs along the upper border of the left gastroepiploic arteries FRANCISCO,
pancreas and lies behind the stomach Notes: The greater curvature of the stomach MD (TOP 9 -
B. The lesser curvature of the stomach receives its blood receives its blood supply from the right and left FEB 2013
supply from the right and left gastroepiploic arteries gastroepiploic arteries. MED
C. The lymph drainage from the gastroesophageal junction BOARDS;
passess to the celiac lymph nodes TOPNOTCH
D. The lesser sac lies behind the stomach MD)

688 Ductus/vas deferens is the main content of the spermatic Answer: D. Ilioinguinal nerve MICHELLE MIDTERM 1
cord. Among these choices, which is not found in the Notes: Ilioinguinal nerve travels along/on the JAY - AUG 2013
spermatic cord? superficial part of the spermatic cord. Hernia sac FRANCISCO,
A. Hernia sac may or may not be found inside the spermatic MD (TOP 9 -
B. Genital branch of the genitofemoral nerve cord. FEB 2013
C. Cremasteric muscle fibers MED
D. Ilioinguinal nerve Mnemonics for contents of the spermatic cord: BOARDS;
Piles Don’t Contribute To A Good Sex Life TOPNOTCH
• Pampiniform venous plexus MD)
• Ductus deferens
• Cremasteric muscle
• Testicular Artery
• Artery of the ductus deferens
• Genital branch of the genitofemoral nerve
• Sympathetic nerve fibers
• Lymphatic vessels

689 You asked a patient to look to the left side during a Answer: A. Right medial longitudinal fasciculus MICHELLE MIDTERM 1
neurologic examination. Patient’s right eye cannot look to his Notes: JAY - AUG 2013
left, with (+) left eye nystagmus, but convergence is still • Left abducens nucleus or right cerebral cortex – FRANCISCO,
intact. In order to produce these symptoms, you suspect that neither eye can look left with a slow drift to the MD (TOP 9 -
the lesion is most probably located in the _____________. right FEB 2013
A. Right medial longitudinal fasciculus • Left abducens nerve – left eye can’t look to the MED
B. Left abducens nucleus left BOARDS;
C. Right cerebral cortex • Right medial longitudinal fasciculus – right eye TOPNOTCH
D. Left abducens nerve can’t look left, left eye nystagmus, and MD)
convergence is intact

690 Name the ocular defect if the lesion is found in the right Answer: A. Left homonymous hemianopsia MICHELLE MIDTERM 1
lateral geniculate body in the thalamus Notes: JAY - AUG 2013
A. Left homonymous hemianopsia • Left optic nerve lesion – left eye anopsia (left FRANCISCO,
B. Left nasal and temporal hemianopsia nasal and temporal hemianopsia) MD (TOP 9 -
C. Bitemporal heteronymous hemianopsia • Right calcarine cortex lesion – left homonymous FEB 2013
D. Right nasal hemianopsia hemianopsia MED
• Right LGB lesion – left homonymous BOARDS;
hemianopsia TOPNOTCH
• Optic chiasm lesion – bitemporal heteronymous MD)
hemianopsia
• Right lateral compression of the optic chiasm (as
in aneurysms of the internal carotid artery) – right
nasal hemianopsia
691 What is true regarding lymphatic anatomy? Lymphatic system begins as network of valveless MICHELLE MIDTERM 1
A. The limb lymphatics are valveless. capillaries in the superficial dermis. RBC & bact. JAY - AUG 2013
B. The lymphatic system begins just below the dermis as a enter lymph vessels by separating endothelial cells FRANCISCO,
network of fine capillaries at their junctions. MD (TOP 9 -
C. RBC and bacteria do not enter the lymphatic capillaries FEB 2013
D. Extrinsic factors (muscle contraction, arterial pulsations, MED
respiratory movement and massage)aid in the movement of BOARDS;
lymph flow. TOPNOTCH
E. All of the above. MD)

TOPNOTCH MEDICAL BOARD PREP ANATOMY SUPEREXAM Page 90 of 94


For inquiries visit www.topnotchboardprep.com.ph or email us at topnotchmedicalboardprep@gmail.com
TOPNOTCH MEDICAL BOARD PREP ANATOMY SUPEREXAM
For inquiries visit www.topnotchboardprep.com.ph or email us at topnotchmedicalboardprep@gmail.com
Item QUESTION EXPLANATION AUTHOR TOPNOTCH
# EXAM
692 A 10-yr old girl is brought to her doctor because of Poland syndrome is a congenital defect of about 1 MICHELLE MIDTERM 1
hypoplasia of her right breast. On closer examination, she is in 30,000 births. Hallmark is chest wall anomaly: JAY - AUG 2013
found to have asymmetry of the chest wall itself. Which of partial absence of sternal head of pec.major m., FRANCISCO,
the following is true regarding this syndrome? hypo/aplasia of breast & SC tissue and possible MD (TOP 9 -
A. This affects i in every 10,000 live births. complete absence of pec.minor m. FEB 2013
B. Deformity of the thoracoacromial joint is characteristic of MED
this syndrome. BOARDS;
C. Absence of the sternum is characteristic of this syndrome TOPNOTCH
D. Absence of the sterna head of the pectoralis muscle is a MD)
characteristic of this syndrome.
E. Pectus excavatum is part of this syndrome..

693 Which of the following statements is most accurate Third arch anomalies are rare & appear in lower MICHELLE MIDTERM 1
regarding branchial cleft anomalies? neck. Second branchial arch anomalies are the JAY - AUG 2013
A. Third arch anomalies are common most common & ends at the tonsillar fossa. FRANCISCO,
B. Type II 1st branchial anomalies are the most common MD (TOP 9 -
over-all. FEB 2013
C. The glossopharyngeal nerve is asso. with the 3rd branchial MED
arch. BOARDS;
D. Second arch anomalies end in pyriform sinus. TOPNOTCH
MD)

694 True of liver anatomy: The right lobe consists of posterior and anterior MICHELLE MIDTERM 1
A. The right lobe extends to the umbilical fissure and segments. The left lobe consists of medial JAY - AUG 2013
falciform ligament. (quadrate) and lateral segment divided by FRANCISCO,
B. The left lobe end at the falciform ligament. falciform ligament. The caudate can be considered MD (TOP 9 -
C. The quadrate lobe is a portion of the medial segment of the anatomically independent from both R/L lobes FEB 2013
right lobe. because it receives portal & arterial blood supply MED
D. The lateral segments of the left lobe in the American from both sides. BOARDS;
system consists of segments II & III. TOPNOTCH
MD)

695 All except one drains into the right atrium through the The anterior cardiac vein together w/ smallest MICHELLE MIDTERM 1
coronary sinus: cardiac opens directly into the right atrium. JAY - AUG 2013
FRANCISCO,
A. small cardiac MD (TOP 9 -
B. middle cardiac FEB 2013
C. anterior cardiac MED
D. great cardiac BOARDS;
TOPNOTCH
MD)
696 The muscle and central tendon of the diaphragm develop Pleuroperitoneal membranes-peripheral area of MICHELLE MIDTERM 1
from which of the following embryonic structures? diaphragmatic pleura & upper/lower pleural JAY - AUG 2013
A. Septum transversum surface; Dorsal mesentery - crura FRANCISCO,
B. Pleuroperitoneal membranes MD (TOP 9 -
C. Dorsal mesentery of esophagus FEB 2013
D. Ventral mesentery of esophagus MED
BOARDS;
TOPNOTCH
MD)
697 The blood supply to the thoracic wall mainly comes from the These 9 pairs of post. intercostal arteries arise MICHELLE MIDTERM 1
_________ artery. from the thoracic aorta, the right arteries, having JAY - AUG 2013
A. Lateral thoracic longer course than the left ones. FRANCISCO,
B. Internal thoracic MD (TOP 9 -
C. Posterior intercostals FEB 2013
D. Subcostal MED
BOARDS;
TOPNOTCH
MD)
698 I.M. injections should be given in the upper outer quadrant of *review ana nerve distribution of lower limbs MICHELLE MIDTERM 1
the buttocks to prevent damage to which of the following JAY - AUG 2013
nerves? FRANCISCO,
A. Sciatic MD (TOP 9 -
B. Obturator FEB 2013
C. Superior gluteal MED
D. Lateral femoral cutaneous BOARDS;
TOPNOTCH
MD)
699 This ligament prevents posterior dislocation of the femur at The rectus femoris as part of quadriceps crosses MICHELLE MIDTERM 1
the knee joint. the hip joint anteriorly and its tendon crosses the JAY - AUG 2013
A. Anterior cruciate knee joint anteriorly resulting in flexion of the FRANCISCO,
B. Ischiofemoral thigh and extension of the leg. MD (TOP 9 -
C. Lateral collateral FEB 2013
D. Posterior cruciate MED
BOARDS;
TOPNOTCH
MD)

TOPNOTCH MEDICAL BOARD PREP ANATOMY SUPEREXAM Page 91 of 94


For inquiries visit www.topnotchboardprep.com.ph or email us at topnotchmedicalboardprep@gmail.com
TOPNOTCH MEDICAL BOARD PREP ANATOMY SUPEREXAM
For inquiries visit www.topnotchboardprep.com.ph or email us at topnotchmedicalboardprep@gmail.com
Item QUESTION EXPLANATION AUTHOR TOPNOTCH
# EXAM
700 A neurologic exam of a 34 yr-old man reveals a direct and MICHELLE MIDTERM 1
consensual light reflex in his left eye but neither in his right JAY - AUG 2013
eye. The nerve involved is the: FRANCISCO,
A. right optic nerve MD (TOP 9 -
B. left optic nerve FEB 2013
C. right oculomotor MED
D. left oculomotor BOARDS;
TOPNOTCH
MD)

TOPNOTCH MEDICAL BOARD PREP ANATOMY SUPEREXAM Page 92 of 94


For inquiries visit www.topnotchboardprep.com.ph or email us at topnotchmedicalboardprep@gmail.com
TOPNOTCH MEDICAL BOARD PREP ANATOMY SUPEREXAM
For inquiries visit www.topnotchboardprep.com.ph or email us at topnotchmedicalboardprep@gmail.com

Item # KEY 87 B 175 D 263 B 351 B


ANSWER 88 A 176 E 264 D 352 C
1 E 89 A 177 C 265 B 353 A
2 E 90 A 178 D 266 C 354 C
3 B 91 B 179 A 267 B 355 A
4 A 92 B 180 C 268 D 356 B
5 C 93 C 181 A 269 B 357 A
6 B 94 D 182 D 270 B 358 C
7 E 95 B 183 C 271 B 359 B
8 D 96 C 184 D 272 A 360 A
9 B 97 B 185 B 273 B 361 C
10 A 98 E 186 C 274 A 362 A
11 C 99 C 187 B 275 A 363 B
12 C 100 C 188 A 276 A 364 C
13 A 101 D 189 C 277 D 365 C
14 E 102 D 190 B 278 D 366 C
15 C 103 A 191 E 279 C 367 D
16 C 104 C 192 D 280 B 368 C
17 D 105 B 193 C 281 A 369 D
18 E 106 D 194 B 282 D 370 D
19 C 107 A 195 B 283 B 371 A
20 D 108 B 196 A 284 B 372 A
21 A 109 A 197 B 285 E 373 B
22 C 110 C 198 D 286 C 374 D
23 D 111 E 199 C 287 C 375 D
24 B 112 D 200 C 288 B 376 D
25 A 113 B 201 C 289 E 377 C
26 B 114 C 202 A 290 A 378 C
27 D 115 D 203 A 291 E 379 C
28 D 116 A 204 C 292 C 380 B
29 C 117 B 205 B 293 E 381 B
30 D 118 E 206 D 294 D 382 C
31 C 119 A 207 B 295 C 383 C
32 C 120 C 208 B 296 A 384 D
33 B 121 C 209 B 297 C 385 E
34 B 122 A 210 A 298 D 386 A
35 C 123 E 211 B 299 B 387 D
36 E 124 E 212 D 300 D 388 D
37 E 125 C 213 C 301 C 389 B
38 A 126 B 214 A 302 A 390 D
39 B 127 A 215 D 303 391 A
40 C 128 B 216 C 304 D 392 C
41 C 129 C 217 B 305 A 393 B
42 B 130 D 218 A 306 B 394 C
43 A 131 B 219 D 307 D 395 B
44 B 132 A 220 B 308 C 396 A
45 A 133 B 221 C 309 C 397 B
46 C 134 C 222 A 310 C 398 B
47 A 135 A 223 C 311 A 399 B
48 E 136 B 224 C 312 B 400 C
49 E 137 D 225 B 313 C 401 D
50 C 138 D 226 A 314 A 402 E
51 D 139 D 227 A 315 C 403 D
52 C 140 E 228 C 316 A 404 C
53 C 141 B 229 C 317 B 405 B
54 A 142 A 230 C 318 E 406 A
55 B 143 A 231 B 319 B 407 B
56 E 144 E 232 A 320 D 408 D
57 B 145 B 233 C 321 C 409 A
58 B 146 B 234 D 322 D 410 C
59 C 147 D 235 C 323 A 411 D
60 B 148 B 236 C 324 E 412 A
61 C 149 A 237 D 325 C 413 D
62 A 150 A 238 C 326 C 414 B
63 D 151 E 239 A 327 B 415 A
64 B 152 B 240 B 328 B 416 E
65 B 153 E 241 A 329 E 417 B
66 BONUS 154 A 242 C 330 B 418 E
67 E 155 B 243 A 331 E 419 B
68 D 156 B 244 C 332 C 420 B
69 B 157 B 245 C 333 C 421 B
70 C 158 A 246 D 334 D 422 A
71 B 159 C 247 C 335 D 423 B
72 B 160 D 248 D 336 C 424 C
73 D 161 D 249 B 337 B 425 D
74 E 162 A 250 B 338 E 426 B
75 B 163 B 251 B 339 B 427 B
76 D 164 B 252 B 340 C 428 D
77 C 165 E 253 C 341 C 429 B
78 A 166 A,B 254 C 342 A 430 B
79 D 167 C 255 D 343 B 431 D
80 C 168 A 256 D 344 B 432 C
81 A 169 E 257 C 345 C 433 A
82 D 170 B 258 B 346 C 434 B
83 B 171 A 259 C 347 D 435 A
84 B 172 B 260 C 348 C 436 D
85 A 173 C 261 E 349 C 437 B
86 D 174 B 262 D 350 B 438 D
TOPNOTCH MEDICAL BOARD PREP ANATOMY SUPEREXAM Page 93 of 94
For inquiries visit www.topnotchboardprep.com.ph or email us at topnotchmedicalboardprep@gmail.com
TOPNOTCH MEDICAL BOARD PREP ANATOMY SUPEREXAM
For inquiries visit www.topnotchboardprep.com.ph or email us at topnotchmedicalboardprep@gmail.com
439 C 528 C 617 A
440 D 529 C 618 D
441 C 530 D 619 C
442 E 531 C 620 B
443 B 532 B 621 C
444 B 533 B 622 B
445 C 534 D 623 C
446 A 535 D 624 C
447 D 536 B 625 E
448 D 537 A 626 B
449 E 538 C 627 C
450 D 539 B 628 C
451 B 540 D 629 D
452 A 541 B 630 C
453 E 542 C 631 A
454 C 543 C 632 C
455 D 544 C 633 B
456 E 545 B 634 B
457 B 546 D 635 A
458 D 547 D 636 D
459 C 548 C 637 B
460 D 549 A 638 C
461 D 550 B 639 D
462 B 551 A 640 C
463 A 552 E 641 B
464 D 553 C 642 E
465 C 554 B 643 A
466 C 555 A 644 A
467 A 556 C 645 C
468 A 557 D 646 D
469 A 558 A 647 A
470 D 559 A 648 D
471 D 560 E 649 B
472 A 561 A 650 B
473 B 562 A 651 A
474 D 563 A 652 E
475 E 564 B 653 D
476 A 565 C 654 B
477 E 566 B 655 C
478 A 567 A 656 C
479 A 568 C 657 E
480 C 569 A 658 E
481 A 570 A 659 C
482 B 571 C 660 E
483 C 572 B 661 B
484 A 573 B 662 B
485 C 574 A 663 C
486 D 575 A 664 B
487 D 576 C 665 A
488 A 577 B 666 E
489 B 578 A 667 D
490 B 579 B 668 D
491 C 580 B 669 D
492 A 581 B 670 A
493 B 582 C 671 C
494 D 583 C 672 C
495 C 584 B 673 D
496 B 585 A 674 A
497 D 586 E 675 C
498 C 587 A 676 B
499 B 588 D 677 C
500 C 589 B 678 A
501 D 590 B 679 C
502 C 591 A 680 E
503 A 592 D 681 B
504 C 593 B 682 B
505 C 594 E 683 D
506 D 595 E 684 A
507 A 596 A 685 B
508 A 597 B 686 A
509 B 598 A 687 B
510 D 599 C 688 D
511 E 600 A 689 A
512 B 601 D 690 A
513 A 602 C 691 D
514 D 603 C 692 D
515 B 604 C 693 C
516 D 605 A 694 D
517 A 606 E 695 C
518 B 607 D 696 A
519 D 608 A 697 C
520 C 609 B 698 A
521 D 610 D 699 A
522 D 611 D 700 C
523 B 612 E
524 C 613 A
525 D 614 E
526 A 615 D
527 A 616 A
TOPNOTCH MEDICAL BOARD PREP ANATOMY SUPEREXAM Page 94 of 94
For inquiries visit www.topnotchboardprep.com.ph or email us at topnotchmedicalboardprep@gmail.com

You might also like